Go Math Grade 3 Answer Key Chapter 7 Division Facts and Strategies Extra Practice

Go Math Grade 3 Answer Key Division Facts and Strategies Extra Practice

Are you looking for Homework Help on Go Math Grade 3 Ch 7? Then, this Go Math Grade 3 Answer Key Chapter 7 Division Facts and Strategies Extra Practice is of great help during your preparation. Assess your preparation standard and understand where you went wrong with Go Math Grade 3 Answer Key Chapter 7 Division Facts and Strategies Extra Practice.

Grade 3 Go Math Answer Key Chapter 7 Division Facts and Strategies Extra Practice

Taking the help of approaches used for solving the Problems in HMH Go Math Grade 3 Chapter 7 Division Facts and Strategies Extra Practice you can get faster results. Get to know about the topics in advance and begin your preparation. Solve the Questions from 3rd Grade Go Math Answer Key Ch 7 Extra Practice and clarify all your queries then and there itself.

Common Core – Page No. 149000

Lessons 7.1–7.2

Find the quotient. You may want to draw a quick picture to help.

Question 1.
8 ÷ 2 = ______

Answer: 4

Question 2.
______ = 14 ÷ 2

Answer: 7

Question 3.
18 ÷ 2 = ______

Answer: 9

Question 4.
______ = 12 ÷ 2

Answer: 6

Question 5.
70 ÷ 10 = ______

Answer: 7

Question 6.
50 ÷ 10 = ______

Answer: 5

Question 7.
40 ÷ 10 = ______

Answer: 4

Question 8.
90 ÷ 10 = ______

Answer: 9

Lessons 7.3–7.4

Find the quotient.

Question 9.
15 ÷ 5 = ______

Answer: 3

Explanation:

5 divides 15 into 3 equal groups. So the quotient of 15 and 5 is 3.

Question 10.
______ = 45 ÷ 5

Answer: 9

Explanation:

5 divides 45 into nine equal groups. Thus the quotient is 9.

Question 11.
______ = 10 ÷ 5

Answer: 2

Explanation:

5 divides 10 two times. So the quotient of 10 and 5 is 2.

Question 12.
40 ÷ 5 = ______

Answer: 8

Explanation:

5 divides 40 into eight equal groups. So the quotient is 8.

Question 13.
6 ÷ 3 = ______

Answer: 2

Explanation:

3 divides 6 into two equal groups. Thus the quotient is 2.

Question 14.
______ = 21 ÷ 3

Answer: 7

Explanation:

3 divides 21 into seven equal groups. So the quotient is 7.

Question 15.
______ = 24 ÷ 3

Answer: 8

Explanation:

3 divides 24 eight times. Thus the quotient is 8.

Question 16.
______ = 18 ÷ 3

Answer: 6

Explanation:

3 divides 18 into six equal parts. Therefore the quotient is 6.

Question 17.
There are 30 balloons arranged in 6 equal groups. How many balloons are in each group?
______ balloons

Answer: 5 balloons

Explanation:

Total number of balloons = 30
Number of balloons arranged in equals groups = 6
Number of balloons in each group = x
Divide the number of balloons by a number of equal groups.
= 30 ÷ 6 = 5 balloons
Therefore number of balloons in each group = 5

Question 18.
Mr. Song spends $27 on sports drinks. Each bottle costs $3. How many bottles does Mr. Song buy?
______ bottles

Answer: 9

Explanation:

Given,
Mr. Song spends $27 on sports drinks. Each bottle costs $3.
Number of bottles he bought = x
x × 3 = 27
x = 27 ÷ 3 = 9
Thus Mr. Song bought 9 bottles.

Lesson 7.5

Find the quotient.

Question 19.
28 ÷ 4 = ______

Answer: 7

Explanation:

4 divides 28 seven times. So the quotient is 7.

Question 20.
______ = 16 ÷ 4

Answer: 4

Explanation:

4 divides 16 four times. The quotient of 16 and 4 is 4.

Question 21.
______ = 20 ÷ 4

Answer: 5

Explanation:

4 divides 20 five times. Thus the quotient is 5.

Question 22.
______ = 32 ÷ 4

Answer: 8

Explanation:

4 divides 32 eight times. So the quotient of 32 and 4 is 8.

Question 23.
4)\(\bar { 3 6 }\)
______

Answer: 9

Explanation:

4 divides 36 nine times. Thus the quotient is 9.

Question 24.
4)\(\bar { 1 2 }\)
______

Answer: 3

12 ÷ 4

4 divides 12 three times. So the quotient of 12 and 4 is 3.

Question 25.
4)\(\bar { 2 4 }\)
______

Answer: 6

Explanation:

4 divides 24 six times. Thus the quotient is 6.
24 ÷ 4 = 6

Question 26.
4)\(\bar { 4 }\)
______

Answer: 1

Explanation:

Any number divides by the same number will be always 1. Thus the quotient is 1.

Find the unknown number.

Question 27.
a = 40 ÷ 4
a = _____

Answer: 10

Explanation:

Let the unknown number be a
4 divides 40 ten times.
a = 40 ÷ 4
a = 40/4 = 10
Therefore a = 10

Question 28.
0 ÷ 4 = b
b = _____

Answer: 0

Explanation:

0 divides by any number are always 0. So the value of b is 0.

Question 29.
c = 36 ÷ 4
c = ______

Answer: 9

Explanation:

Let c be the unknown number
36 ÷ 4 = C
4 divides 36 nine times.
Thus the value of c is 9.

Question 30.
8 ÷ 4 = d
d = ______

Answer: 2

Explanation:

d is the unknown number
d = 8 ÷ 4
4 divides 8 two times.
Thus the value of d is 2.

Common Core – Page No. 150000

Lessons 7.6–7.7

Find the unknown factor and quotient.

Question 1.
7 × ______ = 35 35 ÷ 7 = ______

Answer: 5, 5

Explanation:

Let the unknown factor be x
7 × x = 35
x = 35 ÷ 7
x = 5
Now check whether the related multiplication and division facts are the same.
35 ÷ 7 = 5
Thus the unknown factor and quotient are the same. So the answer is 5.

Question 2.
6 × ______ = 54 54 ÷ 6 = ______

Answer: 9, 9

Explanation:

Let the unknown factor be y
6 × y = 54
y = 54 ÷ 6
6 divides 54 nine times. Thus the unknown factor is 9.
Now check if the related multiplication and division facts are the same.
54 ÷ 6 = 9
Therefore the unknown factor and the quotient are the same I.e., 9

Question 3.
6 × ______ = 18 18 ÷ 6 = ______

Answer: 3, 3

Explanation:

Let the unknown factor be t
6 × t = 18
t = 18/6 = 3
Now check the related multiplication and division facts of the equation
18 ÷ 6 = 3
The related multiplication and division facts are the same.
Thus the unknown factor and quotient are 3.

Question 4.
7 × ______ = 49 49 ÷ 7 = ______

Answer: 7, 7

Explanation:

Let the unknown factor be x
7 × x = 49
x = 49/7 = 7
Check whether the related multiplication and division facts of the equation are the same or not.
49 ÷ 7 = 7
By thus we can say that the related facts are the same. So the unknown factor and quotient are 7.

Find the quotient.

Question 5.
36 ÷ 6 = ______

Answer: 6

Explanation:

First take the factors of 6
Factors of 6 are 3, 2
First divide 36 by 3
36 ÷ 3 = 12
Now divide 12 by 2
12 ÷ 2 = 6
So the quotient is 6.

Question 6.
48 ÷ 6 = ______

Answer: 8

Explanation:

The factors of 6 are 3 and 2
First divide by 3
48 ÷ 3 = 16
Now divide 16 by 2
16 ÷ 2 = 8
So the quotient is 8.

Question 7.
7)\(\bar { 6 3 }\)
______

Answer: 9

Explanation:

The factors of 7 are 1, 7
Divide 63 by 7
7 divides 63 nine times. So the quotient is 9.

Question 8.
7)\(\bar { 5 6 }\)
______

Answer: 8

Explanation:

The factors of 7 are 1, 7
7 divides 56 eight times. So the quotient is 8.

Lessons 7.8–7.9

Find the quotient.

Question 9.
40 ÷ 8 = ______

Answer: 5

Explanation:

Factors of 8 is 4, 2
First divide by 4
40 ÷ 4 = 10
Next divide 10 by 2
10 ÷ 2 = 5
So the quotient is 5.

Question 10.
______ = 24 ÷ 8

Answer: 3

Explanation:

The factors of 8 is 4 and 2
Divide 24 by 4
24 ÷ 4 = 6
Now divide 6 by 2
6 ÷ 2 = 3
So the quotient of 24 ÷ 8 = 3

Question 11.
72 ÷ 9 = ______

Answer: 8

Explanation:

The factors of 9 are 3, 3
First divide 72 by 3
72 ÷ 3 = 24
Next divide 24 by 3
24 ÷ 3 = 8
The quotient of 72 ÷ 9 = 8

Question 12.
______ = 81 ÷ 9

Answer: 9

Explanation:

The factors of 9 are 3, 3
Divide 81 by 3
81 ÷ 3 = 27
Now divide 27 by 3
27 ÷ 3 = 9
The quotient of 81 ÷ 9 = 9

Find the unknown number.

Question 13.
36 ÷ 9 = m
m = ______

Answer: 4

Explanation:

Let m be the unknown number
The factors of 9 are 3, 3
First, divide by 3
36 ÷ 3 = 12
Next divide 12 by 3
12 ÷ 3 = 4
So the value of m is 4.

Question 14.
18 ÷ 9 = ■
■ ______

Answer: 2

Explanation:

Take the factors of 9
Divide 18 by 3
18 ÷ 3 = 6
Now divide 6 by 3
6 ÷ 3 = 2
■ = 2

Question 15.
48 ÷ 8 = b
b = ______

Answer: 6

Let b be the unknown number
The factors of 8 is 4, 2
Divide 48 by 4
48 ÷ 4 = 12
Next divide 12 by 2
12 ÷ 2 = 6
Therefore the value of b = 6

Question 16.
56 ÷ 8 = p
p = ______

Answer: 7

Explanation:

Let p be the unknown number
The factors of 8 are 4, 2
First, divide 56 by 4
56 ÷ 4 = 14
Next divide 14 by 2
14 ÷ 2 = 6
The value of p is 6.

Lesson 7.10

Question 17.
At a store, there are 5 vases. Each vase has the same number of flowers. Sixteen flowers are sold. Now there are 24 flowers left. How many flowers were in each vase?
______ flowers

Answer: 8 flowers

Explanation:

Given that,
Number of vases = 4
Number of flowers sold = 16
Number of flowers left = 24
Total number of flowers = 16 + 24 = 40
To find the number of flowers in each vase you need to divide the total number of flowers by number of vases
= 40 ÷ 5 = 8 flowers
Thus the number of flowers in each vase = 8

Question 18.
Lizzy bought 4 bags of apples. Each bag had the same number of apples. Her mom gave her 8 more apples. Now Lizzy has 36 apples. How many apples were in each bag?
______ bags

Answer: 7 apples

Explanation:

Given,
Lizzy bought 4 bags of apples.
Number of apples her mother gave = 8
Number of apples now Lizzy have = 36
To find the actual number of apples before her mother gave, we need to subtract 8 from 36
36 – 8 = 28
Now divide the number of apples by number of bags
28 ÷ 4 = 7 apples
Therefore the number of apples in each bag = 7 apples

Lesson 7.11

Follow the order of operations to find the unknown number.

Question 19.
10 − 3 + 4 = t
t = ______

Answer: 11

Explanation:

First subtract from left to right and then add
10 – 3 + 4 = 7 + 4 = 11
Therefore t = 11

Question 20.
8 − 3 × 2 = p
p = ______

Answer: 2

Explanation:

First multiple 3 and 2
3 × 2 = 6
And then subtract 6 from 8, you get 2
Thus p = 2

Question 21.
24 ÷ 6 + 2 = w
w = ______

Answer: 6

Explanation:

First, divide 24 and 6
24 ÷ 6 = 4
Now add from left to right
4 + 2 = 6
Therefore the unknown number w = 6

Conclusion

Solve the Problems in Go Math Grade 3 Chapter 7 Division Facts and Strategies Extra Practice and score better grades in your exams. If you want homework help you can always go through Go Math Grade 3 Answer Key Chapter 7 Division Facts and Strategies. Become pro in the concepts of Grade 3 Chapter 7 Division Facts and Strategies with our Step by Step Solution provided here.

Go Math Grade 7 Answer Key Chapter 1 Adding and Subtracting Integers

go-math-grade-7-chapter-1-adding-and-subtracting-integers-answer-key

Are you searching for Go Math Grade 7 Solutions? then what are you waiting for Download Go Math Grade 7 Answer Key Chapter 1 Adding and Subtracting Integers pdf for free of cost. Here we are providing the step by step explanation for all the questions in Go Math Grade 7 Answer Key for Chapter 1 Adding and Subtracting Integers.

Go Math Grade 7 Answer Key Chapter 1 Adding and Subtracting Integers

We suggest the students check out the topics of Grade 7 chapter 1 before you start your preparation for exams. The chapter Adding and Subtracting Integers contains topics such as Adding Integers with the Same Sign, Different sign, Subtracting Integers, Applying Addition and Subtraction of Integers, and so on. Tap the below links and try to solve the questions provided in the Go Math Grade 7 Solution Key Chapter 1 Adding and Subtracting Integers.

Chapter 1 – Adding Integers with the Same Sign

Chapter 1 – Adding Integers with Different Signs

Chapter 1 – Subtracting Integers

Chapter 1 – Applying Addition and Subtraction of Integers

Chapter 1 – MODULE 1

Adding Integers with the Same Sign – Guided Practice – Page No. 10

Find each sum.

Question 1.
-5 + (-1)
Go Math Grade 7 Answer Key Chapter 1 Adding and Subtracting Integers Lesson 1: Adding Integers with the Same Sign img 1
a. How many counters are there?
_______ counters

Answer: 6

Explanation:
By seeing the above pictures we can say that there are 6 counters.

Question 1.
b. Do the counters represent positive or negative numbers?
____________

Answer: negative numbers

Explanation:
The counters are red so they represent negative numbers.

Question 1.
c. -5 + (-1) =
_______

Answer: -6

Explanation:
There are 6 counters so -5 + (-1) = – 6

Question 2.
-2 + (-7)
Go Math Grade 7 Answer Key Chapter 1 Adding and Subtracting Integers Lesson 1: Adding Integers with the Same Sign img 2
a. How many counters are there?
_______ counters

Answer: 9

Explanation:
The above figure shows that there are 9 counters.

Question 2.
b. Do the counters represent positive or negative numbers?
____________

Answer: negative numbers

Explanation:
The counters are red so they represent the negative numbers.

Question 2.
c. -2 + (-7) =
_______

Answer: -9

Explanation:
There are 9 counters so -2 + (-7) = -9
The answer is -9.

Model each addition problem on the number line to find each sum.

Question 3.
-5 + (-2) =
Go Math Grade 7 Answer Key Chapter 1 Adding and Subtracting Integers Lesson 1: Adding Integers with the Same Sign img 3
_______

Answer: -7

Explanation:
Remember if the number being added is positive more number of units going to the right and if the number being added is negative more number of units to the left.
Since we are adding the negative number starting from -5, we move 2 units to the left. This results in -7.

Question 4.
-1 + (-3) =
Go Math Grade 7 Answer Key Chapter 1 Adding and Subtracting Integers Lesson 1: Adding Integers with the Same Sign img 4
_______

Answer: -4

Explanation:
Remember if the number being added is positive more number of units going to the right and if the number being added is negative more number of units to the left.
Since we are adding a negative number starting from -1, we move 3 units to left. This results in -4.

Question 5.
-3 + (-7) =
Go Math Grade 7 Answer Key Chapter 1 Adding and Subtracting Integers Lesson 1: Adding Integers with the Same Sign img 5
_______

Answer: -10

Explanation:
Remember if the number being added is positive more number of units going to the right and if the number being added is negative more number of units to the left.
Since we are adding a negative number starting from -3, we move 7 units to left. This results in -10.

Question 6.
-4 + (-1) =
Go Math Grade 7 Answer Key Chapter 1 Adding and Subtracting Integers Lesson 1: Adding Integers with the Same Sign img 6
_______

Answer: -5

Explanation:
Remember if the number being added is positive more number of units going to the right and if the number being added is negative more number of units to the left.
Since we are adding a negative number starting from -4, we move 1 unit to left. This results in -5.

Question 7.
-2 + (-2) =
Go Math Grade 7 Answer Key Chapter 1 Adding and Subtracting Integers Lesson 1: Adding Integers with the Same Sign img 7
_______

Answer: -4

Explanation:
Remember if the number being added is positive more number of units going to the right and if the number being added is negative more number of units to the left.
Since we are adding the negative number starting -2, we move 2 units to the left which gives the result -4.

Question 8.
-6 + (-8) =
Go Math Grade 7 Answer Key Chapter 1 Adding and Subtracting Integers Lesson 1: Adding Integers with the Same Sign img 8
_______

Answer: -14

Explanation:
Remember if the number being added is positive more number of units going to the right and if the number being added is negative more number of units to the left.
Since we are adding the negative number starting from -6 we have to move 8 units to left which shows the result -14.

Find each sum.

Question 9.
-5 + (-4) =
_______

Answer: -9

Explanation:
In adding two integers with the same signs you add both the integers and keep the common sign.
Since -5 + (-4) has the same sign we add their absolute value and keep the same sign.
-5 + (-4) = -(5 + 4) = -9

Question 10.
-1 + (-10) =
_______

Answer: -11

Explanation:
In adding two integers with the same signs you add both the integers and keep the common sign.
Since -1 + (-10) has the same sign we add their absolute value and keep the same sign.
-1 + (-10) = -(1 + 10)
= -11
So the answer is -11.

Question 11.
-9 + (-1) =
_______

Answer: -10

Explanation:
In adding two integers with the same signs you add both the integers and keep the common sign.
Since -9 + (-1) has the same sign we add their absolute value and keep the same sign.
-9 + -1 = -(9 + 1)
= -10
Thus the answer is -10.

Question 12.
-90 + (-20) =
_______

Answer: -110

Explanation:
In adding two integers with the same signs you add both the integers and keep the common sign.
Since -90 + (-20) has the same sign we add their absolute value and keep the same sign.
-90 + (-20) = -(90 + 20)
= -110
The answer is -110.

Question 13.
-52 + (-48) =
_______

Answer: -100

Explanation:
In adding two integers with the same signs you add both the integers and keep the common sign.
Since -52 + (-48) has the same sign we add their absolute value and keep the same sign.
-52 + (-48) = -(52 + 48)
= -100
The answer is -100.

Question 14.
5 + 198 =
_______

Answer: 203

Explanation:
In adding two integers with the same signs you add both the integers and keep the common sign.
Since 5 + 198 has the same sign we add their absolute value and keep the same sign.
5 + 198 = 203
The answer is 203.

Question 15.
-4 + (-5) + (-6) =
_______

Answer: -15

Explanation:
In adding two integers with the same signs you add both the integers and keep the common sign.
Since -4 + (-5) + (-6) has the same sign we add their absolute value and keep the same sign.
-4 + (-5) + (-6) = -(4 + 5 + 6)
= -15
The answer is -15.

Question 16.
-50 + (-175) + (-345) =
_______

Answer: -570

Explanation:
In adding two integers with the same signs you add both the integers and keep the common sign.
Since -50 + (-175) + (-345) has the same sign we add their absolute value and keep the same sign.
-50 + (-175) + (-345)
= -(50 + 175 + 345)
= -570
The answer for -50 + (-175) + (-345) is -570.

Question 17.
How do you add integers with the same sign?
Type below:
______________

Answer:

First, you should their absolute values and keep the common sign. If both signs are positive, the answer will be positive. If both signs are negative, the answer will be negative.

Adding Integers with the Same Sign – Independent Practice – Page No. 11

Question 18.
Represent Real-World Problems Jane and Sarah both dive down from the surface of a pool. Jane first dives down 5 feet and then dives down 3 more feet. Sarah first dives down 3 feet, and then dives down 5 more feet.
a. Multiple Representations Use the number line to model the equation -5 + (-3) = -3 + (-5).
Go Math Grade 7 Answer Key Chapter 1 Adding and Subtracting Integers Lesson 1: Adding Integers with the Same Sign img 9
Type below:
______________

Answer: -8

Explanation:
Start at -3 and move 5 units down for one number line. Next, start at -5 and move down 3 units for another number line.
Both have a final answer of -8.
So, -5 + (-3) = -3 + (-5) = -8.

Question 18.
b. Does the order in which you add two integers with the same sign affect the sum? Explain.
_______

Answer: no

Explanation:

Based on the results of part a, the order doesn’t matter. Since the commutative properties of addition hold for the sum of two negative numbers.

Question 19.
A golfer has the following scores for a 4-day tournament.
Go Math Grade 7 Answer Key Chapter 1 Adding and Subtracting Integers Lesson 1: Adding Integers with the Same Sign img 10
What was the golfer’s total score for the tournament?
_______

Answer: -11

Explanation:
The total score is sum of each day’s score
= -3 + (-1) + (-5) + (-2)
= -(3 + 1 + 5 + 2)
= -11
Thus the total score for 4 days tournament is -11.

Question 20.
A football team loses 3 yards on one play and 6 yards on another play. Write a sum of negative integers to represent this situation. Find the sum and explain how
it is related to the problem.
The sum = _______

Answer: -9

Explanation:
The negative sum of 3 yards and 6 yards is
-3 + (-6) = -(3 + 6)
= -9
Thus the negative sum is -9.

Question 21.
When the quarterback is sacked, the team loses yards. In one game, the quarterback was sacked four times. What was the total sack yardage?
Go Math Grade 7 Answer Key Chapter 1 Adding and Subtracting Integers Lesson 1: Adding Integers with the Same Sign img 11
_______

Answer: -54

Explanation:
The total sack yardage = -14 + (-5) + (-12) + (-23)
= -(14 + 5 + 12 + 23)
= -54
Therefore the total sack yardage is -54.

Question 22.
Multistep The temperature in Jonestown and Cooperville was the same at 1:00. By 2:00, the temperature in Jonestown dropped 10 degrees, and the temperature in Cooperville dropped 6 degrees. By 3:00, the temperature in Jonestown dropped 8 more degrees, and the temperature in Cooperville dropped 2 more degrees.
a. Write an equation that models the change to the temperature in Jonestown since 1:00.
Type below:
______________

Answer: J = T – 18

Explanation:
Let J be the final temperature and T be the initial temperature. Then the equation is J = T + (-10) + (-8)
J = T – 18

Question 22.
b. Write an equation that models the change to the temperature in Cooperville since 1:00.
Type below:
______________

Answer: C = T – 8

Explanation:
Let C be the final temperature and T be the initial temperature. Then the equation is C = T + (-6) + (-2)
C = T – 8

Question 22.
c. Where was it colder at 3:00, in Jonestown or Cooperville?
__________

Answer: Jonestown

Explanation:
Since they both started at the same temperature and Jonestown dropped a total of 18 degrees while Cooperville dropped a total of 8 degrees, Jonestown is colder.

Adding Integers with the Same Sign – Page No. 12

Question 23.
Represent Real-World Problems Julio is playing a trivia game. On his first turn, he lost 100 points. On his second turn, he lost 75 points. On his third turn, he lost 85 points. Write a sum of three negative integers that models the change to Julio’s score after his first three turns.
Type below:
______________

Answer: -260 points

Explanation:
The change in his total score is the sum of the losses = -100 + (-75) + (-85)
= -(100 + 75 + 85)
= -260 points
Thus Julio’s score after his first three turns is -260 points.

H.O.T. FOCUS ON HIGHER ORDER THINKING

Question 24.
Multistep On Monday, Jan made withdrawals of $25, $45, and $75 from her savings account. On the same day, her twin sister Julie made withdrawals of $35, $55, and $65 from her savings account.
a. Write a sum of negative integers to show Jan’s withdrawals on Monday. Find the total amount Jan withdrew.
Jan withdrew $ _______

Answer: 145

Explanation:
Each withdrawal is represented by a negative integer so find the sum of those negative integers = -25 + (-45) + (-75)
= -(25 + 45 + 75)
= -145
Thus Jan withdrew $145.

Question 24.
b. Write a sum of negative integers to show Julie’s withdrawals on Monday. Find the total amount Julie withdrew.
Julie withdrew $ _______

Answer: 155

Explanation:
Each withdrawal is represented by a negative integer so find the sum of those negative integers
= -35 + (-55) + (-65)
= – (35 + 55 + 65)
= -155
The total amount Julie withdrew is -$155.

Question 24.
c. Julie and Jan’s brother also withdrew money from his savings account on Monday. He made three withdrawals and withdrew $10 more than Julie did. What are three possible amounts he could have withdrawn?
Type below:
______________

Answer:

If he withdrew $10 more than Julie then he withdrew $165 in total. The possible amounts could then be $35, $55, $75.

Question 25.
Communicate Mathematical Ideas Why might you want to use the Commutative Property to change the order of the integers in the following sum before adding?
-80 + (-173) + (-20)
Type below:
______________

Answer: You can add 80 and 20 easily to get 100 which is then easier to add 173. So changing the order makes the problem easier to do mentally.

Question 26.
Critique Reasoning The absolute value of the sum of two different integers with the same sign is 8. Pat says there are three pairs of integers that match this description. Do you agree? Explain.
__________

Answer: Disagree

Explanation:
Pat is saying that x + y = 8 is true for only three pairs of numbers with the same sign. This is not true though. The pairs could be 1, 7, 2 and 6, 3, 5, 4 and -4, -1 and -7, -2 and -6, -3 and -5 and -4, -4.

Adding Integers with Different Signs – Guided Practice – Page No. 16

Use a number line to find each sum.

Question 1.
9 + (-3) =
Go Math Grade 7 Answer Key Chapter 1 Adding and Subtracting Integers Lesson 2: Adding Integers with Different Signs img 12
_______

Answer: 6

Explanation:
Remember if the number being added is positive more number of units going to the right and if the number being added is negative more number of units to the left.
Since we are adding a negative number starting from 9, move 3 units to the left. This results in 6.

Question 2.
-2 + 7 =
Go Math Grade 7 Answer Key Chapter 1 Adding and Subtracting Integers Lesson 2: Adding Integers with Different Signs img 13
_______

Answer: 5

Explanation:
Remember if the number being added is positive more number of units going to the right and if the number being added is negative more number of units to the left.
Since we are adding a positive number starting from -2 we move 7 units to the right. This results in 5.

Question 3.
-15 + 4 =
Go Math Grade 7 Answer Key Chapter 1 Adding and Subtracting Integers Lesson 2: Adding Integers with Different Signs img 14
_______

Answer: -11

Explanation:
Remember if the number being added is positive more number of units going to the right and if the number being added is negative more number of units to the left.
Since we are adding a positive number starting from -15, we move 4 units to the right. This results in -11

Question 4.
1 + (-4) =
Go Math Grade 7 Answer Key Chapter 1 Adding and Subtracting Integers Lesson 2: Adding Integers with Different Signs img 15
_______

Answer: -3

Explanation:
Remember if the number being added is positive more number of units going to the right and if the number being added is negative more number of units to the left.
Since we are adding the negative number starting from 1, we move 4 units to the left. This results in -3.

Circle the zero pairs in each model. Find the sum.

Question 5.
-4 + 5 =
Go Math Grade 7 Answer Key Chapter 1 Adding and Subtracting Integers Lesson 2: Adding Integers with Different Signs img 16
_______

Answer: 1

Explanation:
In adding two integers with the same sign, add their absolute value, and keep the common sign.
When adding two integers with opposite signs, subtract the smaller absolute value from the larger and keep the sign of the number with the larger absolute value.
Above is an illustration of which are the zero pairs and what remains. In this item 1 yellow counter remains which means the sum is 1.

Question 6.
-6 + 6 =
Go Math Grade 7 Answer Key Chapter 1 Adding and Subtracting Integers Lesson 2: Adding Integers with Different Signs img 17
_______

Answer: 0

Explanation:
In adding two integers with the same sign, add their absolute value, and keep the common sign.
When adding two integers with opposite signs, subtract the smaller absolute value from the larger and keep the sign of the number with the larger absolute value.
Above is an illustration of which are the zero pairs and what remains. In this item, there are no counters so the sum is 0.

Question 7.
2 + (-5) =
Go Math Grade 7 Answer Key Chapter 1 Adding and Subtracting Integers Lesson 2: Adding Integers with Different Signs img 18
_______

Answer: -3

Explanation:
In adding two integers with the same sign, add their absolute value, and keep the common sign.
When adding two integers with opposite signs, subtract the smaller absolute value from the larger and keep the sign of the number with the larger absolute value.
Above is an illustration of which are the zero pairs and what remains. In this item, 3 red counters are remaining so the sum is -3.

Question 8.
-3 + 7 =
Go Math Grade 7 Answer Key Chapter 1 Adding and Subtracting Integers Lesson 2: Adding Integers with Different Signs img 19
_______

Answer: 4

Explanation:
In adding two integers with the same sign, add their absolute value, and keep the common sign.
When adding two integers with opposite signs, subtract the smaller absolute value from the larger and keep the sign of the number with the larger absolute value.
Above is an illustration of which are the zero pairs and what remains. In this item, 4 yellow counters remain so the sum is 4.

Find each sum.

Question 9.
-8 + 14 =
_______

Answer: 6

Explanation:
In adding two integers with the same sign, add their absolute value, and keep the common sign.
When adding two integers with opposite signs, subtract the smaller absolute value from the larger and keep the sign of the number with the larger absolute value.
Here we are the opposite number with the negative number.
-8 + 14 = 6
The larger number is having a positive sign so the sum is 6.

Question 10.
7 + (-5) =
_______

Answer: 2

Explanation:
In adding two integers with the same sign, add their absolute value, and keep the common sign.
When adding two integers with opposite signs, subtract the smaller absolute value from the larger and keep the sign of the number with the larger absolute value.
7 + (-5) = 7 – 5 = 2
The larger number is having a positive sign so the sum is 2.

Question 11.
5 + (-21) =
_______

Answer: -16

Explanation:
In adding two integers with the same sign, add their absolute value, and keep the common sign.
When adding two integers with opposite signs, subtract the smaller absolute value from the larger and keep the sign of the number with the larger absolute value.
5 + (-21) = 5 – 21 = -17
The larger number is having a negative number so the sum is -17.

Question 12.
14 + (-14) =
_______

Answer: 0

Explanation:
In adding two integers with the same sign, add their absolute value, and keep the common sign.
When adding two integers with opposite signs, subtract the smaller absolute value from the larger and keep the sign of the number with the larger absolute value.
14 + (-14) =14 – 14 = 0

Question 13.
0 + (-5) =

Answer: -5

Explanation:
In adding two integers with the same sign, add their absolute value, and keep the common sign.
When adding two integers with opposite signs, subtract the smaller absolute value from the larger and keep the sign of the number with the larger absolute value.
0 + (-5) = 0 – 5 = -5
The larger is having the negative sign so the sum is -5.

Question 14.
32 + (-8) =
_______

Answer: 24

Explanation:
In adding two integers with the same sign, add their absolute value, and keep the common sign.
When adding two integers with opposite signs, subtract the smaller absolute value from the larger and keep the sign of the number with the larger absolute value.
32 + (-8) = 32 – 8 = 24
The larger number is having a positive sign so the sum is 24.

Question 15.
Describe how to find the sums -4 + 2 and -4 + ( -2 ) on a number line.
Type below:
____________

Answer: -2

Explanation:
Start at -4 and move 2 units up for one number line. Next, start at -4 and move down 2 units for another number line.
-4 + 2 = -2
-4 – 2 = -6

Adding Integers with Different Signs – Independent Practice – Page No. 17

Find each sum.

Question 16.
-15 + 71 =
_______

Answer: 56

Explanation:
In adding two integers with the same sign, add their absolute value, and keep the common sign.
When adding two integers with opposite signs, subtract the smaller absolute value from the larger and keep the sign of the number with the larger absolute value.
-15 + 71 = |71| – |-15|
= 71 – 15
= 56

Question 17.
-53 + 45 =
_______

Answer: -8

Explanation:
In adding two integers with the same sign, add their absolute value, and keep the common sign.
When adding two integers with opposite signs, subtract the smaller absolute value from the larger and keep the sign of the number with the larger absolute value.
-53 + 45 = |-53| – |45|
53 – 45 = 8
The larger number is having the negative symbol so the answer is -8.

Question 18.
-79 + 79 =
_______

Answer: 0

Explanation:
In adding two integers with the same sign, add their absolute value, and keep the common sign.
When adding two integers with opposite signs, subtract the smaller absolute value from the larger and keep the sign of the number with the larger absolute value.
79 + (-79) = |79| – |-79|
79 – 79 = 0

Question 19.
-25 + 50 =
_______

Answer: 25

Explanation:
In adding two integers with the same sign, add their absolute value, and keep the common sign.
When adding two integers with opposite signs, subtract the smaller absolute value from the larger and keep the sign of the number with the larger absolute value.
-25 + 50 = |50| – |-25|
50 – 25 = 25

Question 20.
18 + (-32) =
_______

Answer: -14

Explanation:
In adding two integers with the same sign, add their absolute value, and keep the common sign.
When adding two integers with opposite signs, subtract the smaller absolute value from the larger and keep the sign of the number with the larger absolute value.
18 + (-32) = |-32| – |18|
32 – 18 = 14
The larger number is having a negative sign so the answer is -14.

Question 21.
5 + (-100) =
_______

Answer: -95

Explanation:
In adding two integers with the same sign, add their absolute value, and keep the common sign.
When adding two integers with opposite signs, subtract the smaller absolute value from the larger and keep the sign of the number with the larger absolute value.
5 + (-100) = |-100| – |5|
100 – 5 = 95
The larger number is having a negative sign so the answer is -95.

Question 22.
-12 + 8 + 7 =
_______

Answer: 3

Explanation:
In adding two integers with the same sign, add their absolute value, and keep the common sign.
When adding two integers with opposite signs, subtract the smaller absolute value from the larger and keep the sign of the number with the larger absolute value.
-12 + 8 + 7 = -12 + (8 + 7)
For the terms have different signs, we subtract the lesser absolute value from the greater absolute value and use the sign of the integer with the greater absolute value for the sum: 3
-12 + 15 = 3

Question 23.
-8 + (-2) + 3 =
_______

Answer: -7

Explanation:
In adding two integers with the same sign, add their absolute value, and keep the common sign.
When adding two integers with opposite signs, subtract the smaller absolute value from the larger and keep the sign of the number with the larger absolute value.
-(8 + 2) + 3
For the terms have different signs, we subtract the lesser absolute value from the greater absolute value and use the sign of the integer with the greater absolute value for the sum: -7
-10 + 3 = -7

Question 24.
15 + (-15) + 200 =
_______

Answer: 200

Explanation:
We are given the expression:
15 + (-15) + 200 = 0 + 200
The sum of the opposite number is 0.
0 + 200 = 200

Question 25.
-500 + (-600) + 1200 =
_______

Answer: 100

Explanation:
We are given the expression:
-500 + (-600) + 1200 = -(500 + 600) + 1200
-1100 + 1200 = +100

Question 26.
A football team gained 9 yards on one play and then lost 22 yards on the next. Write a sum of integers to find the overall change in field position. Explain your answer.
Type below:
____________

Answer: -13

Explanation:
9 + (-22)
Since 9 yards are gained, the field position is changed by +9 and since 22 yards are lost the field position will be changed by -22, so we have:
(+9) + (-22) = -(22 – 9) = -13
We computer the overall change in field position: -13

Question 27.
A soccer team is having a car wash. The team spent $55 on supplies. They earned $275, including tips. The team’s profit is the amount the team made after paying for supplies. Write a sum of integers that represents the team’s profit.
Type below:
____________

Answer: 220

Explanation:
(-55) + (+275)
The money spent on supplies diminish the profit, so they contribute to the profit with -55, while the earned money increase the profit, so they contribute to the profit with +275.
The sum of integers that represents the team’s profit is:
(-55) + (+275) = (275 -55) = 220

Question 28.
As shown in the illustration, Alexa had a negative balance in her checking account before depositing a $47.00 check. What is the new balance of Alexa’s checking account?
Go Math Grade 7 Answer Key Chapter 1 Adding and Subtracting Integers Lesson 2: Adding Integers with Different Signs img 20
$ _______

Answer: 0

Explanation:
(-47) + 47 = 0
The new balance consists of the sum between the old balance and the amount she deposits: 0

Question 29.
The sum of two integers with different signs is 8. Give two possible integers that fit this description.
Type below:
____________

Answer: 10 and -2

Explanation:
10 and 2
10 – 2 = 8
Because the sum of the two numbers is positive and the two numbers have different signs, it means the absolute value of the positive number is 8 units greater than the absolute value of the negative number. First, we find two positive numbers which are different by 8, which will be the positive values of our numbers.
10 and -2
Our positive number will be greater one while our negative number will be the smaller one (-2). So the desired numbers are:
10 + (-2) = 8
12 + (-4) = 8
15 + (-7) = 8

Question 30.
Multistep Bart and Sam played a game in which each player earns or loses points in each turn. A player’s total score after two turns is the sum of his points earned or lost. The player with the greater score after two turns wins. Bart earned 123 points and lost 180 points. Sam earned 185 points and lost 255 points. Which person won the game? Explain.
____________

Answer: Bart

Explanation:
123 + (-180) = -(180 – 123) = -57
The person who has the greatest number of points after 2 turns win.
We find the number of points Bart has, by adding the number of points from the two turns:
185 + (-255) = -(255 – 185) – 70
We find the number of points Sam has, by adding the number of points from the two turns:
The winner is Bart because -57 is greater than -70.

Adding Integers with Different Signs – Page No. 18

H.O.T. FOCUS ON HIGHER ORDER THINKING

Question 31.
Critical Thinking Explain how you could use a number line to show that -4 + 3 and 3 + (-4) have the same value. Which property of addition states that these sums are equivalent?
____________ Property of Addition

Answer: Commutative property of addition

Explanation:
In order to prove that -4 + 3 and 3 + (-4) have the same value we use the number line twice: -1 we start from -4 and we move 3 units in the positive direction to the right we get the sum -1.
We start from 3 and we move 4 units in the negative direction to the left where we find again -1.
The property of addition which states that the sum is the same no matter the order in which we add the terms is called commutative property.

Question 32.
Represent Real-World Problems Jim is standing beside a pool. He drops a weight from 4 feet above the surface of the water in the pool. The weight travels a total distance of 12 feet down before landing on the bottom of the pool. Explain how you can write a sum of integers to find the depth of the water.
Type below:
____________

Answer: 12 + (-4) = 8

Explanation:
Given that,
Jim is standing beside a pool.
He drops weight from 4 feet above the surface of the water in the pool.
The weight travels a total distance of 12 feet down before landing on the bottom of the pool.
12 + (-4) = 12 – 4 = 8
The depth of the water can be calculated by adding to the total distance of 12 feet the negative distance of -4 feet.

Question 33.
Communicate Mathematical Ideas Use counters to model two integers with different signs whose sum is positive. Explain how you know the sum is positive.
Type below:
____________

Answer: The result is positive because there are more positive counters than negative counters.

Explanation:
○○○○○○○
●●●
Let’s model the sum 7 + (-3) using counters we use 7 white counters for the positive numbers and 3 black counters for the negative numbers.
We pair each white counter with a black counter their sum is being 0.
The result is +4 as we are left with 4 white counters.
The result is positive because there are more positive counters than negative counters.

Question 34.
Analyze Relationships You know that the sum of -5 and another integer is a positive integer. What can you conclude about the sign of the other integer? What can you conclude about the value of the other integer? Explain.
Type below:
____________

Answer:
We know that the sum is -5 and another integer is a positive integer. This means that the absolute value of the positive number is greater than the absolute value of -5.
The absolute value of -5 is 5, so the absolute value of the positive integer must be greater than 5. But because the number is positive, its absolute value is the number itself, so the positive number must be greater than 5.
-5 + 7 = 7 – 5 = 2

Subtracting Integers – Guided Practice – Page No. 22

Explain how to find each difference using counters.

Question 1.
5 – 8 =
_______

Answer: -3

Explanation:
5 – 8
We start with 5 black counters.
Since we have to subtract more black counters than we have (5 instead of 8), we add 3 zero pairs:
We subtract the 8 black counters: -3
We are left with 3 white counters, which means the result is -3.

Question 2.
-5 – (-3) =
_______

Answer: -2

Explanation:
-5 – (-3)
We have to find the difference:
We start with 5 black counters.
we subtract 3 black counters from the 5 black counters: -2
We are left with 2 black counters, which means the result is: -2

Use a number line to find each difference.

Question 3.
− 4 − 5 = − 4 + ( _______ ) = _______

Answer: -9

Explanation:
-4 – 5
We have to compute the difference:
-4 – 5 = -(4 + 5)
On a number line, we start from -4 and we go to the left by 5 units:
-4 -5 = -9

Question 4.
1 − 4 = 1 + ( _______ ) = _______

Answer: -3

Explanation:
1 – 4
We have to compute the difference:
1 – 4 = 1 + (-4)
We replace the subtraction by addition with the opposite:
On a number line, we start from 1 and we go to the left by 4 units:
1 – 4 = – 3
The result is -3.

Solve.

Question 5.
8 – 11 =
_______

Answer: -3

Explanation:
8 – 11
We have to perform the subtraction:
8 – 11 = 8 + (-11)
We replace subtraction by addition with the opposite number:
8 + (-11) = -3
We use the rule for adding integers: -3

Question 6.
-3 – (-5) =
_______

Answer: 2

Explanation:
-3 – (-5)
We have to perform the subtraction:
-3 – (-5) = -3 + 5
We replace subtraction by addition with the opposite number:
-3 + 5 = 2
We use the rule for adding integers: 2

Question 7.
15 – 21 =
_______

Answer: -6

Explanation:
15 – 21
We have to perform the subtraction:
15 – 21 = 15 + (21)
We replace subtraction by addition with the opposite number:
15 + (-21) = -6
We use the rule for adding intergers: -6

Question 8.
-17 – 1 =
_______

Answer: -18

Explanation:
We have to perform the subtraction:
-17 – 1 = -17 + (-1)
We replace subtraction by addition with the opposite number:
-17 + (-1) = -18
We use the rule for adding integers: -18

Question 9.
0 – (-5) =
_______

Answer: 5

Explanation:
We have to perform the subtraction:
0 – (-5) = 0 + 5
We replace subtraction b addition with the opposite number:
0 + 5 = 5
We use the rule for adding integers: 5

Question 10.
1 – (-18) =
_______

Answer: 19

Explanation:
We have to perform the subtraction:
1 – (-18) = 1 + 18
We replace subtraction by addition with the opposite number:
1 + 18 = 19
We use the rule for adding integers: 19

Question 11.
15 – 1 =
_______

Answer: 14

Explanation:
We have to perform the subtraction:
15 – 1 = 14
We subtract the numbers directly as in this case it is simpler than to replace subtraction by addition with the opposite: 14

Question 12.
-3 – (-45) =
_______

Answer: 42

Explanation:
We have to perform the subtraction:
-3 – (-45) = -3 + 45
We replace subtraction by addition with the opposite number:
-3 + 45 = 42
We use the rule for adding integers: 42

Question 13.
19 – (-19) =
_______

Answer: 38

Explanation:
We have to perform the subtraction:
19 – (-19) = 19 + 19
We replace subtraction by addition with the opposite number:
19 + 19 = 38
We use the rule for adding integers: 38

Question 14.
-87 – (-87) =
_______

Answer: 0

Explanation:
We have to perform the subtraction:
-87 – (-87) = -87 + 87
We replace subtraction by addition with the opposite number:
-87 + 87 = 0
Ths um of opposite numbers is 0

Question 15.
How do you subtract an integer from another integer without using a number line or counters? Give an example.
Type below:
____________

Answer:
Integers with the same sign: Change to additions values then keep the common sign.
integers with the different signs: Change to addition absolute value from larger value, the keep sign of larger absolute value.

Subtracting Integers – Independent Practice – Page No. 23

Question 16.
Theo had a balance of -$4 in his savings account. After making a deposit, he has $25 in his account. What is the overall change to his account?
$ _______

Answer: $29

Explanation:
Theo had a balance of -$4 in his savings account.
After making a deposit, he has $25 in his account.
25 – (-4)
The overall change to the account is the difference between the amount in the account after making the deposit and the amount before it, so we have to perform the subtraction.
25 – (-4) = 25 + 4
We change subtraction to addition with the opposite number:
25 + 4 = 29
We apply the rules for adding integers: $29

Question 17.
As shown, Suzi starts her hike at an elevation below sea level. When she reaches the end of the hike, she is still below sea level at -127 feet. What was the change in elevation from the beginning of Suzi’s hike to the end of the hike?
Go Math Grade 7 Answer Key Chapter 1 Adding and Subtracting Integers Lesson 3: Subtracting Integers img 21
_______ feet

Answer: 98 feet

Explanation:
127 – (-225)
The change in the elevation from the beginning of Suzi’s hike to the end of the hike is the difference between the elevation at the end of the hike and the elevation at the beginning of it, so we have to perform the subtraction:
-127 – (-225) = -127 + 225
We change subtraction to addition with the opposite number:
-127 + 225 = 98
We apply the rules for adding integers: 98 feet

Question 18.
The record high January temperature in Austin, Texas, is 90 °F. The record low January temperature is -2 °F. Find the difference between the high and low temperatures.
_______ °F

Answer: 92°F

Explanation:
90 – (-2)
We have to find the difference between the high and low temperature, so we have to perform the subtraction:
90 – (-2) = 90 + 2
We change subtraction to addition with the opposite number:
90 + 2 = 92 feet

Question 19.
Cheyenne is playing a board game. Her score was -275 at the start of her turn, and at the end of her turn her score was -425. What was the change in Cheyenne’s score from the start of her turn to the end of her turn?
_______ °C

Answer: -150

Explanation:
-425 – (-275)
The change in Cheyenne’s score from the start of her turn to the end of her turn in the result of the subtraction:
-425 – (-275) = -425 + 275 = -150 points

Question 20.
A scientist conducts three experiments in which she records the temperature of some gases that are being heated. The table shows the initial temperature and the
final temperature for each gas.
Go Math Grade 7 Answer Key Chapter 1 Adding and Subtracting Integers Lesson 3: Subtracting Integers img 22
a. Write a difference of integers to find the overall temperature change for each gas.
Gas A: __________ °C increase
Gas B: __________ °C increase
Gas C: __________ °C increase

Answer:
We determine the overall change of temperature for each gas by subtracting the initial temperature from the final temperature.
Gas A:
-8 – (-21) = -8 + 21 = 13
Gas B:
12 – (-12) = 12 + 12 = 24
Gas C:
-15 – (-19) = -15 + 19 = 4

Question 20.
What If? Suppose the scientist performs an experiment in which she cools the three gases. Will the changes in temperature be positive or negative for this experiment? Why?
__________

Answer: Negative

Explanation:
Cooling the gases means diminishing their temperature, thus their final temperature will be lower than the initial temperature, so the change in temperature will be negative.

Subtracting Integers – Page No. 24

Question 21.
Analyze Relationships For two months, Nell feeds her cat Diet Chow brand cat food. Then for the next two months, she feeds her cat Kitty Diet brand cat food. The table shows the cat’s change in weight over 4 months.
Go Math Grade 7 Answer Key Chapter 1 Adding and Subtracting Integers Lesson 3: Subtracting Integers img 23
Which brand of cat food resulted in the greatest weight loss for Nell’s cat? Explain.
__________

Answer: Diet Chow

Explanation:
(-8) + (-18) = -26
We count the total change of weight resulted after using the diet chow for two months.
We count the total change of weight resulted after using the Kitty Diet for two months:
3 + (-19) = -16
This means that by using the Diet Chow the cat lost 26 oz, while using the Kitty Diet she lost 16 oz, thus the greatest loss of weight resulted in using the Diet Chow food.

FOCUS ON HIGHER ORDER THINKING

Question 22.
Represent Real-World Problems Write and solve a word problem that can be modeled by the difference -4 – 10.
Type below:
____________

Answer:
We have to write and solve a problem using the difference:
-4 – 10
For example:
Yesterday the temperature was -4 degrees. Today the temperature decreased by 10 degrees. What is the temperature today?
– 4 – 10 =- + (-10) = -14

Question 23.
Explain the Error When Tom found the difference -11 – (-4), he got -15. What might Tom have done wrong?
Type below:
____________

Answer:
We have to find the error in computing the difference:
-11 – (-4) = -15
In order to perform subtraction, Tom replaced it by addition, but he was wrong in adding -4 instead of adding its opposite 4.
The correct form is -11 – (-4) = -11 + 4 = -7

Question 24.
Draw Conclusions When you subtract one negative integer from another, will your answer be greater than or less than the integer you started with? Explain your reasoning and give an example.
____________ the integer

Answer: Greater

Explanation:
When we subtract one negative integer from another we will get an integer which is greater than the integer we started with because subtracting a negative integer from the initial number can be replaced by adding the opposite of that negative integer, which is a positive integer, thus the result will definitely be greater than the initial number.
-10 – (-3) = -10 + 3 = -7
-2 – (-7) = -2 + 7 = -5

Question 25.
Look for a Pattern Find the next three terms in the pattern 9, 4, −1, −6, −11, … . Then describe the pattern.
9, 4, -1, -6, -11, _______ , _______ , _______

Answer: -16, -21, -26

Explanation:
We are given the sequence of numbers:
9, 4 , -1, -6, -11,…
We find the next 3 terms:
-11 – 5 = -11 + (-5) = -16
-16 – 5 = 16 + (-5) = -21
-21 – 5 = -21 + (-5) = -26
Thus the next three terms are -16, -21, -26

Applying Addition and Subtraction of Integers – Guided Practice – Page No. 28

Write an expression. Then find the value of the expression.

Question 1.
Tomas works as an underwater photographer. He starts at a position that is 15 feet below sea level. He rises 9 feet, then descends 12 feet to take a photo of a coral reef. Write and evaluate an expression to find his position relative to sea level when he took the photo.
_______ feet below sea level

Answer: 18 feet

Explanation:
When he rises, we add the distance. When he descends, we subtract the distance.
The initial position is -15. We write an expression to find his position relative to sea level when he took the photo:
-15 + 9 – 12 = (-15) + 9 + (-12)
(-15) + (-12) + 9
-(15 + 12) + 9
-27 + 9 = -18
Thus he was 18 feet below sea level when he took the photo.

Question 2.
The temperature on a winter night was -23 °F. The temperature rose by 5 °F when the sun came up. When the sun set again, the temperature dropped by 7 °F. Write and evaluate an expression to find the temperature after the sun set.
_______ °F

Answer: -25

Explanation:
When the temperature rises, we add the temperature. When the temperature drops, we subtract the temperature. The initial temperature is -23.
We write an expression to find the temperature after the sunset:
-23 + 5 – 7 = -(23 + 7) + 5
-30 + 5 = -25
Thus the temperature is -25°F after the sunset.

Question 3.
Jose earned 50 points in a video game. He lost 40 points, earned 87 points, then lost 30 more points. Write and evaluate an expression to find his final score in the video game.
_______ points

Answer: 67 points

Explanation:
When he wins, we add points. When he loses, we subtract points.
The score is 50 points. We write the expression to find the final score:
50 – 40 + 87 – 30
50 + (-40) + 87 + (-30)
50 + 87 – (40 + 30)
137 – 70 = 67
Thus his final is 67 points.

Find the value of each expression.

Question 4.
-6 + 15 + 15 =
_______

Answer: 24

Explanation:
We have to find the value of the expression:
-6 + 15 + 15 = – 6 + 30 = 24
-6 + 15 + 15 = 24

Question 5.
9 – 4 – 17 =
_______

Answer: -12

Explanation:
We have to find the value of the expression:
9 – 4 – 17 = 9 – (4 + 17)
= 9 – 21 = -12

Question 6.
50 – 42 + 10 =
_______

Answer: 18

Explanation:
We have to find the value of the expression:
50 + (-42) + 10 = 60 – 42
We use the commutative property:
60 – 42 = 18

Question 7.
6 + 13 + 7 – 5 =
_______

Answer: 21

Explanation:
We have to find the value of the expression:
6 + 13 + 7 – 5 = 6 + 13 + 7 + (-5)
We use the associative property:
6 + 13 + 7 + (-5)
= (6 + 13 + 7) + (-5)
26 + (-5)
26 – 5 = 21

Question 8.
65 + 43 – 11 =
_______

Answer: 97

Explanation:
We have to find the value of the expression:
65 + 43 – 11 = 65 + 43 + (-11)
We use the associative property:
(65 + 43) – 11 = 97

Question 9.
-35 – 14 + 45 + 31 =
_______

Answer: 27

Explanation:
We have to find the value of the expression:
-35 – 14 + 45 + 31 = -(35 + 14) + 45 + 31
We use the associative property:
-(35 + 14) + 45 + 31
-49 + 76
= 27

Determine which expression has a greater value.

Question 10.
-12 + 6 – 4 or -34 – 3 + 39
___________

Answer:
We have to compare the expressions:
-12 + 6 – 4 or -34 – 3 + 39
We compute the first expression:
-12 + 6 – 4
-(12 + 4) + 6
-16 + 6 = -10
We compute the second expression:
-34 – 3 + 39
-(34 + 3) + 39
-37 + 39 = 2
2 > -10
Since 2 is greater than -10, the second expression is greater than the first expression.

Question 11.
21 – 3 + 8 or -14 + 31 – 6
___________

Answer:
We have to compare the expressions:
21 – 3 + 8 or -14 + 31 – 6
We compute the first expression:
21 – 3 + 8
21 + 8 – 3
21 + 5 = 26
We compute the second expression:
-14 + 31 – 6
31 – (14 + 6)
31 – 20 = 11
26 > 11
Since 26 is greater than 11, the first expression is greater than the second expression.

Question 12.
Explain how you can find the value of the expression -5 + 12 + 10 – 7.
Type below:
___________

Answer: 10

Explanation:
We have to find the value of the expression:
-5 + 12 + 10 – 7 = 12 + 10 – (5 + 7)
22 – 12 = 10

Applying Addition and Subtraction of Integers – Independent Practice – Page No. 29

Question 13.
Sports Cameron is playing 9 holes of golf. He needs to score a total of at most 15 over par on the last four holes to beat his best golf score. On the last four holes, he scores 5 over par, 1 under par, 6 over par, and 1 under par.
a. Write and find the value of an expression that gives Cameron’s score for 4 holes of golf.
Type below:
___________

Answer:
We write the expression that gives Cameron’s score for 4 holes:
5 – 1 + 6 – 1
5 + 6 – (1 + 1)
11 – 2 = 9

Question 13.
b. Is Cameron’s score on the last four holes over or under par?
Type below:
___________

Answer: The result shows that Cameron’s score is over par.

Question 13.
c. Did Cameron beat his best golf score?
_______

Answer:
Since his score of 9 is beaten his best score as 9 > 15.

Question 14.
Herman is standing on a ladder that is partly in a hole. He starts out on a rung that is 6 feet underground, climbs up 14 feet, then climbs down 11 feet. What is Herman’s final position, relative to ground level?
_______ feet underground

Answer: 3 feet underground

Explanation:
Herman is standing on a ladder that is partly in a hole.
He starts out on a rung that is 6 feet underground, climbs up 14 feet, then climbs down 11 feet.
-6 + 14 -11
14 – (11 + 6)
14 – 17 = -3
Therefore the final position is 3 feet underground.

Question 15.
Explain the Error Jerome tries to find the value of the expression 3 – 6 + 5 by first applying the Commutative Property. He rewrites the expression as 3 – 5 + 6. Explain what is wrong with Jerome’s approach.
Type below:
___________

Answer: Jerome is wrong in using the Commutative Property at Subtraction which is not true: this property works for addition.
3 – 6 + 5 = 3 + (-6) + 5
3 + 5 – 6
= 8 – 6 = 2

Question 16.
Lee and Barry play a trivia game in which questions are worth different numbers of points. If a question is answered correctly, a player earns points. If a question is answered incorrectly, the player loses points. Lee currently has -350 points.

a. Before the game ends, Lee answers a 275-point question correctly, a 70-point question correctly, and a 50-point question incorrectly. Write and find the value of an expression to find Lee’s final score.
_______ points

Answer: -55 points

Explanation:
The initial score is -350 points. We write and find the value of an expression to find Lee’s final score:
-350 + 275 + 70 – 50
-(350 + 50) + 275 + 70
-400 + 345 = -55

Question 17.
b. Barry’s final score is 45. Which player had the greater final score?
___________

Answer: Since -55 < 45, it means Barry has a greater final score.

Question 17.
Multistep Rob collects data about how many customers enter and leave a store every hour. He records a positive number for customers entering the store each hour and a negative number for customers leaving the store each hour.
Go Math Grade 7 Answer Key Chapter 1 Adding and Subtracting Integers Lesson 4: Applying Addition and Subtraction of Integers img 24
a. During which hour did more customers leave than arrive?
___________

Answer: 3:00 – 4:00

Explanation:
since in the last column the only positive value is in the last position, the hour in which more customers leave than arrive is 3:00 – 4:00

Question 17.
b. There were 75 customers in the store at 1:00. The store must be emptied of customers when it closes at 5:00. How many customers must leave the store between 4:00 and 5:00?
_______ customers

Answer: 87

Explanation:
75 + 30 – 12 + 14 – 8 + 18 – 30
75 + 30 + 14 + 18 – (12 + 8 + 30)
137 – 50 = 87
Since there are 87 customers in the store at 4:00 and the store must be emptied at 5:00, the number of clients who must leave is 87.

Applying Addition and Subtraction of Integers – Page No. 30

The table shows the changes in the values of two friends’ savings accounts since the previous month.
Go Math Grade 7 Answer Key Chapter 1 Adding and Subtracting Integers Lesson 4: Applying Addition and Subtraction of Integers img 25

Question 18.
Carla had $100 in her account in May. How much money does she have in her account in August?
$ _______

Answer: $51

Explanation:
We are given the data:
100 – 18 + 22 – 53
100 + 22 -(18 + 53)
122 – 71 = 51
Thus Carla saved $51 in her account in August.

Question 19.
Leta had $45 in her account in May. How much money does she have in her account in August?
$ _______

Answer: $24

Explanation:
We are given the data:
45 – 17 – 22 + 18
45 + 18 -(17 + 22)
63 – 39 = 24
Thus Leta saved $24 in her account in August.

Question 20.
Analyze Relationships Whose account had the greatest decrease in value from May to August?
___________

Answer: Carla’s account

Explanation:
Carla had $100 in May and $51 in august, thus her account’s change is:
51 – 100 = -49
Leta had $45 in May and $24 in august, thus her accounts change is:
24 – 45 = -21
Carla’s account had a decrease of $49, while Leta’s account decreased by $21, so the account with the greatest decrease is Carla’s.

FOCUS ON HIGHER ORDER THINKING

Question 21.
Represent Real-World Problems Write and solve a word problem that matches the diagram shown.
Go Math Grade 7 Answer Key Chapter 1 Adding and Subtracting Integers Lesson 4: Applying Addition and Subtraction of Integers img 26
Type below:
___________

A diver leaves from a point situated 1 meter below the sea level. First, he dives 6 meters, then he rises 3 meters and stops. At which level under the sea level does he stop?
We start from the initial point -1, we add distance if he rises and we subtract distance when he dives. We determine the final level under the sea level where he stops:
-1 – 6 + 3
-(1 + 6) + 3
-7 + 3 = -4
-4 or 4 meters below the sea level.

Question 22.
Critical Thinking Mary has $10 in savings. She owes her parents $50. She does some chores and her parents pay her $12. She also gets $25 for her birthday from her grandmother. Does Mary have enough money to pay her parents what she owes them? If not, how much more money does she need? Explain.
_______

Answer:
The initial point is 10. We add money when she is paid for chores, gets presents. We determine,ine the amount of money she has after she gets money from chores and presents:
10 + 12 + 25 = 47
47 < 50
50 – 47 = 3
Thus she needs $3.

Question 23.
Draw Conclusions An expression involves subtracting two numbers from a positive number. Under what circumstances will the value of the expression be negative? Give an example.
Type below:
___________

Answer:
The sum of the two numbers to be subtracted from the positive number is a number, we will study this first. Since we subtract this number from the positive number and we get a negative number, it means that the number is greater than the positive number, therefore mandatory positive. This means the two numbers cannot be both negative.
Example:
10 – (7 + 5) = 10 – 12 = -2
-2 < 0

Module Quiz – Ready to Go On – Page No. 31

Adding Integers with the Same Sign

Add

Question 1.
−8 + (−6) = _______

Answer: -14

Explanation:
In adding two integers with the same signs you add both the integers and keep the common sign.
−8 + (−6) = -(8 + 6) = -14

Question 2.
−4 + (−7) = _______

Answer: -11

Explanation:
In adding two integers with the same signs you add both the integers and keep the common sign.
−4 + (−7) = – 4 – 7
-(4 + 7) = -11
−4 + (−7) = -11

Question 3.
−9 + (−12) = _______

Answer: -21

Explanation:
In adding two integers with the same signs you add both the integers and keep the common sign.
−9 + (−12) = -9 – 12
-(9 + 12) = – 21
Thus −9 + (−12) = -21

Adding Integers with Different Signs

Add

Question 4.
5 + (−2) = _______

Answer: 3

Explanation:
In adding two integers with the same sign, add their absolute value, and keep the common sign.
When adding two integers with opposite signs, subtract the smaller absolute value from the larger and keep the sign of the number with the larger absolute value.
5 + (−2) = 5 – 2 = 3
The larger number is having the positive sign thus the sum is 3

Question 5.
−8 + 4 = _______

Answer: -4

Explanation:
In adding two integers with the same sign, add their absolute value, and keep the common sign.
When adding two integers with opposite signs, subtract the smaller absolute value from the larger and keep the sign of the number with the larger absolute value.
−8 + 4 = (-8) + 4 = -4
The larger number is having a negative sign thus the sum is -4.

Question 6.
15 + (−8) = _______

Answer: 7

Explanation:
In adding two integers with the same sign, add their absolute value, and keep the common sign.
When adding two integers with opposite signs, subtract the smaller absolute value from the larger and keep the sign of the number with the larger absolute value.
15 + (−8) = 15 – 8 = 7
The larger number is having the positive sign thus the sum is 7.

Subtracting Integers

Subtract.

Question 7.
2 − 9 = _______

Answer: -7

Explanation:
2 – 9 = 2 + (-9)
|2| = 2
|-9| = 9
9 – 2 = 7
2 + (-9) = -7

Question 8.
−3 − (−4) = _______

Answer: 1

Explanation:
-3 – (-4) = – 3 + 4
4 – 3 = 1

Question 9.
11 − (−12) = _______

Answer: 23

Explanation:
11 − (−12) = 11 + 12 = 23

Applying Addition and Subtraction of Integers

Question 10.
A bus makes a stop at 2:30, letting off 15 people and letting on 9. The bus makes another stop ten minutes later to let off 4 more people. How many more or fewer people are on the bus after the second stop compared to the number of people on the bus before the 2:30 stop?
_______ people

Answer: 10

Explanation:
Assume that the total passengers on the bus before 2:30 was x
15 passengers got off and 9 got on.
number of passengers = x – 15 + 9
number of passengers = x -6
4 passengers got off the bus
number of passengers = (x-6) – 4
number of passengers = x – 10
The original number of passengers on the bus decreased by 10 after the second stop.

Question 11.
Cate and Elena were playing a card game. The stack of cards in the middle had 24 cards in it to begin with. Cate added 8 cards to the stack. Elena then took 12 cards from the stack. Finally, Cate took 9 cards from the stack. How many cards were left in the stack?
_______ cards

Answer: 11 cards

Explanation:
When cards are put to the stack, we perform addition.
When cards are taken from the stack we perform subtraction.
24 + 8 – 12 – 9
32 – (12 + 9)
32 – 21 = 11
Thus in the end the stack has 11 cards.

ESSENTIAL QUESTION

Question 12.
Write and solve a word problem that can be modeled by addition of two negative integers.
Type below:
_____________

Answer: -25

Explanation:
A football team played two games. During the first game, the team lost 15 points and during the second game, it lost another 10 points. What is the change in the team’s score after these two games?
(-15) + (-10) = -25

Module Quiz – MODULE 1 MIXED REVIEW – Page No. 32

Assessment Readiness

Selected Response

Question 1.
Which expression has the same value as -3 + (-5):
Options:
a. -3 – (-5)
b. -3 + 5
c. -5 + (-3)
d. -5 – (-3)

Answer: -5 + (-3)

Explanation:
a. -3 – (-5)
-3 + 5 = 2
b. -3 + 5
5 – 3 = 2
c. -5 + (-3)
– 5 – 3 = -8
d. -5 – (-3)
-5 + 3 = -2
Thus the correct answer is option C.

Question 2.
A diver’s elevation is -30 feet relative to sea level. She dives down 12 feet. What is her elevation after the dive?
Options:
a. 12 feet
b. 18 feet
c. -30 feet
d. -42 feet

Answer: -42 feet

Explanation:
A diver’s elevation is -30 feet relative to sea level. She dives down 12 feet.
-30 -12 = (-30) + (-12) = -42 feet
Thus the correct answer is option D.

Question 3.
Which number line models the expression -3 + 5?
Options:
a. Go Math Grade 7 Answer Key Chapter 1 Adding and Subtracting Integers Module Quiz img 27
b. Go Math Grade 7 Answer Key Chapter 1 Adding and Subtracting Integers Module Quiz img 28
c. Go Math Grade 7 Answer Key Chapter 1 Adding and Subtracting Integers Module Quiz img 29
d. Go Math Grade 7 Answer Key Chapter 1 Adding and Subtracting Integers Module Quiz img 30

Answer: Go Math Grade 7 Answer Key Chapter 1 Adding and Subtracting Integers Module Quiz img 28

Explanation:
-3 + 5
On the numeric line, his is modeled by starting at -3 and going right by 5 units. The number which models this is is:
Go Math Grade 7 Answer Key Chapter 1 Adding and Subtracting Integers Module Quiz img 28
Thus the correct answer is option B.

Question 4.
Which number can you add to 5 to get a sum of 0?
Options:
a. -10
b. -5
c. 0
d. 5

Answer: -5

Explanation:
The number we can add to 5 to get a sum of 0 is its opposite:
5 + (-5) = 0
The correct answer is option B.

Question 5.
The temperature in the morning was -3 °F. The temperature dropped 11 degrees by night. What was the temperature at night?
Options:
a. -14 °F
b. -8 °F
c. 8 °F
d. 14 °F

Answer: -14 °F

Explanation:
The temperature in the morning was -3 °F. The temperature dropped 11 degrees by night.
-3 + (-11) = -3 – 11 = -14°F
Therefore the correct answer is option A.

Question 6.
Which of the following expressions has the greatest value?
Options:
a. 3 – 7 + (-10)
b. 3 + 7 – (-10)
c. 3 – 7 – (-10)
d. 3 + 7 + (-10)

Answer: 3 + 7 – (-10)

Explanation:
a. 3 – 7 + (-10)
3 – 7 – 10 = 3 -(7 + 10) = 3 – 17 = -14
b. 3 + 7 – (-10)
3 + 7 + 10 = 20
c. 3 – 7 – (-10)
3 – 7 + 10 = 13 – 7 = 6
d. 3 + 7 + (-10)
10 – 10 = 0
Thus the correct answer is option B.

Mini-Task

Question 7.
At the end of one day, the value of a share of a certain stock was $12. Over the next three days, the change in the value of the share was -$1, then, -$1, and then $3.
a. Write an expression that describes the situation.
Type below:
____________

Answer:
We write an expression that describes the changes in the value of the share:
12 – 1 – 1 + 3

Question 7.
b. Evaluate the expression.
______

Answer: 13

Explanation:
12 – 1 – 1 + 3
12 + 3 – (1 + 1)
15 – 2 = 13

Question 7.
c. What does your answer to part b mean in the context of the problem?
Type below:
____________

Answer: After 3 days, the value of the share changed from $12 to $13.

MODULE 1

MIXED REVIEW

Assessment Readiness

Look at each expression. Does it have the same value as -6 – 4?

Select Yes or No for expressions A–C.

Question 8.
A. -6 + (-4)
______

Answer: Yes

Explanation:
-6 + (-4) = – 6 – 4
-6 + (-4) has the same value as – 6 – 4

Question 8.
B. -4 + (-6)
______

Answer: Yes

Explanation:
-4 + (-6) = -4 – 6
-4 + (-6) has the same value as – 6 – 4

Question 8.
C. 6 + (-4)
______

Answer: No

Explanation:
6 + (-4) = 6 – 4
6 – 4 ≠ – 6 – 4
So, 6 – 4 does not have the same value as – 6 – 4

Choose True or False for A–C.

Question 9.
A. x = 4 is the solution for x + 4 = 0.
i. True
ii. False

Answer: False

Explanation:
x + 4 = 0
x = 4
4 + 4 = 0
8 ≠ 0
So, the statement is false.

Question 9.
B. x = 24 is the solution for \(\frac{x}{3}\) = 8.
i. True
ii. False

Answer: True

Explanation:
\(\frac{x}{3}\) = 8
x = 24
24/3 = 8
8 = 8
Thus the statement is true.

Question 9.
C. x = 6 is the solution for 6x = 1
i. True
ii. False

Answer: False

Explanation:
6x = 1
x = 6
6(6) = 1
36 ≠ 1
Thus the statement is false.

Module 1 Review – Adding and Subtracting Integers – Page No. 103

EXERCISES

Question 1.
−10 + (−5) =
________

Answer: -15

Explanation:
In adding two integers with the same sign, add their absolute value, and keep the common sign.
When adding two integers with opposite signs, subtract the smaller absolute value from the larger and keep the sign of the number with the larger absolute value.
-10 – 5 = -(10 + 5) = -15

Question 2.
9 + (−20) =
________

Answer: -11

Explanation:
In adding two integers with the same sign, add their absolute value, and keep the common sign.
When adding two integers with opposite signs, subtract the smaller absolute value from the larger and keep the sign of the number with the larger absolute value.
9 + (-20) = 9 – 20 = -11

Question 3.
−13 + 32 =
________

Answer: 19

Explanation:
In adding two integers with the same sign, add their absolute value, and keep the common sign.
When adding two integers with opposite signs, subtract the smaller absolute value from the larger and keep the sign of the number with the larger absolute value.
-13 + 32 = 32 + (-13)
32 – 13 = 19

Question 4.
−12 − 5 =
________

Answer: -17

Explanation:
In adding two integers with the same sign, add their absolute value, and keep the common sign.
When adding two integers with opposite signs, subtract the smaller absolute value from the larger and keep the sign of the number with the larger absolute value.
-12 – 5 = -(12 + 5) = -17

Question 5.
25 − (−4) =
________

Answer: 29

Explanation:
In adding two integers with the same sign, add their absolute value, and keep the common sign.
When adding two integers with opposite signs, subtract the smaller absolute value from the larger and keep the sign of the number with the larger absolute value.
25 − (−4) = 25 + 4 = 29

Question 6.
−3 − (−40) =
________

Answer: 37

Explanation:
In adding two integers with the same sign, add their absolute value, and keep the common sign.
When adding two integers with opposite signs, subtract the smaller absolute value from the larger and keep the sign of the number with the larger absolute value.
-3 – (-40) = -3 + 40 = 37

Question 7.
Antoine has $13 in his checking account. He buys some school supplies and ends up with $5 in his account. What was the overall change in Antoine’s account?
$ ________

Answer: $8

Explanation:
The overall change in his account is given by the difference between the final amount of money and the initial amount of money
5 – 13 = 5 + (-13) = -8
The amount in his account is decreased by $8.

Conclusion:
We believe that the solutions provided in Go Math Answer Key Grade 7 Chapter 1 Adding and Subtracting Integers are helpful for you. Also, share the Go Math Grade 7 Key Chapter 1 Adding and Subtracting Integers with your dear ones to help to overcome the issues in solving the integer problems. It helps to learn the simple techniques to solve the adding and subtracting integer problems. If you have any doubts you can clarify them by posting the comments in the below section.

Go Math Grade 4 Answer Key Homework FL Chapter 12 Relative Sizes of Measurement Units Review/Test

go-math-grade-4-chapter-12-relative-sizes-of-measurement-units-review-test-answer-key

Improve your subject skills and problem-solving skills by practicing the questions of Chapter 12 Relative Sizes of Measurement Units from Go Math Grade 4 Answer Key Homework FL Chapter 12 Relative Sizes of Measurement Units Review/Test. By taking the chapter review or chapter test you can easily assess the level of preparation and easily fill up the knowledge gap by HMH Go Math Grade 4 Review/Test Answer Key. Finally, you can score good marks in the exam.

Go Math Grade 4 Answer Key Homework FL Chapter 12 Relative Sizes of Measurement Units Review/Test

Go Math Grade 4 Solution Key Homework FL Review/Test comprises all the concepts in Chapter 12 Relative Sizes of Measurement Units. After testing your kid’s knowledge by giving the question from the Review/Test you can easily identify their preparation & subject skills. Simply tap on the respective link and Download Go Math Grade 4 Solution Key Homework FL Chapter 12 Relative Sizes of Measurement Units Review/Test pdf for free to practice well.

Chapter 12 – Review/Test

Review/Test – Page No. 491

Choose the best term from the box to complete the sentence.
Go Math Grade 4 Answer Key Homework FL Chapter 12 Relative Sizes of Measurement Units Review Test img 1

Question 1.
A ___________ is a metric unit for measuring length or distance.
________

Answer: millimeter
A millimeter is a metric unit for measuring length or distance.

Question 2.
A ___________ is a metric unit for measuring liquid volume.
________

Answer: milliliter
A milliliter is a metric unit for measuring liquid volume.

Question 3.
A ___________ is a graph that shows the frequency of data along a number line.
________

Answer: line plot
A line plot is a graph that shows the frequency of data along a number line.

Question 4.
A ___________ is a customary unit for measuring liquid volume.
________

Answer: quart
A quart is a customary unit for measuring liquid volume.

Complete.

Question 5.
9 feet = _____ inches

Answer: 108 inches

Explanation:
Convert from feet to inches
1 feet = 12 inches
9 feet = 9 × 12 inches = 108 inches
Thus 9 feet = 108 inches

Question 6.
7 tons = _____ pounds

Answer: 14,000 pounds

Explanation:
Converting from tons to pounds
1 ton = 2000 pounds
7 tons = 7 × 2000 pounds = 14,000 pounds
Thus 7 tons = 14,000 pounds

Question 7.
10 pints = _____ cups

Answer: 20 cups

Explanation:
Converting from pints to cups.
1 pint = 2 cups
10 pints = 10 × 2 cups = 20 cups
Thus 10 pints = 20 cups

Question 8.
4 decimeters = _____ centimeters

Answer: 40 centimeters

Explanation:
Converting from decimeters to centimeters.
1 decimeter = 10 centimeter
4 decimeters = 4 × 10 centimeter = 40 centimeters
Thus 4 decimeters = 40 centimeters

Question 9.
8 liters = _____ milliliters

Answer: 8000 millimeters

Explanation:
Converting from liters to milliliters.
1 liter = 1000 milliliters
8 liters = 8 × 1000 milliliters
= 8000 milliliters
Thus 8 liters = 8000 milliliters

Question 10.
5 weeks = _____ days

Answer: 35 days

Explanation:
Converting from weeks to days.
1 week = 7 days
5 weeks = 5 × 7 days = 35 days
Thus 5 weeks = 35 days

Compare using <, >, or =.

Question 11.
3 yards _____ 36 inches

Answer: >

Explanation:
Converting from yards to inches.
1 yard = 36 inches
3 yards = 108 inches
Thus 3 yards > 36 inches

Question 12.
10 cups _____ 80 fluid ounces

Answer: =

Explanation:
Converting from cups to fluid ounces.
1 cup = 8 fluid ounces
10 cups = 8 × 10 = 80 fluid oiunces
Thus 10 cups = 80 fluid ounces

Question 13.
4 pounds _____ 96 ounces

Answer: <

Explanation:
Converting from pounds to ounces.
1 pound = 16 ounces
4 pounds = 4 × 16 ounces = 64 ounces
64 ounces is less than 96 ounces
Thus, 4 pounds < 96 ounces

Question 14.
8 meters _____ 700 centimeters

Answer: >

Explanation:
Converting from meters to centimeters.
1 meter = 100 centimeters
8 meters = 8 × 100 centimeters = 800 centimeters
800 centimeters is greater than 700 centimeters.
Thus, 8 meters > 700 centimeters

Question 15.
6 liters _____ 6,500 milliliters

Answer: <

Explanation:
Converting from liters to milliliters.
1 liter = 1000 milliliters
6 liters = 6 × 1000 milliliters
6000 milliliters is less than 6500 milliliters.
Thus, 6 liters < 6,500 milliliters.

Question 16.
9 kilograms _____ 9,000 grams

Answer: =

Explanation:
Converting from kilograms to grams.
1 kilogram = 1000 grams
9 kilograms = 9 × 1000 grams = 9000 grams
9 kilograms = 9,000 grams

Add or subtract.

Question 17.
8 hr 30 min
− 6 hr 25 min
————————–
_____ hr _____ min

Answer: 2 hr 5 min

Explanation:
8 hr 30 min
-6 hr 25 min
2 hr 5 min

Question 18.
7 c 4 fl oz
+4 c 3 fl oz
———————–
_____ c _____ fl oz

Answer: 11c 7 fl oz

Explanation:
7 c 4 fl oz
+4 c 3 fl oz
11 c 7 fl oz.

Question 19.
9 yd 1 ft
−5 yd 2 ft
———————–
_____ yd _____ ft

Answer: 3 yd 2 ft

Explanation:
First, convert from the yard to feet.
1 yard = 3 ft
9 yd 1 ft = 8 yd 4 ft
8 yd 4 ft
-5 yd 2 ft
3 yd 2 ft

Review/Test – Page No. 492

Fill in the bubble completely to show your answer.

Question 20.
Maya’s band rehearsal started at 10:30 A.M. It ended 1 hour and 40 minutes later. At what time did Maya’s band rehearsal end?
Options:
a. 12:10 A.M.
b. 8:50 A.M.
c. 12:10 P.M.
d. 11:10 P.M.

Answer: 12:10 P.M.

Explanation:
Given,
Maya’s band rehearsal started at 10:30 A.M. It ended 1 hour and 40 minutes later.
10 hr 30 min
+1 hr 40 min
11 hr 70 min
Now convert 70 min to hours.
70 min = 1 hr 10 min
11 hr 70 min = 12:10 P.M.
Thus the correct answer is option C.

Question 21.
Darlene is making punch. She pours 4 quarts 2 cups of apple juice into a bowl. Then she pours 3 quarts 1 cup of grape juice into the bowl. How much juice is in the bowl now?
Options:
a. 1 quart 1 cup
b. 7 quarts 1 cup
c. 7 quarts 3 cups
d. 8 quarts 1 cup

Answer: 7 quarts 3 cups

Explanation:
Given,
Darlene is making punch. She pours 4 quarts 2 cups of apple juice into a bowl.
Then she pours 3 quarts 1 cup of grape juice into the bowl.
4 quarts 2 cups
+3 quarts 1 cup
7 quarts 3 cups
Thus the correct answer is option c.

Question 22.
Kainoa bought a brick of modeling clay that was labeled 2 kilograms. He needs to separate the clay into balls that are measured in grams. How many grams does he have?
Options:
a. 20 grams
b. 200 grams
c. 2,000 grams
d. 20,000 grams

Answer: 2,000 grams

Explanation:
Given,
Kainoa bought a brick of modeling clay that was labeled 2 kilograms.
He needs to separate the clay into balls that are measured in grams.
Convert from kilograms to grams.
1 kilogram = 1000 grams
2 kilograms = 2 × 1000 grams = 2000 grams
Thus the correct answer is option c.

Question 23.
A truck driver’s truck weighs 3 tons. A weigh station measures the weight in pounds. How many pounds does the truck weigh?
Options:
a. 600 pounds
b. 2,000 pounds
c. 3,000 pounds
d. 6,000 pounds

Answer: 6,000 pounds

Explanation:
Given,
A truck driver’s truck weighs 3 tons. A weigh station measures the weight in pounds.
Convert from tons to pounds.
1 ton = 2000 pounds
3 tons = 3 × 2000 pounds = 6000 pounds
Thus the correct answer is option d.

Review/Test – Page No. 493

Fill in the bubble completely to show your answer.

Question 24.
Brody and Amanda canoed for 1 hour and 20 minutes before stopping to fish at 1:15 P.M. At what time did they start canoeing?
Options:
a. 11:55 A.M.
b. 12:05 P.M.
c. 2:35 P.M.
d. 11:55 P.M.

Answer: 11:55 A.M.

Explanation:
Given,
Brody and Amanda canoed for 1 hour and 20 minutes before stopping to fish at 1:15 P.M.
13 hr 15 min
-1 hr 20 min
11 hr 55 min
Thus they start canoeing at 11:55 A.M.
Thus the correct answer is option d.

Question 25.
Lewis fills his thermos with 2 liters of water. Garret fills his thermos with 1 liter of water. How many more milliliters of water does Lewis have than Garret?
Options:
a. 1 more milliliter
b. 100 more milliliters
c. 1,000 more milliliters
d. 2,000 more milliliters

Answer: 1,000 more milliliters

Explanation:
Given,
Lewis fills his thermos with 2 liters of water. Garret fills his thermos with 1 liter of water.
2 liters
-1 liters
1 liter
Convert from liters to milliliters.
1 liter = 1000 milliliters
Thus the correct answer is option c.

Question 26.
Lola won the 100-meter freestyle event at her swim meet. How many decimeters did Lola swim?
Options:
a. 1 decimeter
b. 10 decimeter
c. 100 decimeter
d. 1,000 decimeter

Answer: 1,000 decimeter

Explanation:
Given,
Lola won the 100-meter freestyle event at her swim meet.
Convert from meter to decimeter.
1 meter = 10 decimeter
100 meter = 100 × 10 decimeter = 1000 decimeter
Thus the correct answer is option d.

Question 27.
What is the best estimate for the length of an ant’s leg?
Options:
a. 2 millimeters
b. 2 centimeters
c. 2 decimeters
d. 2 meters

Answer: 2 centimeters

Explanation:
The best estimation for the length of an ant’s leg is 2 centimeters.
Thus the correct answer is option b.

Review/Test – Page No. 494

Question 28.
Sabita made this table to relate two customary units of liquid volume. List the number pairs for the table. Describe the relationship between the numbers in each pair.
Go Math Grade 4 Answer Key Homework FL Chapter 12 Relative Sizes of Measurement Units Review Test img 2
Type below:
________

Answer: The relationship between the numbers in each pair is pints and cups.

Question 29.
Label the columns of the table. Explain your answer.
Type below:
________

Answer:
Go-Math-Grade-4-Answer-Key-Homework-FL-Chapter-12-Relative-Sizes-of-Measurement-Units-Review-Test-img-2

Question 30.
Landon borrowed a book from the library. The data show the lengths of time Landon read the book each day until he finished it.
Go Math Grade 4 Answer Key Homework FL Chapter 12 Relative Sizes of Measurement Units Review Test img 3
A. Make a tally table and a line plot to show the data.
Go Math Grade 4 Answer Key Homework FL Chapter 12 Relative Sizes of Measurement Units Review Test img 4
Go Math Grade 4 Answer Key Homework FL Chapter 12 Relative Sizes of Measurement Units Review Test img 5
Type below:
________

Answer:
Go-Math-Grade-4-Answer-Key-Homework-FL-Chapter-12-Relative-Sizes-of-Measurement-Units-Review-Test-img-4

Question 30.
B. Explain how you used the tally table to label the numbers and plot the Xs on the line plot.
Type below:
________

Answer:
I used the number of tallys to the plot the Xs on the line plot.

Question 30.
C. What is the difference between the longest time and shortest time Landon spent reading the book?
\(\frac{□}{□}\) hour

Answer: \(\frac{3}{4}\) hour

Explanation:
The shortest time Landon spent reading the book is 1/4
The longest time Landon spent reading the book is 1
1 – 1/4 = 3/4
Thus the difference between the longest time and shortest time Landon spent reading the book is \(\frac{3}{4}\) hour.

Conclusion:

Keep in touch with our Go Math Grade 4 Answer Key in pdf format & learn the explanations of each and every question from ch 12. For more questions and assistance take help from the other two articles ie, Go Math Grade 4 Answer Key Chapter 12 Relative Sizes of Measurement Units and Go Math Grade 4 Solution Key Homework practice FL Chapter 12 pdf. All The Best!!!

Go Math Grade 7 Answer Key Chapter 3 Rational Numbers

go-math-grade-7-chapter-3-rational-numbers-answer-key

Get expert verified solutions in Go Math Grade 7 Answer Key Chapter 3 Rational Numbers here. So, the students of the 7th class can Download Go Math Grade 7 Answer Key Chapter 3 Rational Numbers pdf for free. Our Go Math 7th Grade Chapter 3 Rational Numbers helps the students to complete their homework in time and also score the highest marks in the exams.

Go Math Grade 7 Answer Key Chapter 3 Rational Numbers

All concepts are covered in one place along with answers for Go Math Grade 7 Chapter 3 Rational Numbers. Follow the steps to Download HMH Go Math Chapter 3 Grade 7 Answer Key pdf to learn simple methods to solve the problems. The quick way of solving problems will help the students to save time. Hence, check the question and find out the complete answers and explanations for every problem.

Chapter 3 – Rational Numbers and Decimals

Chapter 3 – Adding Rational Numbers

Chapter 3 – Subtracting Rational Numbers

Chapter 3 – Multiply Rational Numbers

Chapter 3 – Divide Rational Numbers

Chapter 3 – Applying Rational Number Operations

Chapter 3 – Module Review

Chapter 3 – Performance Tasks

Chapter 3 – MIXED REVIEW

Rational Numbers and Decimals – Guided Practice – Page No. 64

Write each rational number as a decimal. Then tell whether each decimal is a terminating or a repeating decimal.

Question 1.
\(\frac{3}{5}\) =
___________ decimals

Answer: terminating

Explanation:
To convert fraction decimals, we have to divide the numerator to the denominator. If the quotient goes on and on, then it is a repeating decimal, and to write this as a decimal, put a bar on top of the repeating digits.

\(\frac{3}{5}\) = 3 ÷ 5
3/5 = 0.6
The decimal is not repeating so it is a terminating decimal which is 0.6

Question 2.
\(\frac{89}{100}\) =
___________ decimals

Answer: terminating

Explanation:
To convert fraction decimals, we have to divide the numerator to the denominator. If the quotient goes on and on, then it is a repeating decimal, and to write this as a decimal, put a bar on top of the repeating digits.
\(\frac{89}{100}\) = 0.89
The decimal is not repeating so it is a terminating decimal which is 0.89

Question 3.
\(\frac{4}{12}\) =
___________ decimals

Answer: repeating

Explanation:
To convert fraction decimals, we have to divide the numerator to the denominator. If the quotient goes on and on, then it is a repeating decimal, and to write this as a decimal, put a bar on top of the repeating digits.
\(\frac{4}{12}\) = 4 ÷ 12
4/12 = 0. 333….
The quotient is a repeating decimal which is 0.33…

Question 4.
\(\frac{25}{99}\) =
___________ decimals

Answer: repeating

Explanation:
To convert fraction decimals, we have to divide the numerator to the denominator. If the quotient goes on and on, then it is a repeating decimal, and to write this as a decimal, put a bar on top of the repeating digits.
\(\frac{25}{99}\) = 0.2525…
The quotient is a repeating decimal which is 0.2525…

Question 5.
\(\frac{7}{9}\) =
___________ decimals

Answer: repeating

Explanation:
To convert fraction decimals, we have to divide the numerator to the denominator. If the quotient goes on and on, then it is a repeating decimal, and to write this as a decimal, put a bar on top of the repeating digits.
\(\frac{7}{9}\) = 0.77…
The quotient is a repeating decimal which is 0.77…

Question 6.
\(\frac{9}{25}\) =
___________ decimals

Answer: terminating

Explanation:
To convert fraction decimals, we have to divide the numerator to the denominator. If the quotient goes on and on, then it is a repeating decimal, and to write this as a decimal, put a bar on top of the repeating digits.
\(\frac{9}{25}\) = 0.36
The decimal is not repeating so it is a terminating decimal which is 0.36

Question 7.
\(\frac{1}{25}\) =
___________ decimals

Answer: terminating

Explanation:
To convert fraction decimals, we have to divide the numerator to the denominator. If the quotient goes on and on, then it is a repeating decimal, and to write this as a decimal, put a bar on top of the repeating digits.
\(\frac{1}{25}\) = 0.04
The decimal is not repeating so it is a terminating decimal which is 0.04

Question 8.
\(\frac{25}{176}\) =
___________ decimals

Answer: repeating

Explanation:
To convert fraction decimals, we have to divide the numerator to the denominator. If the quotient goes on and on, then it is a repeating decimal, and to write this as a decimal, put a bar on top of the repeating digits.
\(\frac{25}{176}\) = 0.14204545454
The quotient is a repeating decimal which is 0.14204545454

Question 9.
\(\frac{12}{1000}\) =
___________ decimals

Answer: terminating

Explanation:
To convert fraction decimals, we have to divide the numerator to the denominator. If the quotient goes on and on, then it is a repeating decimal, and to write this as a decimal, put a bar on top of the repeating digits.
\(\frac{12}{1000}\) =0.012
The decimal is not repeating so it is a terminating decimal which is 0.012

Write each mixed number as a decimal.

Question 10.
11 \(\frac{1}{6}\) =
___________ decimals

Answer: repeating

Explanation:
To convert fraction decimals, we have to divide the numerator to the denominator. If the quotient goes on and on, then it is a repeating decimal, and to write this as a decimal, put a bar on top of the repeating digits.
11 \(\frac{1}{6}\) = 11.1666666667
The quotient is a repeating decimal which is 11.1666666667

Question 11.
2 \(\frac{9}{10}\) =
___________ decimals

Answer: terminating

Explanation:
To convert fraction decimals, we have to divide the numerator to the denominator. If the quotient goes on and on, then it is a repeating decimal, and to write this as a decimal, put a bar on top of the repeating digits.
First, convert the mixed fraction to the improper fraction.
2 \(\frac{9}{10}\) = \(\frac{29}{10}\) = 2.9
Thus, the decimal is not repeating so it is a terminating decimal which is 2.9

Question 12.
8 \(\frac{23}{100}\) =
___________ decimals

Answer: terminating

Explanation:
To convert fraction decimals, we have to divide the numerator to the denominator. If the quotient goes on and on, then it is a repeating decimal, and to write this as a decimal, put a bar on top of the repeating digits.
First, convert the mixed fraction to the improper fraction.
8 \(\frac{23}{100}\) = \(\frac{823}{100}\) = 8.23
Thus, the decimal is not repeating so it is a terminating decimal which is 8.23

Question 13.
7 \(\frac{3}{15}\) =
___________ decimals

Answer: terminating

Explanation:
To convert fraction decimals, we have to divide the numerator to the denominator. If the quotient goes on and on, then it is a repeating decimal, and to write this as a decimal, put a bar on top of the repeating digits.
First, convert the mixed fraction to the improper fraction.
7 \(\frac{3}{15}\) = \(\frac{108}{15}\) = 7.2
Thus, the decimal is not repeating so it is a terminating decimal which is 7.2

Question 14.
54 \(\frac{3}{11}\) =
___________ decimals

Answer: repeating

Explanation:
To convert fraction decimals, we have to divide the numerator to the denominator. If the quotient goes on and on, then it is a repeating decimal, and to write this as a decimal, put a bar on top of the repeating digits.
First, convert the mixed fraction to the improper fraction.
54 \(\frac{3}{11}\) = \(\frac{597}{11}\) = 54.2727…
The quotient is a repeating decimal which is 54.2727…

Question 15.
3 \(\frac{1}{18}\) =
___________ decimals

Answer: repeating

Explanation:
To convert fraction decimals, we have to divide the numerator to the denominator. If the quotient goes on and on, then it is a repeating decimal, and to write this as a decimal, put a bar on top of the repeating digits.
First, convert the mixed fraction to the improper fraction.
3 \(\frac{1}{18}\) = \(\frac{55}{18}\) = 3.055..
The quotient is a repeating decimal which is 3.055..

Question 16.
Maggie bought 3 \(\frac{2}{3}\) lb of apples to make some apple pies. What is the weight of the apples written as a decimal?
3 \(\frac{2}{3}\) =
___________ decimal

Answer: repeating

Explanation:
To convert fraction decimals, we have to divide the numerator to the denominator. If the quotient goes on and on, then it is a repeating decimal, and to write this as a decimal, put a bar on top of the repeating digits.
First, convert the mixed fraction to the improper fraction.
3 \(\frac{2}{3}\) = \(\frac{11}{3}\) = 3.66..
The quotient is a repeating decimal which is 3.66..

Question 17.
Harry’s dog weighs 12 \(\frac{7}{8}\) pounds. What is the weight of Harry’s dog written as a decimal?
12 \(\frac{7}{8}\) =
___________ decimals

Answer: terminating

Explanation:
Given that,
Harry’s dog weighs 12 \(\frac{7}{8}\) pounds.
To convert fraction decimals, we have to divide the numerator to the denominator. If the quotient goes on and on, then it is a repeating decimal, and to write this as a decimal, put a bar on top of the repeating digits.
First, convert the mixed fraction to the improper fraction.
12 \(\frac{7}{8}\) = \(\frac{103}{8}\) = 12.875

Essential Question Check-In

Question 18.
Tom is trying to write \(\frac{3}{47}\) as a decimal. He used long division and divided until he got the quotient 0.0638297872, at which point he stopped. Since the decimal doesn’t seem to terminate or repeat, he concluded that \(\frac{3}{47}\) is not rational. Do you agree or disagree? Why?
___________

Answer: disagree

Explanation:
We are given the number:
{0, 1, 2, 3, ……45, 46}
When dividing a number by 47 the possible remainders at each step are:
This means that after at most 47 steps we get a remainder which repeats. This means that process and which repeats. This means that the process stops and we get a repeating decimal.

Rational Numbers and Decimals – Independent Practice – Page No. 65

Use the table for 19–23. Write each ratio in the form \(\frac{a}{b}\) and then as a decimal. Tell whether each decimal is a terminating or a repeating decimal.
Go Math Grade 7 Answer Key Chapter 3 Rational Numbers Lesson 1: Rational Numbers and Decimals img 1

Question 19.
Basketball players to football players
___________ decimal

Answer: Repeating

Explanation:
To convert fraction decimals, we have to divide the numerator to the denominator. If the quotient goes on and on, then it is a repeating decimal, and to write this as a decimal, put a bar on top of the repeating digits.
Since the item is asking us to write basketball players to football players, we write the number of basketball players (5) in the numerator and the number of football players (11) in the denominator.
5/11 = 0.4545..
This is a repeating decimal with 45 as the repeating digits.

Question 20.
Hockey players to lacrosse players
___________ decimal

Answer: terminating

Explanation:
To convert fraction decimals, we have to divide the numerator to the denominator. If the quotient goes on and on, then it is a repeating decimal, and to write this as a decimal, put a bar on top of the repeating digits.
Since the item is asking us to write hockey players to lacrosse players, we write the number of hockey players (6) in the numerator and the number of lacrosse players (10) in the denominator.
Now convert the fraction into the decimal
6/10 = 0.6
This is a terminating decimal which is 0.6.

Question 21.
Polo players to football players
___________ decimal

Answer: Repeating

Explanation:
To convert fraction decimals, we have to divide the numerator to the denominator. If the quotient goes on and on, then it is a repeating decimal, and to write this as a decimal, put a bar on top of the repeating digits.
Since the item is asking us to write polo players to football players, we write the number of polo players (4) in the numerator and the number of football players (11) in the denominator.
Now we convert this as a decimal.
4/11 = 0.36..
This is a repeating decimal with 36 as the repeating digits.

Question 22.
Lacrosse players to rugby players
___________ decimal

Answer: Repeating

Explanation:
To convert fraction decimals, we have to divide the numerator to the denominator. If the quotient goes on and on, then it is a repeating decimal, and to write this as a decimal, put a bar on top of the repeating digits.
Since the item is asking us to write lacrosse players to rugby players, we write the number of lacrosse players (10) in the numerator and the number of rugby players (15) in the denominator.
10/15 = 0.66..
This is a repeating decimal with 6 as the repeating digit.

Question 23.
Football players to soccer players
___________ decimal

Answer: terminating

Explanation:
To convert fraction decimals, we have to divide the numerator to the denominator. If the quotient goes on and on, then it is a repeating decimal, and to write this as a decimal, put a bar on top of the repeating digits
Since the item is asking us to write football players to soccer players, we write the number of football players (11) in the numerator and the number of soccer players (11) in the denominator.
11/11 = 1
This is a terminating decimal which is 1.

Question 24.
Look for a Pattern Beth said that the ratio of the number of players in any sport to the number of players on a lacrosse team must always be a terminating decimal. Do you agree or disagree? Why?
___________

Answer: agree

Explanation:
The ratios of the number of players in any sport to the number of players on a lacrosse team are:
{9/10, 5/10, 11/10, 6/10, 10/10, 4/10, 15/10, 11/10}
All these ratios are terminating decimals as all numerators divided by 10 lead to a terminating decimal.

Question 25.
Yvonne bought 4 \(\frac{7}{8}\) yards of material to make a dress.
a. What is 4 \(\frac{7}{8}\) written as an improper fraction?
\(\frac{□}{□}\)

Answer:
To convert fraction decimals, we have to divide the numerator to the denominator. If the quotient goes on and on, then it is a repeating decimal, and to write this as a decimal, put a bar on top of the repeating digits.
Convert from mixed fraction to the improper fraction.
4 \(\frac{7}{8}\) = (8 × 4) + 7 = 32 + 7 = 39/8

Question 25.
b. What is 4 \(\frac{7}{8}\) written as a decimal?
______

Answer:
Remember that we need to add the whole number and just convert the fraction part to decimal.
7/8 = 0.875
The fraction is a terminating decimal. Combining the whole number and the decimal part we get,
4 + 0.875 = 4.875

Question 25.
c. Communicate Mathematical Ideas If Yvonne wanted to make 3 dresses that use 4 \(\frac{7}{8}\) yd of fabric each, explain how she could use estimation to make sure she has enough fabric for all of them.
Type below:
_____________

Answer:
Using estimation, we say that 4 \(\frac{7}{8}\) ≈ 5.
We can now multiply 3 by 5, and therefore, she needs 15 yards of fabric.

Rational Numbers and Decimals – Page No. 66

Question 26.
Vocabulary A rational number can be written as the ratio of one _______ to another and can be represented by a repeating or ______ decimal.
Type below:
_____________

Answer: A rational number can be written as the ratio of one integer to another and can be represented by a repeating or terminating decimal.

Question 27.
Problem Solving Marcus is 5 \(\frac{7}{24}\) feet tall. Ben is 5 \(\frac{5}{16}\) feet tall. Which of the two boys is taller? Justify your answer.
_____________

Answer:
Explanation:
To convert fraction decimals, we have to divide the numerator to the denominator. If the quotient goes on and on, then it is a repeating decimal, and to write this as a decimal, put a bar on top of the repeating digits.
To determine who is taller, we convert both to decimals. Remember that we need to add the whole number and just convert the fraction part to decimal.
For Marcus:
7/24 = 0.29166..
Combine the whole number and the decimal part we get 5.29166..
For Ben:
5/16 = 0.3125
Combine the whole number and the decimal part we get 5.1325
Hence Ben is taller.

Question 28.
Represent Real-World Problems If one store is selling \(\frac{3}{4}\) of a bushel of apples for 9 dollars, and another store is selling \(\frac{2}{3}\) of a bushel of apples for 9 dollars, which store has the better deal? Explain your answer.
_____________

Answer:
To convert fraction decimals, we have to divide the numerator to the denominator. If the quotient goes on and on, then it is a repeating decimal, and to write this as a decimal, put a bar on top of the repeating digits.
To determine which store has a better deal, we convert both fractions to decimals.
For the first store:
3/4 = 0.75
For the second store:
2/3 = 0.666..
Since the first store offers 0.75 of a bushel of apples, this store has a better deal.

Question 29.
Analyze Relationships You are given a fraction in simplest form. The numerator is not zero. When you write the fraction as a decimal, it is a repeating decimal. Which numbers from 1 to 10 could be the denominator?
Type below:
_____________

Answer: {3, 6, 7, 9}

Explanation:
Since the only numbers which can be factors of the denominators lead to a terminating decimal are 1, 2, and 5 and combinations of them, it means that if the denominator has at least one of the other numbers at the denominator, the decimal form will be a repeating decimal.
Among the numbers from 1 to 10, the presence of any of these numbers in the denominator will lead to a repeating decimal:
{3, 6, 7, 9}

Question 30.
Communicate Mathematical Ideas Julie got 21 of the 23 questions on her math test correct. She got 29 of the 32 questions on her science test correct. On which test did she get a higher score? Can you compare the fractions \(\frac{21}{23}\) and \(\frac{29}{32}\) by comparing 29 and 21? Explain. How can Julie compare her scores?
_____________

Answer:
To convert fraction decimals, we have to divide the numerator to the denominator. If the quotient goes on and on, then it is a repeating decimal, and to write this as a decimal, put a bar on top of the repeating digits.
For the math test:
21/23 = 0.9130
For the science test:
29/32 = 0.9063
Therefore she got a higher score in her math test.
Julie got a higher score in her math test. We cannot compare the fractions by comparing the numerators. Instead, we can compare her scores if the denominators of the fractions are the same.

Question 31.
Look for a Pattern Look at the decimal 0.121122111222.… If the pattern continues, is this a repeating decimal? Explain.
_____________

Answer: The number is not a repeating decimal.

Adding Rational Numbers – Guided Practice – Page No. 72

Use a number line to find each sum.

Question 1.
−3 + (−1.5) =
______

Answer: -4.5

Explanation:
Remember if the number being added is positive, move the number of units going to the right and if the number being added is negative, move the number of units to the left.
Since we are adding a negative number, starting from -3, we move 1.5 units to the left. This results in -4.5.

Question 2.
1.5 + 3.5 =
______

Answer: 5

Explanation:
Remember if the number being added is positive, move the number of units going to the right and if the number being added is negative, move the number of units to the left.
Since we are adding a positive number, starting from 1.5 we move 3.5 units to the right. This results in 5.

Question 3.
\(\frac{1}{4}+\frac{1}{2}\) =
\(\frac{□}{□}\)

Answer: \(\frac{3}{4}\)

Explanation:
Remember if the number being added is positive, move the number of units going to the right and if the number being added is negative, move the number of units to the left.
Since we are adding a positive number, starting from 1/4, we move 1/2 or 2/4, units to the right. This results in 3/4.

Question 4.
−1 \(\frac{1}{2}\) + (−1 \(\frac{1}{2}\)) =
______

Answer: -3

Explanation:
Remember if the number being added is positive, move the number of units going to the right and if the number being added is negative, move the number of units to the left.
Since we are adding a negative number, starting from −1 \(\frac{1}{2}\), we move 1 1/2 units to the left. This is results in -3.

Question 5.
3 + (−5) =
______

Answer: -2

Explanation:
Remember if the number being added is positive, move the number of units going to the right and if the number being added is negative, move the number of units to the left.
Since we are adding a negative number, starting from 3 we move 5 units to the left. This results in -2.

Question 6.
(−1.5) + 4 =
______

Answer: 2.5

Explanation:
Remember if the number being added is positive, move the number of units going to the right and if the number being added is negative, move the number of units to the left.
Since we are adding a positive number, starting from 1.5 we move 4 units to the left. This results in 2.5

Question 7.
Victor borrowed 21.50 dollars from his mother to go to the theater. A week later, he paid her 21.50 dollars back. How much does he still owe her?
______

Answer: 0

Explanation:
We use positive numbers for the money he receives and negative numbers for the money he returns.
21.50 – 21.50 = 0
The result is zero. This means he doesn’t owe anything to his mother.

Question 8.
Sandra used her debit card to buy lunch for 8.74 on Monday. On Tuesday, she deposited 8.74 back into her account. What is the overall increase or decrease in her bank account?
______

Answer: 0

Explanation:
We use positive numbers for the money she deposits and negative numbers for the money she spends.
-8.74 + 8.74 = 0
The result is zero. This means her bank account didn’t increase or decrease.

Find each sum without using a number line.

Question 9.
2.75 + (−2) + (−5.25) =
______

Answer: -4.50

Explanation:
We are given the expression:
2.75 + (-2) + (-5.25)
We group numbers with the same sign using the associative property.
2.75 – 7.25 = -4.50

Question 10.
−3 + (1 \(\frac{1}{2}\)) + (2 \(\frac{1}{2}\)) =
______

Answer: 1

Explanation:
We are given the expression
-3 + (1 \(\frac{1}{2}\)) + (2 \(\frac{1}{2}\))
-3 + 1.5 + 2.5
We group numbers with the same sign using the associative property.
-3 + 4 = 1
The larger number is having a positive sign so the sum is 1.

Question 11.
−12.4 + 9.2 + 1 =
______

Answer: -2.2

Explanation:
We are given the expression
-12.4 + 9.2 + 1
We group numbers with the same sign using the associative property.
-12.4 + 10.2 = -2.2
The larger number is having a negative sign so the answer is -2.2.

Question 12.
−12 + 8 + 13 =
______

Answer: 9

Explanation:
We are given the expression|
-12 + 8 + 13
We group numbers with the same sign using the associative property.
-12 + 21 = 9
The larger number is having the positive sign so the answer is 9.

Question 13.
4.5 + (−12) + (−4.5) =
______

Answer: -12

Explanation:
We are given the expression
4.5 + (-12) + (-4.5)
We group numbers with the same sign using the associative property.
0 – 12 = -12
The larger number is having the negative sign so the answer is -12.

Question 14.
\(\frac{1}{4}\) + (− \(\frac{3}{4}\)) =
– \(\frac{□}{□}\)

Answer: -0.50

Explanation:
We are given the expression
\(\frac{1}{4}\) + (− \(\frac{3}{4}\))
Convert the fraction to Decimal.
0.25 – 0.75 = -0.50
The larger number is having the negative sign so the sum is -0.50

Question 15.
−4 \(\frac{1}{2}\) + 2 =
– \(\frac{□}{□}\)

Answer: -2.5

Explanation:
We  = are given the expression
−4 \(\frac{1}{2}\) + 2
Convert from fraction to decimal.
-4.5 + 2 = -2.5
The larger number is having the negative sign so the sum is -2.5.

Question 16.
−8 + (−1 \(\frac{1}{8}\)) =
– \(\frac{□}{□}\)

Answer: -9.125

Explanation:
We are given the expression
−8 + (−1 \(\frac{1}{8}\))
Convert from fraction to decimal.
-8 + (-1.125) = – 9.125

Question 17.
How can you use a number line to find the sum of -4 and 6?
Type below:
____________

Answer: 6

Explanation:
Remember if the number being added is positive, move the number of units going to the right and if the number being added is negative, move the number of units to the left.
Since we are adding a positive number, starting from -4 we move 6 units to the right. This results in 2.

Adding Rational Numbers – Independent Practice – Page No. 73

Question 18.
Samuel walks forward 19 steps. He represents this movement with a positive 19. How would he represent the opposite of this number?
_______

Answer: -19
He would represent the opposite of 19 by a negative 19.

Question 19.
Julia spends 2.25 on gas for her lawn mower. She earns 15.00 mowing her neighbor’s yard. What is Julia’s profit?
_______

Answer: $12.75

Explanation:
We use positive numbers for the money she earns and negative numbers for the money she spends.
-2.25 + 15 = 12.75
Thus her profit is $12.75

Question 20.
A submarine submerged at a depth of -35.25 meters dives an additional 8.5 meters. What is the new depth of the submarine?
_______

Answer: In adding two integers with same sign add their absolute value and keep the common sign.
When adding two integers with opposite sign subtract the smaller absolute value from the larger and keep the sign of the number with the larger absolute value.
Since the submarine dove 32.25 meters down this can be interrupted as -32.25. And because it dove an additional 8.5 meters down, we can add -8.5 meters to the previous distance.
Add 32.25 and 8.5 meters
32.25 + 8.5 = 43.75 meters
Thus the submarines new depth is 43.75 meters deep or -43.75 meters.

Question 21.
Renee hiked for 4 \(\frac{3}{4}\) miles. After resting, Renee hiked back along the same route for 3 \(\frac{1}{4}\) miles. How many more miles does Renee need to hike to return to the place where she started?
_______ \(\frac{□}{□}\)

Answer:
Given that
Renee hiked for 4 \(\frac{3}{4}\) miles. After resting, Renee hiked back along the same route for 3 \(\frac{1}{4}\) miles.
4 \(\frac{3}{4}\) + (-3 \(\frac{1}{4}\)) = 1 \(\frac{1}{2}\)
Thus Renee needs to hike to return to the place where she started is 1 \(\frac{1}{2}\) or 1.5 miles.

Question 22.
Geography
The average elevation of the city of New Orleans, Louisiana, is 0.5 m below sea level. The highest point in Louisiana is Driskill Mountain at about 163.5 m higher than New Orleans. How high is Driskill Mountain?
_______

Answer: 163 meters

Explanation:
We use the positive numbers for the elevation above the sea level and negative numbers for the elevation below the sea level.
163.5 – 0.5 = 163 meters
Thus the height of the Driskill mountain is 163 meters.

Question 23.
Problem Solving
A contestant on a game show has 30 points. She answers a question correctly to win 15 points. Then she answers a question incorrectly and loses 25 points. What is the contestant’s final score?
_______

Answer: 20

Explanation:
We use positive numbers for won points and negative numbers for lost points.
30 + 15 + (-25) = 20
Thus the final score is 20.

Go Math Grade 7 Answer Key Chapter 3 Rational Numbers Lesson 2: Adding Rational Numbers img 2

Financial Literacy

Use the table for 24–26. Kameh owns a bakery. He recorded the bakery income and expenses in a table.

Question 24.
In which months were the expenses greater than the income? Name the month and find how much money was lost.
Type below:
___________

Answer:
We count the balance for January
1205 + (-1290.60)  = -85.60
We count the balance for February
1183 + (-1345.44) = -162.44
January: $85.60
February: $162.44

Question 25.
In which months was the income greater than the expenses? Name the months and find how much money was gained.
Type below:
___________

Answer:
The income was greater than the expenses in the months:
We count the balance for june:
2413 + (-2106.23) = 306.77
We count the balance for july:
2260 + (-1958.50) = 301.5
We count the balance for august:
2183 + (-1845.12) = 337.88
June: $306.77 gained
July: $301.5 gained
August: $337.88 gained

Question 26.
Communicate Mathematical Ideas
If the bakery started with an extra $250 from the profits in December, describe how to use the information in the table to figure out the profit or loss of money at the bakery by the end of August. Then calculate the profit or loss.
Balance: $ _______

Answer: 948.71

Explanation:
If the bakery started with an extra $250 from the profits in December.
We will add this amount to January’s income.
250 + 1205 + 1183 + 1664 + 2413 + 2260 + 2183 = 11,158
We compute the expenses during the 6 months
(-1290) + (-1345.44) + (-1664) + (-2106.24) + (-1958.50) + (-1845.12) = -10209.29
11158 -10209.29 = 948.71
Since the result is a positive number, the bakery has profit.

Adding Rational Numbers – Independent Practice – Page No. 74

Question 27.
Vocabulary
-2 is the ________ of 2.
__________

Answer: additive inverse

Explanation:
When the sum of two numbers with opposite signs is 0, then they are additive inverses of eacj other.
Therefore, -2 is the additive inverse of 2.

Question 28.
The basketball coach made up a game to play where each player takes 10 shots at the basket. For every basket made, the player gains 10 points. For every basket missed, the player loses 15 points.
a. The player with the highest score sank 7 baskets and missed 3. What was the highest score?
_______ points

Answer: 25

Explanation:
We use the positive numbers for won points and negative numbers for lost points.
We determine the highest score:
7(10) + 3(-15) = 70 + (-45) = 25

Question 28.
b. The player with the lowest score sank 2 baskets and missed 8. What was the lowest score?
_______ points

Answer: -100

Explanation:
We determine the lowest score:
2(10) + 8(-15) = 20 + (-120) = -100

Question 28.
c. Write an expression using addition to find out what the score would be if a player sank 5 baskets and missed 5 baskets.
Type below:
__________

Answer: -25

Explanation:
We determine the score for 5 baskets and 5 missed baskets:
5(10) + 5(-15) = 50 + (-75)
50 – 75 = -25

H.O.T

FOCUS ON HIGHER ORDER THINKING

Question 29.
Communicate Mathematical Ideas
Explain the different ways it is possible to add two rational numbers and get a negative number.
Type below:
__________

Answer:
The sum of two rational numbers is negative either if both numbers are negative, or they have different signs, but the negative number is the one with the greater absolute value.

Question 30.
Explain the Error
A student evaluated -4 + x for x = -9 \(\frac{1}{2}\) and got an answer of 5 \(\frac{1}{2}\). What might the student have done wrong?
Type below:
__________

Answer:
We expect about 95% of all possible samples to have a 95% confidence interval that contains the population proportion who favor such an amendment.

Question 31.
Draw Conclusions
Can you find the sum [5.5 + (-2.3)] + (-5.5 + 2.3) without performing any additions?
_______

Answer:
Yes, we can find the sum without performing any computation if we notice that the two numbers from each set of brackets are the opposites of the numbers in the other set  of bracelets, thus the sum is zero:
[5.5 + (-2.3)] + (-5.5 + 2.3)
5.5 – 2.3 – 5.5 + 2.3 = 0

Subtracting Rational Numbers – Guided Practice – Page No. 79

Use a number line to find each difference.

Question 1.
5 − (−8) =
_______

Answer: 13

Explanation:
Remember if the number being subtracted is positive, move the number of units going to the left and if the number being subtracted is negative, move the number of units to the right.
Since we are subtracting a negative number, starting from 5, we move 8 units to the right. This results in 13.

Question 2.
−3 \(\frac{1}{2}\) − 4 \(\frac{1}{2}\) =
_______

Answer: -8

Explanation:
Remember if the number being subtracted is positive, move the number of units going to the left and if the number being subtracted is negative, move the number of units to the right.
Since we are subtracting a positive number, starting from −3 \(\frac{1}{2}\), we move 4 \(\frac{1}{2}\) units to the left. This results in -8.

Question 3.
−7 − 4 =
_______

Answer: -11

Explanation:
Remember if the number being subtracted is positive, move the number of units going to the left and if the number being subtracted is negative, move the number of units to the right.
Since we are subtracting a positive number, starting from -7, we move 4 units to the left. This results in -11.

Question 4.
−0.5 − 3.5 =
_______

Answer: -4

Explanation:
Remember if the number being subtracted is positive, move the number of units going to the left and if the number being subtracted is negative, move the number of units to the right.
Since we are subtracting a positive number, starting from -0.5, we move 3.5 units to the left. This results in -4

Find each difference.

Question 5.
−14 − 22 =
_______

Answer: -36

Explanation:
We have to determine the difference
-14 – 22 = (-14) + (-22) = -36
−14 − 22 = -36

Question 6.
−12.5 − (−4.8) =
_______

Answer: -7.7

Explanation:
-12.5 – (-4.8)
We convert subtraction into addition with the opposite number
-12.5 – (-4.8) = -12.5 + 4.8 = -7.7
So, the answer is -7.7

Question 7.
\(\frac{1}{3}\) − (−\(\frac{2}{3}\)) =
_______

Answer: 1

Explanation:
\(\frac{1}{3}\) − (−\(\frac{2}{3}\))
\(\frac{1}{3}\) + \(\frac{2}{3}\) = \(\frac{3}{3}\) = 1
The result is 1.

Question 8.
65 − (−14) =
_______

Answer: 79

Explanation:
We convert subtraction into addition with the opposite number
65 − (−14) = 65 + 14 = 79
The answer is 79.

Question 9.
− \(\frac{2}{9}\) − (−3) =
_______ \(\frac{□}{□}\)

Answer: 2 \(\frac{7}{9}\)

Explanation:
We convert subtraction into addition with the opposite number
− \(\frac{2}{9}\) − (−3) = − \(\frac{2}{9}\) + 3 = 2 \(\frac{7}{9}\)
The answer is 2 \(\frac{7}{9}\)

Question 10.
24 \(\frac{3}{8}\) − (−54 \(\frac{1}{8}\)) =
_______ \(\frac{□}{□}\)

Answer: 78 \(\frac{1}{2}\)

Explanation:
We convert subtraction into addition with the opposite number.
24 \(\frac{3}{8}\) − (−54 \(\frac{1}{8}\)) = 24 \(\frac{3}{8}\) + 54 \(\frac{1}{8}\) = 78 \(\frac{1}{2}\)
Thus the result is 78 \(\frac{1}{2}\).

Question 11.
A girl is snorkeling 1 meter below sea level and then dives down another 0.5 meter. How far below sea level is the girl?
_______

Answer: 1.5 meter

Explanation:
1 m below sea level is represented by the number -14. Since she is diving down 0.5 m, you must subtract -1 – 0.5 = -1.5 m
Thus the girl is 1.5 m long.

Question 12.
The first play of a football game resulted in a loss of 12 \(\frac{1}{2}\) yards. Then a penalty resulted in another loss of 5 yards. What is the total loss or gain?
_______

Answer: 17 \(\frac{1}{2}\) yards

Explanation:
The first play of a football game resulted in a loss of 12 \(\frac{1}{2}\) yards. Then a penalty resulted in another loss of 5 yards.
-12 \(\frac{1}{2}\) – 5 = -17 \(\frac{1}{2}\) yards
It is a loss of 17 \(\frac{1}{2}\) yards

Question 13.
A climber starts descending from 533 feet above sea level and keeps going until she reaches 10 feet below sea level. How many feet did she descend?
_______

Answer: 543 feet

Explanation:
Given,
A climber starts descending from 533 feet above sea level and keeps going until she reaches 10 feet below sea level.
533 feet + 10 feet = 543 feet

Question 14.
Eleni withdrew 45.00 dollars from her savings account. She then used her debit card to buy groceries for 30.15 dollars. What was the total amount Eleni took out of her account?
_______

Answer: $75.15

Explanation:
Given that,
Eleni withdrew 45.00 dollars from her savings account. She then used her debit card to buy groceries for 30.15 dollars.
$45 + $30.15 = $75.15
Thus Eleni took $75.15 out of her account.

Question 15.
Mandy is trying to subtract 4 – 12, and she has asked you for help. How would you explain the process of solving the problem to Mandy, using a number line?
Type below:
____________

Answer: Start at 4 on the number line. Then move 12 places to the left since you are subtracting. This gives -8.

Subtracting Rational Numbers – Independent Practice – Page No. 80

Question 16.
Science
At the beginning of a laboratory experiment, the temperature of a substance is -12.6 °C. During the experiment, the temperature of the substance decreases 7.5 °C. What is the final temperature of the substance?
_______

Answer: -20.1°C

Explanation:
Remember if the number being subtracted is positive, move the number of units going to the left and if the number being subtracted is negative, move the number of units to the right.
Since the temperature of the substance is -12.6 and it decreases further by 7.5, we can create the expression -12.6 – 7.5.
-12.6 – 7.5 = -20.1
Thus the final temperature is -20.1°C

Question 17.
A diver went 25.65 feet below the surface of the ocean, and then 16.5 feet further down, he then rose 12.45 feet. Write and solve an expression to find the diver’s new depth.
_______

Answer: -29.7 feet

Explanation:
Remember if the number being subtracted is positive, move the number of units going to the left and if the number being subtracted is negative, move the number of units to the right.
Since the diver went down 25.65 feet then dove again further by 16.5 feet then rose up by 12.45 feet, we can create the expression -25.65 – 16.5 + 12.45 = -29.7 feet
The diver’s new depth is -29.7 feet.

Question 18.
A city known for its temperature extremes started the day at -5 degrees Fahrenheit. The temperature increased by 78 degrees Fahrenheit by midday, and then dropped 32 degrees by nightfall.
a. What expression can you write to find the temperature at nightfall?
Type below:
____________

Answer:
The temperature started at -5 degrees then increased 78 degrees and then dropped 32 degrees.
The expression is -5 + 78 – 32

Question 18.
b. What expression can you write to describe the overall change in temperature? Hint: Do not include the temperature at the beginning of the day since you only
Type below:
____________

Answer: The overall change is the increase and decrease combined.
The expression is 78 – 32

Question 18.
c. What is the final temperature at nightfall? What is the overall change in temperature?
Type below:
____________

Answer:
Use the first expression -5 + 78 – 32 = 73 – 32 = 41 degrees
78 – 32 = 46 degrees

Question 19.
Financial Literacy
On Monday, your bank account balance was -$12.58. Because you didn’t realize this, you wrote a check for $30.72 for groceries.
a. What is the new balance in your checking account?
$ _______

Answer:
Subtract the check amount from the initial balance.
-$12.58 – $30.72 = -$43.30

Question 19.
b. The bank charges a $25 fee for paying a check on a negative balance. What is the balance in your checking account after this fee?
$ _______

Answer:
Subtract 25 from the balance from part a.
-$43.30 – $25 = -$68.30

Question 19.
c. How much money do you need to deposit to bring your account balance back up to $0 after the fee?
$ _______

Answer:
Since the account balance is -$68.30, a deposit of $68.30 is required to make the balance $0.

Astronomy

Use the table for problems 20–21.
Go Math Grade 7 Answer Key Chapter 3 Rational Numbers Lesson 3: Subtracting Rational Numbers img 3

Question 20.
How much deeper is the deepest canyon on Mars than the deepest canyon on Venus?
_______

Answer: -16,500 feet deper

Explanation:
Subtract the lowest elevations of Mars and Venus.
-26,000 – (-9500) = -16,500

Question 21.
Persevere in Problem Solving
What is the difference between Earth’s highest mountain and its deepest ocean canyon? What is the difference between Mars’ highest mountain and its deepest canyon? Which difference is greater? How much greater is it?
Type below:
____________

Answer:
Subtract the highest elevation and the lowest elevation on Earth.
29,035 – (-36,198) = 65,233
Subtract the highest elevation and the lowest elevation on Mars.
96,000 – 65,233 = 30,767
96,000 is greater than 65,233 so the difference for Mars is greater. subtract these two numbers to get how much greater.

Subtracting Rational Numbers – Page No. 81

Question 22.
Pamela wants to make some friendship bracelets for her friends. Each friendship bracelet needs 5.2 inches of string.
a. If Pamela has 20 inches of string, does she have enough to make bracelets for 4 of her friends?
a. _______

Answer: no

Explanation:
Each bracelet needs 5.2 inches so multiply 4 and 5.2 inches to see how many total inches she needs this is greater than 20 so she does not have enough.
4 × 5.2 = 20.8 inches

Question 22.
b. If so, how much string would she had left over? If not, how much more string would she need?
_______ in.

Answer: She needs 0.8 inches more

Question 23.
Jeremy is practicing some tricks on his skateboard. One trick takes him forward 5 feet, then he flips around and moves backwards 7.2 feet, then he moves forward again for 2.2 feet.
a. What expression could be used to find how far Jeremy is from his starting position when he finishes the trick?
Type below:
___________

Answer: 5 – 7.2 + 2.2

Explanation:
He moves 5 feet forward, back 7.2 feet, and then forward 2.2 feet.

Question 23.
b. How far from his starting point is he when he finishes the trick? Explain
_______ ft.

Answer: 0 ft

Explanation:
Since the distance just pulls hi back and forth at the same amount of distance.
5 – 7.2 + 2.2 = 0 ft

Question 24.
Esteban has $20 from his allowance. There is a comic book he wishes to buy that costs $4.25, a cereal bar that costs $0.89, and a small remote control car that costs $10.99.
a. Does Esteban have enough to buy everything?
a. _______

Answer:
Find the total amount of money he wants to spend this is less than 20 so he has enough
4.25 + 0.89 + 10.99 = 16.13
Thus Esteban had enough money.

Question 24.
b. If so, how much will he have left over? If not, how much does he still need?
$ _______

Answer:
Subtract the amount he wants to spend from the amount he has to find how much he has left.
20 – 16.13 = 3.87
Thus $3.87 left.

Subtracting Rational Numbers – H.O.T – Page No. 82

Focus on Higher Order Thinking

Question 25.
Look for a Pattern
Show how you could use the Commutative Property to simplify the evaluation of the expression \(-\frac{7}{16}-\frac{1}{4}-\frac{5}{16}\).
_______

Answer:
\(-\frac{7}{16}-\frac{1}{4}-\frac{5}{16}\)
-12/16 – 1/4
= -3/4 -1/4
= -4/4 = -1

Question 26.
Problem Solving
The temperatures for five days in Kaktovik, Alaska, are given below.
-19.6 °F, -22.5 °F, -20.9 °F, -19.5 °F, -22.4 °F
Temperatures for the following week are expected to be twelve degrees lower every day. What are the highest and lowest temperatures expected for the corresponding 5 days next week?
Type below:
____________

Answer:
The highest temperature for the first five days was -19.5 degrees so the highest temperature the following week is 12 degrees less than that. the lowest temperature the first week was -22.9 degree so the lowest temperature the second week is 12 degree below that
high: -19.5 – 12 = -31.5°F
low: -22.5 – 12 = -34.5°F

Question 27.
Make a Conjecture
Must the difference between two rational numbers be a rational number? Explain.
_______

Answer:
Yes, the difference between two rational numbers must be rational. Subtracting two fractions equals a fraction of an integer. Integers are rational numbers so even if the answer isn’t a fraction, it is still a rational number.

Question 28.
Look for a Pattern
Evan said that the difference between two negative numbers must be negative. Was he right? Use examples to illustrate your answer.
_______

Answer:
He is not correct. The difference between -2 and -5 is -2- (-5) = -2 + 5 = 3
which is not negative.

Multiply Rational Numbers – Guided Practice – Page No. 86

Use a number line to find each product.

Question 1.
5(−\(\frac{2}{3}\)) =
_______ \(\frac{□}{□}\)

Answer: -3 \(\frac{1}{3}\)

Explanation:
Remember if the number being multiplied is positive, starting from zero move the number of units by how many times it is multiplied going to the right and if the number being multiplied is negative, starting from zero, move the number of units by how many times it is multiplied going to the left.
Since we are multiplying −\(\frac{2}{3}\) by 5, starting from 0, we move \(\frac{2}{3}\) units to the left five times. This results in -3 \(\frac{1}{3}\)

Question 2.
3(−\(\frac{1}{4}\)) =
\(\frac{□}{□}\)

Answer: –\(\frac{3}{4}\)

Explanation:
Remember if the number being multiplied is positive, starting from zero move the number of units by how many times it is multiplied going to the right and if the number being multiplied is negative, starting from zero, move the number of units by how many times it is multiplied going to the left.
Since we are multiplying −\(\frac{1}{4}\) by 3, starting from 0, we move −\(\frac{1}{4}\) units to the left three times. This results in –\(\frac{3}{4}\).

Question 3.
−3(−\(\frac{4}{7}\)) =
_______ \(\frac{□}{□}\)

Answer: 1 \(\frac{5}{7}\)

Explanation:
Remember if the number being multiplied is positive, starting from zero move the number of units by how many times it is multiplied going to the right and if the number being multiplied is negative, starting from zero, move the number of units by how many times it is multiplied going to the left.
Since we are multiplying −\(\frac{4}{7}\) by -3, let us first multiply −\(\frac{4}{7}\) by 3. Starting from 0, we move \(\frac{4}{7}\) units to the left three times.
This results in -1 \(\frac{5}{7}\)
Therefore the opposite of this is 1 \(\frac{5}{7}\).

Question 4.
−\(\frac{3}{4}\)(−4) =
______

Answer: 3

Explanation:
Remember if the number being multiplied is positive, starting from zero move the number of units by how many times it is multiplied going to the right and if the number being multiplied is negative, starting from zero, move the number of units by how many times it is multiplied going to the left.
Since we are multiplying −\(\frac{3}{4}\) by -4, let us first multiply −\(\frac{3}{4}\) by 4. Starting from 0, we move \(\frac{3}{4}\) units to the left three times. This results in -3. Therefore the opposite of this is 3.

Question 5.
4(−3) =
______

Answer: -12

Explanation:
Remember if the number being multiplied is positive, starting from zero move the number of units by how many times it is multiplied going to the right and if the number being multiplied is negative, starting from zero, move the number of units by how many times it is multiplied going to the left.
Since we are multiplying -3 by 4, starting from 0, we move 3 units to the left four times. This results in -12.

Question 6.
(−1.8)5 =
______

Answer: -9

Explanation:
Remember if the number being multiplied is positive, starting from zero move the number of units by how many times it is multiplied going to the right and if the number being multiplied is negative, starting from zero, move the number of units by how many times it is multiplied going to the left.
Since we are multiplying -1.8 by 5, starting from 0, we move 1.8 units to the left five times. This results in -9.

Question 7.
−2(−3.4) =
______

Answer: 6.8

Explanation:
Remember if the number being multiplied is positive, starting from zero move the number of units by how many times it is multiplied going to the right and if the number being multiplied is negative, starting from zero, move the number of units by how many times it is multiplied going to the left.
Since we are multiplying -2 by -3.4, starting from 0, starting from 0, we move 3.4 units to the left two times. This results in -6.8. Therefore, the opposite of this is 6.8.

Question 8.
0.54(8) =
______

Answer: 4.32

Explanation:
Remember if the number being multiplied is positive, starting from zero move the number of units by how many times it is multiplied going to the right and if the number being multiplied is negative, starting from zero, move the number of units by how many times it is multiplied going to the left.
Since we are multiplying 0.54 by 8, starting from 0, we move 0.54 units to the right eight times. This results in 4.32.

Question 9.
−5(−1.2) =
______

Answer: 6

Explanation:
Remember if the number being multiplied is positive, starting from zero move the number of units by how many times it is multiplied going to the right and if the number being multiplied is negative, starting from zero, move the number of units by how many times it is multiplied going to the left.
Since we are multiplying -1.2 by -5, Starting from 0, we move 1.2 units to the left five times. This results in -6. Therefore the opposite of this is 6.

Question 10.
−2.4(3) =
______

Answer: -7.2

Explanation:
Remember if the number being multiplied is positive, starting from zero move the number of units by how many times it is multiplied going to the right and if the number being multiplied is negative, starting from zero, move the number of units by how many times it is multiplied going to the left.
Since we are multiplying -2.4 by 3, starting from 0, we move 2.4 units to the left three times. This results in -7.2

Multiply.

Question 11.
\(\frac{1}{2} \times \frac{2}{3} \times \frac{3}{4}\) = □ × \(\frac{3}{4}\) =
\(\frac{□}{□}\)

Answer: \(\frac{1}{4}\)

Explanation:
\(\frac{1}{2} \times \frac{2}{3} \times \frac{3}{4}\) = □ × \(\frac{3}{4}\)
1/3 × 3/4 = 1/4
\(\frac{1}{2} \times \frac{2}{3} \times \frac{3}{4}\) = □ × \(\frac{3}{4}\) = \(\frac{1}{4}\)

Question 12.
\(-\frac{4}{7}\left(-\frac{3}{5}\right)\left(-\frac{7}{3}\right)\) = □ × \(\left(-\frac{7}{3}\right)\) =
\(\frac{□}{□}\)

Answer: – \(\frac{4}{5}\)

Explanation:
Multiply the first two fractions by multiplying the top numbers together and multiply the bottom numbers together.
Remember that two negatives make a positive so the product of the first two fractions is positive.
\(-\frac{4}{7}\left(-\frac{3}{5}\right)\left(-\frac{7}{3}\right)\) = □ × \(\left(-\frac{7}{3}\right)\)
12/35 × -7/3 = -4/5

Question 13.
\(-\frac{1}{8} \times 5 \times \frac{2}{3}\) =
\(\frac{□}{□}\)

Answer: –\(\frac{5}{12}\)

Explanation:
Use the commutative property to switch the order of the first two fractions.
\(-\frac{1}{8} \times 5 \times \frac{2}{3}\) = –\(\frac{1}{8}\) × \(\frac{2}{3}\) × 5
–\(\frac{1}{12}\) × 5 = –\(\frac{5}{12}\)

Question 14.
\(-\frac{2}{3}\left(\frac{1}{2}\right)\left(-\frac{6}{7}\right)\) =
\(\frac{□}{□}\)

Answer: \(\frac{2}{7}\)

Explanation:
Multiply the first two fractions by cancelling the 2s.
\(-\frac{2}{3}\left(\frac{1}{2}\right)\left(-\frac{6}{7}\right)\) = –\(\frac{1}{3}\)(-\(\frac{6}{7}\))
Multiply by cancelling the 3 and 6 to get a 2 in the numerator two negatives make a positive.
So the answer is \(\frac{2}{7}\)

Question 15.
The price of one share of Acme Company declined $3.50 per day for 4 days in a row. What is the overall change in price of one share?
$ _______

Answer: -$14

Explanation:
Given that,
The price of one share of Acme Company declined $3.50 per day for 4 days in a row.
-$3.50 × 4 = -$14.00
Thus the overall change in the price of one share is -$14.

Question 16.
In one day, 18 people each withdrew $100 from an ATM machine. What is the overall change in the amount of money in the ATM machine?
$ _______

Answer: The overall change in the amount of money in the ATM machine is the product of the amount people withdrew times the number of people. This gives -100(18) = -1800.
Therefore the overall change in the amount of money in the ATM machine is -$1800.

Question 17.
Explain how you can find the sign of the product of two or more rational numbers.
Type below:
____________

Answer: If the product has an even number of negative signs, then the product is positive. If the product has an odd number of negative signs, then the product is negative.

Multiply Rational Numbers – Independent Practice – Page No. 87

Question 18.
Financial Literacy
Sandy has $200 in her bank account.
a. If she writes 6 checks for exactly $19.98, what expression describes the change in her bank account?
_______

Answer: The change in her bank account is equal to the product of the check amounts and the number of checks.
This gives the expressions 6(-19.98)

Question 18.
b. What is her account balance after the checks are cashed?
$ _______

Answer: She started with $200 and her account balance changes by 6(-19.98) dollars so her account balance is 200 – 6(-19.98) = 200 – 119.88 = 80.12

Question 19.
Communicating Mathematical Ideas
Explain, in words, how to find the product of -4(-1.5) using a number line. Where do you end up?
Type below:
____________

Answer:
First, find the value of -4(-1.5) by starting at 0 on the number line and moving 1.5 units left four times.
This gives a value of 4(-1.5) = -6
Since -4(-1.5) is the opposite of 4(-1.5), the answer is 6.

Question 20.
Greg sets his watch for the correct time on Wednesday. Exactly one week later, he finds that his watch has lost 3 \(\frac{1}{4}\) minutes. If his watch continues to lose time at the same rate, what will be the overall change in time after 8 weeks?
_______ minutes

Answer: 26 minutes

Explanation:
Given,
Greg sets his watch for the correct time on Wednesday.
Exactly one week later, he finds that his watch has lost 3 \(\frac{1}{4}\) minutes.
8(3 \(\frac{1}{4}\)) = 8 \(\frac{13}{4}\)
= 2 × 13 = 26 minutes.
Therefore the overall change in time after 8 weeks is 26 minutes.

Question 21.
A submarine dives below the surface, heading downward in three moves. If each move downward was 325 feet, where is the submarine after it is finished diving?
_______ feet

Answer: 975

Explanation:
Moving downward is represented by a negative number. Multiply the distance traveled down by the number of moves.
3 × -325 feet = -975
The submarine is 975 feet below the surface.

Question 22.
Multistep
For Home Economics class, Sandra has 5 cups of flour. She made 3 batches of cookies that each used 1.5 cups of flour. Write and solve an expression to find the amount of flour Sandra has left after making the 3 batches of cookies.
_______ cups

Answer: 0.5 cups

Explanation:
Sandra has a total of 5 cups of flour. Since she used 1.5 cups per batch of the cookie, and there are 3 batches, we can subtract the product of the cups and the number of batches
1.5 × 3 = 4.5
Therefore the expression should be 5 – 4.5 = 0.5
Thus Sandra has 0.5 cups of flour left.

Question 23.
Critique Reasoning
In class, Matthew stated,“I think that a negative is like the opposite. That is why multiplying a negative times a negative equals a positive. The opposite of negative is positive, so it is just like multiplying the opposite of a negative twice, which is two positives.”
Do you agree or disagree with his reasoning? What would you say in response to him?
_______

Answer: I agree with him. The product of two negatives is positive because the product of two positives is positive and negatives are opposites of positives.

Question 24.
Kaitlin is on a long car trip. Every time she stops to buy gas, she loses 15 minutes of travel time. If she has to stop 5 times, how late will she be getting to her destination?
_______ minutes

Answer: 75 minutes

Explanation:
Multiply the number of stops by the length of each stop to find the time she will be late.
5 × 15 = 75
Thus Kaitlin will be 75 minutes late to reach her destination.

Multiply Rational Numbers – Page No. 88

Question 25.
The table shows the scoring system for quarterbacks in Jeremy’s fantasy football league. In one game, Jeremy’s quarterback had 2 touchdown passes, 16 complete passes, 7 incomplete passes, and 2 interceptions. How many total points did Jeremy’s quarterback score?
Go Math Grade 7 Answer Key Chapter 3 Rational Numbers Lesson 4: Multiply Rational Numbers img 4
_______ pts

Answer: 13.5 points

Explanation:
Write the expression for the total number of points
2(6) + 16(0.5) + 7(-0.5) + 2(-1.5)
= 12 + 8 – 3.5 – 3
= 20 – 6.5
= 13.5
Thus Jeremy’s quarterback scored 13.5 points.

H.O.T

Focus On Higher Order Thinking

Question 26.
Represent Real-World Problems
The ground temperature at Brigham Airport is 12 °C. The temperature decreases by 6.8 °C for every increase of 1 kilometer above the ground. What is the overall change in temperature outside a plane flying at an altitude of 5 kilometers above Brigham Airport?
_______ °C

Answer: -22°C

Explanation:
Remember if the number being multiplied is positive, starting from zero moves the number of units by how many times it is multiplied going to the right and if the number being multiplied is negative, starting from zero, move the number of units by how many times it is multiplied going to the left.
Note that the ground temperature is 12°C. Since the temperature decreases by 6.8°C for every kilometer above ground, and the given height of the plane is 5 kilometers,
We can subtract the product of the temperature and the distance 5(6.8) from the ground temperature.
Therefore the expression should be 12 – 5(6.8)
= 12 – 34
= -22
Thus the temperature outside a plane flying at an altitude of 5 kilometers above Brigham Airport is -22°C

Question 27.
Identify Patterns
The product of four numbers, a, b, c, and d, is a negative number. The table shows one combination of positive and negative signs of the four numbers that could produce a negative product. Complete the table to show the seven other possible combinations.
Go Math Grade 7 Answer Key Chapter 3 Rational Numbers Lesson 4: Multiply Rational Numbers img 5
Type below:
_____________

Answer:
In multiplying numbers, an odd number of negative signs produces a negative product.
Go-Math-Grade-7-Answer-Key-Chapter-3-Rational-Numbers-img-5

Question 28.
Reason Abstractly
Find two integers whose sum is -7 and whose product is 12. Explain how you found the numbers.
Type below:
_____________

Answer: -3 and -4

Explanation:
Let x and y be the two numbers. Write the equations using the given information
x + y = -7
xy = 12
Since the two numbers multiply to a positive and add to a negative the two numbers must be negative. Find the pairs of negative numbers that multiply to 12.
-1 and -12, -2 and -6 and -3 and -4.
Thus the pairs that have a sum as -7 and product as 12 is -3 and -4.

Divide Rational Numbers – Guided Practice – Page No. 92

Find each quotient.

Question 1.
\(\frac{0.72}{-0.9}\) =
_______

Answer: -0.8

Explanation:
We have to find the quotient:
\(\frac{0.72}{-0.9}\)
We determine the sign of the quotient.
The quotient will be negative because the numbers have different signs.
\(\frac{0.72}{-0.9}\) = -0.8

Question 2.
\(\left(-\frac{\frac{1}{5}}{\frac{7}{5}}\right)\) =
\(\frac{□}{□}\)

Answer: – \(\frac{1}{7}\)

Explanation:
We have to find the quotient:
\(\left(-\frac{\frac{1}{5}}{\frac{7}{5}}\right)\)
We determine the sign of the quotient.
The quotient will be negative because the numbers have different signs.
\(\left(-\frac{\frac{1}{5}}{\frac{7}{5}}\right)\) = – \(\frac{5}{35}\) = – \(\frac{1}{7}\)

Question 3.
\(\frac{56}{-7}\) =
_______

Answer: -8

Explanation:
We have to find the quotient:
\(\frac{56}{-7}\)
We determine the sign of the quotient.
The quotient will be negative because the numbers have different signs.
7 divides 56 eight times.
Thus the quotient of \(\frac{56}{-7}\) = -8

Question 4.
\(\frac{251}{4} \div\left(-\frac{3}{8}\right)\) =
\(\frac{□}{□}\)

Answer: –\(\frac{502}{3}\)

Explanation:
We have to find the quotient:
\(\frac{251}{4} \div\left(-\frac{3}{8}\right)\)
We determine the sign of the quotient.
The quotient will be negative because the numbers have different signs.
Rewrite using multiplication by multiplying with the reciprocal.
\(\frac{251}{4}\) × –\( \frac{8}{3}\) = –\(\frac{2008}{12}\)
–\(\frac{2008}{12}\) = –\(\frac{502}{3}\)
The quotient of \(\frac{251}{4} \div\left(-\frac{3}{8}\right)\) is –\(\frac{502}{3}\)

Question 5.
\(\frac{75}{-\frac{1}{5}}\) =
_______

Answer: -375

Explanation:
We have to find the quotient:
\(\frac{75}{-\frac{1}{5}}\)
We determine the sign of the quotient.
The quotient will be negative because the numbers have different signs.
75 ÷ 1/5
75 × -5 = -375
Thus the quotient of \(\frac{75}{-\frac{1}{5}}\) is -375.

Question 6.
\(\frac{-91}{-13}\) =
_______

Answer: 7

Explanation:
We have to find the quotient:
\(\frac{-91}{-13}\)
We determine the sign of the quotient.
The quotient will be positive because the numbers have the same signs.
13 divides 91 seven times.
\(\frac{-91}{-13}\) = 7
Thus the quotient is 7.

Question 7.
\(\frac{-\frac{3}{7}}{\frac{9}{4}}\) =
\(\frac{□}{□}\)

Answer: –\(\frac{4}{21}\)

Explanation:
We have to find the quotient:
\(\frac{-\frac{3}{7}}{\frac{9}{4}}\)
We determine the sign of the quotient.
The quotient will be negative because the numbers have different signs.
\(\frac{-\frac{3}{7}}{\frac{9}{4}}\) = -3/7 × 4/9 = -12/63
-12/63 = -4/21
\(\frac{-\frac{3}{7}}{\frac{9}{4}}\) = –\(\frac{4}{21}\)

Question 8.
– \(\frac{12}{0.03}\) =
_______

Answer: -400

Explanation:
We have to find the quotient:
– \(\frac{12}{0.03}\)
We determine the sign of the quotient.
The quotient will be negative because the numbers have different signs.
– \(\frac{12}{0.03}\) = -400
So the quotient is -400.

Question 9.
A water pail in your backyard has a small hole in it. You notice that it has drained a total of 3.5 liters in 4 days. What is the average change in water volume each day?
_______ liters per day

Answer: -0.875 litres/day

Explanation:
Given that,
A water pail in your backyard has a small hole in it. You notice that it has drained a total of 3.5 liters in 4 days.
The average change of water volume each day is the quotient.
So, divide -3.5 by 4.
The quotient will be negative because the numbers have different signs.
-3.5/4 = -0.875
Thus the water volume diminishes by 0.875 liters each day.

Question 10.
The price of one share of ABC Company declined a total of $45.75 in 5 days. What was the average change of the price of one share per day?
$ _______

Answer: -$9.15

Explanation:
The price of one share of ABC Company declined a total of $45.75 in 5 days.
We use negative numbers for the price going down.
The average change in the price of one share per day is the quotient.
-45.75/5
We determine the sign of the quotient.
The quotient will be negative because the numbers have different signs.
-45.75/5 = -9.15
Thus the price of one share diminishes by $9.15 per day.

Question 11.
To avoid a storm, a passenger-jet pilot descended 0.44 mile in 0.8 minutes. What was the plane’s average change of altitude per minute?
_______

Answer: -0.55 miles/min

Explanation:
We use negative numbers for the altitude going down.
The plane’s average change of altitude per minute is the quotient:
-0.44/0.8
We determine the sign of the quotient
The quotient will be negative because the numbers have different signs.
-0.44/0.8 = -0.55
Therefore the plane descends by 0.55 miles per minute.

Essential Question Check-In

Question 12.
Explain how you would find the sign of the quotient \(\frac{32 \div(-2)}{-16 \div 4}\).
Type below:
___________

Answer: positive

Explanation:
Given,
\(\frac{32 \div(-2)}{-16 \div 4}\)
Since all the operations are of multiplication and division, the sign is given by the number of negative signs.
If the number of negative signs is even, the quotient is positive while if the number of negative signs is odd, the quotient is negative.
In this case, the number of negative signs is 2, therefore even, so the quotient is positive.
\(\frac{32 \div(-2)}{-16 \div 4}\) = -16/-4 = 4
Thus the solution is positive.

Divide Rational Numbers – Independent Practice – Page No. 93

Question 13.
\(\frac{5}{-\frac{2}{8}}\) =
_______

Answer: -20

Explanation:
We are given the expression:
\(\frac{5}{-\frac{2}{8}}\)
The quotient will be negative because the numbers have different signs.
\(\frac{5}{-\frac{2}{8}}\) = 5 ÷ (-2/8)
We rewrite using the multiplication by multiplying with the reciprocal:
5 × -8/2 = 5 × -4 = -20
Thus the quotient for \(\frac{5}{-\frac{2}{8}}\) is -20.

Question 14.
\(5 \frac{1}{3} \div\left(-1 \frac{1}{2}\right)\) =
\(\frac{□}{□}\)

Answer: – \(\frac{32}{9}\)

Explanation:
We have to find the quotient:
\(5 \frac{1}{3} \div\left(-1 \frac{1}{2}\right)\)
We determine the sign of the quotient.
The quotient will be negative because the numbers have different signs.
16/3 ÷ -3/2
16/3 × -2/3 = -32/9
Thus the quotient for \(5 \frac{1}{3} \div\left(-1 \frac{1}{2}\right)\) is – \(\frac{32}{9}\)

Question 15.
\(\frac{(-120)}{(-6)}\) =
_______

Answer: 20

Explanation:
We have to find the quotient:
\(\frac{(-120)}{(-6)}\)
We determine the sign of the quotient.
The quotient will be positive because the numbers have the same signs.
6 divides 120 twenty times.
\(\frac{(-120)}{(-6)}\) = 20
Thus the quotient for \(\frac{(-120)}{(-6)}\) is 20.

Question 16.
\(\frac{-\frac{4}{5}}{-\frac{2}{3}}\) =
\(\frac{□}{□}\)

Answer: \(\frac{6}{5}\)

Explanation:
We have to find the quotient:
\(\frac{-\frac{4}{5}}{-\frac{2}{3}}\)
We determine the sign of the quotient.
The quotient will be positive because the numbers have the same signs.
(-4/5) × (-3/2) = 12/10 = 6/5
Thus the quotient for \(\frac{-\frac{4}{5}}{-\frac{2}{3}}\) is \(\frac{6}{5}\)

Question 17.
1.03 ÷ (−10.3) =
_______

Answer: -0.1

Explanation:
We have to find the quotient:
1.03 ÷ (−10.3)
We determine the sign of the quotient.
The quotient will be negative because the numbers have different signs.
1.03 ÷ (-10.3) = -0.1

Question 18.
\(\frac{(-0.4)}{80}\) =
_______

Answer: -0.005

Explanation:
We have to find the quotient:
\(\frac{(-0.4)}{80}\)
We determine the sign of the quotient.
The quotient will be negative because the numbers have different signs.
\(\frac{(-0.4)}{80}\) = -0.005
Thus the quotient for \(\frac{(-0.4)}{80}\) is -0.005.

Question 19.
\(1 \div \frac{9}{5}\) =
\(\frac{□}{□}\)

Answer: \(\frac{5}{9}\)

Explanation:
We have to find the quotient:
\(1 \div \frac{9}{5}\)
We determine the sign of the quotient.
The quotient will be positive because the numbers have the same signs
\(1 \div \frac{9}{5}\) = 1 × 5/9 = 5/9
Thus the quotient for \(1 \div \frac{9}{5}\) is \(\frac{5}{9}\)

Question 20.
\(\frac{\frac{-1}{4}}{\frac{23}{0.4}}\) =
\(\frac{□}{□}\)

Answer: –\(\frac{6}{23}\)

Explanation:
We have to find the quotient:
\(\frac{\frac{-1}{4}}{\frac{23}{0.4}}\)
We determine the sign of the quotient.
The quotient will be negative because the numbers have different signs.
\(\frac{\frac{-1}{4}}{\frac{23}{0.4}}\) = (-1/4) . (24/23) = -24/92
-24/92 = -6/23
Thus the quotient for \(\frac{\frac{-1}{4}}{\frac{23}{0.4}}\) is –\(\frac{6}{23}\)

Question 21.
\(\frac{-10.35}{-2.3}\) =
_______

Answer: 4.5

Explanation:
We have to find the quotient:
\(\frac{-10.35}{-2.3}\)
We determine the sign of the quotient.
The quotient will be positive because the numbers have the same signs
\(\frac{-10.35}{-2.3}\) = 4.5
So, the quotient for \(\frac{-10.35}{-2.3}\) is 4.5

Question 22.
Alex usually runs for 21 hours a week, training for a marathon. If he is unable to run for 3 days, describe how to find out how many hours of training time he loses, and write the appropriate integer to describe how it affects his time.
_______ hours

Answer: -9

Explanation:
Alex usually runs for 21 hours a week, training for a marathon.
If he runs 21 hours a week, he runs 21/3 = 3 hours.
If he doesn’t run for 3 days, then he is losing 3(3) = 9 hours of training time.
Since he is losing hours, the integer is negative so the answer is -9.

Question 23.
The running back for the Bulldogs football team carried the ball 9 times for a total loss of 15 \(\frac{3}{4}\) yards. Find the average change in field position on each run.
\(\frac{□}{□}\) yards per run

Answer: 1 \(\frac{3}{4}\) yards per run

Explanation:
The running back for the Bulldogs football team carried the ball 9 times for a total loss of 15 \(\frac{3}{4}\) yards
Convert from mixed fractions to the improper fraction.
15 \(\frac{3}{4}\) = \(\frac{63}{4}\)
\(\frac{63}{4}\) × \(\frac{1}{9}\)
Divide 63 and 9 by 9 and then multiply the remaining factors.
\(\frac{7}{4}\) × 1 = \(\frac{7}{4}\)
Rewrite as a mixed fraction.
\(\frac{7}{4}\) = 1 \(\frac{3}{4}\) yards per run.

Question 24.
The 6:00 a.m. temperatures for four consecutive days in the town of Lincoln were -12.1 °C, -7.8°C, -14.3°C, and -7.2°C. What was the average 6:00 a.m. temperature for the four days?
_______ °C

Answer: -10.35°C

Explanation:
The average is the sum of the temperatures divided by the number of temperatures.
(-12.1 – 7.8 – 14.3 – 7.2)/4
= 41.4/4 = -10.35°C

Question 25.
Multistep
A seafood restaurant claims an increase of $1,750.00 over its average profit during a week where it introduced a special of baked clams.
a. If this is true, how much extra profit did it receive per day?
$ _______ per day

Answer:
There are 7 days in a week so divide the total profit of $1750 by 7 to find the extra profit per day.
1750/7 = 250
Thus he receive $250 extra profit per day.

Question 25.
b. If it had, instead, lost $150 per day, how much money would it have lost for the week?
$ _______

Answer:
Multiply the daily loss of $150 by 7 to get the weekly loss.
150 × 7 = $1050

Question 25.
c. If its total loss was $490 for the week, what was its average daily change?
$ _______ per day

Answer:
Since the company lost $490, its income changed by -$490.
Divide the change in income by 7 to find the average daily change.
-$490/7 = -$70
Thus the average daily change is -$70 per day.

Question 26.
A hot air balloon descended 99.6 meters in 12 seconds. What was the balloon’s average rate of descent in meters per second?
_______ m/s

Answer: 8.3 meters per second

Explanation:
Given that,
A hot air balloon descended 99.6 meters in 12 seconds.
99.6/12 = 8.3 meters per second.
Thus the balloon’s average rate of descent is 8.3 meters per second.

Divide Rational Numbers – Page No. 94

Question 27.
Sanderson is having trouble with his assignment. His shown work is as follows:
\(\frac{-\frac{3}{4}}{\frac{4}{3}}=-\frac{3}{4} \times \frac{4}{3}=-\frac{12}{12}=-1\)
However, his answer does not match the answer that his teacher gives him. What is Sanderson’s mistake? Find the correct answer.
\(\frac{□}{□}\)

Answer: – \(\frac{9}{16}\)

Explanation:
Sanderson made the mistake of not flipping the bottom fraction when he rewrote the problem as multiplication. The correct work is
\(\frac{-\frac{3}{4}}{\frac{4}{3}}\) = -3/4 × 4/3 = – \(\frac{9}{16}\)

Question 28.
Science
Beginning in 1996, a glacier lost an average of 3.7 meters of thickness each year. Find the total change in its thickness by the end of 2012.
_______ meters

Answer: 59.2 meters

Explanation:
Beginning in 1996, a glacier lost an average of 3.7 meters of thickness each year.
1996 to 2012 is 16 years so the total change in thickness is 16(3.7) = 59.2 meters.

H.O.T

Focus On Higher Order Thinking

Question 29.
Represent Real-World Problems
Describe a real-world situation that can be represented by the quotient -85 ÷ 15. Then find the quotient and explain what the quotient means in terms of the real-world situation.
Quotient: _______

Answer: -5.67

Explanation:
A possible real-world situation could be:
Sam has withdrawn $85 from his bank account over a period of 15 days. Find the average change in his account balance per day.
Answer: -85/15 = -5.67
So, the average rate of change in his account balance is -$5.67 per day.

Question 30.
Construct an Argument
Divide 5 by 4. Is your answer a rational number? Explain.
_______

Answer: The quotient is a rational number because it is a fraction.

Question 31.
Critical Thinking
Should the quotient of an integer divided by a nonzero integer always be a rational number? Why or why not?
_______

Answer:
Remember that in dividing and simplifying rational numbers, the quotient is positive if the signs of the numbers are the same, and negative if the signs of the numbers are different.
The quotient should be a rational number. This is because since the integers can be expressed as a quotient of two integers, then it is a rational number.

Applying Rational Number Operations – Guided Practice – Page No. 98

Question 1.
Mike hiked to Big Bear Lake in 4.5 hours at an average rate of 3 \(\frac{1}{5}\) miles per hour. Pedro hiked the same distance at a rate of 3 \(\frac{3}{5}\) miles per hour. How long did it take Pedro to reach the lake?
_______ hours

Answer: 4 hours

Explanation:
Given that,
Mike hiked to Big Bear Lake in 4.5 hours at an average rate of 3 \(\frac{1}{5}\) miles per hour. Pedro hiked the same distance at a rate of 3 \(\frac{3}{5}\) miles per hour.
4.5h × 3 \(\frac{3}{5}\) miles per hour = 4.5 × 3.2 miles = 14.4 miles
Plug in the distance you found in step 1 and the given rate in the problem to find the number of hours for Pedro.
14.4 miles ÷ 3 \(\frac{3}{5}\) miles per hour = 14.4 ÷ 3.6 hours = 4 hours

Question 2.
Until this year, Greenville had averaged 25.68 inches of rainfall per year for more than a century. This year’s total rainfall showed a change of −2 \(\frac{3}{8}\)% with respect to the previous average. How much rain fell this year?
_______ inches

Answer: 25.0701 inches

Explanation:
Greenville had averaged 25.68 inches of rainfall per year for more than a century.
This year’s total rainfall showed a change of −2 \(\frac{3}{8}\)% with respect to the previous average.
−2 \(\frac{3}{8}\)% = -2.375% = -0.02375
25.68 × 0.02375 ≈ 0.6099 inches
Find this year’s total rainfall
25.68 inches – 0.6099 inches = 25.0701 inches

Essential Question Check-In

Question 3.
Why is it important to consider using tools when you are solving a problem?
Type below:
___________

Answer: It is important to consider using tools, such as a calculator, when solving problems because some problems involve multiplying and dividing decimals that are too time consuming to do by hand.

Applying Rational Number Operations – Independent Practice – Page No. 99

Solve, using appropriate tools.

Question 4.
Three rock climbers started a climb with each person carrying 7.8 kilograms of climbing equipment. A fourth climber with no equipment joined the group. The group divided the total weight of climbing equipment equally among the four climbers. How much did each climber carry?
_______ kilograms

Answer: 5.85 kilograms

Explanation:
Given,
Three rock climbers started a climb with each person carrying 7.8 kilograms of climbing equipment.
A fourth climber with no equipment joined the group.
3 × 7.8 = 23.4
The group divided the total weight of climbing equipment equally among the four climbers.
23.4/4 = 5.85 kilograms
Thus each climber carry 5.85 kilograms

Question 5.
Foster is centering a photo that is 3 \(\frac{1}{2}\) inches wide on a scrapbook page that is 12 inches wide. How far from each side of the page should he put the picture?
________ \(\frac{□}{□}\) inches

Answer: 4 \(\frac{1}{4}\) inches

Explanation:
Given,
Foster is centering a photo that is 3 \(\frac{1}{2}\) inches wide on a scrapbook page that is 12 inches wide.
Let x be how far the photo is from each side of the page.
Since the photo is 3 \(\frac{1}{2}\) inches wide, then the total width of the page is
x + 3 \(\frac{1}{2}\) + x = 2x + 3 \(\frac{1}{2}\)
2x + 3 \(\frac{1}{2}\) = 12
2x + 7/2 = 12
2x = 17/2
x = 17/4
Convert the fraction to the mixed fraction.
x = 4 \(\frac{1}{4}\) inches

Question 6.
Diane serves breakfast to two groups of children at a daycare center. One box of Oaties contains 12 cups of cereal. She needs \(\frac{1}{3}\) cup for each younger child and \(\frac{3}{4}\) cup for each older child. Today’s group includes 11 younger children and 10 older children. Is one box of Oaties enough for everyone? Explain.
________

Answer: Yes

Explanation:
11 × \(\frac{1}{3}\) + 10 × \(\frac{3}{4}\)
\(\frac{11}{3}\) + \(\frac{15}{2}\)
\(\frac{22}{6}\) + \(\frac{45}{6}\) = \(\frac{67}{6}\)
= 11 \(\frac{1}{6}\)

Question 7.
The figure shows how the yard lines on a football field are numbered. The goal lines are labeled G. A referee was standing on a certain yard line as the first quarter ended. He walked 41 \(\frac{3}{4}\) yards to a yard line with the same number as the one he had just left. How far was the referee from the nearest goal line?
1Go Math Grade 7 Answer Key Chapter 3 Rational Numbers Lesson 6: Applying Rational Number Operations img 6
________ \(\frac{□}{□}\)

Answer: 29 \(\frac{1}{8}\)

Explanation:
The American football field is 100 yds long, 53 1/3 yards wide, and has 10-yard touchdown zones at each end of the field.
Let x = distance of the referee at the end of the quarter from the nearest goal.
The distance between the same yard lines on either side of the centerline is
100 – 2x
This distance is the 41 3/4 yards that the referee walked. Therefore
100 – 2x = 41.75
-2x = 41.75 – 100 = -58.25
x = 29.125 yd
Convert from decimal to fraction.
x = 29 \(\frac{1}{8}\) yards

In 8–10, a teacher gave a test with 50 questions, each worth the same number of points. Donovan got 39 out of 50 questions right. Marci’s score was 10 percentage points higher than Donovan’s.

Question 8.
What was Marci’s score? Explain.
________ %

Answer: 88 %

Explanation:
39/50 = 78/100
78/100 + 10/100 = 88/100 = 44/50
88/100 = 88%

Question 9.
How many more questions did Marci answer correctly? Explain.
________ questions

Answer: 5 questions

Explanation:
Marci got 44 correct and Donovan got 39 correct so she got 44 – 39 = 5 more questions correct.

Question 10.
Explain how you can check your answers for reasonableness.
Type below:
_____________

Answer:
You can check your answers for reasonableness by using estimates.
Donovan scored 39/50 which is about 40/50 = 80/100 = 80%
Ten percentage points higher is then 80% + 10% = 90% = 90/100 = 45/50.
Since Marci’s score was 44/50, it is a reasonable answer.

Applying Rational Number Operations – Page No. 100

For 11–13, use the expression 1.43 × \(\left(-\frac{19}{37}\right)\)

Question 11.
Critique Reasoning
Jamie says the value of the expression is close to −0.75. Does Jamie’s estimate seem reasonable? Explain.
_______

Answer: Yes

Explanation:
Jamie is correct. 1.43 is about 1.5 and -19/37 ≈ 1/2.
Since 1.5 × – 1/2 = -0.75
Jamie’s estimation is reasonable.

Question 12.
Find the product. Explain your method.
_______

Answer:
Using a calculator, you get that 1.43 × (-19/37) ≈ -0.734

Question 13.
Does your answer to Exercise 12 justify your answer to Exercise 11?
_______

Answer: Yes

Explanation:
-0.734 is close to the estimate of -0.75 so the answer to Exercise 12 justifies the answer to Exercise 11.

H.O.T

Focus On Higher Order Thinking

Question 14.
Persevere in Problem Solving
A scuba diver dove from the surface of the ocean to an elevation of −79 \(\frac{9}{10}\) feet at a rate of -18.8 feet per minute. After spending 12.75 minutes at that elevation, the diver ascended to an elevation of −28 \(\frac{9}{10}\) feet. The total time for the dive so far was 19 \(\frac{1}{8}\) minutes. What was the rate of change in the diver’s elevation during the ascent?
_______ ft/min

Answer: 24 ft/min

Explanation:
Given that,
A scuba diver dove from the surface of the ocean to an elevation of −79 \(\frac{9}{10}\) feet at a rate of -18.8 feet per minute.
After spending 12.75 minutes at that elevation, the diver ascended to an elevation of −28 \(\frac{9}{10}\) feet. The total time for the dive so far was 19 \(\frac{1}{8}\) minutes.
−79 \(\frac{9}{10}\) ÷ -18.8 = 4.25 minutes
Find the time it took to ascend by subtracting the descent time and time spent at the descent elevation from the total dive time.
19 \(\frac{1}{8}\) – 4.25 – 12.75 = 2 1/8 minutes
-28 \(\frac{9}{10}\) – (-−79 \(\frac{9}{10}\)) = 51 feet
Find the rate of change by dividing the distance in feet divided by the time.
51/2 1/8 = 24 feet per minute

Question 15.
Analyze Relationships
Describe two ways you could evaluate 37% of the sum of 27 \(\frac{3}{5}\) and 15.9. Tell which method you would use and why
Type below:
___________

Answer:
Method 1:
Rewrite numbers in fraction form and evaluate algebraically.
37% (27 \(\frac{3}{5}\) + 15.9)
37/100 (27 3/5 + 15 9/10)
37/100 (138/5 + 159/10)
37/100 (435/10)
37/100 × 87/2 = 3219/200 = 16.095
Method 2:
Rewrite numbers in decimal form and evaluate with a calculator
37% (27 \(\frac{3}{5}\) + 15.9)
0.37(27.6 + 15.9)
0.37 × 43.5 = 16.095

Question 16.
Represent Real-World Problems
Describe a real-world problem you could solve with the help of a yardstick and a calculator.
Type below:
___________

Answer:
Finding the perimeter of the table. Using the yardstick you can get the side length of the table and add these measurements to get the perimeter.

Module Quiz – 3.1 Rational Numbers and Decimals – Page No. 101

Write each mixed number as a decimal.

Question 1.
4 \(\frac{1}{5}\) =
_______

Answer: 4.2

Explanation:
To convert fractions to decimals, simply divide the numerator to the denominator. If the quotient goes on and on, it is a repeating decimal, and to write this as a decimal, put a bar on top of the repeating digits.
\(\frac{1}{5}\) = 0.2
4 + 0.2 = 4.2
4 \(\frac{1}{5}\) = 4.2

Question 2.
12 \(\frac{14}{15}\) =
_______

Answer: 12.933..

Explanation:
To convert fractions to decimals, simply divide the numerator to the denominator. If the quotient goes on and on, it is a repeating decimal, and to write this as a decimal, put a bar on top of the repeating digits.
\(\frac{14}{15}\) = 0.933..
12 + 0.933 = 12.933..
12 \(\frac{14}{15}\) = 12.933..

Question 3.
5 \(\frac{5}{32}\) =
_______

Answer: 5.15625

Explanation:
To convert fractions to decimals, simply divide the numerator to the denominator. If the quotient goes on and on, it is a repeating decimal, and to write this as a decimal, put a bar on top of the repeating digits.
\(\frac{5}{32}\) = 0.15625
5 + 0.15625 = 5.15625
5 \(\frac{5}{32}\) = 5.15625

3.2 Adding Rational Numbers

Find each sum.

Question 4.
4.5 + 7.1 =
_______

Answer: 11.6

Explanation:
To add or subtract numbers, make sure to align the digits vertically before doing the operation.
Make sure to align ones, tens, hundreds and thousands digits before adding
4.5
+7.1
 11.6

Question 5.
\(5 \frac{1}{6}+\left(-3 \frac{5}{6}\right)\) =
_______ \(\frac{□}{□}\)

Answer: 1 \(\frac{1}{3}\)

Explanation:
\(5 \frac{1}{6}+\left(-3 \frac{5}{6}\right)\) =
\(5 \frac{1}{6}\) – [/latex]3 \frac{5}{6}\right)[/latex] = 4 7/6 – 3 5/6
1 2/6 = 1 1/3
Thus \(5 \frac{1}{6}+\left(-3 \frac{5}{6}\right)\) = 1 \(\frac{1}{3}\)

3.3 Subtracting Rational Numbers

Find each difference.

Question 6.
\(-\frac{1}{8}-\left(6 \frac{7}{8}\right)\) =
_______

Answer: -7

Explanation:
Both numbers are negative. so add the opposites of each number and write the answer as a negative.
\(-\frac{1}{8}-\left(6 \frac{7}{8}\right)\)
-(\(\frac{1}{8}+\left(6 \frac{7}{8}\right)\))
= – 6 \(\frac{8}{8}\) = -6 – 1 = -7

Question 7.
14.2 − (−4.9) =
_______

Answer: 19.1

Explanation:
14.2 − (−4.9)
= 14.2 + 4.9 = 19.1

3.4 Multiplying Rational Numbers

Multiply.

Question 8.
\(-4\left(\frac{7}{10}\right)\) =
\(\frac{□}{□}\)

Answer: –\(\frac{14}{5}\)

Explanation:
Multiply the whole number with the numerator. write this product in the numerator and keep the same denominator.
\(-4\left(\frac{7}{10}\right)\) = –\(\frac{14}{5}\)

Question 9.
−3.2(−5.6)(4) =
_______

Answer: 71.68

Explanation:
Multiply the first two numbers. There are two negative signs so the answer will be positive.
−3.2(−5.6)(4) = 17.92 × 4 = 71.68

3.5 Dividing Rational Numbers

Find each quotient.

Question 10.
\(-\frac{19}{2} \div \frac{38}{7}\) =
\(\frac{□}{□}\)

Answer: –\(\frac{7}{4}\)

Explanation:
\(-\frac{19}{2} \div \frac{38}{7}\)
-19/2 × 7/38 = -7/2 × 1/2
= –\(\frac{7}{4}\)

Question 11.
\(\frac{-32.01}{-3.3}\) =
_______

Answer: 9.7

Explanation:
Given,
\(\frac{-32.01}{-3.3}\)
Remember that dividing two negatives gives a positive answer.
-32.01 ÷ -3.3 = 9.7

3.6 Applying Rational Number Operations

Question 12.
Luis bought stock at $83.60. The next day, the price increased by 15.35 dollars. This new price changed by −4 \(\frac{3}{4}\)% the following day. What was the final stock price? Is your answer reasonable? Explain.
$ _______

Answer: $94.25

Explanation:
83.60 + 15.35 = 98.95
98.95 × −4 \(\frac{3}{4}\)% = 98.95 × -0.0475 = 4.70
98.95 – 4.70 = $94.25

Essential Question

Question 13.
How can you use negative numbers to represent real-world problems?
Type below:
___________

Answer:
Negative numbers can be used in real-world problems to represents decreases or values that are below a level considered to be 0.

MODULE 3 MIXED REVIEW – Selected Response – Page No. 102

Question 1.
What is −7 \(\frac{5}{12}\) written as a decimal?
Options:
a. -7.25
b. -7.333…
c. -7.41666…
d. -7.512

Answer: -7.41666…

Explanation:
Given,
−7 \(\frac{5}{12}\)
Convert from fraction to decimal.
5 ÷ 12 = 0.4166..
−7 \(\frac{5}{12}\) = -7.4166….
Thus the correct answer is option C.

Question 2.
Glenda began the day with a golf score of -6 and ended with a score of -10. Which statement represents her golf score for that day?
Options:
a. -6 – (-10) = 4
b. -10 – (-6) = -4
c. -6 + (-10) = -16
d. -10 + (-6) = -16

Answer: -10 – (-6) = -4

Explanation:
Given,
Her golf score for the day can be found by subtracting her ending score and her beginning score which gives
-10 – (-6) = -10 + 6 = -4
So, the correct answer is option B.

Question 3.
A submersible vessel at an elevation of -95 feet descends to 5 times that elevation. What is the vessel’s new elevation?
Options:
a. -475 ft
b. -19 ft
c. 19 ft
d. 475 ft

Answer: -475 ft

Explanation:
Given,
A submersible vessel at an elevation of -95 feet descends to 5 times that elevation.
-95 feet × 5 = -475 feet
Thus the correct answer is option A.

Question 4.
The temperature at 7 P.M. at a weather station in Minnesota was -5 °F. The temperature began changing at the rate of -2.5 °F per hour. What was the temperature at 10 P.M.?
Options:
a. -15 °F
b. -12.5 °F
c. 2.5 °F
d. 5 °F

Answer: -12.5 °F

Explanation:
Find the total change in temperature by multiplying the rate of change per hour times the number of hours from 7 pm to 10 pm.
-5 + (-7.5) = -12.5°F
Thus the correct answer is option B.

Question 5.
What is the sum of -2.16 and -1.75?
Options:
a. 0.41
b. 3.91
c. -0.41
d. -3.91

Answer: -3.91

Explanation:
Both numbers are negative so add their opposites and make the answer negative.
-2.16 + (-1.75) = -(2.16 + 1.75) = -3.91
So, the correct answer is option D.

Question 6.
On Sunday, the wind chill temperature reached -36 °F. On Monday, the wind chill temperature only reached \(\frac{1}{4}\) of Sunday’s wind chill temperature. What was the lowest wind chill temperature on Monday?
Options:
a. -9 °F
b. -36 \(\frac{1}{4}\) °F
c. -40 °F
d. -144 °F

Answer: -9 °F

Explanation:
Given that,
On Sunday, the wind chill temperature reached -36 °F.
On Monday, the wind chill temperature only reached \(\frac{1}{4}\) of Sunday’s wind chill temperature.
-36 × \(\frac{1}{4}\) = -9°F
Thus the correct answer is option A.

Question 7.
The level of a lake was 8 inches below normal. It decreased 1 \(\frac{1}{4}\) inches in June and 2 \(\frac{3}{8}\) inches more in July. What was the new level with respect to the normal level?
Options:
a. -11 \(\frac{5}{8}\) in.
b. -10 \(\frac{5}{8}\) in.
c. -9 \(\frac{1}{8}\) in.
d. -5 \(\frac{3}{8}\) in.

Answer: -11 \(\frac{5}{8}\) in.

Explanation:
The level of a lake was 8 inches below normal. It decreased 1 \(\frac{1}{4}\) inches in June and 2 \(\frac{3}{8}\) inches more in July.
The initial level is below normal so it is represented by a negative number. The level continued to decrease in June and July so those changes are also represented by negative numbers.
Find the sum of these values to find what the new level was with respect to the normal level.
-8 – 1 \(\frac{1}{4}\) – 2 \(\frac{3}{8}\)
= -8 – \(\frac{5}{4}\) – \(\frac{19}{8}\)
= – \(\frac{93}{8}\)
= – 11 \(\frac{5}{8}\)
Thus the correct answer is option A.

Mini-Task

Question 8.
The average annual rainfall for a town is 43.2 inches.
a. What is the average monthly rainfall?
________

Answer:
If the average rainfall us 43.2 inches then the monthly rainfall is 43.2/12 = 3.6 inches since there are 12 months in a year.

Question 8.
Go Math Grade 7 Answer Key Chapter 3 Rational Numbers MODULE 3 MIXED REVIEW img 7
b. The difference of a given month’s rainfall from the average monthly rainfall is called the deviation. What is the deviation for each month shown?
May: ___________ inch
June: ___________ inches
July: ___________ inches

Answer:
The deviation for May is 2 3/5 – 3.6 = 2.6 – 3.6 = -1 inches.
The deviation for June is 7/8 – 3.6 = -2.725 inches.
The deviation for July 4 1/4 – 3.6 = 0.65 inches.

Question 8.
c. The average monthly rainfall for the previous 9 months was 4 inches. Did the town exceed its average annual rainfall? If so, by how much?
________

Answer:
If is rained 4 inches for 9 months, the total amount of rain over the 12 month period is than 9(4) + 2 3/5 + 7/8 + 4 1/4
= 36 + 2.6 + 0.875 + 4.25 = 43.725.
Since this is greater than the average annual rainfall of 43.2, the town did exceed is average annual rainfall.
the difference of 43.725 and 43.2 is
43.725 – 43.2 = 0.525
so, it exceeded it by 0.525 inches.

Module 3 Review – Rational Numbers – Page No. 106

EXERCISES

Write each mixed number as a whole number or decimal. Classify each number according to the group(s) to which it belongs: rational numbers, integers, or whole numbers.

Question 1.
\(\frac{3}{4}\)
________

Answer: 0.75, rational

Explanation:
Write as a decimal by dividing 3 by 4. A shortcut with fourths is to think of the fractions in terms of money. 4 quarters make a dollar and 3 quarters is $0.75 so three-fourths is 0.75 in decimal form.
Since \(\frac{3}{4}\) could not be written as a whole number or integer, it is a rational number.

Question 2.
\(\frac{8}{2}\)
________

Answer: 4

Explanation:
\(\frac{8}{2}\) = 4
Since 4 doesn’t have a decimal and is positive, it is a whole number. All whole numbers are also integers and rational numbers so 4 is a rational number, integer, and a whole number.

Question 3.
\(\frac{11}{3}\)
________

Answer: 3.66

Explanation:
Rewrite as a mixed number and then divide 2 by 3 to get the decimal part of the number.
\(\frac{11}{3}\) = 3 2/3 = 3.666…
Since 3.66.. has a decimal, it is not an integer or whole number. Therefore it is a rational number only.

Question 4.
\(\frac{5}{2}\)
________

Answer: 2.5

Explanation:
Write as a mixed number and then divide 1 by 2 to get the decimal part of the number a shortcut is to think of the fraction in terms of money. Half a dollar is $0.50 so one half equals 0.50 = 0.50
Since 2.5 has a decimal, it is not an integer or whole number. Therefore 2.5 is a rational number only.

Find each sum or difference.

Question 5.
−5 + 9.5
________

Answer: 4.5

Explanation:
Rewrite as subtraction and then subtract.
-5 + 9.5 = 4.5

Question 6.
\(\frac{1}{6}\) + (−\(\frac{5}{6}\))
\(\frac{□}{□}\)

Answer: –\(\frac{2}{3}\)

Explanation:
Rewrite as subtraction and then subtract.
\(\frac{1}{6}\) + (−\(\frac{5}{6}\))
\(\frac{1}{6}\) −\(\frac{5}{6}\)
= –\(\frac{4}{6}\) = –\(\frac{2}{3}\)

Question 7.
−0.5 + (−8.5)
________

Answer: -9

Explanation:
Both numbers are negative so add their opposites and write the answers as a negative.
−0.5 + (−8.5) = -(0.5 + 8.5) = -9

Question 8.
−3 − (−8)
________

Answer: 5

Explanation:
Rewrite as addition since subtracting a negative is the same as adding a positive.
−3 − (−8) = -3 + 8 = 5

Question 9.
5.6 − (−3.1)
________

Answer: 8.7

Explanation:
Rewrite as addition since subtracting a negative is the same as adding a positive.
5.6 − (−3.1) = 5.6 + 3.1 = 8.7

Question 10.
3 \(\frac{1}{2}\) − 2 \(\frac{1}{4}\)
\(\frac{□}{□}\)

Answer: 1 \(\frac{1}{4}\)

Explanation:
Get common denominator.
3 \(\frac{1}{2}\) − 2 \(\frac{1}{4}\)
3 \(\frac{2}{4}\) − 2 \(\frac{1}{4}\) = 1 \(\frac{1}{4}\)

Find each product or quotient

Question 11.
−9 × (−5)
________

Answer: 45

Explanation:
Multiply two negative numbers make a positive number.
−9 × (−5) = 45

Question 12.
0 × (−7)
________

Answer: 0

Explanation:
Any number multiplied with 0 will be zero.
So, the product is 0.

Question 13.
−8 × 8
________

Answer: -64

Explanation:
Multiply since there is only one negative the answer is negative.
-8 × 8 = -64

Question 14.
\(\frac{-56}{8}\)
________

Answer: -7

Explanation:
Divide since there is only one negative the answer is negative.
8 divides 56 seven times.
\(\frac{-56}{8}\) = -7

Question 15.
\(\frac{-130}{-5}\)

Answer: 26

Explanation:
Divide since there are two negative signs the answer is positive.
\(\frac{-130}{-5}\) = 26

Question 16.
\(\frac{34.5}{1.5}\)
________

Answer: 23

Explanation:
Divide since both the numbers are positive the answer will be positive.
\(\frac{34.5}{1.5}\) = 23
1.5 divides 34.5 23 times.
So, the quotient is 23.

Question 17.
\(-\frac{2}{5}\left(-\frac{1}{2}\right)\left(-\frac{5}{6}\right)\)
\(\frac{□}{□}\)

Answer: –\(\frac{1}{6}\)

Explanation:
Multiply by cancelling the 2s and 5s an odd number of negatives makes a negative so the answer is negative.
\(-\frac{2}{5}\left(-\frac{1}{2}\right)\left(-\frac{5}{6}\right)\) = –\(\frac{1}{6}\)

Question 18.
\(\frac{1}{5}\left(-\frac{5}{7}\right)\left(\frac{3}{4}\right)\)
\(\frac{□}{□}\)

Answer: –\(\frac{3}{28}\)

Explanation:
\(\frac{1}{5}\left(-\frac{5}{7}\right)\left(\frac{3}{4}\right)\)
multiply by cancelling the 5s
\(\frac{1}{5}\left(-\frac{5}{7}\right)\left(\frac{3}{4}\right)\) = – 3/7×4 = -3/28
Thus \(\frac{1}{5}\left(-\frac{5}{7}\right)\left(\frac{3}{4}\right)\) = –\(\frac{3}{28}\)

Question 19.
Lei withdrew $50 from her bank account every day for a week. What was the change in her account in that week?
$ ________

Answer: -$350

Explanation:
Lei withdrew $50 from her bank account every day for a week.
Convert from week to days
1 week = 7 days
7 × -50 = -350
The change in her account is -$350.

Question 20.
Dan is cutting 4.75 foot lengths of twine from a 240 foot spool of twine. He needs to cut 42 lengths, and says that 40.5 feet of twine will remain. Show that this is reasonable.
Type below:
__________

Answer:
The estimation of 4.75 is 5 and 42 is 40.
5 × 40 = 200
So he will be using about 200 feet.
He has 240 feet so he will have about 240-200 = 40 feet remaining.
Since 40 ≈ 40.5
The answer is reasonable.

Unit 1 Performance Tasks – Page No. 107

Question 1.
Armand is an urban planner, and he has proposed a site for a new town library. The site is between City Hall and the post office on Main Street.
Go Math Grade 7 Answer Key Chapter 3 Rational Numbers Unit 1 Performance Tasks img 8
The distance between City Hall and the post office is 612 miles. City Hall is 114 miles closer to the library site than it is to the post office.
a. Write 6 \(\frac{1}{2}\) miles and 1 \(\frac{1}{4}\) miles as decimals
6 \(\frac{1}{2}\) = __________
1 \(\frac{1}{4}\) = __________

Answer:
Write as decimal by dividing 1 by 2 and dividing 1 by 4 a shortcut is to think about ur in terms of money. Half of a dollae is $0.50 and a quarter is $0.25
So 1/2 = 0.50 = 0.5
1/4 = 0.25
6 1/2 = 6.5 and 1 1/4 = 1.25

Question 1.
b. Let d represent the distance from City Hall to the library site. Write an expression for the distance from the library site to the post office.
__________

Answer:
The library is closer to City Hall than the post office is so d is the difference between the distance from City Hall to the Post Office and the distance between City Hall and the Library Site.
d = 6 1/2 – 1 1/4

Question 1.
c. Write an equation that represents the following statement: The distance from City Hall to the library site plus the distance from the library site to the post office is equal to the distance from City Hall to the post office.
Type below:
__________

Answer:
The distance from the City Hall to the library is d, the distance from the library to the post office is 1 1/4 since the library is 1 1/4 miles closer to City Hall than the post office is, the distance from City Hall to the Post Office is 6 1/4
d + 1 1/4 = 6 1/4

Question 1.
d. Solve your equation from part c to determine the distance from City Hall to the library site, and the distance from the post office to the library site.
City Hall to library site: __________ miles
Library site to post office: __________ miles

Answer:
d = 6 1/2 – 1 1/4
d = 6 2/4 – 1 1/4
d = 5 1/4
Thus the distance is 5 1/4 miles.

Question 2.
Sumaya is reading a book with 288 pages. She has already read 90 pages. She plans to read 20 more pages each day until she finishes the book.
a. Sumaya writes the equation 378 = -20d to find the number of days she will need to finish the book. Identify the errors that Sumaya made.
Type below:
__________

Answer:
She made the mistake of using -20 in the equation instead of a positive 20. The negative can’t be used since she is not reading a negative number of pages per day.
She also made the mistake of adding 90 to 288 instead of subtracting.
Since she has already read 90 pages she has less than 288 pages left to read, not more.
288 – 90 = 198
The correct equation is 198 = 20d

Question 2.
b. Write and solve an equation to determine how many days Sumaya will need to finish the book. In your answer, count part of a day as a full day. Show that your answer is reasonable.
______ days

Answer:

198 = 20d is dividing both sides by 20 gives d = 198/20 = 9.9
Rounding this up gives 10 days.
This answer is reasonable since the books is about 300 pages and she has read about 100 pages of the book leaving about 200 pages left to read.
She is reading 20 pages per day and 20 × 10 = 200
So it would take 10 days to read about 200 pages.

Question 2.
c. Estimate how many days you would need to read a book about the same length as Sumaya’s book. What information did you use to find the estimate?
Type below:
__________

Answer:
Sumaya’s book is about 300 pages. Reading 20 pages a day would mean it would take about 300/20 = 15 days to read the book.

Unit 1 Performance Tasks – Page No. 108

Question 3.
Jackson works as a veterinary technician and earns $12.20 per hour.
a. Jackson normally works 40 hours a week. In a normal week, what is his total pay before taxes and other deductions?
$ ______

Answer: $488

Explanation:
Jackson works as a veterinary technician and earns $12.20 per hour.
Jackson normally works 40 hours a week.
40 × $12.20 = $488
Thus the total pay before taxes and other deductions is $488.

Question 3.
b. Last week, Jackson was ill and missed some work. His total pay before deductions was $372.10. Write and solve an equation to find the number of hours Jackson worked.
______ hours

Answer: 30.5 hours

Explanation:
Jackson works as a veterinary technician and earns $12.20 per hour.
His total pay before deductions was $372.10.
$12.20h = $372.10
h = 372.10/12.20
h = 30.5 hours

Question 3.
c. Jackson records his hours each day on a time sheet. Last week when he was ill, his time sheet was incomplete. How many hours are missing? Show your work. Then show that your answer is reasonable.
Go Math Grade 7 Answer Key Chapter 3 Rational Numbers Unit 1 MIXED REVIEW img 9
______ hours

Answer: 6.75 hours

Explanation:
8 + 7.25 + 8.5 = 23.75
30.5 – 23.75 = 6.75 hours

Question 3.
d. When Jackson works more than 40 hours in a week, he earns 1.5 times his normal hourly rate for each of the extra hours. Jackson worked 43 hours one week. What was his total pay before deductions? Justify your answer.
$ __________________

Answer: $542.90

Explanation:
When Jackson works more than 40 hours in a week, he earns 1.5 times his normal hourly rate for each of the extra hours.
Jackson worked 43 hours one week.
40 × 12.20 + 3 × 1.5 × 12.20 = $488 + $54.90 = $542.90

Question 3.
e. What is a reasonable range for Jackson’s expected yearly pay before deductions? Describe any assumptions you made in finding your answer.
$ __________________

Answer:

Assuming he works between 40 and 45 hours per week, his weekly pay range is between 40 × 12.20 = $488
40 × 12.20 + 5 × 1.5 × 12.20 = 488 + 91.50 = $579.50
Since there are 52 weeks in a year, his yearly pay is between 52 × 488 ≈ $25,000
and 52 × $579.50 ≈ $30,000.

Unit 1 MIXED REVIEW – Selected Response – Page No. 109

Question 1.
What is −6 \(\frac{9}{16}\) written as a decimal?
Options:
a. -6.625
b. -6.5625
c. -6.4375
d. -6.125

Answer: -6.5625

Explanation:
−6 \(\frac{9}{16}\)
Divide 9 by 16 to get 9/16 = 0.5625.
6 \(\frac{9}{16}\) = 6 + 0.5625 = 6.5625
−6 \(\frac{9}{16}\) = -6.5625
Thus the correct answer is option B.

Question 2.
Working together, 6 friends pick 14 \(\frac{2}{5}\) pounds of pecans at a pecan farm. They divide the pecans equally among themselves. How many pounds does each friend get?
Options:
a. 20 \(\frac{2}{5}\) pounds
b. 8 \(\frac{2}{5}\) pounds
c. 2 \(\frac{3}{5}\) pounds
d. 2 \(\frac{2}{5}\) pounds

Answer: \(\frac{2}{5}\) pounds

Explanation:
Divide the number of pounds by the number of friends to get the number of pounds each friend gets.
14 \(\frac{2}{5}\)/6 = 14.4/6 = 2.4 pounds.
2.4 = 2 \(\frac{2}{5}\) pounds
Thus the correct answer is option D.

Question 3.
What is the value of (−3.25)(−1.56)?
Options:
a. -5.85
b. -5.07
c. 5.07
d. 5.85

Answer: 5.07

Explanation:
Multiply two negatives make a positive.
So the answer is positive.
(−3.25)(−1.56) = 5.07
The answer is option C.

Question 4.
Mrs. Rodriguez is going to use 6 \(\frac{1}{3}\) yards of material to make two dresses. The larger dress requires 3 \(\frac{2}{3}\) yards of material. How much material will Mrs. Rodriguez have left to use on the smaller dress?
Options:
a. 1 \(\frac{2}{3}\) yards
b. 2 \(\frac{1}{3}\) yards
c. 2 \(\frac{2}{3}\) yards
d. 3 \(\frac{1}{3}\) yards

Answer: 2 \(\frac{2}{3}\) yards

Explanation:
Subtract the yards of material for the larger dress from the total yards of material.
6 \(\frac{1}{3}\) yards – 3 \(\frac{2}{3}\) yards = 2 \(\frac{2}{3}\) yards
Thus the correct answer is option C.

Question 5.
Jaime had $37 in his bank account on Sunday. The table shows his account activity for the next four days. What was the balance in Jaime’s account after his deposit on Thursday?
Go Math Grade 7 Answer Key Chapter 3 Rational Numbers Unit 1 MIXED REVIEW img 10
Options:
a. $57.49
b. $59.65
c. $94.49
d. $138.93

Answer: $94.49

Explanation:
Add up all the deposits and withdrawals to his original balance make sure deposits are represented by positive numbers and withdrawals are represented by negative numbers.
37 + 17.42 – 12.60 – 9.62 + 62.29 = 94.49
Thus the correct answer is option C.

Question 6.
A used motorcycle is on sale for $3,600. Erik makes an offer equal to \(\frac{3}{4}\) of this price. How much does Erik offer for the motorcycle?
Options:
a. $4800
b. $2700
c. $2400
d. $900

Answer: $2700

Explanation:
Given that,
A used motorcycle is on sale for $3,600. Erik makes an offer equal to \(\frac{3}{4}\) of this price.
\(\frac{3}{4}\) × 3600 = 2700
Thus the correct answer is option B.

Question 7.
Ruby ate \(\frac{1}{3}\) of a pizza, and Angie ate \(\frac{1}{5}\) of the pizza. How much of the pizza did they eat in all?
Options:
a. 1 \(\frac{1}{5}\) of the pizza
b. \(\frac{1}{8}\) of the pizza
c. \(\frac{3}{8}\) of the pizza
d. \(\frac{8}{15}\) of the pizza

Answer: \(\frac{8}{15}\) of the pizza

Explanation:
Ruby ate \(\frac{1}{3}\) of a pizza, and Angie ate \(\frac{1}{5}\) of the pizza.
\(\frac{1}{3}\) = \(\frac{1}{5}\) = \(\frac{5}{15}\) + \(\frac{3}{15}\) = \(\frac{8}{15}\)
Thus the correct answer is option D.

Unit 1 MIXED REVIEW – Page No. 110

Question 8.
Winslow buys 1.2 pounds of bananas. The bananas cost $1.29 per pound. To the nearest cent, how much does Winslow pay for the bananas?
Options:
a. $1.08
b. $1.20
c. $1.55
d. $2.49

Answer: $1.55

Explanation:
Winslow buys 1.2 pounds of bananas. The bananas cost $1.29 per pound.
1.2 × $1.29 = $1.548 ≈ $1.55
Thus the correct answer is option C.

Question 9.
The temperature was -10 °F and dropped by 16 °F. Which statement represents the resulting temperature in degrees Fahrenheit?
Options:
a. -10 – (-16) = -6
b. -10 – 16 = -26
c. 10 – (-16) = 26
d. -10 + 16 = 6

Answer: -10 – 16 = -26

Explanation:
The temperature was -10 °F and dropped by 16 °F.
-10 + (-16) = -26°F.
So, the correct answer is option B.

Question 10.
A scuba diver at a depth of -12 ft (12 ft below sea level), dives down to a coral reef that is 3.5 times the diver’s original depth. What is the diver’s new depth?
Options:
a. -420 ft
b. -42 ft
c. 42 ft
d. about 3.4 ft

Answer: -42 ft

A scuba diver at a depth of -12 ft, dives down to a coral reef that is 3.5 times the diver’s original depth.
-12 × 3.5 = -42 ft
So, the correct answer is option B.

Question 11.
The school Spirit Club spent $320.82 on food and took in 643.59 selling the food. How much did the Spirit Club make?
Options:
a. -$322.77
b. -$964.41
c. $322.77
d. $964.41

Answer: $322.77

Explanation:
The school Spirit Club spent $320.82 on food and took in 643.59 selling the food.
$643.59 – $320.82 = $322.77
So, the answer is option C.

Question 12.
Lila graphed the points -2 and 2 on a number line. What does the distance between these two points represent?
Options:
a. the sum of -2 and 2
b. the difference of 2 and -2
c. the difference of -2 and 2
d. the product of -2 and 2

Answer: the difference of 2 and -2

Explanation:
Distance is found by subtracting the larger number and the smaller number so it is the difference of 2 and -2.
Thus the correct answer is option B.

Question 13.
What is a reasonable estimate of −3 \(\frac{4}{5}\) + (−5.25) and the actual value?
Options:
a. -4 + (-5) = -9; −9 \(\frac{1}{20}\)
b. -3 + (-5) = -8; −8 \(\frac{1}{20}\)
c. -4 + (-5) = -1; −8 \(\frac{9}{20}\)
d. -3 + (-5) = 8; 8 \(\frac{1}{20}\)

Answer: -4 + (-5) = -9; −9 \(\frac{1}{20}\)

Explanation:
−3 \(\frac{4}{5}\) + (−5.25)
−3 \(\frac{4}{5}\) ≈ -4
−5.25 ≈ -5
So the sum is about -4 + -5 = -9.
The estimated answer is -9.

Mini-Task

Question 14.
Juanita is watering her lawn using the water stored in her rainwater tank. The water level in the tank drops \(\frac{1}{3}\) inch every 10 minutes she waters.
a. What is the change in the tank’s water level after 1 hour?
______ inches

Answer: -2 inches

Explanation:
Juanita is watering her lawn using the water stored in her rainwater tank.
There are six 10 minute intervals in 1 hour so change is
6 × –\(\frac{1}{3}\) = -2 inches
Therefore, the tank’s water level after 1 hour is -2 inches.

Question 14.
b. What is the expected change in the tank’s water level after 2.25 hours?
______ inches

Answer: -4.5 inches

Explanation:
Since the water level drops 2 inches every hour, in 2.25 hours the water level change will be -2 × 2.25 = -4.5 inches
Thus the expected change in the tank’s water level after 2.25 hours is -4.5 inches.

Question 14.
c. If the tank’s water level is 4 feet, how many days can Juanita water if she waters for 15 minutes each day?
______ days

Answer: 96 days

Explanation:
15 minutes is 1/4 of an hour so in 15 minutes the water level will have dropped by 2 × 1/4 = 1/2 inches.
Since the water level is initially 4 feet = 48 inches
She can water for 48/1/2 = 48 × 2 = 96 days
It takes 96 days if she waters for 15 minutes.

Conclusion:

Hope the info prevailed in this article is beneficial for all the students. Keep in touch with us to get the latest updates regarding Go Math Grade 7 Answer Key Chapter 3 Rational Numbers. Also, the students of 4th grade can get the solutions for all the chapters on Go Math Grade 7 Answer Key page.

Go Math Grade 3 Answer Key Chapter 2 Represent and Interpret Data Extra Practice

go-math-grade-3-chapter-2-represent-and-interpret-data-extra-practice-answer-key

Go Math Grade 3 Answer Key Chapter 2 Represent and Interpret Data Extra Practice helps you clarify all your concerns easily. Know the Tips and Tricks to Solve the Problems on Go Math Grade 3 Ch 2 from Go Math Grade 3 Answer Key. Cross check your solutions with the Go Math Grade 3 Chapter 2 Answer Key Extra Practice and assess your preparation standard and bridge the knowledge gap accordingly.

Go Math Grade 3 Answer Key Chapter 2 Represent and Interpret Data Extra Practice

You can solve problems in Go Math Grade 3 Chapter 2 Represent and Interpret Data using different approaches. But to help you solve them easily we have listed the step by step solutions for all the exercise questions in HMH Go Math Grade 3 Chapter 2.  Learn all of them easily and enhance your subject knowledge on the corresponding topics instantly. Utilize the Go Math Solution Key Chapter 2 Represent and Interpret Data Extra Practice for better understanding.

Common Core – Page No. 45000

Lesson 2.1

Use the Pets tables for 1–2.

Question 1.
Manny collected data about pets owned by students in his class. Complete Manny’s tally table and frequency table.

Pets
Pets Tally
Cat ________
Dog ________
Bird ________
Fish ________

Answer:

Pets
Pets Tally
Cat ||||
Dog ||
Bird |
Fish |

Question 1.

Pets
Pets Frequency
______ 4
______ 2
______ 1
______ 1

Answer:

Pets
Pets Frequency
Cat 4
Dog 2
Bird 1
Fish 1

Question 2.
How many more students have cats than have dogs and birds combined?
______ students

Answer: 1

Explanation:

Number of students who have cats = 4
Number of students who have Dogs = 2
Number of students who have Birds = 1
Total students who have Dogs and birds combined = 2 + 1 = 3
To know how many more students have cats than have dogs and birds combined, we need to subtract students who have Dogs and birds combined from the Number of students who have cats.
4 – 3 = 1
Therefore the answer is 1.

Lessons 2.2–2.3

Use the Seashells picture graph for 3–5.
Go Math Grade 3 Answer Key Chapter 2 Represent and Interpret Data Extra Practice Common Core img 1

Question 3.
Maggie has a picture graph that shows the seashells she collected. How many seashells did Maggie collect in all?
______ seashells

Answer: 85

Explanation:

Number of Cockle shells =4  Go Math Grade 3 Answer Key Chapter 2 Represent and Interpret Data Extra Practice Common Core img 2
Each Go Math Grade 3 Answer Key Chapter 2 Represent and Interpret Data Extra Practice Common Core img 2 = 10 shells
So, Number of Cockle shells = 10 + 10 + 10 + 10 = 40 shells
Number of Conch Shells = 2 Go Math Grade 3 Answer Key Chapter 2 Represent and Interpret Data Extra Practice Common Core img 2
Each Go Math Grade 3 Answer Key Chapter 2 Represent and Interpret Data Extra Practice Common Core img 2 = 10 shells
= 10 + 10 = 20 shells
Number of Lightning Whelk = 2 and a half Go Math Grade 3 Answer Key Chapter 2 Represent and Interpret Data Extra Practice Common Core img 2
Each Go Math Grade 3 Answer Key Chapter 2 Represent and Interpret Data Extra Practice Common Core img 2 = 10 shells
= 10 + 10 + 5 = 25 shells

Total number of shells that Maggie collected = 40 + 20 + 25 = 85 shells

Question 4.
How many more cockle shells did she collect than lightning whelks?
______ shells.

Answer: 15

Explanation:

Number of Cockle shells =4  Go Math Grade 3 Answer Key Chapter 2 Represent and Interpret Data Extra Practice Common Core img 2
Each Go Math Grade 3 Answer Key Chapter 2 Represent and Interpret Data Extra Practice Common Core img 2 = 10 shells
So, Number of Cockle shells = 10 + 10 + 10 + 10 = 40 shells
Number of Lightning Whelk = 2 and a half Go Math Grade 3 Answer Key Chapter 2 Represent and Interpret Data Extra Practice Common Core img 2
= 10 + 10 + 5 = 25 shells
Subtract Number of Lightning Whelk shells from Number of Cockle shells
= 40 – 25 = 15 shells

Question 5.
What if the key were “Each Go Math Grade 3 Answer Key Chapter 2 Represent and Interpret Data Extra Practice Common Core img 2 = 5 shells?” How many pictures would there be for conch?
______ pictures

Answer: 4 pictures

Explanation:

Number of Conch Shells = 2 Go Math Grade 3 Answer Key Chapter 2 Represent and Interpret Data Extra Practice Common Core img 2
If each Go Math Grade 3 Answer Key Chapter 2 Represent and Interpret Data Extra Practice Common Core img 2 = 5 shells
Then the number of Conch shells will be 4 Go Math Grade 3 Answer Key Chapter 2 Represent and Interpret Data Extra Practice Common Core img 2
= 5 + 5 + 5 + 5 = 20 shells

Page No. 46000

Common Core

Lessons 2.4–2.6

Use the Bicycle Rides frequency table for 1–3.
Go Math Grade 3 Answer Key Chapter 2 Represent and Interpret Data Extra Practice Common Core img 3

Question 1.
The frequency table shows the number of miles Sean rode on his bicycle. Use the data in the frequency table to complete the bar graph.
Type below:
_________

Answer:

Question 2.
How many more miles did Sean ride on Saturday than on Monday?
______ miles

Answer: 8 miles

Explanation:

Number of miles that Sean ride on Saturday = 12
Number of miles that Sean ride on Monday = 4
To find how many more miles did Sean ride on Saturday than on Monday
Subtract the number of miles that Sean ride on Monday from Number of miles that Sean ride on Saturday
12 – 4 = 8 miles

Question 3.
Write a number sentence to show how many miles in all Sean rode on his bicycle.
______ miles

Answer: 25 miles

Explanation:

Number of miles that Sean ride on Saturday = 12
Number of miles that Sean ride on Monday = 4
Number of miles that Sean ride on Wednesday = 9
Total number of miles that Sean ride = 12 + 9 + 4 = 25 miles

Lesson 2.7

Use the Number of Beads line plot for 4–6.
Go Math Grade 3 Answer Key Chapter 2 Represent and Interpret Data Extra Practice Common Core img 4

Question 4.
Kim is making bead necklaces. She records the number of beads on the different necklaces on a line plot. How many necklaces have exactly 50 beads?
______ necklaces

Answer: 5 necklaces

Explanation:

The line plot shows that there are 5 necklaces that have exactly 50 beads.

Question 5.
How many necklaces have fewer than 40 beads?
______ necklaces

Answer: 3 necklaces

Explanation:

The above figure shows that the number of necklaces that have less than 40 beads = 3

Question 6.
How many necklaces have 50 or more beads?
______ necklaces

Answer: 7 necklaces

Explanation:

Number of necklaces that have 50 beads = 5
Number of necklaces that have 60 beads = 2
Total necklaces that have more than 50 beads = 5 + 2 = 7 necklaces.

Conclusion

Represent and Interpret Data chapter includes bar graphs, line plots tally marks, etc. Look at the topics in Chapter 2 before you go ahead with your preparation.  To know more such questions Download Go Math Answer Key Grade 3 Chapter 2 Represent and Interpret Data pdf and prepare whenever you want.

Go Math Grade 4 Answer Key Homework Practice FL Chapter 9 Relate Fractions and Decimals

go-math-grade-4-chapter-9-relate-fractions-and-decimals-pages-169-185-answer-key

Go Math Grade 4 Answer Key Homework Practice FL Chapter 9 Relate Fractions and Decimals are provided here in pdf format for easy access. In HMH Go Math Answer Key Grade 4, students can find all chapters questions and solutions such as Relate Tenths and Decimals, Hundredths and Decimals, Equivalent Fractions, and Decimals, Relate Fractions, Decimals, and Money, Add Fractional Parts of 10 and 100, and many more. To help you out, we have complied Go Math Grade 4 Solution Key Homework Practice FL Chapter 9 Relate Fractions and Decimals for all chapters in a better way.

Go Math Grade 4 Answer Key Homework Practice FL Chapter 9 Relate Fractions and Decimals

Browse all 9th chapter topics from Go Math Grade 4 Solution Key easily and learn the basics for further studies. It will be tough to understand the Chapter 9 Fractions and decimals concepts without a lot of practice. So, we have given the best preparation resource on this page ie., Go Math Grade 4 Answer Key Homework Practice FL Chapter 9 Relate Fractions and Decimals pdf. Download the Go Math HMH 4th Grade Answer Key of chapter 9 homework practice FL and learn the concepts properly to improve math skills.

Lesson: 1 – Relate Tenths and Decimals

Lesson: 2 – Relate Hundredths and Decimals

Lesson: 3 – Equivalent Fractions and Decimals

Lesson: 4 – Relate Fractions, Decimals, and Money

Lesson: 5 – Problem Solving Money

Lesson: 6 – Add Fractional Parts of 10 and 100

Lesson: 7 – Compare Decimals

Lesson: 8

Common Core – Relate Fractions and Decimals – Page No. 171

Relate Tenths and Decimals

Write the fraction or mixed number and the decimal shown by the model.

Question 1.
Go Math Grade 4 Answer Key Homework Practice FL Chapter 9 Relate Fractions and Decimals Common Core - Relate Fractions and Decimals img 1

Question 2.
Go Math Grade 4 Answer Key Homework Practice FL Chapter 9 Relate Fractions and Decimals Common Core - Relate Fractions and Decimals img 2
Type below:
_________
Answer:
1 2/10

Explanation:
The model is divided into 10 equal parts. Each part represents one-tenth.
1 2/10 is 1 whole and 2 tenths.

Question 3.
Go Math Grade 4 Answer Key Homework Practice FL Chapter 9 Relate Fractions and Decimals Common Core - Relate Fractions and Decimals img 3
Type below:
_________

Answer:
2 3/10 = 2.3

Explanation:
grade 4 chapter 9 Common Core Image 1 499
By seeing the above number line we can say that the decimal is 2.3

Question 4.
Go Math Grade 4 Answer Key Homework Practice FL Chapter 9 Relate Fractions and Decimals Common Core - Relate Fractions and Decimals img 4
Type below:
_________

Answer:
4810 = 4.8

Explanation:
grade 4 chapter 9 Common Core Image 2 499

Write the fraction or mixed number as a decimal.

Question 5.
\(\frac{4}{10}\)
_____

Answer:
0.4

Explanation:
Write down 4 with the decimal point 1 space from the right (because 10 has 1 zero)
0.4
The decimal form for the fraction \(\frac{4}{10}\) is 0.4

Question 6.
3 \(\frac{1}{10}\)
_____

Answer:
3.1

Explanation:
Multiply 3 x 10 = 30.
Add 30 + 1 = 31.
So, 31/10.
Write down 31 with the decimal point 1 space from the right (because 10 has 1 zero)
3.1

Question 7.
\(\frac{7}{10}\)
_____

Answer:
0.7

Explanation:
Write down 7 with the decimal point 1 space from the right (because 10 has 1 zero)
0.7
The decimal form for the fraction \(\frac{7}{10}\) is 0.7

Question 8.
6 \(\frac{5}{10}\)
_____

Answer:
6.5

Explanation:
Multiply 6 x 10 = 60.
Add 60 + 5 = 65.
So, 65/10.
Write down 35 with the decimal point 1 space from the right (because 10 has 1 zero)
6.5

Question 9.
\(\frac{9}{10}\)
_____

Answer:
0.9

Explanation:
Write down 9 with the decimal point 1 space from the right (because 10 has 1 zero)
0.9
The decimal form for the fraction \(\frac{9}{10}\) is 0.9

Problem Solving

Question 10.
There are 10 sports balls in the equipment closet. Three are kickballs. Write the portion of the balls that are kickballs as a fraction, as a decimal, and in word form.
Type below:
_________

Answer:
3/10 = 0.3 = three tenths

Explanation:
Given,
There are 10 sports balls in the equipment closet.
Three are kickballs.
So, 3/10 kickballs are available.

Question 11.
Peyton has 2 pizzas. Each pizza is cut into 10 equal slices. She and her friends eat 14 slices. What part of the pizzas did they eat? Write your answer as a decimal.
_________

Answer:
1.4 pizzas

Explanation:
Peyton has 2 pizzas. Each pizza is cut into 10 equal slices.
So, total number of slices = 2 x 10 = 20.
She and her friends eat 14 slices.
So, they ate 1 whole pizza and 4 parts out of 10 slices in the second pizza.
1 4/10 = 14/10 = 1.4 pizzas.
Therefore the decimal form of the part of the pizzas they eat is 1.4 pizzas.

Common Core – Relate Fractions and Decimals – Page No. 172

Lesson Check

Question 1.
Valerie has 10 CDs in her music case. Seven of the CDs are pop music CDs. What is this amount written as a decimal?
Options:
a. 70.0
b. 7.0
c. 0.7
d. 0.07

Answer:
c. 0.7

Explanation:
Valerie has 10 CDs in her music case. Seven of the CDs are pop music CDs.
Seven CDs out of 10 CDs = 7/10 =0.7
Thus the correct answer is option c.

Question 2.
Which decimal amount is modeled below?
Go Math Grade 4 Answer Key Homework Practice FL Chapter 9 Relate Fractions and Decimals Common Core - Relate Fractions and Decimals img 5
Options:
a. 140.0
b. 14.0
c. 1.4
d. 0.14

Answer:
c. 1.4

Explanation:
1 4/10
Multiply 10 x 1 = 10.
Add 10 + 4 = 14.
So, 14/10 = 1.4.
Thus the correct answer is option c.

Spiral Review

Question 3.
Which number is a factor of 13?
Options:
a. 1
b. 3
c. 4
d. 7

Answer:
a. 1

Explanation:
13 has 1 and 13 as its factors.
Thus the correct answer is option a.

Question 4.
An art gallery has 18 paintings and 4 photographs displayed in equal rows on a wall, with the same number of each type of art in each row. Which of the following could be the number of rows?
Options:
a. 2 rows
b. 3 rows
c. 4 rows
d. 6 rows

Answer:
a. 2 rows

Explanation:
An art gallery has 18 paintings and 4 photographs displayed in equal rows on a wall, with the same number of each type of art in each row. So, 18 paintings and 4 photographs need to be divided into equal parts.
18/2 = 9; 4/2 = 2.
2 rows can be possible with 9 pictures and 2 pictures in each row.
Thus the correct answer is option a.

Question 5.
How do you write the mixed number shown as a fraction greater than 1?
Go Math Grade 4 Answer Key Homework Practice FL Chapter 9 Relate Fractions and Decimals Common Core - Relate Fractions and Decimals img 6
Options:
a. \(\frac{32}{5}\)
b. \(\frac{14}{4}\)
c. \(\frac{6}{4}\)
d. \(\frac{4}{4}\)

Answer:
b. 14/4
Explanation:
324 = 14/4. 14 divided by 4 is equal to 3 with a remainder of 2. The 3 is greater than 1. So, 14/4 > 1.
Thus the correct answer is option b.

Question 6.
Which of the following models has an amount shaded that is equivalent to the fraction \(\frac{1}{5}\)?
Options:
a. Go Math Grade 4 Answer Key Homework Practice FL Chapter 9 Relate Fractions and Decimals Common Core - Relate Fractions and Decimals img 7
b. Go Math Grade 4 Answer Key Homework Practice FL Chapter 9 Relate Fractions and Decimals Common Core - Relate Fractions and Decimals img 8
c. Go Math Grade 4 Answer Key Homework Practice FL Chapter 9 Relate Fractions and Decimals Common Core - Relate Fractions and Decimals img 9
d. Go Math Grade 4 Answer Key Homework Practice FL Chapter 9 Relate Fractions and Decimals Common Core - Relate Fractions and Decimals img 10

Answer:
c. Go Math Grade 4 Answer Key Chapter 9 Relate Fractions and Decimals Common Core - New img 9

Explanation:
a. 2/3
b. 5/10 = 1/2
c. 2/10 = 1/5
d. 1/10
Thus the correct answer is option c.

Common Core – Relate Fractions and Decimals – Page No. 173

Relate Hundredths and Decimals

Write the fraction or mixed number and the decimal shown by the model.

Question 1.
Go Math Grade 4 Answer Key Homework Practice FL Chapter 9 Relate Fractions and Decimals Common Core - Relate Fractions and Decimals img 11

Question 2.
Go Math Grade 4 Answer Key Homework Practice FL Chapter 9 Relate Fractions and Decimals Common Core - Relate Fractions and Decimals img 12
Type below:
_________

Answer:
29/100 = 0.29

Explanation:
0.20 names the same amount as 20/100.
So, the given point is at 29/100 = 0.29

Question 3.
Go Math Grade 4 Answer Key Homework Practice FL Chapter 9 Relate Fractions and Decimals Common Core - Relate Fractions and Decimals img 13
Type below:
_________

Answer:
1 54/100 = 1.54

Explanation:
From the given image, one model is one whole and another model 54 boxes shaded out of 100. So, the answer is 1 54/100 = 1.54

Question 4.
Go Math Grade 4 Answer Key Homework Practice FL Chapter 9 Relate Fractions and Decimals Common Core - Relate Fractions and Decimals img 14
Type below:
_________

Answer:
4 62/100 = 4.62

Explanation:
4.60 names the same amount as 460100. So, the given point is at 4 62/100 = 4.62

Write the fraction or mixed number as a decimal.

Question 5.
\(\frac{37}{100}\)
_____

Answer:
0.37

Explanation:
Write down 37 with the decimal point 2 spaces from the right (because 100 has 2 zeros). 0.37

Question 6.
8 \(\frac{11}{100}\)
_____

Answer:
8.11

Explanation:
8 11/100 = 811/100
Write down 811 with the decimal point 2 spaces from the right (because 100 has 2 zeros). So, 8.11 is the answer.

Question 7.
\(\frac{98}{100}\)
_____

Answer:
0.98

Explanation:
Write down 98 with the decimal point 2 spaces from the right (because 100 has 2 zeros). 0.98

Question 8.
25 \(\frac{50}{100}\)
_____

Answer:
25.50

Explanation:
25 50/100 = 2550/100
Write down 2550 with the decimal point 2 spaces from the right (because 100 has 2 zeros). So, 25.50 is the answer.

Question 9.
\(\frac{6}{100}\)
_____

Answer:
0.06

Explanation:
Write down 6 with the decimal point 2 spaces from the right (because 100 has 2 zeros). 0.06

Problem Solving

Question 10.
There are 100 pennies in a dollar. What fraction of a dollar is 61 pennies? Write it as a fraction, as a decimal, and in word form.
Type below:
_________

Answer:
61/100 pennies = 0.61 = sixty-one hundredths

Explanation:
There are 100 pennies in a dollar. So, for 61 pennies, there are 61100 pennies = 0.61 = sixty-one hundredths.

Question 11.
Kylee has collected 100 souvenir thimbles from different places she has visited with her family. Twenty of the thimbles are carved from wood. Write the fraction of thimbles that are wooden as a decimal.
_________

Answer:
It is easier to work with decimals then fractions because it is like adding whole numbers in a normal way.

Common Core – Relate Fractions and Decimals – Page No. 174

Lesson Check

Question 1.
Which decimal represents the shaded section of the model below?
Go Math Grade 4 Answer Key Homework Practice FL Chapter 9 Relate Fractions and Decimals Common Core - Relate Fractions and Decimals img 15
Options:
a. 830.0
b. 83.0
c. 8.30
d. 0.83

Answer:
d. 0.83

Explanation:
The model is divided into 100 equal parts. Each part represents one hundredth. 83 boxes are shaded out of 100.
So, the answer is 83/100 = 0.83
Thus the correct answer is option d.

Question 2.
There were 100 questions on the unit test. Alondra answered 97 of the questions correctly. What decimal represents the fraction of questions Alondra answered correctly?
Options:
a. 0.97
b. 9.70
c. 90.70
d. 970.0

Answer:
a. 0.97

Explanation:
There were 100 questions on the unit test. Alondra answered 97 of the questions correctly. So, 97/100 questions answered correctly. = 0.97
Thus the correct answer is option a.

Spiral Review

Question 3.
Which is equivalent to \(\frac{7}{8}\) ?
Options:
a. \(\frac{5}{8}+\frac{3}{8}\)
b. \(\frac{4}{8}+\frac{1}{8}+\frac{1}{8}\)
c. \(\frac{3}{8}+\frac{2}{8}+\frac{2}{8}\)
d. \(\frac{2}{8}+\frac{2}{8}+\frac{1}{8}+\frac{1}{8}\)

Answer:
c. 3/8+2/8+2/8
Explanation:
c. 3/8+2/8+2/8 = 7/8
Thus the correct answer is option c.

Question 4.
What is \(\frac{9}{10}-\frac{6}{10}\)?
Go Math Grade 4 Answer Key Homework Practice FL Chapter 9 Relate Fractions and Decimals Common Core - Relate Fractions and Decimals img 16
Options:
a. \(\frac{1}{10}\)
b. \(\frac{3}{10}\)
c. \(\frac{4}{10}\)
d. \(\frac{6}{10}\)

Answer:
b. 3/10
Explanation:
9/10−6/10. From 9 parts, 6 parts are removed. So, remaining parts are 3.
Thus the correct answer is option b.

Question 5.
Misha used 14 of a carton of 12 eggs to make an omelet. How many eggs did she use?
Options:
a. 2
b. 3
c. 4
d. 6

Answer:
b. 3

Explanation:
Misha used 14 of a carton of 12 eggs to make an omelet. 14 x 12 = 3 eggs.
Thus the correct answer is option b.

Question 6.
Kurt used the rule add 4, subtract 1 to generate a pattern. The first term in his pattern is 5. Which number could be in Kurt’s pattern?
Options:
a. 4
b. 6
c. 10
d. 14

Answer:
d. 14

Explanation:
Kurt used the rule add 4, subtract 1 to generate a pattern.
The first term in his pattern is 5.
The pattern numbers are 5, 8, 11, 14, 17, 20, etc.
So, the answer is 14.
Thus the correct answer is option d.

Common Core – Relate Fractions and Decimals – Page No. 175

Equivalent Fractions and Decimals

Write the number as hundredths in fraction form and decimal form.

Question 1.
\(\frac{5}{10}\)
\(\frac{5}{10}\) = \(\frac{5 \times 10}{10 \times 10}=\frac{50}{100}\)
Go Math Grade 4 Answer Key Homework Practice FL Chapter 9 Relate Fractions and Decimals Common Core - Relate Fractions and Decimals img 17
Think: 5 tenths is the same as 5 tenths and 0 hundredths. Write 0.50.

Question 2.
\(\frac{9}{10}\)
Type below:
_________

Answer:
90/100; 0.90

Explanation:
9/10 = 9×10/10×10 = 90/100
9 tenths is the same as 9 tenths and 0 hundredths. Write 0.90

Question 3.
0.2
Type below:
_________

Answer:
20100
0.20

Explanation:
2 tenths is the same as 2 tenths and 0 hundredths. Write 0.20.
grade 4 chapter 9 Relate Fractions and Decimals Image 6 509
2/10 = 2×10/10×10 = 20/100

Question 4.
0.8
Type below:
_________

Answer:
80100 = 0.80

Explanation:
8 tenths is the same as 8 tenths and 0 hundredths. Write 0.80.
grade 4 chapter 9 Relate Fractions and Decimals Image 5 509
8/10 = 8×10/10×10 = 80/100

Write the number as tenths in fraction form and decimal form.

Question 5.
\(\frac{40}{100}\)
Type below:
_________

Answer:
4/10 = 0.4

Explanation:
10 is a common factor of the numerator and the denominator.
40/100 = 40÷10/100÷10 = 4/10
= 0.4

Question 6.
\(\frac{10}{100}\)
Type below:
_________

Answer:
1/10 = 0.1

Explanation:
10 is a common factor of the numerator and the denominator.
10/100 = 10÷10/100÷10 = 1/10
= 0.1

Question 7.
0.60
Type below:
_________

Answer:
6/10 = 0.6

Explanation:
0.60 is 60 hundredths.
60/100.
10 is a common factor of the numerator and the denominator.
60/100 = 60÷10/100÷10 = 6/10
= 0.6

Problem Solving

Question 8.
Billy walks \(\frac{6}{10}\) mile to school each day. Write \(\frac{6}{10}\) as hundredths in fraction form and in decimal form.
Type below:
_________

Answer:
60100
0.60

Explanation:
Billy walks 6/10 mile to school each day.
6/10 = 6×10/10×10 = 60/100

Question 9.
Four states have names that begin with the letter A. This represents 0.08 of all the states. Write 0.08 as a fraction.
\(\frac{□}{□}\)

Answer:
8/100
Explanation:
0.08 is 8 hundredths. So, the fraction is 8/100

Common Core – Relate Fractions and Decimals – Page No. 176

Lesson Check

Question 1.
The fourth-grade students at Harvest School make up 0.3 of all students at the school. Which fraction is equivalent to 0.3?
Options:
a. \(\frac{3}{10}\)
b. \(\frac{30}{10}\)
c. \(\frac{3}{100}\)
d. \(\frac{33}{100}\)

Answer:
a. 3/10
Explanation:
0.3 is same as the 3 tenths.
So, the answer is 3/10
Thus the correct answer is option a.

Question 2.
Kyle and his brother have a marble set. Of the marbles, 12 are blue. This represents \(\frac{50}{100}\) of all the marbles. Which decimal is equivalent to \(\frac{50}{100}\)?
Options:
a. 50
b. 5.0
c. 0.50
d. 5,000

Answer:
c. 0.50

Explanation:

Write down 50 with the decimal point 2 spaces from the right (because 100 has 2 zeros).
So, 0.50 is the answer.
Thus the correct answer is option c.

Spiral Review

Question 3.
Jesse won his race by 3 \(\frac{45}{100}\) seconds. What is this number written as a decimal?
Options:
a. 0.345
b. 3.45
c. 34.5
d. 345

Answer:
b. 3.45

Explanation:
3 45/100 = 345/100. Write down 345 with the decimal point 2 spaces from the right (because 100 has 2 zeros). So, 3.45 is the answer.
Thus the correct answer is option b.

Question 4.
Marge cut 16 pieces of tape for mounting pictures on poster board. Each piece of tape was \(\frac{3}{8}\) inch long. How much tape did Marge use?
Options:
a. 2 inches
b. 4 inches
c. 5 inches
d. 6 inches

Answer:
d. 6 inches

Explanation:
3/8 x 16 = 6 inches
Thus the correct answer is option d.

Question 5.
Of Katie’s pattern blocks, \(\frac{9}{12}\) are triangles. What is \(\frac{9}{12}\) in simplest form?
Options:
a. \(\frac{1}{4}\)
b. \(\frac{2}{3}\)
c. \(\frac{3}{4}\)
d. \(\frac{9}{12}\)

Answer:
c. 3/4
Explanation:
9/12 is divided by 3. So, 3/4 is the answer.
Thus the correct answer is option c.

Question 6.
A number pattern has 75 as its first term. The rule for the pattern is subtract 6. What is the sixth term?
Options:
a. 39
b. 45
c. 51
d. 69

Answer:
b. 45

Explanation:
75 is the first term.
75 – 6 =69
69 – 6 = 63
63 – 6 = 57
57 – 6 = 51
51 – 6 = 45.
The sixth term is 45.
Thus the correct answer is option b.

Common Core – Relate Fractions and Decimals – Page No. 177

Relate Fractions, Decimals, and Money

Write the total money amount. Then write the amount as a fraction or a mixed number and as a decimal in terms of dollars.

Question 1.
Go Math Grade 4 Answer Key Homework Practice FL Chapter 9 Relate Fractions and Decimals Common Core - Relate Fractions and Decimals img 18

Answer:
$0.18 = 18/100 = 0.18

Explanation:
Given that 3 Pennies + 3 Nickels = 3/100 + 15/100 = 18/100

Question 2.
Go Math Grade 4 Answer Key Homework Practice FL Chapter 9 Relate Fractions and Decimals Common Core - Relate Fractions and Decimals img 19
Type below:
_________

Answer:
$0.56 = 56/100 = 0.56

Explanation:
Given that 1 Quarter + 3 dime + 1 Pennies = 25/100 + 30/100 + 1/100 = 56/100

Write as a money amount and as a decimal in terms of dollars.

Question 3.
\(\frac{25}{100}\)
Dollars: _____ Decimal: _____

Answer:
Dollars: 1 quarter = $0.25; Decimal: 0.25

Explanation:
25 our of 100 dollars = 1 quarter.
So, 25/100 = 0.25

Question 4.
\(\frac{79}{100}\)
Dollars: _____ Decimal: _____

Answer:
amount: $0.79 decimal: 0.79 of a dollar

Explanation:
79/100 = 0.79

Question 5.
\(\frac{31}{100}\)
Dollars: _____ Decimal: _____

Answer:
amount: $0.31 decimal: 0.31 of a dollar

Explanation:
31/100 = 0.31

Question 6.
\(\frac{8}{100}\)
Dollars: _____ Decimal: _____

Answer:
amount: $0.08 decimal: 0.08 of a dollar

Explanation:
81/00 = 0.08

Question 7.
\(\frac{42}{100}\)
Dollars: _____ Decimal: _____

Answer:
amount: $0.42 decimal: 0.42 of a dollar

Explanation:
42/100 = 0.42

Write the money amount as a fraction in terms of dollars.

Question 8.
$0.87
\(\frac{□}{□}\)

Answer:
87/100 of a dollar

Explanation:
grade 4 chapter 9 Relate Fractions and Decimals Image 1 517
$0.87 = 87 pennies
There are 100 pennies in 1 dollar.
So, $0.87 = 87/100 of a dollar.

Question 9.
$0.03
\(\frac{□}{□}\)

Answer:
3/100
Explanation:
grade 4 chapter 9 Relate Fractions and Decimals Image 2 517
$0.03 = 3 pennies
There are 100 pennies in 1 dollar.
So, $0.03 = 3/100.

Question 10.
$0.66
\(\frac{□}{□}\)

Answer:
66/100

Explanation:
grade 4 chapter 9 Relate Fractions and Decimals Image 3 517
$0.66 = 66 pennies
There are 100 pennies in 1 dollar.
So, $0.66 = 66/100.

Question 11.
$0.95
\(\frac{□}{□}\)

Answer:
95/100

Explanation:
grade 4 chapter 9 Relate Fractions and Decimals Image 4 517
$0.95 = 95 pennies
There are 100 pennies in 1 dollar.
So, $0.95 = 95/100.

Question 12.
$1.00
\(\frac{□}{□}\)

Answer:
100/100

Explanation:
grade 4 chapter 9 Relate Fractions and Decimals Image 5 517
$1.00 = 1 dollar
There are 100 pennies in 1 dollar.
So, $1.00 = 100/100.

Write the total money amount. Then write the amount as a fraction and as a decimal in terms of dollars.

Question 13.
2 quarters 2 dimes
Type below:
_________

Answer:
money amount: $0.70; fraction: 70/100; decimal: 0.70

Explanation:
Given that 2 quarters 2 dimes = (2 x 25/100) + (2 x 10/100) = 50/100 + 20/100 = 70/100

Question 14.
3 dimes 4 pennies
Type below:
_________

Answer:
money amount: $0.34; fraction: 34/100; decimal: 0.34

Explanation:
Given that 3 dimes 4 pennies = (3 x 10/100) + (4 x 1/100) = 30/100 + 4/100 = 34/100

Question 15.
8 nickels 12 pennies
Type below:
_________

Answer:
money amount: $0.57; fraction: 57/100; decimal: 0.57

Explanation:
Given that 8 nickels 12 pennies = (8 x 5/100) + (12 x 1/100) = 45/100 + 12/100 = 57/100

Problem Solving

Question 16.
Kate has 1 dime, 4 nickels, and 8 pennies. Write Kate’s total amount as a fraction in terms of a dollar.
\(\frac{□}{□}\)

Answer:
fraction: 38/100
Explanation:
Kate has 1 dime, 4 nickels, and 8 pennies.
10/100 + (4 x 5/100) + (8/100) = 10/100 + 20/100 + 8/100 = 38/100

Question 17.
Nolan says he has \(\frac{75}{100}\) of a dollar. If he only has 3 coins, what are the coins?
__________

Answer:
3 quarters

Explanation:
3 quarters = 25/100 + 25/100 + 25/100 = 75/100

Common Core – Relate Fractions and Decimals – Page No. 178

Lesson Check

Question 1.
Which of the following names the total money amount shown as a fraction in terms of a dollar?
Go Math Grade 4 Answer Key Homework Practice FL Chapter 9 Relate Fractions and Decimals Common Core - Relate Fractions and Decimals img 20
Options:
a. \(\frac{43}{1}\)
b. \(\frac{43}{10}\)
c. \(\frac{43}{57}\)
d. \(\frac{43}{100}\)

Answer:
d. 43/100

Explanation:
Given that 1 quarter + 1 nickel + 1 dime + 3 pennies = 25/100 + 5/100 + 10/100 + 3/100 = 43/100
Thus the correct answer is option d.

Question 2.
Crystal has \(\frac{81}{100}\) of a dollar. Which of the following could be the coins Crystal has?
Options:
a. 3 quarters, 1 dime, 1 penny
b. 2 quarters, 6 nickels, 1 penny
c. 2 quarters, 21 pennies
d. 1 quarter, 4 dimes, 1 nickel, 1 penny

Answer:
b. 2 quarters, 6 nickels, 1 penny

Explanation:
2 quarters, 6 nickels, 1 penny = (2 x 25/100) + (6 x 5/100) + 1/100 = 50/100 + 30/100 + 1/100 = 81/100
Thus the correct answer is option b.

Spiral Review

Question 3.
Joel gives \(\frac{1}{3}\) of his baseball cards to his sister. Which fraction is equivalent to \(\frac{1}{3}\)?
Options:
a. \(\frac{3}{5}\)
b. \(\frac{2}{6}\)
c. \(\frac{8}{9}\)
d. \(\frac{4}{10}\)

Answer:
b. 2/6
Explanation:
2/6 is divided by 2. The remaining answer after the dividion is 1/3.
Thus the correct answer is option b.

Question 4.
Penelope bakes pretzels. She salts \(\frac{3}{8}\) of the pretzels. Which fraction is equivalent to \(\frac{3}{8}\) ?
Options:
a. \(\frac{9}{24}\)
b. \(\frac{15}{20}\)
c. \(\frac{3}{16}\)
d. \(\frac{1}{5}\)

Answer:
a. 9/24
Explanation:
a. 9/24 is divided by 3. The remaining fraction after the division is 3/8.
Thus the correct answer is option a.

Question 5.
Which decimal is shown by the model?
Go Math Grade 4 Answer Key Homework Practice FL Chapter 9 Relate Fractions and Decimals Common Core - Relate Fractions and Decimals img 21
Options:
a. 10.0
b. 1.0
c. 0.1
d. 0.01

Answer:
d. 0.01

Explanation:
1 box is shaded out of 100. So, the fraction is 1/100 = 0.01.
Thus the correct answer is option d.

Question 6.
Mr. Guzman has 100 cows on his dairy farm. Of the cows, 57 are Holstein. What decimal represents the portion of cows that are Holstein?
Options:
a. 0.43
b. 0.57
c. 5.7
d. 57.0

Answer:
b. 0.57

Explanation:
Mr. Guzman has 100 cows on his dairy farm. Of the cows, 57 are Holstein. So, 57/100 Holstein cows are available.
57/100 = 0.57
Thus the correct answer is option b.

Common Core – Relate Fractions and Decimals – Page No. 179

Problem Solving Money

Use the act it out strategy to solve.

Question 1.
Carl wants to buy a bicycle bell that costs $4.50. Carl has saved $2.75 so far. How much more money does he need to buy the bell?
Use 4 $1 bills and 2 quarters to model $4.50. Remove bills and coins that have a value of $2.75. First, remove 2 $1 bills and 2 quarters.
Next, exchange one $1 bill for 4 quarters and remove 1 quarter.
Count the amount that is left. So, Carl needs to save $1.75 more.
Go Math Grade 4 Answer Key Homework Practice FL Chapter 9 Relate Fractions and Decimals Common Core - Relate Fractions and Decimals img 22

Answer:
Go Math Grade 4 Answer Key Chapter 9 Relate Fractions and Decimals Common Core - New img 42

Question 2.
Together, Xavier, Yolanda, and Zachary have $4.44. If each person has the same amount, how much money does each person have?
$ _________

Answer:
$1.11

Explanation:
Together, Xavier, Yolanda, and Zachary have $4.44. If each person has the same amount, $4.44/4 = $1.11

Question 3.
Marcus, Nan, and Olive each have $1.65 in their pockets. They decide to combine the money. How much money do they have altogether?
$ _________

Answer:
$4.95

Explanation:
Marcus, Nan, and Olive each have $1.65 in their pockets. They decide to combine the money. So, $1.65 + $1.65 + $1.65 = $4.95

Question 4.
Jessie saves $6 each week. In how many weeks will she have saved at least $50?
_________ weeks

Answer:
9 weeks

Explanation:
Jessie saves $6 each week. To save $50, $50/$6 = 9 weeks (approximately)

Question 5.
Becca has $12 more than Cece. Dave has $3 less than Cece. Cece has $10. How much money do they have altogether?
$ _________

Answer:
$39

Explanation:
Cece has $10.
Becca has $12 more than Cece = $10 + $12 = $22.
Dave has $3 less than Cece = $10 – $3 = $7.
All together = $10 + $22 + $7 = $39.

Common Core – Relate Fractions and Decimals – Page No. 180

Lesson Check

Question 1.
Four friends earned $5.20 for washing a car. They shared the money equally. How much did each friend get?
Options:
a. $1.05
b. $1.30
c. $1.60
d. $20.80

Answer:
b. $1.30

Explanation:
Four friends earned $5.20 for washing a car. They shared the money equally.
$5.20/4 = $1.30
Thus the correct answer is option b.

Question 2.
Which represents the value of one $1 bill and 5 quarters?
Options:
a. $1.05
b. $1.25
c. $1.50
d. $2.25

Answer:
d. $2.25

Explanation:
one $1 bill and 5 quarters. 5 quarters = 5 x 0.25 = 1.25.
$1 + $1.25 = $2.25
Thus the correct answer is option d.

Spiral Review

Question 3.
Bethany has 9 pennies. What fraction of a dollar is this?
Options:
a. \(\frac{9}{100}\)
b. \(\frac{9}{10}\)
c. \(\frac{90}{100}\)
d. \(\frac{99}{100}\)

Answer:
a. 9/100

Explanation:
1 dollar = 100 pennies.
So, 9 pennies = 9/100 of a dollar
Thus the correct answer is option a.

Question 4.
Michael made \(\frac{9}{12}\) of his free throws at practice. What is \(\frac{9}{12}\) in simplest form?
Options:
a. \(\frac{1}{4}\)
b. \(\frac{3}{9}\)
c. \(\frac{1}{2}\)
d. \(\frac{3}{4}\)

Answer:
d. 3/4

Explanation:
9/12 is divided by 3 that is equal to d. 3/4.
Thus the correct answer is option d.

Question 5.
I am a prime number between 30 and 40. Which number could I be?
Options:
a. 31
b. 33
c. 36
d. 39

Answer:
a. 31

Explanation:
31 has fractions 1 and 31.
Thus the correct answer is option a.

Question 6.
Georgette is using the benchmark \(\frac{1}{2}\) to compare fractions. Which statement is correct?
Options:
a. \(\frac{3}{8}>\frac{1}{2}\)
b. \(\frac{2}{5}<\frac{1}{2}\)
c. \(\frac{7}{12}<\frac{1}{2}\)
d. \(\frac{9}{10}=\frac{1}{2}\)

Answer:
b. 2/5<1/2

Explanation:
From the given details, 2/5<1/2 is the correct answer.
Thus the correct answer is option b.

Common Core – Relate Fractions and Decimals – Page No. 181

Add Fractional Parts of 10 and 100

Find the sum.

Question 1.
\(\frac{2}{10}+\frac{43}{100}\) Think: Write \(\frac{2}{10}\) as a fraction with a denominator of 100:
\(\frac{2 \times 10}{10 \times 10}=\frac{20}{100}\)
Go Math Grade 4 Answer Key Homework Practice FL Chapter 9 Relate Fractions and Decimals Common Core - Relate Fractions and Decimals img 23
Answer:
63/100

Explanation:
Think: Write 2/10 as a fraction with a denominator of 100:  2×10/10×10=20/100
Go Math Grade 4 Answer Key Chapter 9 Relate Fractions and Decimals Common Core - New img 49

Question 2.
\(\frac{17}{100}+\frac{6}{10}\)
\(\frac{□}{□}\)

Answer:
77/100

Explanation:
17/100+6/10.
6×10/10×10=60/100
17/100+60/100 = 77/100

Question 3.
\(\frac{9}{100}+\frac{4}{10}\)
\(\frac{□}{□}\)
Answer:
49/100
Explanation:
9/100+4/10.
4×10/10×10=40/100
9/100+40/100 = 49/100

Question 4.
\(\frac{7}{10}+\frac{23}{100}\)
\(\frac{□}{□}\)

Answer:
93/100

Explanation:
7/10+23/100.
7×10/10×10=70/100
70/100+23/100 = 93/100

Question 5.
$0.48 + $0.30
$ _____

Answer:
$0.78

Explanation:
Think $0.48 as 48/100.
Think $0.30 as 30/100.
48/100+30/100 = 78/100 = $0.78

Question 6.
$0.25 + $0.34
$ _____

Answer:
$0.59

Explanation:
Think $0.25 as 25/100.
Think $0.34 as 34/100.
25/100+34/100 = 59/100 = $0.59

Question 7.
$0.66 + $0.06
$ _____

Answer:
$0.72

Explanation:
Think $0.66 as 66/100.
Think $0.06 as 6/100.
66/100+6/100 = 72/100 = $0.72

Problem Solving

Question 8.
Ned’s frog jumped \(\frac{38}{100}\) meter. Then his frog jumped \(\frac{4}{10}\) meter. How far did Ned’s frog jump in all?
\(\frac{□}{□}\)

Answer:
78/100 meter

Explanation:
Ned’s frog jumped 38/100 meter. Then his frog jumped 4/10 meter.
So, together 38/100 + 4/10 jumped.
4/10 = 4×10/10×10=40/100.
38/100 + 40/100 = 78/100.

Question 9.
Keiko walks \(\frac{5}{10}\) kilometer from school to the park. Then she walks \(\frac{19}{100}\) kilometer from the park to her home. How far does Keiko walk in all?
\(\frac{□}{□}\)

Answer:
69/100 kilometer

Explanation:
Keiko walks 5/10 kilometer from school to the park. Then she walks 19/100 kilometer from the park to her home.
Total = 5/10 + 19/100 kilometer.
5/10 = 5×10/10×10=50/100.
50/100 + 19/100 = 69/100.

Common Core – Relate Fractions and Decimals – Page No. 182

Lesson Check

Question 1.
In a fish tank, \(\frac{2}{10}\) of the fish were orange and \(\frac{5}{100}\) of the fish were striped. What fraction of the fish were orange or striped?
Options:
a. \(\frac{7}{10}\)
b. \(\frac{52}{100}\)
c. \(\frac{25}{100}\)
d. \(\frac{7}{100}\)

Answer:
c. 25/100

Explanation:
In a fish tank, 2/10 of the fish were orange and 5/100 of the fish were striped.
To find the raction of the fish were orange or striped Add 2/10 and 5/100.
2/10 = 2×10/10×10=20/100.
20/100 + 5/100 = 25/100.
Thus the correct answer is option c.

Question 2.
Greg spends $0.45 on an eraser and $0.30 on a pen. How much money does Greg spend in all?
Options:
a. $3.45
b. $0.75
c. $0.48
d. $0.15

Answer:
b. $0.75

Explanation:
Think $0.45 as 45/100.
Think $0.30 as 30/100.
45/100+30/100 = 75/100 = $0.75.
Thus the correct answer is option b.

Spiral Review

Question 3.
Phillip saves $8 each month. How many months will it take him to save at least $60?
Options:
a. 6 months
b. 7 months
c. 8 months
d. 9 months

Answer:
c. 8 months

Explanation:
Phillip saves $8 each month.
To save at least $60, 60/8 = 8 months (approximately).
Thus the correct answer is option c.

Question 4.
Ursula and Yi share a submarine sandwich. Ursula eats \(\frac{2}{8}\) of the sandwich. Yi eats \(\frac{3}{8}\) of the sandwich. How much of the sandwich do the two friends eat?
Options:
a. \(\frac{1}{8}\)
b. \(\frac{4}{8}\)
c. \(\frac{5}{8}\)
d. \(\frac{6}{8}\)

Answer:
c. 5/8

Explanation:
Ursula and Yi share a submarine sandwich. Ursula eats 2/8 of the sandwich. Yi eats 3/8 of the sandwich.
Two friends eat 2/8 + 3/8 = 5/8
Thus the correct answer is option c.

Question 5.
A carpenter has a board that is 8 feet long. He cuts off two pieces. One piece is 3 \(\frac{1}{2}\) feet long and the other is 2 \(\frac{1}{3}\) feet long. How much of the board is left?
Options:
a. 2 \(\frac{1}{6}\) feet
b. 2 \(\frac{5}{6}\) feet
c. 3 \(\frac{1}{6}\) feet
d. 3 \(\frac{5}{6}\) feet

Answer:
a. 2 1/6
Explanation:
3 1/2 = 7/2.
2 1/3 = 7/3.
A carpenter has a board that is 8 feet long. He cuts off two pieces. One piece is 3 1/2 feet long and the other is 2 1/3 feet long.
7/2 + 7/3 = 7×3/2×3+\(7×2/3×2=[latex]2/16 + 14/6 = 35/6 = 5 5/6.
He left 8 – 55/6.
7 6/6 – 5 5/6 = 2 1/6
Thus the correct answer is option a.

Question 6.
Jeff drinks [latex]\frac{2}{3}\) of a glass of juice. Which fraction is equivalent to \(\frac{2}{3}\) ?
Options:
a. \(\frac{1}{3}\)
b. \(\frac{3}{2}\)
c. \(\frac{3}{6}\)
d. \(\frac{8}{12}\)

Answer:
d. 8/12
Explanation:
8/12 is divided by 4. So, 8/12 = 2/3.
Thus the correct answer is option d.

Common Core – Relate Fractions and Decimals – Page No. 183

Compare Decimals

Compare. Write <. >, or =.

Question 1.
Go Math Grade 4 Answer Key Homework Practice FL Chapter 9 Relate Fractions and Decimals Common Core - Relate Fractions and Decimals img 24
Think: 3 tenths is less than 5 tenths. So, 0.35 < 0.53

Answer:
0.35 < 0.53

Explanation:
3 tenths is less than 5 tenths. So, 0.35 < 0.53

Question 2.
0.6 ____ 0.60

Answer:
0.6 = 0.60

Explanation:
0.6 is 6 tenths can write as 6 tenths and 0 hundredths. So, 0.6 = 0.60.

Question 3.
0.24 ____ 0.31

Answer:
0.24 < 0.31

Explanation:
2 tenths is less than 3 tenths. So, 0.24 < 0.31.

Question 4.
0.94 ____ 0.9

Answer:
0.94 > 0.9

Explanation:
The digits of tenths are equal. So, compare hundredths. 4 hundredths is greater than 0 hundredths. So, 0.94 > 0.9.

Question 5.
0.3 ____ 0.32

Answer:
0.3 < 0.32

Explanation:
The digits of tenths are equal. So, compare hundredths. 0 hundredths is less than 2 hundredths. So, 0.3 < 0.32.

Question 6.
0.45 ____ 0.28

Answer:
0.45 > 0.28

Explanation:
4 tenths is greater than 2 tenths. So, 0.45 > 0.28.

Question 7.
0.39 ____ 0.93

Answer:
0.39 < 0.93

Explanation:
3 tenths is less than 9 tenths. So, 0.39 < 0.93.

Use the number line to compare. Write true or false.
Go Math Grade 4 Answer Key Homework Practice FL Chapter 9 Relate Fractions and Decimals Common Core - Relate Fractions and Decimals img 25

Question 8.
0.8 > 0.78
_____

Answer:
true

Explanation:
0.78 is in between 0.7 and 0.8 that is less than 0.8. So, 0.8 > 0.78.

Question 9.
0.4 > 0.84
_____

Answer:
false

Explanation:
0.4 is less than 0.84 and the left side of the number line. So, 0.4 < 0.84. The answer is false.

Question 10.
0.7 < 0.70
_____

Answer:
false

Explanation:
0.7 is 7 tenths and 70 hundredths. 0.7 = 0.70. So, the answer is false.

Question 11.
0.4 > 0.04
_____

Answer:
true

Explanation:
0.04 is less than 0.4 and it is left side of the 0.1 on the number line. 0.1 is less than 0.4. So, the given answer is true.

Compare. Write true or false.

Question 12.
0.09 > 0.1
_____

Answer:
false

Explanation:
0 tenths is less than 1 tenths. So, 0.09 < 0.1. So, the answer is false.

Question 13.
0.24 = 0.42
_____

Answer:
false

Explanation:
2 tenths is less than 4 tenths. So, 0.24 < 0.42. So, the answer is false.

Question 14.
0.17 < 0.32
_____

Answer:
true

Explanation:
1 tenth is less than 3 tenths. So, 0.17 < 0.32. So, the answer is true.

Question 15.
0.85 > 0.82
_____

Answer:
true

Explanation:
The digits of tenths are equal. So, compare hundredths. 5 hundredths is greater than 2 hundredths. So, 0.85 > 0.82.

Question 16.
Kelly walks 0.7 mile to school. Mary walks 0.49 mile to school. Write an inequality using <, > or = to compare the distances they walk to school.
0.7 _____ 0.49

Answer:
0.7 > 0.49

Explanation:
7 tenths is greater than 4 tenths. So, 0.7 > 0.49.

Question 17.
Tyrone shades two decimal grids. He shades 0.03 of the squares on one grid blue. He shades 0.3 of another grid red. Which grid has the greater part shaded?
0.03 _____ 0.3

Answer:
0.03 < 0.3

Explanation:
0.03 is 3 hundredths.
0.3 is 3 tenths, which is equal to 30 hundredths.
3 hundredths < 30 hundredths. So, 0.03 < 0.3.

Common Core – Relate Fractions and Decimals – Page No. 184

Lesson Check

Question 1.
Bob, Cal, and Pete each made a stack of baseball cards. Bob’s stack was 0.2 meter high. Cal’s stack was 0.24 meter high. Pete’s stack was 0.18 meter high.
Which statement is true?
Options:
a. 0.02 > 0.24
b. 0.24 > 0.18
c. 0.18 > 0.2
d. 0.24 = 0.2

Answer:
b. 0.24 > 0.18

Explanation:
2 tenths is greater than 1 tenth. So, 0.24 > 0.18.
Thus the correct answer is option b.

Question 2.
Three classmates spent money at the school supplies store. Mark spent 0.5 dollar, Andre spent 0.45 dollar, and Raquel spent 0.52 dollar. Which
statement is true?
Options:
a. 0.45 > 0.5
b. 0.52 < 0.45
c. 0.5 = 0.52
d. 0.45 < 0.5

Answer:
d. 0.45 < 0.5

Explanation:
4 tenths is less than 5 tenth. So, 0.45 > 0.5.
Thus the correct answer is option d.

Spiral Review

Question 3.
Pedro has $0.35 in his pocket. Alice has $0.40 in her pocket. How much money do Pedro and Alice have in their pockets altogether?
Options:
a. $0.05
b. $0.39
c. $0.75
d. $0.79

Answer:
c. $0.75

Explanation:
Pedro has $0.35 in his pocket. Alice has $0.40 in her pocket.
Together = $0.35 + $0.40 = $0.75.
Thus the correct answer is option c.

Question 4.
The measure 62 centimeters is equivalent to \(\frac{62}{100}\) meter. What is this measure written as a decimal?
Options:
a. 62.0 meters
b. 6.2 meters
c. 0.62 meter
d. 0.6 meter

Answer:
c. 0.62 meter

Explanation:
The decimal form of 62/100 = 0.62 meter.
Thus the correct answer is option c.

Question 5.
Joel has 24 sports trophies. Of the trophies, \(\frac{1}{8}\) are soccer trophies. How many soccer trophies does Joel have?
Options:
a. 2
b. 3
c. 4
d. 6

Answer:
b. 3

Explanation:
Joel has 24 sports trophies. Of the trophies, 18 are soccer trophies.
So, 18 × 24 = 3 soccer trophies.
Thus the correct answer is option b.

Question 6.
Molly’s jump rope is 6 \(\frac{1}{3}\)feet long. Gail’s jump rope is 4 \(\frac{2}{3}\)feet long. How much longer is Molly’s jump rope?
Options:
a. 1 \(\frac{1}{3}\) feet
b. 1 \(\frac{2}{3}\) feet
c. 2 \(\frac{1}{3}\) feet
d. 2 \(\frac{2}{3}\) feet

Answer:
b. 1 2/3 feet

Explanation:
6 1/3 feet = 193 feet.
4 2/3 feet = 143 feet.
19/3 – 14/3 = 5/3 feet = b. 1 2/3 feet.
Thus the correct answer is option b.

Common Core – Relate Fractions and Decimals – Page No. 185

Lessons 9.1 –9.2

Write the fraction or mixed number and the decimal shown by the model.

Question 1.
Go Math Grade 4 Answer Key Homework Practice FL Chapter 9 Relate Fractions and Decimals Common Core - Relate Fractions and Decimals img 26
Type below:
_________

Answer:
Go Math Grade 4 Answer Key Chapter 9 Relate Fractions and Decimals Common Core - New img 1

Question 2.
Go Math Grade 4 Answer Key Homework Practice FL Chapter 9 Relate Fractions and Decimals Common Core - Relate Fractions and Decimals img 27
Type below:
_________

Answer:
1 2/10

Explanation:
The model is divided into 10 equal parts. Each part represents one-tenth.
1 2/10 is 1 whole and 2 tenths.

Question 3.
Go Math Grade 4 Answer Key Homework Practice FL Chapter 9 Relate Fractions and Decimals Common Core - Relate Fractions and Decimals img 28
Type below:
_________

Answer:
2 3/10 = 2.3

Explanation:
grade 4 chapter 9 Common Core Image 1 499

Lesson 9.3

Write the number as hundredths in fraction form and decimal form.

Question 4.
\(\frac{8}{10}\)
Type below:
_________

Answer:
80/100
grade 4 chapter 9 Relate Fractions and Decimals Image 5 509
0.8

Explanation:
Write 8/10 as an equivalent fraction.
8/10 =8×10/10×10 = 80/100
8 tenths is the same as 8 tenths 0 hundredths. So the decimal form = 0.8

Question 5.
0.1
Type below:
_________

Answer:
50/100
grade 4 chapter 9 Relate Fractions and Decimals Image 7 509

0.50

Explanation:
Write 0.1 = 1/10 as an equivalent fraction.
1/10 =1×10/10×10 = 10/100
1 tenth is the same as 1 tenth 0 hundredths and also 0.1

Question 6.
\(\frac{3}{10}\)
Type below:
_________

Answer:
grade-4-chapter-9-Relate-Fractions-and-Decimals-Image-7-509

Write 0.1 = 1/10 as an equivalent fraction.
3/10 =3×10/10×10 = 30/100
3 tenth is the same as 3 tenth 0 hundredths and also 0.3

Write the number as tenths in fraction form and decimal form.

Question 7.
\(\frac{60}{100}\)
Type below:
_________

Answer:
6/10
grade 4 chapter 9 Relate Fractions and Decimals Image 8 509
0.6

Explanation:
10 is a common factor of the numerator and the denominator.
60/100 = 60÷10/100÷10 = 6/10
0.6
Thus the decimal form of the fraction \(\frac{60}{100}\) is 0.6

Question 8.
\(\frac{70}{100}\)
Type below:
_________

Answer:
grade-4-chapter-9-Relate-Fractions-and-Decimals-Image-7-509-2

Explanation:
10 is a common factor of the numerator and the denominator.
70/100 = 70÷10/100÷10 = 7/10
0.7
Thus the decimal form of the fraction \(\frac{70}{100}\) is 0.7

Question 9.
0.20
Type below:
_________

Answer: \(\frac{20}{100}\)

Explanation:
The fraction form of 0.20 is \(\frac{20}{100}\)

Lesson 9.4

Write as a money amount and as a decimal in terms of dollars.

Question 10.
\(\frac{30}{100}\)
Dollars: $ _____ Decimal: _____

Answer:
amount: $0.3 decimal: 0.3 of a dollar

Explanation:
30/100 = 0.3
Thus the decimal form of the fraction \(\frac{30}{100}\) is 0.3

Question 11.
\(\frac{91}{100}\)
Dollars: $ _____ Decimal: _____

Answer:
amount: $0.91 decimal: 0.91 of a dollar

Explanation:
91/100 = 0.91
Thus the decimal form of the fraction \(\frac{91}{100}\) is 0.91

Question 12.
\(\frac{5}{100}\)
Dollars: $ _____ Decimal: _____

Answer:
amount: $0.05 decimal: 0.05 of a dollar

Explanation:
5/100 = 0.05
Thus the decimal form of the fraction \(\frac{5}{100}\) is 0.05

Write the total money amount. Then write the amount as a fraction and as a decimal in terms of dollars.

Question 13.
4 dimes, 9 pennies

Answer:
money amount: $0.49; fraction: 49/100; decimal: 0.49

Explanation:
Given that 4 dimes 9 pennies = (4 x 10/100) + (9 x 1/100) = 40/100 + 9/100 = 49/100

Question 14.
3 quarters, 1 dime

Answer:
money amount: $0.85; fraction: 85/100; decimal: 0.85

Explanation:
Given that 3 quarters 1 dime = (3 x 25/100) + (1 x 10/100) = 75/100 + 10/100 = 85/100

Question 15.
7 nickels, 2 pennies

Answer:
money amount: $0.37; fraction: 37100; decimal: 0.37

Explanation:
Given that 7 nickels 2 pennies = (7 x 5/100) + (2 x 1/100) = 35/100 + 2/100 = 37/100

Common Core – Relate Fractions and Decimals – Page No. 186

Lesson 9.5

Question 1.
Camila, Jocelyn, and Audrey each earned $2.55. How much did the three girls earn altogether?
$ _____

Answer: $7.65

Explanation:
Given Camila, Jocelyn, and Audrey each earned $2.55
so Multiply $2.55 with 3 = 3 × $2.55
we get three girls to earn altogether is $7.65

Question 2.
Elijah, Xavier, and Adrian earned a total of $8.34. The boys shared the earnings equally. How much did each boy get?
$ _____

Answer: $2.78

Explanation:
Given Elijah, Xavier, and Adrian earned a total of $8.34
so divide the total of $8.34 by 3 = 8.34/3
then we get the boys shared the earnings equally is $2.78

Question 3.
Anthony saves $7 each week. In how many weeks will he have saved at least $40?
_____ weeks

Answer: 6 weeks

Explanation:
Given that,
Anthony saves $7 each week.
We have to find how many weeks will he have saved at least $40
$40/$7 = 6 (approx).
Thus it takes 6 weeks to save at least $40.

Question 4.
Brianna has $2 less than Victoria. Victoria has $11 more than Damian. Damian has $6. How much money do they have in all?
$ _____

Answer: $38

Explanation:
Given,
Brianna has $2 less than Victoria. Victoria has $11 more than Damian.
This means that Victoria has 11 more than Damian, and since Damian has 6, Victoria has 17. Plug this into the fact that Brianna has 2 less than Victoria, or 15, to get 6 + 17 + 15 = 38 dollars.

Lesson 9.6

Find the sum.

Question 5.
\(\frac{6}{10}+\frac{39}{100}\)
\(\frac{□}{□}\)

Answer:
99/100
Explanation:
6/10+39/100.
Write the addends as fractions with a common denominator
6/10 = 6X10/10X10 = 60/100.
60/100+39/100 = 99/100

Question 6.
\(\frac{14}{100}+\frac{8}{10}\)
\(\frac{□}{□}\)

Answer:
94/100
Explanation:
14/100+8/10.
Write the addends as fractions with a common denominator
8/10 = 8X10/10X10 = 80/100.
14/100+80/100 = 94/100.

Question 7.
\(\frac{4}{10}+\frac{18}{100}\)
\(\frac{□}{□}\)

Answer:
58/100

Explanation:
4/10+18/100.
Write the addends as fractions with a common denominator
4/10 = 4X10/10X10 = 40/100.
18/100+40/100 = 58/100

Question 8.
\(\frac{5}{10}+\frac{16}{100}\)
\(\frac{□}{□}\)

Answer:
58/100

Explanation:
5/10+16/100.
Write the addends as fractions with a common denominator
5/10 = 5X10/10X10 = 50/100.
16/100+50/100 = 66/100

Question 9.
$0.43 + $0.20
$ _____

Answer:
$0.63

Explanation:
Think 0.43 as 43 hundredths = 43/100.
Think 0.20 as 20 hundredths = 20/100.
Write the addends as fractions with a common denominator
43/100 + 20/100 = 63/100 = 0.63

Question 10.
$0.07 + $0.35
$ _____

Answer:
$0.42

Explanation:
Think 0.07 as 07 hundredths = 7/100.
Think 0.35 as 35 hundredths = 35/100.
Write the addends as fractions with a common denominator
7/100 + 35/100 = 42/100 = 0.42

Question 11.
$0.80 + $0.15 =
$ _____

Answer:
$0.95

Explanation:
Think 0.80 as 80 hundredths = 80/100.
Think 0.15 as 15 hundredths = 15/100.
Write the addends as fractions with a common denominator
80/100 + 15/100 = 95/100 = 0.95

Question 12.
$0.52 + $0.28
$ _____

Answer:
$0.80

Explanation:
Think 0.52 as 52 hundredths = 52/100.
Think 0.28 as 28 hundredths = 28/100.
Write the addends as fractions with a common denominator
52/100 + 28/100 = 80/100 = 0.80

Lesson 9.7

Compare. Write<, >, or =.

Question 13.
0.3 _____ 0.39

Answer:
0.3 < 0.39.

Explanation:
0.3 is 3 tenths, which is equivalent to 30 hundredths.
0.39 is 39 hundredths.
30 hundredths < 39 hundredths. So, 0.3 < 0.39.

Question 14.
0.9 _____ 0.90

Answer:
0.9 = 0.90

Explanation:
0.9 is 9 tenths, which is equivalent to 90 hundredths.
0.90 is 90 hundredths.
90 hundredths = 90 hundredths. So, 0.9 = 0.90.

Question 15.
0.54 _____ 0.45

Answer:
0.54 > 0.45

Explanation:
0.54 is 5.4 tenths, which is equivalent to 54 hundredths.
0.45 is 45 hundredths.
54 hundredths > 45 hundredths. So, 0.54 > 0.45.

Question 16.
0.04 _____ 0.06

Answer:
0.04 < 0.06

Explanation:
0.04 is 0.4 tenths, which is equivalent to 4 hundredths.
0.06 is 0.6 hundredths.
0.4 hundredths < 0.6 hundredths. So, 0.04 < 0.06

Question 17.
0.7 _____ 0.70

Answer:
0.7 = 0.70

Explanation:
0.7 is 7 tenths, which is equivalent to 70 hundredths.
0.70 is 70 hundredths.
70 hundredths = 70 hundredths. So, 0.7 = 0.70.

Question 18.
0.36 _____ 0.51

Answer:
0.36 < 0.51.

Explanation:
0.36 is 3.6 tenths, which is equivalent to 36 hundredths.
0.51 is 51 hundredths.
36 hundredths < 51 hundredths. So, 0.36 < 0.51.

Question 19.
0.8 _____ 0.67

Answer:
0.8 > 0.67.

Explanation:
0.8 is 8.0 tenths, which is equivalent to 80 hundredths.
0.67 is 67 hundredths.
80 hundredths > 67 hundredths. So, 0.80 > 0.67.

Question 20.
0.63 _____ 0.48

Answer:
0.63 > 0.48.

Explanation:
0.63 is 6.3 tenths, which is equivalent to 63 hundredths.
0.48 is 48 hundredths.
63 hundredths > 48 hundredths. So,0.63 > 0.48.

Compare. Write true or false.

Question 21.
0.32 > 0.23
_____

Answer:
True

Explanation:
0.32 is Greater than 0.23 and the left side of the number line. So, 0.32 < 0.23. The answer is True.

Question 22.
0.86 = 0.9
_____

Answer:
false

Explanation:
86 tenths is less than 90 tenths. So, 0.86 < 0.9. So, the answer is false.

Question 23.
0.68 < 0.83
_____

Answer:
true

Explanation:
6 tenths is less than 8 tenths. So, 0.68 < 0.83. So, the answer is true.

Question 24.
0.97 > 0.94
_____

Answer: true

Explanation:
The digits of tenths are equal. So, compare hundredths. 7 hundredths is greater than 4 hundredths.
So, the answer is 0.97 > 0.94.

Conclusion:

We hope this best resource, Go Math Grade 4 Answer Key Chapter 9 Relate Fractions and Decimals can help you learn the mathematical concepts completely at your fingertips. Make the most out of these Go math 4th grade solutions and grasp the difficult questions & concepts easily and become pro in it. All The Best!!

Go Math Grade 4 Answer Key Homework Practice FL Chapter 10 Two-Dimensional Figures

go-math-grade-4-chapter-10-two-dimensional-figures-pages-187-204-answer-key

Browse & Download Go Math Grade 4 Answer Key Homework Practice FL Chapter 10 Two-Dimensional Figures pdf from here & start your preparation. Students of 4th grade can easily grasp the Two-dimensional figures notations by using the CCSSMathAnswers provided HMH Go Math Answer Key Grade 4 Homework Practice FL 10 Two-Dimensional Figures pdf. Also, this guide is used as homework help and let the students complete the homework or any assessments in no time.

Go Math Grade 4 Answer Key Homework Practice FL Chapter 10 Two-Dimensional Figures

Download Chapter 10 Go Math HMH Grade 4 Solution Key and practice all Two-Dimensional Figures covered concepts like Lines, Rays and Angles, Classify Triangles, Parallel Lines and Perpendicular Lines, Classify Quadrilaterals, Line Symmetry, etc in an easy manner. Simply click on the provided pdf links and ace up your preparation. Also, find some more questions from Go Math Grade 4 Answer Key Chapter 10 Two-Dimensional Figures to score maximum marks in the exams.

Lesson: 1 – Lines, Rays, and Angles

Lesson: 2 – Classify Triangles

Lesson: 3 – Parallel Lines and Perpendicular Lines

Lesson: 4 – Classify Quadrilaterals

Lesson: 5 – Line Symmetry

Lesson: 6 – Find and Draw Lines of Symmetry

Lesson: 7 – Problem Solving Shape Patterns

Chapter 10 – Extra Practice

Common Core – Two-Dimensional Figures – Page No. 189

Lines, Rays, and Angles

Draw and label an example of the figure.

Question 1.
obtuse ∠ABC
Go Math Grade 4 Answer Key Homework Practice FL Chapter 10 Two-Dimensional Figures Common Core - Two-Dimensional Figures img 1
Think: An obtuse angle is greater than a right angle. The middle letter, B, names the vertex of the angle.
________

Answer:
grade 4 chapter 10 Lines, Rays, and Angles image 1 553

Explanation:
An obtuse angle is greater than a right angle. The middle letter, B, names the vertex of the angle.

Question 2.
\(\overrightarrow{G H}\)

Answer:
grade 4 chapter 10 Lines, Rays, and Angles image 2 553

Explanation:
GH is a ray that has one endpoint and continues without an end in one direction.

Question 3.
acute ∠JKL
________

Answer:
grade 4 chapter 10 Lines, Rays, and Angles image 3 553

Explanation:
Angle JKL is an acute angle that is less than a right angle.

Question 4.
\(\overline{B C}\)

Answer:
grade 4 chapter 10 Lines, Rays, and Angles image 4 553

Explanation:
BC is a line that continues without an end in both directions.

Use the figure for 5–8.
Go Math Grade 4 Answer Key Homework Practice FL Chapter 10 Two-Dimensional Figures Common Core - Two-Dimensional Figures img 2

Question 5.
Name a line segment.

Answer:
line segment EF

Explanation:
EF line is a straight path of points that continues without an end in both directions.

Question 6.
Name a right angle.
∠ _____

Answer:
∠EJF

Explanation:
EJF is a right angle that forms a square corner.

Question 7.
Name an obtuse angle.
obtuse ∠ _____

Answer:
∠CEJ

Explanation:
CEJ is an obtuse angle that is greater than a right angle.

Problem Solving

Use the figure at the right for 9–11.
Go Math Grade 4 Answer Key Homework Practice FL Chapter 10 Two-Dimensional Figures Common Core - Two-Dimensional Figures img 3

Question 9.
Classify ∠AFD
________

Answer:
Obtuse Angle

Explanation:
AFD is an obtuse angle that is greater than a right angle.

Question 10.
Classify ∠CFE.
________

Answer:
Right Angle

Explanation:
∠CFE is a right angle that forms a square corner.

Question 11.
Name two acute angles.
acute ∠ _____ acute ∠ _____

Answer:
∠AFB and ∠DFE

Explanation:
∠AFB and ∠DFE are two acute angles with less than a right angle.

Common Core – Two-Dimensional Figures – Page No. 190

Lesson Check

Question 1.
The hands of a clock show the time 12:25.
Go Math Grade 4 Answer Key Homework Practice FL Chapter 10 Two-Dimensional Figures Common Core - Two-Dimensional Figures img 4
Which best describes the angle between the hands of the clock?
Options:
a. acute
b. right
c. obtuse
d. straight

Answer:
c. obtuse

Explanation:
The hands of the time 12:25 are forming greater than a right angle. So, the answer is the Obtuse angle.
Thus the correct answer is option c.

Question 2.
Which of the following name two different figures?
Options:
a. \(\overline{A B} \text { and } \overline{B A}\)
b. \(\overleftrightarrow { AB } \) and \(\overleftrightarrow { BA } \)
c. \(\overrightarrow { AB } \) and \(\overrightarrow { BA } \)
d. ∠ABC and ∠CBA

Answer: \(\overrightarrow { AB } \) and \(\overrightarrow { BA } \)

Explanation:
In \(\overrightarrow { AB } \) A is an end point and B continues without end in one direction.
In \(\overrightarrow { BA } \) B is an end point and A continues without an end in one direction.
Thus the correct answer is option c.

Spiral Review

Question 3.
Jan’s pencil is 8.5 cm long. Ted’s pencil is longer. Which could be the length of Ted’s pencil?
Options:
a. 0.09 cm
b. 0.8 cm
c. 8.4 cm
d. 9.0 cm

Answer:
d. 9.0 cm

Explanation:
9 ones is greater than 8 ones. So, 9.0 cm > 8.5 cm
Thus the correct answer is option d.

Question 4.
Kayla buys a shirt for $8.19. She pays with a $10 bill. How much change should she receive?
Options:
a. $1.81
b. $1.89
c. $2.19
d. $2.81

Answer:
a. $1.81

Explanation:
Kayla buys a shirt for $8.19. She pays with a $10 bill. To find the change she received, $10 – $8.19 = 1.81
Thus the correct answer is option a.

Question 5.
Sasha donated \(\frac{9}{100}\) of her class’s entire can collection for the food drive. Which decimal is equivalent to \(\frac{9}{100}\) ?
Options:
a. 9
b. 0.99
c. 0.9
d. 0.09

Answer:
d. 0.09

Explanation:
\(\frac{9}{100}\) is 9 hundredths. So, the decimal is 0.09.
Thus the correct answer is option d.

Question 6.
Jose jumped 8 \(\frac{1}{3}\) feet. This was 2 \(\frac{2}{3}\) feet farther than Lila jumped. How far did Lila jump?
Options:
a. 5 \(\frac{1}{3}\)
b. 5 \(\frac{2}{3}\)
c. 6 \(\frac{1}{3}\)
d. 11

Answer: 5 \(\frac{2}{3}\)

Explanation:
Jose jumped 8 \(\frac{1}{3}\) feet. This was 2 \(\frac{2}{3}\) feet farther than Lila jumped.
8 \(\frac{1}{3}\) feet – 2 \(\frac{2}{3}\)
= 25/3 – 8/3
= 7/3
= 5 \(\frac{2}{3}\)
Thus the correct answer is option b.

Common Core – Two-Dimensional Figures – Page No. 191

Classify Triangles

Classify each triangle. Write acute, right, or obtuse.

Question 1.
Go Math Grade 4 Answer Key Homework Practice FL Chapter 10 Two-Dimensional Figures Common Core - Two-Dimensional Figures img 5
Think: Angles A and C are both acute.
Angle B is obtuse.

Answer:
Obtuse triangle;
Angle A and Angle C are both acute.
Angle B is obtuse.

Explanation:
From triangle, ABC, Angle A, and Angle C are both acute with less than a right angle. Angle B is an obtuse angle that is greater than a right angle.

Question 2.
Go Math Grade 4 Answer Key Homework Practice FL Chapter 10 Two-Dimensional Figures Common Core - Two-Dimensional Figures img 6
________

Answer:
Right Triangle; Triangle DEF;
∠D and ∠F are acute angles.
∠E is Right angle

Explanation:
∠D and ∠F are acute angles with less than a right angle. ∠E is the Right angle that forms a square corner. A triangle that has one right angle is called a right triangle.

Question 3.
Go Math Grade 4 Answer Key Homework Practice FL Chapter 10 Two-Dimensional Figures Common Core - Two-Dimensional Figures img 7
________

Answer:
Acute triangle;
Angle G, Angle J, and Angle H are acute angles.

Explanation:
From triangle GJH, Angle G, Angle J, and Angle H are acute angles with less than a right angle. A triangle with three acute angles called an acute triangle. So, the given triangle is an acute triangle.

Question 4.
Go Math Grade 4 Answer Key Homework Practice FL Chapter 10 Two-Dimensional Figures Common Core - Two-Dimensional Figures img 8
________

Answer:
Obtuse triangle;
Angle L and Angle N are both acute.
Angle M is obtuse.

Explanation:
From triangle LMN, Angle L and Angle N are both acute with less than a right angle. Angle M is an obtuse angle that is greater than a right angle. A triangle with an obtuse angle is called an obtuse triangle.

Problem Solving

Question 5.
Use figure ABCD below. Draw a line segment from point B to point D. Name and classify the triangles formed.
Go Math Grade 4 Answer Key Homework Practice FL Chapter 10 Two-Dimensional Figures Common Core - Two-Dimensional Figures img 9
Two _______ triangles
△ _______
△ _______

Answer:

grade 4 chapter 10 Lines, Rays, and Angles image 1 559

Two Acute triangles.
△ ABD
△ BCD

Explanation:
If we draw a line segment from point B to point D, then there are two traingles formed with less than right angles. They are △ ABD and △ BCD.

Question 6.
Use figure ABCD below. Draw a line segment from point A to point C. Name and classify the triangles formed.
Go Math Grade 4 Answer Key Homework Practice FL Chapter 10 Two-Dimensional Figures Common Core - Two-Dimensional Figures img 10
Two _______ triangles
△ _______
△ _______

Answer:
grade 4 chapter 10 Lines, Rays, and Angles image 2 559

Two Acute triangles.
△ ABC
△ ADC

Explanation:
If we draw a line segment from point A to point C, then there are two triangles formed with less than right angles. They are △ ABC and △ ADC.

Common Core – Two-Dimensional Figures – Page No. 192

Lesson Check

Question 1.
Stephen drew this triangle. How many obtuse angles does the triangle have?
Go Math Grade 4 Answer Key Homework Practice FL Chapter 10 Two-Dimensional Figures Common Core - Two-Dimensional Figures img 11
Options:
a. 0
b. 1
c. 2
d. 3

Answer: a. 0

Explanation:
The given image has three acute angles. So, there are 0 obtuse angles.
Thus the correct answer is option a.

Question 2.
Joan was asked to draw a right triangle. How many right angles are in a right triangle?
Options:
a. 0
b. 1
c. 2
d. 3

Answer:
b. 1

Explanation:
A right triangle has only one right angle.
Thus the correct answer is option b.

Spiral Review

Question 3.
Oliver drew the figure below to show light traveling from the sun to Earth. Name the figure he drew.
Go Math Grade 4 Answer Key Homework Practice FL Chapter 10 Two-Dimensional Figures Common Core - Two-Dimensional Figures img 12
Options:
a. segment SE
b. ray SE
c. line SE
d. ray ES

Answer:
b. ray SE

Explanation:
SE is a ray that has one endpoint and continues without an end in one direction.
Thus the correct answer is option b.

Question 4.
Armon added \(\frac{1}{10}\) and \(\frac{8}{100}\). Which is the correct sum?
Options:
a. \(\frac{18}{10}\)
b. \(\frac{9}{10}\)
c. \(\frac{9}{100}\)
d. \(\frac{18}{100}\)

Answer: d. \(\frac{18}{100}\)

Explanation:
\(\frac{10}{100}\) + \(\frac{8}{100}\) = \(\frac{18}{100}\)
Thus the correct answer is option d.

Question 5.
Sam counted out loud by 6s. Jorge counted out loud by 8s. What are the first three numbers both students said?
Options:
a. 8, 16, 24
b. 14, 28, 42
c. 24, 48, 72
d. 48, 96, 144

Answer:
c. 24, 48, 72

Explanation:
Sam counted out loud by 6s = 6, 12, 18, 24, 30, 36, 42, 48, 54, 60, 66, 72.
Jorge counted out loud by 8s = 8, 16, 24, 32, 40, 48, 56, 64, 72, 80.
Both students said the first three numbers are 24, 48, 72.
Thus the correct answer is option c.

Question 6.
A basketball team averaged 105 points per game. How many points did the team score in 6 games?
Options:
a. 605 points
b. 630 points
c. 900 points
d. 6,030 points

Answer:
b. 630 points

Explanation:
A basketball team averaged 105 points per game.
They score in 6 games = 6 x 105 = 630 points.
Thus the correct answer is option b.

Common Core – Two-Dimensional Figures – Page No. 193

Parallel Lines and Perpendicular Lines

Use the figure for 1–3.
Go Math Grade 4 Answer Key Homework Practice FL Chapter 10 Two-Dimensional Figures Common Core - Two-Dimensional Figures img 13

Question 1.
Name a pair of lines that appear to be perpendicular.
Think: Perpendicular lines form right angles.
\(\overleftrightarrow { AB } \) and \(\overleftrightarrow { EF } \) appear to form right angles.
\(\overleftrightarrow { AB } \) and \(\overleftrightarrow { EF } \)

Answer:
\(\overleftrightarrow { AB } \) and \(\overleftrightarrow { EF } \)

Explanation:
Perpendicular lines form right angles.
\(\overleftrightarrow { AB } \) and \(\overleftrightarrow { EF } \) appear to form right angles.
\(\overleftrightarrow { AB } \) and \(\overleftrightarrow { EF } \)

Question 2.
Name a pair of lines that appear to be parallel.
____ and ____

Answer:
\(\overleftrightarrow { AB } \) and \(\overleftrightarrow { CD } \)

Explanation:
Parallel lines never interest each other. \(\overleftrightarrow { AB } \) and \(\overleftrightarrow { CD } \) are parallel lines.

Question 3.
Name another pair of lines that appear to be perpendicular.
____ and ____

Answer:
\(\overleftrightarrow { CD } \) and \(\overleftrightarrow { EF } \)

Explanation:
Perpendicular lines form right angles.
\(\overleftrightarrow { CD } \) and \(\overleftrightarrow { EF } \) appear to form right angles.
\(\overleftrightarrow { CD } \) and \(\overleftrightarrow { EF } \)

Draw and label the figure described.

Question 4.
\(\overleftrightarrow { MN } \) and \(\overleftrightarrow { PQ } \) intersecting at point R

Answer:
grade 4 chapter 10 Lines, Rays, and Angles image 1 565

Explanation:
MN and PQ are two lines and interesting at point R.

Question 5.
\(\overleftrightarrow { WX } \) || \(\overleftrightarrow { YZ } \)

Answer:
grade 4 chapter 10 Lines, Rays, and Angles image 2 565

Explanation:
WX and YZ are parallel lines and they never intersect with each other.

Question 6.
\(\overleftrightarrow { FH } \) ⊥ \(\overleftrightarrow { JK } \)

Answer:
grade 4 chapter 10 Lines, Rays, and Angles image 3 565

Explanation:
FH and JK are two lines and intersecting each other to form four right angles.

Problem Solving

Use the street map for 7–8.
Go Math Grade 4 Answer Key Homework Practice FL Chapter 10 Two-Dimensional Figures Common Core - Two-Dimensional Figures img 14

Question 7.
Name two streets that intersect but do not appear to be perpendicular.
Type below:
_______

Answer:
Maple and Oak or Oak and Birch

Explanation:
Maple and Oak or Oak and Birch; They are intersecting with each other and not perpendicular.

Question 8.
Name two streets that appear to be parallel to each other.
Type below:
_______

Answer:
Maple and Birch

Explanation:
Maple and Birch are streets and not intersect with each other. They appear to be parallel to each other.

Common Core – Two-Dimensional Figures – Page No. 194

Lesson Check

Question 1.
Which capital letter appears to have perpendicular line segments?
Options:
a. N
b. O
c. T
d. V

Answer:
c. T

Explanation:
T has two lines and interesting to form four right angles.

Question 2.
In the figure, which pair of line segments appear to be parallel?
Go Math Grade 4 Answer Key Homework Practice FL Chapter 10 Two-Dimensional Figures Common Core - Two-Dimensional Figures img 15
Options:
a. \(\overline{F G} \text { and } \overline{G H}\)
b. \(\overline{F J} \text { and } \overline{G H}\)
c. \(\overline{F G} \text { and } \overline{J H}\)
d. \(\overline{J H} \text { and } \overline{F J}\)

Answer:
c. \(\overline{F G} \text { and } \overline{J H}\)

Explanation:
\(\overline{F G} \text { and } \overline{J H}\) are parallel lines that never intersect

Spiral Review

Question 3.
Nolan drew a right triangle. How many acute angles did he draw?
Options:
a. 0
b. 1
c. 2
d. 3

Answer:
c. 2

Explanation:
A triangle with one right angle will have two acute angles.

Question 4.
Mike drank more than half the juice in his glass. What fraction of the juice could Mike have drunk?
Options:
a. \(\frac{1}{3}\)
b. \(\frac{2}{5}\)
c. \(\frac{3}{6}\)
d. \(\frac{5}{8}\)

Answer:
d. \(\frac{5}{8}\)

Explanation:
Mike drank more than half the juice in his glass. He drunk \(\frac{5}{8}\) of the juice.

Question 5.
A school principal ordered 1,000 pencils. He gave an equal number to each of 7 teachers until he had given out as many as possible. How many pencils were left?
Options:
a. 2
b. 4
c. 6
d. 142

Answer:
c. 6

Explanation:
A school principal ordered 1,000 pencils. He gave an equal number to each of the 7 teachers until he had given out as many as possible. He shared 142 pencils for each of the 7 teachers. So, 142 × 7 = 994. The remaining pencils are 6.

Question 6.
A carton of juice contains 64 ounces. Ms. Wilson bought 6 cartons of juice. How many ounces of juice did she buy?
Options:
a. 364 ounces
b. 370 ounces
c. 384 ounces
d. 402 ounces

Answer:
c. 384 ounces

Explanation:
A carton of juice contains 64 ounces. Ms. Wilson bought 6 cartons of juice. 64 X 6 = 384 ounces juice she can buy.

Common Core – Two-Dimensional Figures – Page No. 195

Classify Quadrilaterals

Classify each figure as many ways as possible. Write quadrilateral, trapezoid, parallelogram, rhombus, rectangle, or square.

Question 1.
Go Math Grade 4 Answer Key Homework Practice FL Chapter 10 Two-Dimensional Figures Common Core - Two-Dimensional Figures img 16
Type below:
________
Answer:
Quadrilateral, Parallelogram, and rhombus.

Explanation:
2 pairs of parallel sides
4 sides of equal length
0 right angles
Quadrilateral, Parallelogram, and rhombus.

Question 2.
Go Math Grade 4 Answer Key Homework Practice FL Chapter 10 Two-Dimensional Figures Common Core - Two-Dimensional Figures img 17
Type below:
________

Answer:
Quadrilateral, Parallelogram, Rectangle

Explanation:
2 pairs of parallel sides
2 pairs of sides of equal length
4 right angles
Quadrilateral, Parallelogram, Rectangle

Question 3.
Go Math Grade 4 Answer Key Homework Practice FL Chapter 10 Two-Dimensional Figures Common Core - Two-Dimensional Figures img 18
Type below:
________

Answer:

Explanation:
1 pair of parallel sides
2 sides of equal length
0 right angles
Quadrilateral, Trapezoid

Question 4.
Go Math Grade 4 Answer Key Homework Practice FL Chapter 10 Two-Dimensional Figures Common Core - Two-Dimensional Figures img 19
Type below:
________

Answer:
Quadrilateral

Explanation:
0 pair of parallel sides
0 sides of equal length
0 right angles
Quadrilateral

Question 5.
Go Math Grade 4 Answer Key Homework Practice FL Chapter 10 Two-Dimensional Figures Common Core - Two-Dimensional Figures img 20
Type below:
________

Answer:
Quadrilateral, Parallelogram, and rhombus

Explanation:
2 pairs of parallel sides
4 sides of equal length
0 right angles
Quadrilateral, Parallelogram, and rhombus

Question 6.
Go Math Grade 4 Answer Key Homework Practice FL Chapter 10 Two-Dimensional Figures Common Core - Two-Dimensional Figures img 21
Type below:
________

Answer:

Explanation:
1 pair of parallel sides
0 sides of equal length
2 right angles
Quadrilateral, Trapezoid

Question 7.
Go Math Grade 4 Answer Key Homework Practice FL Chapter 10 Two-Dimensional Figures Common Core - Two-Dimensional Figures img 22
Type below:
________

Answer:

Explanation:
2 pairs of parallel sides
2 pairs of sides of equal length
0 right angles
Quadrilateral, Parallelogram

Problem Solving

Question 8.
Alan drew a polygon with four sides and four angles. All four sides are equal. None of the angles are right angles. What figure did Alan draw?
________

Answer:
Quadrilateral or rhombus

Explanation:
Alan drew a polygon with four sides and four angles. All four sides are equal. None of the angles are right angles. Alan drew Quadrilateral or rhombus

Question 9.
Teresa drew a quadrilateral with 2 pairs of parallel sides and 4 right angles. What quadrilateral could she have drawn?
________

Answer:
square or rectangle

Explanation:
2 pairs of parallel sides and 4 right angles. she could draw a square or rectangle.

Common Core – Two-Dimensional Figures – Page No. 196

Lesson Check

Question 1.
Joey is asked to name a quadrilateral that is also a rhombus. What should be his answer?
Options:
a. square
b. rectangle
c. parallelogram
d. trapezoid

Answer:
a. square

Explanation:
The quadrilateral square is also called a rhombus. Both square and rhombus have 2 pairs of parallel sides and 4 sides of equal length.
Thus the correct answer is option a.

Question 2.
Which quadrilateral has exactly one pair of parallel sides?
Options:
a. square
b. rhombus
c. parallelogram
d. trapezoid

Answer:
d. trapezoid

Explanation:
A trapezoid has exactly one pair of parallel sides.
Thus the correct answer is option d.

Spiral Review

Question 3.
Terrence has 24 eggs to divide into equal groups. What are all the possible numbers of eggs that Terence could put in each group?
Options:
a. 1, 2, 3, 4
b. 2, 4, 6, 8, 12
c. 1, 2, 3, 4, 6, 8, 12, 24
d. 24, 48, 72, 96

Answer:
c. 1, 2, 3, 4, 6, 8, 12, 24

Explanation:
Terrence has 24 eggs to divide into equal groups. Terence could put in each group in 1, 2, 3, 4, 6, 8, 12, 24 ways.
Thus the correct answer is option c.

Question 4.
In a line of students, Jenna is number 8. The teacher says that a rule for a number pattern is add 4. The first student in line says the first term, 7. What number
should Jenna say?
Options:
a. 31
b. 35
c. 39
d. 43

Answer:
b. 35

Explanation:
In a line of students, Jenna is number 8. The teacher says that a rule for a number pattern is to add 4. The first student in line says the first term, 7.
7 + 4 = 11
11 + 4 = 15
15 + 4 = 19
19 + 4 = 23
23 + 4 = 27
27 + 4 = 31
31 + 4 = 35.
Jenna says 35.
Thus the correct answer is option b.

Question 5.
Lou eats \(\frac{6}{8}\) of a pizza. What fraction of the pizza is left over?
Options:
a. \(\frac{1}{8}\)
b. \(\frac{1}{4}\)
c. \(\frac{1}{2}\)
d. \(\frac{3}{4}\)

Answer:
b. 1/4
Explanation:
Lou eats 6/8 of a pizza. So, 6 parts of the pizza are finished and the remaining 2 parts of the pizza have remained. So, the left over pizza is 2/8 = 1/4.
Thus the correct answer is option b.

Question 6.
Which capital letter appears to have parallel lines?
Options:
a. D
b. L
c. N
d. T

Answer:
c. N

Explanation:
N has two parallel lines and never intersect each other.
Thus the correct answer is option c.

Common Core – Two-Dimensional Figures – Page No. 197

Line Symmetry

Tell if the dashed line appears to be a line of symmetry. Write yes or no.

Question 1.
Go Math Grade 4 Answer Key Homework Practice FL Chapter 10 Two-Dimensional Figures Common Core - Two-Dimensional Figures img 23
yes

Answer:
Yes

Explanation:
The line of symmetry divides a shape into two parts that are the same size and shape.

Question 2.
Go Math Grade 4 Answer Key Homework Practice FL Chapter 10 Two-Dimensional Figures Common Core - Two-Dimensional Figures img 24
____

Answer:
No

Explanation:
The line of symmetry divides a shape into two parts that are not of the same size and shape.

Question 3.
Go Math Grade 4 Answer Key Homework Practice FL Chapter 10 Two-Dimensional Figures Common Core - Two-Dimensional Figures img 25
____

Answer:
Yes

Explanation:
The line of symmetry divides a shape into two parts that are the same size and shape.

Question 4.
Go Math Grade 4 Answer Key Homework Practice FL Chapter 10 Two-Dimensional Figures Common Core - Two-Dimensional Figures img 26
____

Answer:
No

Explanation:
The line of symmetry divides a shape into two parts that are not of the same size and shape.

Question 5.
Go Math Grade 4 Answer Key Homework Practice FL Chapter 10 Two-Dimensional Figures Common Core - Two-Dimensional Figures img 27
____

Answer:
No

Explanation:
The line of symmetry divides a shape into two parts that are not of the same size and shape.

Question 6.
Go Math Grade 4 Answer Key Homework Practice FL Chapter 10 Two-Dimensional Figures Common Core - Two-Dimensional Figures img 28
____

Answer:
Yes

Explanation:
The line of symmetry divides a shape into two parts that are the same size and shape.

Question 7.
Go Math Grade 4 Answer Key Homework Practice FL Chapter 10 Two-Dimensional Figures Common Core - Two-Dimensional Figures img 29
____

Answer:
No

Explanation:
The line of symmetry divides a shape into two parts that are not of the same size and shape.

Question 8.
Go Math Grade 4 Answer Key Homework Practice FL Chapter 10 Two-Dimensional Figures Common Core - Two-Dimensional Figures img 30
____

Answer:
Yes

Explanation:
The line of symmetry divides a shape into two parts that are the same size and shape.

Complete the design by reflecting over the line of symmetry.

Question 9.
Go Math Grade 4 Answer Key Homework Practice FL Chapter 10 Two-Dimensional Figures Common Core - Two-Dimensional Figures img 31

Answer:

grade 4 chapter 10 Lines, Rays, and Angles image 4 578

Question 10.
Go Math Grade 4 Answer Key Homework Practice FL Chapter 10 Two-Dimensional Figures Common Core - Two-Dimensional Figures img 32

Answer:

grade 4 chapter 10 Lines, Rays, and Angles image 6 578

Problem Solving

Question 11.
Kara uses the pattern below to make paper dolls. The dashed line represents a line of symmetry. A complete doll includes the reflection of the pattern over the line of symmetry. Complete the design to show what one of Kara’s paper dolls looks like.
Go Math Grade 4 Answer Key Homework Practice FL Chapter 10 Two-Dimensional Figures Common Core - Two-Dimensional Figures img 33

Answer:

grade 4 chapter 10 Lines, Rays, and Angles image 7 578

Common Core – Two-Dimensional Figures – Page No. 198

Lesson Check

Question 1.
Which best describes the line of symmetry in the letter D?
Go Math Grade 4 Answer Key Homework Practice FL Chapter 10 Two-Dimensional Figures Common Core - Two-Dimensional Figures img 34
Options:
a. horizontal
b. vertical
c. diagonal
d. half turn

Answer:
a. horizontal

Explanation:
The horizontal line of symmetry in the letter D can exactly separate two parts equally.

Question 2.
Which shape has a correctly drawn line of symmetry?
Options:
a. Go Math Grade 4 Answer Key Homework Practice FL Chapter 10 Two-Dimensional Figures Common Core - Two-Dimensional Figures img 35
b. Go Math Grade 4 Answer Key Homework Practice FL Chapter 10 Two-Dimensional Figures Common Core - Two-Dimensional Figures img 36
c. Go Math Grade 4 Answer Key Homework Practice FL Chapter 10 Two-Dimensional Figures Common Core - Two-Dimensional Figures img 37
d. Go Math Grade 4 Answer Key Homework Practice FL Chapter 10 Two-Dimensional Figures Common Core - Two-Dimensional Figures img 38

Answer:
b.Go Math Grade 4 Answer Key Chapter 10 Two-Dimensional Figures Common Core - New img 86

Spiral Review

Question 3.
The class has 360 unit cubes in a bag. Johnnie divides the unit cubes equally among 8 groups. How many unit cubes will each group get?
Options:
a. 40
b. 44
c. 45
d. 48

Answer:
c. 45

Explanation:
The class has 360 unit cubes in a bag. Johnnie divides the unit cubes equally among 8 groups. 360/8= 45.

Question 4.
There are 5,280 feet in one mile. How many feet are there in 6 miles?
Options:
a. 30,680
b. 31,260
c. 31,608
d. 31,680

Answer:
d. 31,680

Explanation:
There are 5,280 feet in one mile. So, for 6 miles = 6 x 5, 280 = 31,680.

Question 5.
Sue has 4 pieces of wood. The lengths of her pieces of wood are \(\frac{1}{3}\) foot, \(\frac{2}{5}\) foot, \(\frac{3}{10}\) foot, and \(\frac{1}{4}\) foot. Which piece of wood is the shortest?
Options:
a. the \(\frac{1}{3}\) foot piece
b. the \(\frac{2}{5}\) foot piece
c. the \(\frac{3}{10}\) foot piece
d. the \(\frac{1}{4}\) foot piece

Answer:
d. the 1/4 foot piece

Explanation:
The lengths of 1/4 foot piece is less compared to other lengths.

Question 6.
Alice has \(\frac{1}{5}\) as many miniature cars as Sylvester has. Sylvester has 35 miniature cars. How many miniature cars does Alice have?
Options:
a. 7
b. 9
c. 40
d. 175

Answer:
a. 7

Explanation:
Alice has 1/5 as many miniature cars as Sylvester has. Sylvester has 35 miniature cars. Alice has 1/5 × 35 = 7 miniature cars.

Common Core – Two-Dimensional Figures – Page No. 199

Find and Draw Lines of Symmetry

Tell whether the shape appears to have zero lines, 1 line, or more than 1 line of symmetry. Write zero, 1, or more than 1.

Question 1.
Go Math Grade 4 Answer Key Homework Practice FL Chapter 10 Two-Dimensional Figures Common Core - Two-Dimensional Figures img 39
1

Answer:
more than 1

Explanation:
There is more than 1 line of symmetry that separates two parts equally.

Question 2.
Go Math Grade 4 Answer Key Homework Practice FL Chapter 10 Two-Dimensional Figures Common Core - Two-Dimensional Figures img 40
________

Answer:
more than 1

Explanation:
There is more than 1 line of symmetry that separates two parts equally.

Question 3.
Go Math Grade 4 Answer Key Homework Practice FL Chapter 10 Two-Dimensional Figures Common Core - Two-Dimensional Figures img 41
________

Answer:
Zero

Explanation:
There are 0 lines of symmetries.

Question 4.
Go Math Grade 4 Answer Key Homework Practice FL Chapter 10 Two-Dimensional Figures Common Core - Two-Dimensional Figures img 42
________

Answer:
more than 1

Explanation:

grade 4 chapter 10 Lines, Rays, and Angles image 2 585
There is more than 1 line of symmetry that separates two parts equally.

Does the design have line symmetry? Write yes or no.
If your answer is yes, draw all lines of symmetry.

Question 5.
Go Math Grade 4 Answer Key Homework Practice FL Chapter 10 Two-Dimensional Figures Common Core - Two-Dimensional Figures img 43
____

Answer:
Yes
grade 4 chapter 10 Lines, Rays, and Angles image 3 585

Question 6.
Go Math Grade 4 Answer Key Homework Practice FL Chapter 10 Two-Dimensional Figures Common Core - Two-Dimensional Figures img 44
____

Answer:
Yes

grade 4 chapter 10 Lines, Rays, and Angles image 5 585

Question 7.
Go Math Grade 4 Answer Key Homework Practice FL Chapter 10 Two-Dimensional Figures Common Core - Two-Dimensional Figures img 45
____

Answer:
No

Question 8.
Go Math Grade 4 Answer Key Homework Practice FL Chapter 10 Two-Dimensional Figures Common Core - Two-Dimensional Figures img 46
____

Answer:
Yes
grade 4 chapter 10 Lines, Rays, and Angles image 6 585

Draw a shape for the statement. Draw the line or lines of symmetry.

Question 9.
zero lines of symmetry
Go Math Grade 4 Answer Key Homework Practice FL Chapter 10 Two-Dimensional Figures Common Core - Two-Dimensional Figures img 47

Answer:

grade 4 chapter 10 Lines, Rays, and Angles image 7 585

Question 10.
1 line of symmetry
Go Math Grade 4 Answer Key Homework Practice FL Chapter 10 Two-Dimensional Figures Common Core - Two-Dimensional Figures img 48

Answer:
grade 4 chapter 10 Lines, Rays, and Angles image 9 585

Question 11.
2 lines of symmetry
Go Math Grade 4 Answer Key Homework Practice FL Chapter 10 Two-Dimensional Figures Common Core - Two-Dimensional Figures img 49

Answer:

grade 4 chapter 10 Lines, Rays, and Angles image 10 585

Problem Solving

Use the chart for 12–13.
Go Math Grade 4 Answer Key Homework Practice FL Chapter 10 Two-Dimensional Figures Common Core - Two-Dimensional Figures img 50

Question 12.
Which number or numbers appear to have only 1 line of symmetry?
____

Answer:
3

Explanation:
The number 3 has only 1 line of symmetry.

Question 13.
Which number or numbers appear to have 2 lines of symmetry?
____

Answer:
0 and 8

Explanation:
The numbers 0 and 8 appear to have 2 lines of symmetry.

Common Core – Two-Dimensional Figures – Page No. 200

Lesson Check

Question 1.
How many lines of symmetry does this shape appear to have?
Go Math Grade 4 Answer Key Homework Practice FL Chapter 10 Two-Dimensional Figures Common Core - Two-Dimensional Figures img 51
Options:
a. 0
b. 2
c. 6
d. 12

Answer:
c. 6

Explanation:

grade 4 chapter 10 Lines, Rays, and Angles image 1 586
The given shape has 6 lines of symmetry.
Thus the correct answer is option c.

Question 2.
Which of the following shapes appears to have exactly 1 line of symmetry?
Options:
a. Go Math Grade 4 Answer Key Homework Practice FL Chapter 10 Two-Dimensional Figures Common Core - Two-Dimensional Figures img 52
b. Go Math Grade 4 Answer Key Homework Practice FL Chapter 10 Two-Dimensional Figures Common Core - Two-Dimensional Figures img 53
c.Go Math Grade 4 Answer Key Homework Practice FL Chapter 10 Two-Dimensional Figures Common Core - Two-Dimensional Figures img 54
d. Go Math Grade 4 Answer Key Homework Practice FL Chapter 10 Two-Dimensional Figures Common Core - Two-Dimensional Figures img 55

Explanation:
grade 4 chapter 10 Lines, Rays, and Angles image 2 586

The trapezoid has exactly 1 line of symmetry.
Thus the correct answer is option d.

Spiral Review

Question 3.
Richard practiced each of 3 piano solos for \(\frac{5}{12}\) hour. How long did he practice in all?
Options:
a. \(\frac{2}{3}\) hours
b. 1 \(\frac{1}{4}\) hours
c. 1 \(\frac{1}{3}\) hours
d. 1 \(\frac{5}{12}\) hours

Answer:
b. 1 1/4 hours

Explanation:
Richard practiced each of 3 piano solos for 5/12 hour. 5/12 hour = 1 1/4 hours hours.
Thus the correct answer is option b.

Question 4.
Which of the following decimals is equivalent to three and ten hundredths?
Options:
a. 0.30
b. 0.31
c. 3.01
d. 3.1

Answer:
d. 3.1

Explanation:
three and ten hundredths = 310 hundredths = 3.1
Thus the correct answer is option d.

Question 5.
Lynne used \(\frac{3}{8}\) cup of flour and \(\frac{1}{3}\) cup of sugar in a recipe. Which number below is a common denominator for \(\frac{3}{8}\) and \(\frac{1}{3}\)?
Options:
a. 8
b. 12
c. 16
d. 24

Answer:
d. 24

Explanation:
Lynne used 3/8 cup of flour and 1/3 cup of sugar in a recipe. To find the common denominator for 3/8 and 1/3, multiply 8 X3 and 3 X 8 = 24.
Thus the correct answer is option d.

Question 6.
Kevin draws a figure that has four sides. All sides have the same length. His figure has no right angles. What figure does Kevin draw?
Options:
a. square
b. trapezoid
c. rhombus
d. rectangle

Answer:
c. rhombus

Common Core – Two-Dimensional Figures – Page No. 201

Problem Solving Shape Patterns

Solve each Problem.

Question 1.
Marta is using this pattern to decorate a picture frame. Describe the pattern. Draw what might be the next three figures in the pattern.
Go Math Grade 4 Answer Key Homework Practice FL Chapter 10 Two-Dimensional Figures Common Core - Two-Dimensional Figures img 56
Possible answer: the pattern repeats: one trangle followed by two squares.

Answer:
Go Math Grade 4 Answer Key Chapter 10 Two-Dimensional Figures Common Core - New img 127

The pattern repeats one triangle followed by two squares.

Question 2.
Describe the pattern. Draw what might be the next three figures in the pattern. How many circles are in the sixth figure in the pattern?
Go Math Grade 4 Answer Key Homework Practice FL Chapter 10 Two-Dimensional Figures Common Core - Two-Dimensional Figures img 57
____ circles

Answer:
grade 4 chapter 10 Lines, Rays, and Angles image 1 591

Add one more column with 1 more circle than in the previous column; 21.

Question 3.
Larry stencils this pattern to make a border at the top of his bedroom walls. Describe the pattern. Draw what might be the missing figure in the pattern.
Go Math Grade 4 Answer Key Homework Practice FL Chapter 10 Two-Dimensional Figures Common Core - Two-Dimensional Figures img 58

Answer:
grade 4 chapter 10 Lines, Rays, and Angles image 2 591

2 triangles placed side to side followed by 2 sets of 2 triangles placed vertex to vertex

Common Core – Two-Dimensional Figures – Page No. 202

Lesson Check

Question 1.
What might be the next three figures in this pattern?
Go Math Grade 4 Answer Key Homework Practice FL Chapter 10 Two-Dimensional Figures Common Core - Two-Dimensional Figures img 59
Options:
a. Go Math Grade 4 Answer Key Homework Practice FL Chapter 10 Two-Dimensional Figures Common Core - Two-Dimensional Figures img 60
b. Go Math Grade 4 Answer Key Homework Practice FL Chapter 10 Two-Dimensional Figures Common Core - Two-Dimensional Figures img 61
c. Go Math Grade 4 Answer Key Homework Practice FL Chapter 10 Two-Dimensional Figures Common Core - Two-Dimensional Figures img 62
d. Go Math Grade 4 Answer Key Homework Practice FL Chapter 10 Two-Dimensional Figures Common Core - Two-Dimensional Figures img 63

Answer:
a. Go Math Grade 4 Answer Key Chapter 10 Two-Dimensional Figures Common Core - New img 131

Explanation:
the pattern has odd numbers of up arrows then even number of down arrows. So, the next three figures areGo Math Grade 4 Answer Key Chapter 10 Two-Dimensional Figures Common Core - New img 131

Question 2.
Which might be the missing figure in the following pattern?
Go Math Grade 4 Answer Key Homework Practice FL Chapter 10 Two-Dimensional Figures Common Core - Two-Dimensional Figures img 64
Options:
a. Go Math Grade 4 Answer Key Homework Practice FL Chapter 10 Two-Dimensional Figures Common Core - Two-Dimensional Figures img 65
b. Go Math Grade 4 Answer Key Homework Practice FL Chapter 10 Two-Dimensional Figures Common Core - Two-Dimensional Figures img 66
c. Go Math Grade 4 Answer Key Homework Practice FL Chapter 10 Two-Dimensional Figures Common Core - Two-Dimensional Figures img 67
d.Go Math Grade 4 Answer Key Homework Practice FL Chapter 10 Two-Dimensional Figures Common Core - Two-Dimensional Figures img 68

Answer:
a. Go Math Grade 4 Answer Key Chapter 10 Two-Dimensional Figures Common Core - New img 136

Explanation:
From the pattern, the missing image will have a vertical rectangle with the circle and X mark in it.

Spiral Review

Question 3.
Chad has two pieces of wood. One piece is \(\frac{7}{12}\) foot long. The second piece is \(\frac{5}{12}\) foot longer than the first piece. How long is the second piece?
Options:
a. \(\frac{2}{12}\) foot
b. \(\frac{1}{2}\) foot
c. \(\frac{12}{18}\) foot
d. 1 foot

Answer:
d. 1 foot

Explanation:
7/12 + 5/12 = 12/12 = 1 foot.

Question 4.
Olivia finished a race in 40.64 seconds. Patty finished the race in 40.39 seconds. Miguel finished the race in 41.44 seconds. Chad finished the race in 40.46 seconds. Who finished the race in the least time?
Options:
a. Olivia
b. Patty
c. Miguel
d. Chad

Answer:
b. Patty

Explanation:
Patty finished the race in 40.39 seconds that is the least time compared to others.

Question 5.
Justin bought 6 ribbons for an art project. Each ribbon is \(\frac{1}{4}\) yard long. How many yards of ribbon did Justin buy?
Options:
a. \(\frac{2}{3}\) yard
b. 1 \(\frac{1}{4}\) yards
c. 1 \(\frac{1}{2}\) yards
d. 1 \(\frac{3}{4}\) yards

Answer:
c. 1 1/2 yards

Explanation:
Justin bought 6 ribbons for an art project. Each ribbon is 1/4 yard long. So, 6 X 1/4 = 3/2 = 1 1/2 yards.

Question 6.
Kyle and Andrea were asked to make a list of prime numbers.
Kyle: 1, 3, 7, 19, 23
Andrea: 2, 3, 5, 7, 11
Whose list is correct?
Options:
a. Only Kyle’s list
b. Only Andrea’s list
c. Both lists are correct.
d. Neither list is correct.

Answer:
b. Only Andrea’s list

Explanation:
1 is not a prime number. So, the answer is Only Andrea’s list is correct.

Common Core – Two-Dimensional Figures – Page No. 203

Lesson 10.1

Draw and label an example of the figure.

Question 1.
acute ∠MNP
Type below:
_________

Answer:

Question 2.
\(\overline{Q R}\)
Type below:
_________

Question 3.
\(\overrightarrow { TS } \)
Type below:
_________

Lesson 10.2

Classify each triangle. Write acute, right, or obtuse.

Question 4.
Go Math Grade 4 Answer Key Homework Practice FL Chapter 10 Two-Dimensional Figures Common Core - Two-Dimensional Figures img 69
_____

Answer: Acute

Explanation:
The above triangle is less than 90º, thus the above figure is an acute angle triangle.

Question 5.
Go Math Grade 4 Answer Key Homework Practice FL Chapter 10 Two-Dimensional Figures Common Core - Two-Dimensional Figures img 70
_____

Answer: Obtuse

Explanation:
The above triangle is greater than 90º, thus the above figure is an obtuse angle triangle.

Question 6.
Go Math Grade 4 Answer Key Homework Practice FL Chapter 10 Two-Dimensional Figures Common Core - Two-Dimensional Figures img 71
_____

Answer: Right

Explanation:
The above figure has 90º, thus the above figure is an right angle triangle.

Lesson 10.3

Use the street map for 1–2.
Go Math Grade 4 Answer Key Homework Practice FL Chapter 10 Two-Dimensional Figures Common Core - Two-Dimensional Figures img 72

Question 7.
Name two streets that appear to be parallel.
_________

Answer: Oak and elm

Explanation:
By seeing the above figure we say that Oak and Elm are two non-intersecting lines. Thus the two streets that appear to be parallel are Oak and Elm.

Question 8.
Name two streets that appear to be perpendicular.
Type below:
_________

Answer: Park and Oak or Park and Elm

Explanation:
Park and Oak, Park and Elm are intersecting lines, thus the two streets that appear to be perpendicular are Park and Oak or Park and Elm.

Lesson 10.4

Classify each figure as many ways as possible. Write quadrilateral, trapezoid, parallelogram, rhombus, rectangle, or square.

Question 9.
Go Math Grade 4 Answer Key Homework Practice FL Chapter 10 Two-Dimensional Figures Common Core - Two-Dimensional Figures img 73
Type below:
_________

Answer:
Quadrilateral, Parallelogram, Rectangle

Explanation:
2 pairs of parallel sides
2 pairs of sides of equal length
4 right angles
Quadrilateral, Parallelogram, Rectangle

Question 10.
Go Math Grade 4 Answer Key Homework Practice FL Chapter 10 Two-Dimensional Figures Common Core - Two-Dimensional Figures img 74
Type below:
_________

Answer: Quadrilateral, Trapezoid

Explanation:
1 pair of parallel sides
2 sides of equal length
0 right angles
Quadrilateral, Trapezoid

Common Core – Two-Dimensional Figures – Page No. 204

Lesson 10.5

Tell if the dashed line appears to be a line of symmetry.
Write yes or no.

Question 1.
Go Math Grade 4 Answer Key Homework Practice FL Chapter 10 Two-Dimensional Figures Common Core - Two-Dimensional Figures img 75
_____

Answer: Yes

Explanation:
The line of symmetry divides a shape into two parts that are the same size and shape.

Question 2.
Go Math Grade 4 Answer Key Homework Practice FL Chapter 10 Two-Dimensional Figures Common Core - Two-Dimensional Figures img 76
_____

Answer: Yes

Explanation:
The line of symmetry divides a shape into two parts that are the same size and shape.

Question 3.
Go Math Grade 4 Answer Key Homework Practice FL Chapter 10 Two-Dimensional Figures Common Core - Two-Dimensional Figures img 77
_____

Answer: Yes

Explanation:
The line of symmetry divides a shape into two parts that are the same size and shape.

Lesson 10.6

Does the design have line symmetry? Write yes or no.

If your answer is yes, draw all lines of symmetry.

Question 4.
Go Math Grade 4 Answer Key Homework Practice FL Chapter 10 Two-Dimensional Figures Common Core - Two-Dimensional Figures img 78
_____

Answer: Yes

Explanation:
The line of symmetry divides a shape into two parts that are not with the same size and shape. The above figure is not symmetrical.

Question 5.
Go Math Grade 4 Answer Key Homework Practice FL Chapter 10 Two-Dimensional Figures Common Core - Two-Dimensional Figures img 79
_____

Answer: Yes

The above figure is symmetrical.

Question 6.
Go Math Grade 4 Answer Key Homework Practice FL Chapter 10 Two-Dimensional Figures Common Core - Two-Dimensional Figures img 80
_____

Answer: Yes

Explanation: The above figure is symmetrical.

Lesson 10.7

Question 7.
Sonia made a pattern. The first nine shapes are shown below. Describe the pattern. Draw what might be the next three shapes in Sonia’s pattern.
Go Math Grade 4 Answer Key Homework Practice FL Chapter 10 Two-Dimensional Figures Common Core - Two-Dimensional Figures img 81
Type below:
_________

Answer: The pattern repeats circle, square, circle.

Question 8.
Leo makes a pattern with triangles. Draw what might be the next figure in the pattern. How can you describe the pattern?
Go Math Grade 4 Answer Key Homework Practice FL Chapter 10 Two-Dimensional Figures Common Core - Two-Dimensional Figures img 82
Type below:
_________

Answer: The pattern grows by one triangle each time.

Conclusion:

Access the links and avail the grade 4 Ch 10 Go Math Answer Key a quick learning guide with clear-cut explanations to understand the concepts. Also, you can refer to the Go Math Grade 4 Answer Key Chapter 10 Two-Dimensional Figures for better idea of the questions.

Go Math Grade 4 Answer Key Chapter 11 Angles

go-math-grade-4-chapter-11-angles-answer-key

Examine your Preparation and understanding level towards chapter 11 concepts with the help of the Go Math Grade 4 Answer Key Chapter 11 Angles Assessment Test. All you need to do is just click on the links provided here and assess your weak areas and strong areas. So that you can allot time accordingly & fill up the knowledge gaps using the 4th Grade HMH Go Math Homework Practice FL Answer Key Ch 11 Angles. With regular practice, you can secure more marks in your exam.

Go Math Grade 4 Answer Key Chapter 11 Angles

Go Math Grade 4 Answer Key Chapter 11 includes all standard topics of angles. Improve your math skills by taking help from 4th Grade HMH Go Math Solutions Key Chapter 11 Angles Practice FL, Homework practice FL. Educators and Instructors must use of these Grade 4 Go Math Chapter 11 Angles Answer Key & educate their students to understand the topics clearly. Step by step Solutions are given for all the concepts of angles and you can refer to them to easily and clear all your doubts. Utilize these links and begin your practice right away.

Lesson 1:

Lesson 2:

Lesson 3: Measure and Draw Angles

Mid-Chapter Checkpoint

Lesson 4:

Common Core – New

Lesson 5:

Common Core – New

Chapter 11 Review/Test

Common Core – New – Page No. 605

Angles and Fractional Parts of a Circle

Tell what fraction of the circle the shaded angle represents.

Question 1.
Go Math Grade 4 Answer Key Chapter 11 Angles Common Core - New img 1

The figure shows that the \(\frac{1}{4}\)th part of the circle is shaded. So, the fraction of the shaded angle is \(\frac{1}{4}\)

Question 2.
Go Math Grade 4 Answer Key Chapter 11 Angles Common Core - New img 2
\(\frac{□}{□}\)

Answer: \(\frac{1}{2}\)

Explanation:

Half of the circle is shaded. Thus the fraction of the shaded angle is \(\frac{1}{2}\)

Question 3.
Go Math Grade 4 Answer Key Chapter 11 Angles Common Core - New img 3
\(\frac{□}{□}\)

Answer: \(\frac{1}{1}\)

Explanation:

From the above figure, we can observe that the complete circle is shaded. So, the fraction of the shaded angle is \(\frac{1}{1}\) or 1.

Tell whether the angle on the circle shows a \(\frac{1}{4}, \frac{1}{2}, \frac{3}{4}\), or 1 full turn clockwise or counterclockwise.

Question 4.
Go Math Grade 4 Answer Key Chapter 11 Angles Common Core - New img 4
\(\frac{□}{□}\)

Answer: \(\frac{1}{2}\) turn counter clockwise

Explanation:

From the figure, we can see that the circle is rotating in the anti-clockwise direction. And it has completed the half turn.
Thus the fraction is \(\frac{1}{2}\) turn counter clockwise

Question 5.
Go Math Grade 4 Answer Key Chapter 11 Angles Common Core - New img 5
\(\frac{□}{□}\)

Answer: \(\frac{3}{4}\) turn clockwise

Explanation:

The arrow is turned in a clockwise direction. It has completed \(\frac{3}{4}\) turn. So, the angle with direction is \(\frac{3}{4}\) turn clockwise.

Question 6.
Go Math Grade 4 Answer Key Chapter 11 Angles Common Core - New img 6
_________

Answer: 1 full turn counter clockwise

Explanation:

From the above picture, we can observe that the circle has completed the full turn in the counter clockwise direction.

Problem Solving

Question 7.
Shelley exercised for 15 minutes. Describe the turn the minute hand made.
Go Math Grade 4 Answer Key Chapter 11 Angles Common Core - New img 7
Type below:
_________

Answer: The minute hand made a turn of \(\frac{1}{4}\) clockwise.

Explanation:

Given that,

Shelley exercised for 15 minutes.
So, the fraction of the minute hand made is \(\frac{1}{4}\).
The direction of the minute hand made is clockwise.
So, the answer is the minute hand made a turn of \(\frac{1}{4}\) clockwise.

Question 8.
Mark took 30 minutes to finish lunch. Describe the turn the minute hand made.
Go Math Grade 4 Answer Key Chapter 11 Angles Common Core - New img 8
Type below:
_________

Answer: The minute hand made a turn of \(\frac{1}{2}\) clockwise.

Explanation:

Given, Mark took 30 minutes to finish lunch.
The minute hand made a turn in the clockwise direction from 12 to 6.
That means the fraction of the angle is \(\frac{1}{2}\).
Thus the turn minute hand made is \(\frac{1}{2}\) clockwise.

Common Core – New – Page No. 606

Lesson Check

Question 1.
What fraction of the circle does the shaded angle represent
Go Math Grade 4 Answer Key Chapter 11 Angles Common Core - New img 9
Options:
a. \(\frac{1}{1}\) or 1
b. \(\frac{3}{4}\)
c. \(\frac{1}{2}\)
d. \(\frac{1}{4}\)

Answer: \(\frac{1}{4}\)

Explanation:

From the figure we can say that the fraction of the shaded angle is \(\frac{1}{4}\).
Thus the answer is option D.

Question 2.
Which describes the turn shown below?
Go Math Grade 4 Answer Key Chapter 11 Angles Common Core - New img 10
Options:
a. \(\frac{1}{4}\) turn clockwise
b. \(\frac{1}{2}\) turn clockwise
c. \(\frac{1}{4}\) turn counterclockwise
d. \(\frac{1}{2}\) turn counterclockwise

Answer: \(\frac{1}{2}\) turn clockwise

Explanation:

From the figure, we can see that the circle is rotating in the clockwise direction. And it has completed the half turn.
So, the answer is \(\frac{1}{2}\) turn clockwise.

Spiral Review

Question 3.
Which shows \(\frac{2}{3}\) and \(\frac{3}{4}\) written as a pair of fractions with a common denominator?
Options:
a. \(\frac{2}{3} \text { and } \frac{4}{3}\)
b. \(\frac{6}{9} \text { and } \frac{6}{8}\)
c. \(\frac{2}{12} \text { and } \frac{3}{12}\)
d. \(\frac{8}{12} \text { and } \frac{9}{12}\)

Answer: \(\frac{8}{12} \text { and } \frac{9}{12}\)

Explanation:

\(\frac{2}{3}\) and \(\frac{3}{4}\)
The denomintors are different here. So you have to make the denominators common.
\(\frac{2}{3}\) × \(\frac{4}{4}\) = \(\frac{8}{12}\)
\(\frac{3}{4}\) × \(\frac{3}{3}\) = \(\frac{9}{12}\)
So the answer is option D.

Question 4.
Raymond bought \(\frac{3}{4}\) of a dozen rolls. How many rolls did he buy?
Options:
a. 3
b. 6
c. 7
d. 9

Answer: 9

Explanation:

Raymond bought \(\frac{3}{4}\) of a dozen rolls.
Dozen = 12
\(\frac{3}{4}\) × 12 = 9
Thus the correct answer is option D.

Question 5.
Which of the following lists all the factors of 18?
Options:
a. 1, 2, 4, 9, 18
b. 1, 2, 3, 6, 9, 18
c. 2, 3, 6, 9
d. 1, 3, 5, 9, 18

Answer: 1, 2, 3, 6, 9, 18

Explanation:

The factors of 18 are
1 × 18 = 18
2 × 9 = 18
3 × 6 = 18
6 × 3 = 18
9 × 2 = 18
18 × 1 = 18
Thus the correct answer is option B.

Question 6.
Jonathan rode 1.05 miles on Friday, 1.5 miles on Saturday, 1.25 miles on Monday, and 1.1 miles on Tuesday. On which day did he ride the shortest distance?
Options:
a. Monday
b. Tuesday
c. Friday
d. Saturday

Answer: Friday

Explanation:

Jonathan rode 1.05 miles on Friday, 1.5 miles on Saturday, 1.25 miles on Monday, and 1.1 miles on Tuesday.
The shortest among all is 1.05 miles.
Therefore the answer is option C.

Page No. 609

Question 1.
Find the measure of the angle.
Go Math Grade 4 Answer Key Chapter 11 Angles img 11
Through what fraction of a circle does the angle turn?
\(\frac{1}{3}=\frac{■}{360}\)
Think: 3 × 12 = 36, so 3 × _____ = 360.
So, the measure of the angle is _____.
_____ degrees

Answer: 120°

Explanation:

The fraction of the shaded angle is \(\frac{1}{3}\)
To measure the angle we have to multiply the fraction of the shaded angle with the total angle.
That means, \(\frac{1}{3}\) × 360
360/3 = 120 degrees.
Thus the angle of the shaded part is 120°

Tell the measure of the angle in degrees.

Question 2.
Go Math Grade 4 Answer Key Chapter 11 Angles img 12
____ °

Answer: 45°

Explanation:

The fraction of the shaded angle is \(\frac{45}{360}\)
Multiply the fraction with the complete angle
\(\frac{45}{360}\) × 360° = 45°
Thus the angle of the above figure is 45°

Question 3.
Go Math Grade 4 Answer Key Chapter 11 Angles img 13
____ degrees

Answer: 30°

Explanation:

The figure shows the fraction of the shaded angle is \(\frac{1}{12}\)
Multiply the fraction with the complete angle
\(\frac{1}{12}\) × 360° = 30°
Therefore the measure of the shaded angle is 30°

Tell the measure of the angle in degrees.

Question 4.
Go Math Grade 4 Answer Key Chapter 11 Angles img 14
____ °

Answer: 360°

Explanation:

We observe that the circle is shaded completely.
\(\frac{360}{360}\) × 360° = 360°
Thus the above figure is the complete angle.

Question 5.
Go Math Grade 4 Answer Key Chapter 11 Angles img 15
____ °

Answer: 36°

Explanation:

The fraction of the shaded angle is \(\frac{1}{10}\)
Multiply the fraction with the complete angle
\(\frac{1}{10}\) × 360° = 36°
Therefore the measure of the shaded angle is 36°

Classify the angle. Write acute, obtuse, right, or straight.

Question 6.
Go Math Grade 4 Answer Key Chapter 11 Angles img 16
_________

Answer: Obtuse

An obtuse angle has a measurement greater than 90 degrees but less than 180 degrees. However, A reflex angle measures more than 180 degrees but less than 360 degrees.

Question 7.
Go Math Grade 4 Answer Key Chapter 11 Angles img 17
_________

Answer: Right

A right angle is an angle of exactly 90° (degrees), corresponding to a quarter turn. If a ray is placed so that its endpoint is on a line and the adjacent angles are equal, then they are right angles.

Question 8.
Go Math Grade 4 Answer Key Chapter 11 Angles img 18
_________

Answer: Acute

The acute angle is the small angle which is less than 90°.

Question 9.
Go Math Grade 4 Answer Key Chapter 11 Angles img 19
_________

Answer: Straight

A straight angle is 180 degrees. A straight angle changes the direction to point the opposite way.

Question 10.
Is this an obtuse angle? Explain.
Go Math Grade 4 Answer Key Chapter 11 Angles img 20
Type below:
_________

Answer: Obtuse

An obtuse angle has a measurement greater than 90 degrees but less than 180 degrees. However, A reflex angle measures more than 180 degrees but less than 360 degrees.

Question 11.
Alex cut a circular pizza into 8 equal slices. He removed 2 of the slices of pizza. What is the measure of the angle made by the missing slices of pizza?
Go Math Grade 4 Answer Key Chapter 11 Angles img 21
____ °

Answer: 90°

Explanation:

Alex cut a circular pizza into 8 equal slices.
He removed 2 of the slices of pizza.
The fraction of the missing slices = \(\frac{2}{8}\) = \(\frac{1}{4}\)
The fraction of the missing slices is \(\frac{1}{4}\)
To know the angle we have to multiply the fraction with complete angle i.e., 360°
\(\frac{1}{4}\) × 360° = 90°
Thus the angle of the missing slices is 90°

Page No. 610

Question 12.
Ava started reading at 3:30 p.m. She stopped for a snack at 4:15 p.m. During this time, through what fraction of a circle did the minute hand turn? How many degrees did the minute hand turn?
Go Math Grade 4 Answer Key Chapter 11 Angles img 22
a. What are you asked to find?
Type below:
_________

Answer: I am asked to find the fraction of a circle did the minute hand turn and how many degrees did the minute hand turn

Question 12.
b. What information can you use to find the fraction of a circle through which the minute hand turned?
Type below:
_________

Answer: The fraction of a circle through which the minute hand-turned \(\frac{3}{4}\) Clockwise.

Question 12.
c. How can you use the fraction of a circle through which the minute hand turned to find how many degrees it turned?
Type below:
_________

Answer:

The figure shows that the fraction of a circle through which the minute hand turned is \(\frac{3}{4}\) Clockwise.
Let the shaded part be x
And the nonshaded part is 90°
x + 90° = 360°
x = 360°- 90°
x = 270°
Therefore the minute hand turns 270° clockwise.

Question 12.
d. Show the steps to solve the problem.
Step 1:
\(\frac{3 × ■}{4 × ■}=\frac{?}{360}\)
Step 2:
\(\frac{3 × 90}{4 × 90}=\frac{■}{360}\)
Type below:
_________

Answer:
\(\frac{3 × 90}{4 × 90}=\frac{■}{360}\)
\(\frac{270}{360} = \frac{■}{360}\)
If the denominators are equal then the numerators must be equated.
■ = 270

Question 12.
e. Complete the sentences. From 3:30 p.m. to 4:15 p.m., the minute hand made a ______ turn clockwise. The minute hand turned ______ degrees.
Type below:
_________

Answer:
From 3:30 p.m. to 4:15 p.m., the minute hand made a \(\frac{3}{4}\) turn clockwise. The minute hand turned 270 degrees.

Question 13.
An angle represents \(\frac{1}{15}\) of a circle. Select the number to show how to find the measure of the angle in degrees.
Go Math Grade 4 Answer Key Chapter 11 Angles img 23
Go Math Grade 4 Answer Key Chapter 11 Angles img 24
\(\frac{1}{15}=\frac{1 × □}{15 × □}=\frac{□}{360}\)
Type below:
_________

Answer: 24°
\(\frac{1}{15} × 360° = 24°

Common Core – New – Page No. 611

Degrees

Tell the measure of the angle in degrees.

Question 1.
Go Math Grade 4 Answer Key Chapter 11 Angles Common Core - New img 25
Answer: 60°

Explanation:

Given that the fraction of the shaded angle is [latex]\frac{60}{360}\)
\(\frac{60}{360}\) × 360 = 60°
Thus the angle for the above figure is 60°

Question 2.
Go Math Grade 4 Answer Key Chapter 11 Angles Common Core - New img 26
____ °

Answer: 180°

Explanation:

Half of the circle is shaded. The fraction of the shaded angle is \(\frac{1}{2}\)
\(\frac{1}{2}\) × 360 = 360/2 = 180°

Question 3.
Go Math Grade 4 Answer Key Chapter 11 Angles Common Core - New img 27
____ °

Answer: 90°

Explanation:

The fraction of the shaded angle is \(\frac{1}{4}\)
To find the angle we need to multiply the fraction with the total angle.
\(\frac{1}{4}\) × 360° = 90°

Classify the angle. Write acute, obtuse, right, or straight.

Question 4.
Go Math Grade 4 Answer Key Chapter 11 Angles Common Core - New img 28
_________

Answer: Acute

Explanation:

25° < 90°
So, the above figure is an acute angle.

Question 5.
Go Math Grade 4 Answer Key Chapter 11 Angles Common Core - New img 29
_________

Answer: Obtuse

Explanation:

110° > 90°
So, the figure shown above is an obtuse angle.

Question 6.
Go Math Grade 4 Answer Key Chapter 11 Angles Common Core - New img 30
_________

Answer: Acute

Explanation:

60° < 90°
Acute angles measure less than 90 degrees. Thus the above angle is an acute angle.

Classify the triangle. Write acute, obtuse, or right.

Question 7.
Go Math Grade 4 Answer Key Chapter 11 Angles Common Core - New img 31
_________

Answer: Right

Explanation:

65 + 25 = 90
The sum of two angles = 90°
Thus the above figure is a right-angled triangle.

Question 8.
Go Math Grade 4 Answer Key Chapter 11 Angles Common Core - New img 32
_________

Answer: Obtuse

Explanation:

110° > 90°
So, the above triangle is an obtuse angle.

Question 9.
Go Math Grade 4 Answer Key Chapter 11 Angles Common Core - New img 33
_________

Answer: Acute

Explanation:

50° is less than 90°
Thus the above triangle is an acute angle triangle.

Problem Solving

Ann started reading at 4:00 P.M. and finished at 4:20 P.M.
Go Math Grade 4 Answer Key Chapter 11 Angles Common Core - New img 34

Question 10.
Through what fraction of a circle did the minute hand turn?
\(\frac{□}{□}\)

Answer: \(\frac{1}{3}\) turn clockwise

Explanation:

The fraction of the shaded clock is \(\frac{12}{4}\)
\(\frac{12}{4}\) = \(\frac{1}{3}\)
The minute hand turn clockwise direction.
So, the answer is \(\frac{1}{3}\) turn clockwise

Question 11.
How many degrees did the minute hand turn?
____ °

Answer: 120°

Explanation:

The fraction of the minute hand turn is \(\frac{1}{3}\)
\(\frac{1}{3}\) × 360° = 120°
The minute hand turn 120°

Common Core – New – Page No. 612

Lesson Check

Question 1.
What kind of angle is shown?
Go Math Grade 4 Answer Key Chapter 11 Angles Common Core - New img 35
Options:
a. acute
b. obtuse
c. right
d. straight

Answer: straight

A straight angle is 180 degrees. This is a straight angle. A straight angle changes the direction to point the opposite way.
So, the answer is option D.

Question 2.
How many degrees are in an angle that turns through \(\frac{1}{4}\) of a circle?
Options:
a. 45°
b. 90°
c. 180°
d. 270°

Answer: 90°

Explanation:

\(\frac{1}{4}\) × 360°
\(\frac{360}{4}\) = 90°
Thus the correct answer is option B.

Spiral Review

Question 3.
Mae bought 15 football cards and 18 baseball cards. She separated them into 3 equal groups. How many sports cards are in each group?
Options:
a. 5
b. 6
c. 11
d. 12

Answer: 11

Explanation:

Mae bought 15 football cards and 18 baseball cards.
She separated them into 3 equal groups.
Total number of cards = 15 + 18 = 33
33/3 = 11
There are 11 sports cards in each group.

Question 4.
Each part of a race is \(\frac{1}{10}\) mile long. Marsha finished 5 parts of the race. How far did Marsha race?
Options:
a. \(\frac{1}{10}\) mile
b. \(\frac{5}{12}\) mile
c. \(\frac{1}{2}\) mile
d. 5 \(\frac{1}{10}\) miles

Answer: \(\frac{1}{2}\) mile

Explanation:

Each part of a race is \(\frac{1}{10}\) mile long.
Marsha finished 5 parts of the race.
\(\frac{1}{10}\) × 5 = 5/10 = \(\frac{1}{2}\) mile
Thus the correct answer is option C.

Question 5.
Jeff said his city got \(\frac{11}{3}\) inches of snow. Which shows this fraction written as a mixed number?
Options:
a. 3 \(\frac{2}{3}\)
b. 3 \(\frac{1}{3}\)
c. 2 \(\frac{2}{3}\)
d. 1 \(\frac{2}{3}\)

Answer: 3 \(\frac{2}{3}\)

Explanation:

Jeff said his city got \(\frac{11}{3}\) inches of snow.
The mixed fraction of \(\frac{11}{3}\) is 3 \(\frac{2}{3}\)
The correct answer is option A.

Question 6.
Amy ran \(\frac{3}{4}\) mile. Which decimal shows how many miles she ran?
Options:
a. 0.25 mile
b. 0.34 mile
c. 0.5 mile
d. 0.75 mile

Answer: 0.75 mile

Explanation:

Amy ran \(\frac{3}{4}\) mile.
\(\frac{3}{4}\) = \(\frac{75}{100}\)
The decimal form of \(\frac{75}{100}\) is 0.75
So, the answer is option D.

Page No. 615

Question 1.
Measure ∠ABC.
Go Math Grade 4 Answer Key Chapter 11 Angles img 36
Place the center of the protractor on point ____.
Align ray BC with ____ .
Read where ____ intersects the same scale.
So, m∠ABC is _____.
Type below:
_________

Answer: 65°

Use a protractor to find the angle measure.

Question 2.
Go Math Grade 4 Answer Key Chapter 11 Angles img 37
m∠ONM = ____ °

Answer: 55°

Question 3.
Go Math Grade 4 Answer Key Chapter 11 Angles img 38
m∠TSR = ____ °

Answer: 105°

Use a protractor to draw the angle.

Question 4.
170°
Type below:
_________

Answer:

Go Math grade 4 chapter 11 angles answer key image_1

Question 5.
78°
Type below:
_________

Answer:

Go Math Grade 4 Chapter 11 Answer Key image_2

Use a protractor to find the angle measure.

Question 6.
Go Math Grade 4 Answer Key Chapter 11 Angles img 39
m∠QRS = ____ °

Answer: 90°

Question 7.
Go Math Grade 4 Answer Key Chapter 11 Angles img 40
m∠XYZ = ____ °

Answer: 155°

Use a protractor to draw the angle.

Question 8.
115°
Type below:
_________

Answer:

Go Math Grade 5 Solution Key Angles image_3

Question 9.
67°
Type below:
_________

Answer:

Draw an example of each. Label the angle with its measure.

Question 10.
an acute angle
Type below:
_________

Answer:

Go Math Grade 4 Answer Key Chapter 11 Angles img 18

Question 11.
an obtuse angle
Type below:
_________

Answer:

Go Math Grade 4 Answer Key Chapter 11 Angles Common Core - New img 29

Question 12.
Elizabeth is making a quilt with scraps of fabric. What is the difference between m∠ABC and m∠DEF?
Go Math Grade 4 Answer Key Chapter 11 Angles img 41
____ °

Answer: 15°

Question 13.
Draw an angle with a measure of 0°.
Describe your drawing.
Type below:
_________

Answer:

HMH Go Math Grade 4 Key Chapter 11 image_4

Page No. 616

Question 14.
Hadley wants to divide this angle into three angles with equal measure. What will the measure of each angle be?
Go Math Grade 4 Answer Key Chapter 11 Angles img 42
____ °

Answer: 30°

Explanation:

Given,
Hadley wants to divide this angle into three angles with equal measure.
The above figure is a right angle = 90°
If he divides into three equal angles
90/3 = 30°
So, the measure of angle will be 30°

Question 15.
Tracy measured an angle as 50° that was actually 130°. Explain her error.
Type below:
_________

Answer: She has measured the angle in the counterclockwise direction. So, that is why she got 50°.

Question 16.
Choose the word or number to complete a true statement about ∠QRS.
Go Math Grade 4 Answer Key Chapter 11 Angles img 43
∠QRS is a(n) Go Math Grade 4 Answer Key Chapter 11 Angles img 44 angle that has a measure of Go Math Grade 4 Answer Key Chapter 11 Angles img 45
Type below:
_________

Answer: ∠QRS is an obtuse angle that has a measure of 135°.

Earth’s Axis Earth revolves around the sun yearly. The Northern Hemisphere is the half of Earth that is north of the equator. The seasons of the year are due to the tilt of Earth’s axis.

Use the diagrams and a protractor for 17–18.
Go Math Grade 4 Answer Key Chapter 11 Angles img 46

Question 17.
In the Northern Hemisphere, Earth’s axis is tilted away from the sun on the first day of winter, which is often on December 21. What is the measure of the marked angle on the first day of winter, the shortest day of the year?
____ °

Answer: 115°

Explanation:

By seeing the above figure we can say that the angle is an obtuse angle. The mark is above 90° and the marked angle is 115°.
Therefore the measure of the marked angle on the first day of winter, the shortest day of the year is 115°.

Question 18.
Earth’s axis is not tilted away from or toward the sun on the first days of spring and fall, which are often on March 20 and September 22. What is the measure of the marked angle on the first day of spring or fall?
____ °

Answer: 90°

Explanation:

The mark is exactly 90°. So, the angle on the first day of spring or fall is 90°

Common Core – New – Page No. 617

Measure and Draw Angles

Use a protractor to find the angle measure.

Question 1.
Go Math Grade 4 Answer Key Chapter 11 Angles Common Core - New img 47
m∠ABC = 120°

By using the protractor we can measure the angle m∠ABC i.e., 120°

Question 2.
Go Math Grade 4 Answer Key Chapter 11 Angles Common Core - New img 48
m∠MNP = ____ °

Answer: m∠MNP = 90°

By observing the above figure we can say that the angle of m∠MNP is 90°

Question 3.
Go Math Grade 4 Answer Key Chapter 11 Angles Common Core - New img 49
m∠RST = ____ °

Answer: m∠RST = 65°
By using the protractor we can measure m∠RST = 65°

Use a protractor to draw the angle.

Question 4.
40°

Answer:

Go Math Grade 4 Answer Key Chapter 11 image_5

Question 5.
170°

Answer:

Go Math grade 4 chapter 11 angles answer key image_1

Draw an example of each. Label the angle with its measure.

Question 6.
a right angle

Answer:

A right angle is an angle of exactly 90°

Go Math Grade 4 Answer Key Chapter 11 Angles img 39

Question 7.
an acute angle

Answer:

The acute angle is the small angle which is less than 90°.

Go Math Grade 4 Answer Key Chapter 11 Angles Common Core - New img 28

Problem Solving

The drawing shows the angles a stair tread makes with a support board along a wall. Use your protractor to measure the angles.
Go Math Grade 4 Answer Key Chapter 11 Angles Common Core - New img 50

Question 8.
What is the measure of ∠A?
____ °

Answer: 45°

By using the protractor we can measure the angle for A = 45°

Question 9.
What is the measure of ∠B?
____ °

Answer: 135°

The same process is used to measure ∠B = 135°

Common Core – New – Page No. 618

Lesson Check

Question 1.
What is the measure of ∠ABC?
Go Math Grade 4 Answer Key Chapter 11 Angles Common Core - New img 51
Options:
a. 15°
b. 25°
c. 155°
d. 165°

Answer: 15°

Explanation:

Step 1: Place the center point of the protractor on the point B.
Step 2: Align the 0° mark on the scale of the protractor with ray BC.
Step 3: Find the point where AC meet. Read the angle measure on that scale.
So, the measure of ∠ABC is 15°
Thus the correct answer is option A.

Question 2.
What is the measure of ∠XYZ?
Go Math Grade 4 Answer Key Chapter 11 Angles Common Core - New img 52
Options:
a. 20°
b. 30°
c. 150°
d. 160°

Answer: 150°

Explanation:

Step 1: Place the center point of the protractor on the point Y.
Step 2: Align the 0° mark on the scale of the protractor with ray XY.
Step 3: Find the point where YZ meet. Read the angle measure on that scale.
So, ∠XYZ = 150°
Therefore the correct answer is option C.

Spiral Review

Question 3.
Derrick earned $1,472 during the 4 weeks he had his summer job. If he earned the same amount each week, how much did he earn each week?
Options:
a. $360
b. $368
c. $3,680
d. $5,888

Answer: $368

Explanation:

Given that, Derrick earned $1,472 during the 4 weeks he had his summer job.
Let the amount he earned per week = x
x × 4 = $1,472
x = 1472 ÷ 4
x = 1472/4 = 368
So, Derrick earned $368 per week.

Question 4.
Arthur baked 1 \(\frac{7}{12}\) dozen muffins. Nina baked 1 \(\frac{1}{12}\) dozen muffins. How many dozen muffins did they bake in all?
Options:
a. 3 \(\frac{2}{3}\)
b. 2 \(\frac{2}{3}\)
c. 2 \(\frac{1}{2}\)
d. \(\frac{6}{12}\)

Answer: 2 \(\frac{2}{3}\)

Explanation:

Given:
Arthur baked 1 \(\frac{7}{12}\) dozen muffins. Nina baked 1 \(\frac{1}{12}\) dozen muffins.
Add both the fractions
Convert mixed fraction into normal fractions
1 \(\frac{7}{12}\) = \(\frac{19}{12}\)
1 \(\frac{1}{12}\) = \(\frac{13}{12}\)
\(\frac{19}{12}\) + \(\frac{13}{12}\) = \(\frac{32}{12}\)
= \(\frac{8}{3}\)
Convert \(\frac{8}{3}\) into mixed fraction = 2 \(\frac{2}{3}\)
So, the answer is option B.

Question 5.
Trisha drew the figure below. What figure did she draw?
Go Math Grade 4 Answer Key Chapter 11 Angles Common Core - New img 53
Options:
a. line segment ST
b. ray ST
c. ray TS
d. line TS

Answer: ray TS

A ray can be defined as a part of a line that has a fixed starting point but no endpoint.
Here the point starts from T and ends at S.
So, the figure Trisha drew is ray TS.
The correct answer is option C.

Question 6.
Which describes the turn shown by the angle?
Go Math Grade 4 Answer Key Chapter 11 Angles Common Core - New img 54
Options:
a. 1 full turn clockwise
b. \(\frac{3}{4 }\) turn clockwise
c. \(\frac{1}{2}\) turn clockwise
d. \(\frac{1}{4}\) turn clockwise

Answer: \(\frac{1}{4}\) turn clockwise

Explanation:

The figure shows that the point turned \(\frac{1}{4}\) in clockwise direction.
So, the answer is option D.

Page No. 619

Choose the best term from the box.
Go Math Grade 4 Answer Key Chapter 11 Angles img 55

Question 1.
The unit used to measure an angle is called a ________.
________

Answer: The unit used to measure an angle is called a degree.

Question 2.
________ is the opposite of the direction in which the hands of a clock move.
________

Answer: Counterclockwise is the opposite of the direction in which the hands of a clock move.

Question 3.
A ________ is a tool for measuring the size of an angle.
________

Answer: A protractor is a tool for measuring the size of an angle.

Tell whether the angle on the circle shows a \(\frac{1}{4}, \frac{1}{2}, \frac{3}{4}\), or 1 full turn clockwise or counterclockwise.

Question 4.
Go Math Grade 4 Answer Key Chapter 11 Angles img 56
\(\frac{□}{□}\)

Answer: \(\frac{1}{4}\) turn clockwise
The figure shows that the angle turn \(\frac{1}{4}\) in the clockwise direction.

Question 5.
Go Math Grade 4 Answer Key Chapter 11 Angles img 57
\(\frac{□}{□}\)

Answer: \(\frac{1}{2}\) turn counterclockwise
From the above figure, we can see that the angle turn \(\frac{1}{2}\) in the counterclockwise direction.

Question 6.
Go Math Grade 4 Answer Key Chapter 11 Angles img 58
\(\frac{□}{□}\)

Answer: \(\frac{3}{4}\) turn clockwise
The figure shows that the angle turn \(\frac{3}{4}\) in the clockwise direction.

Question 7.
Go Math Grade 4 Answer Key Chapter 11 Angles img 59
____

Answer: \(\frac{1}{1}\) or 1 turn counterclockwise
From the above figure, we can see that the angle turn \(\frac{1}{1}\) or 1 in the counterclockwise direction.

Tell the measure of the angle in degrees.

Question 8.
Go Math Grade 4 Answer Key Chapter 11 Angles img 60
____ °

Answer: 100°

\(\frac{100}{360}\) × 360° = 100°

Question 9.
____ °

Use a protractor to draw the angle.

Question 10.
75°
Type below:
________

HMH Go Math Key Chapter 11 Angles Image_6

Question 11.
127°
Type below:
________

Chapter 11 Go Math Grade 4 Answer Key Angles Image_7

Page No. 620

Question 12.
Phillip watched a beach volleyball game from 1:45 p.m. to 2:00 p.m. How many degrees did the minute hand turn?
Go Math Grade 4 Answer Key Chapter 11 Angles img 61
____ °

Answer: 90°

Explanation:

Phillip watched a beach volleyball game from 1:45 p.m. to 2:00 p.m.
The minute hand turned for 15 minutes.
That means \(\frac{1}{4}\) turn clockwise.
Complete angle = 360°
\(\frac{1}{4}\) × 360° = 360°/4 = 90°
Therefore the minute hand turn 90°

Question 13.
What angle does this piece of pie form?
Go Math Grade 4 Answer Key Chapter 11 Angles img 62
____ °

Answer: 180°

From the above figure, we can see that half of the pie is completed.
Complete angle = 360°
\(\frac{1}{2}\) × 360°
= 180°
The angle for the piece of pie form is 180°

Question 14.
What is m∠CBT? Use a protractor to help you.
Go Math Grade 4 Answer Key Chapter 11 Angles img 63
____ °

Answer: 60°

By using the protractor we can say that the angle for the above figure is 60°

Question 15.
Matt cut a circle into 8 equal sections. He drew an angle that measures the same as the total measure of 3 of the sections in the circle. What is the measure of the angle Matt drew?
____ °

Answer: 135°

Explanation:

Matt cut a circle into 8 equal sections.
He drew an angle that measures the same as the total measure of 3 of the sections in the circle.
Complete angle = 360°
Divide the total number of sections by 360°
\(\frac{360}{8}\) = 45°
So, the angle for each section is 45°
The angle for 3 of the sections in the circle = 3 × 45° = 135°
Thus the measure of the angle Matt drew is 135°

Page No. 623

Add to find the measure of the angle. Write an equation to record your work.

Question 1.
Go Math Grade 4 Answer Key Chapter 11 Angles img 64
∠PQT = ____ °

Answer: 80°

To find the ∠PQT you have to add 43° and 37°
∠PQT = 43° + 37°
∠PQT = 80°

Question 2.
Go Math Grade 4 Answer Key Chapter 11 Angles img 65
∠JKL = ____ °

Answer: 100°
Let ∠JKL = x°
∠JKL = 90° + 10°
∠JKL = 100°

Question 3.
Go Math Grade 4 Answer Key Chapter 11 Angles img 66
∠RHT = ____ °

Answer:
Let ∠RHT = x°
x = 55° + 27° + 78°
x = 160°
Therefore ∠RHT = 160°

Use a protractor to find the measure of each angle. Label each angle with its measure.
Write the sum of the angle measures as an equation.

Question 4.
Go Math Grade 4 Answer Key Chapter 11 Angles img 67
Type below:
________

Answer:

By using the protractor we can measure the angles of the above figures.
m∠KLM = 160°
m∠KLJ = 80°
m∠LMJ = 120°

Question 5.
Go Math Grade 4 Answer Key Chapter 11 Angles img 68
Type below:
________

Answer:

By using the protractor we can measure the angles of the above figures.

m∠WVZ = 90°
m∠YVZ = 90°
m∠WVX = 140°
m∠YVX = 40°

Question 6.
Use Diagrams What is m∠QRT?
Go Math Grade 4 Answer Key Chapter 11 Angles img 69
∠QRT = ____ °

Answer: 20°

The above figure is a straight angle.
∠QRT + ∠LRD + ∠RLT = 180
∠QRT + 75° + 85° = 180°
∠QRT + 160° = 180°
∠QRT = 180°- 160°
∠QRT = 20°

Question 7.
Look back at Exercise 1. Suppose you joined an angle measuring 10° to ∠PQT. Draw the new angle, showing all three parts. What type of angle is formed?
Type below:
________

Page No. 624

Question 8.
Stephanie, Kay, and Shane each ate an equal-sized piece of a pizza. The measure of the angle of each piece was 45°. When the pieces were together, what is the measure of the angle they formed?
Go Math Grade 4 Answer Key Chapter 11 Angles img 70
a. What are you asked to find?
Type below:
________

Answer: What is the measure of the angle for the pizza leftover?

Question 8.
b. What information do you need to use?
Type below:
________

Answer: I need the information about the angle for each piece of pizza.

Question 8.
c. Tell how you can use addition to solve the problem.
Type below:
________

Answer:
The measure of the angle of each piece was 45°
There are 3 pieces of pizza = 45° + 45° + 45° = 135°

Question 8.
d. Complete the sentence. The three pieces of pizza formed a _________ angle.
________

Answer: Obtuse angle

Question 9.
What is the measure of ∠XZW?
Go Math Grade 4 Answer Key Chapter 11 Angles img 71
____ °

Answer: 113°

Explanation:

∠XZW = ∠XZY + ∠YZW
∠XZY = 42°
∠YZW = 71°
∠XZW = 42° + 71°
∠XZW = 113°

Question 10.
What is m∠PRS? Use equations to explain and check your answer.
Go Math Grade 4 Answer Key Chapter 11 Angles img 72
____ °

Answer: 12°

Explanation:

The above figure is a straight angle.
The sum of the three angles must be equal to 180°
m∠PRS + m∠PRN + m∠TRN = 180°
m∠PRS + 90° + 78° = 180°
m∠PRS = 180° – 90° – 78°
m∠PRS = 12°

Common Core – New – Page No. 625

Join and Separate Angles

Add to find the measure of the angle. Write an equation to record your work.

Question 1.
Go Math Grade 4 Answer Key Chapter 11 Angles Common Core - New img 73
50°+75° = 125°
m∠ABD = 125°

Explanation:

m∠ABC = 50°
m∠CBD = 75°
To find the measure of m∠ABD we have to add m∠ABC and m∠CBD
m∠ABD = 50°+75°
m∠ABD = 125°

Question 2.
Go Math Grade 4 Answer Key Chapter 11 Angles Common Core - New img 74
____ ° + ____ ° = ____ ° ;   m∠FGJ = ____ °

Answer: 140° + 20° = 160°
m∠FGJ = 160°

Explanation:

m∠FGH = 140°
m∠JGH = 20°
To find the measure of m∠FGJ we need to add m∠FGH and m∠JGH
m∠FGJ = 140° + 20°
m∠FGJ = 160°

Question 3.
Go Math Grade 4 Answer Key Chapter 11 Angles Common Core - New img 75
____ ° + ____ ° = ____ ° ; m∠KLN = ____ °

Answer: 30° + 90° + 45° = 165°
m∠KLN = 165°

Explanation:

m∠KLM = 30°
m∠MLP = 90°
m∠PLN = 45°
To find the measure of m∠KLN we need to add m∠KLM, m∠MLP and m∠PLN
m∠KLN = 30° + 90° + 45° = 165°
m∠KLN = 165°

Use a protractor to find the measure of each angle in the circle.
Go Math Grade 4 Answer Key Chapter 11 Angles Common Core - New img 76

Question 4.
m∠ABC = ____ °

Answer: 115°

Question 5.
m∠DBE = ____ °

Answer: 90°

Question 6.
m∠CBD = ____ °

Answer: 75°

Question 7.
m∠EBA = ____ °

Answer: 80°

Question 8.
Write the sum of the angle measures as an equation.
____ ° + ____ ° + ____ ° + ____ ° = ____ °

Answer:

Sum all the angles = m∠DBE + m∠ABC + m∠CBD + m∠EBA
= 115° + 90° + 75° + 80° = 360°

Problem Solving
Go Math Grade 4 Answer Key Chapter 11 Angles Common Core - New img 77

Question 9.
Ned made the design at the right. Use a protractor. Find and write the measure of each of the 3 angles.
____ ° ; ____ ° ; ____ ° ;

Answer: 50°; 60°; 70°

The above figure is a straight angle.
By using the protractor we can measure the angles of the above figure.
The angle of above 3 shades is 50°; 60°; 70°

Question 10.
Write an equation to find the measure of the total angle.
____ ° + ____ ° + ____ ° = ____ °

Answer: Sum of three angles = 50° + 60° + 70° = 180°

Common Core – New – Page No. 626

Lesson Check

Question 1.
What is the measure of m∠WXZ?
Go Math Grade 4 Answer Key Chapter 11 Angles Common Core - New img 78
Options:
a. 32°
b. 83°
c. 88°
d. 97°

Answer: 83°

Explanation:

m∠WXZ = m∠WXY + m∠YXZ
Let m∠WXZ be x°
x° = 58° + 25°
x° = 83°
Thus the correct answer is option B.

Question 2.
Which equation can you use to find the m∠MNQ?
Go Math Grade 4 Answer Key Chapter 11 Angles Common Core - New img 79
Options:
a. 148° – 24° = ■
b. 148° × 24° = ■
c. 148° ÷ 24° = ■
d. 148° + 24° = ■

Answer: 148° + 24° = ■

Explanation:

To measure the unknown angle we need to add both the angles
m∠MNQ = m∠MNP + m∠PNQ
■ = 148° + 24°
So, the correct answer is option D.

Spiral Review

Question 3.
Joe bought 6 packages of envelopes. Each package contains 125 envelopes. How many envelopes did he buy?
Options:
a. 750
b. 723
c. 720
d. 650

Answer: 750

Explanation:

Joe bought 6 packages of envelopes.
Each package contains 125 envelopes.
To find the total number of envelopes in all 6 packages
We have to multiply number of packages and number of envelopes
= 125 × 6 =750
Thus Joe bought 750 envelopes.
The correct answer is option A.

Question 4.
The Lake Trail is \(\frac{3}{10}\) mile long and the Rock Trail is \(\frac{5}{10}\) long. Bill hiked each trail once. How many miles did he hike in all?
Options:
a. \(\frac{1}{5}\) mile
b. \(\frac{4}{10}\) mile
c. \(\frac{1}{2}\) mile
d. \(\frac{8}{10}\) mile

Answer: \(\frac{8}{10}\) mile

Explanation:

The Lake Trail is \(\frac{3}{10}\) mile long and the Rock Trail is \(\frac{5}{10}\) long. Bill hiked each trail once.
We need to both the fractions
\(\frac{3}{10}\) +\(\frac{5}{10}\)
The denominators are common. So add the numerators.
= \(\frac{8}{10}\)
Bill hiked \(\frac{8}{10}\) miles in all.
Thus the correct answer is option D.

Question 5.
Ron drew a quadrilateral with 4 right angles and 4 sides with the same length. Which best describes the figure he drew?
Options:
a. square
b. rhombus
c. trapezoid
d. parallelogram

Answer: square

Explanation:

A square has got 4 sides of equal length and 4 right angles (right angle = 90 degrees).
So, the answer is option A.

Question 6.
How many degrees are in an angle that turns through \(\frac{3}{4}\) of a circle?
Options:
a. 45°
b. 90°
c. 180°
d. 270°

Answer: 270°

Explanation:

Complete angle = 360°
To measure the angle that turns through is \(\frac{3}{4}\)
multiply \(\frac{3}{4}\) with 360°
360° × \(\frac{3}{4}\) = 270°
So, the answer is option D.

Page No. 629

Question 1.
Laura cuts a square out of scrap paper as shown. What is the angle measure of the piece left over?
First, draw a bar model to represent the problem.
Go Math Grade 4 Answer Key Chapter 11 Angles img 80

Type below:
_________

Go Math Grade 4 Chapter 11 Answer Key image_11

Question 1.
Next, write the equation you need to solve.
Type below:
_________

Answer:

m∠MNQ + m∠QNP = m∠MNP
x + 90° = 115°
x = 115° – 90°

Question 1.
Last, find the angle measure of the piece left over.
m∠MNQ =
So, the angle measure of the piece left over is _____.
____ °

Answer:
x + 90° = 115°
x = 115° – 90°
x = 25°
So, the angle measure of the piece left over is 25°

Question 2.
Jackie trimmed a piece of scrap metal to make a straight edge as shown. What is the measure of the piece she trimmed off?
Go Math Grade 4 Answer Key Chapter 11 Angles img 81
x = ____ °

Answer:
x + 180° = 225°
x = 225°- 180°
x = 45°
Thus the measure of the piece she trimmed off is 45°

Question 3.
What if Laura cut a smaller square as shown? Would m∠MNQ be different? Explain.
Go Math Grade 4 Answer Key Chapter 11 Angles img 82
Type below:
_________

Answer: No
m∠MNQ would still be 25°. Only the size of the square changed the angle will be the same.
m∠PNQ and m∠MNP did not change.

Question 4.
The map shows Marco’s paper route. When Marco turns right onto Center Street from Main Street, what degree turn does he make? Hint: Draw a dashed line to extend Oak Street to form a 180° angle.
Go Math Grade 4 Answer Key Chapter 11 Angles img 83

Answer:

x° + 125° + 180° = 360°
x° = 360° – 125° – 180°
x° = 360° – 215°
x° = 145°

Page No. 630

Question 5.
Write an Equation Two angles form a straight angle. One angle measures 89°. What is the measure of the other angle? Explain.
____ °

Answer: 91°

A straight angle measures 180°, so you can subtract 89° from 180°
180° – 89° = 91°

Question 6.
Pose a Problem Look back at Problem 5. Write a similar problem about two angles that form a right angle.
____ °

Answer: Two angles form a right angle. The measure of one angle is 25°. What is the measure of the other angle?
x + 25° = 90°
x °= 90° – 25°
x° = 65°
The measure of other angle is 65°

Question 7.
Sam paid $20 for two T-shirts. The price of each T-shirt was a multiple of 5. What are the possible prices of the T-shirts?
Type below:
_________

Answer:
Sam paid $20 for two T-shirts.
The price of each T-shirt was a multiple of 5.
$20 – 2 T-shirts
x – 1 T-shirt
x = $10
The possible prices of the T-shirts are $10, $10
Another possible price of the T-shirts are $5, $15

Question 8.
Zayna has 3 boxes with 15 art books in each box. She has 2 bags with 11 math books in each bag. If she gives 30 books away, how many art and math books does she have left?
_____ books

Answer: 37 art and math books

Explanation:

Zayna has 3 boxes with 15 art books in each box = 15 × 3 = 45
She has 2 bags with 11 math books in each bag = 11 × 2 = 22
Total number of books = 45 + 22 = 67
If she gives 30 books away, then we have to subtract 30 from 67
67 – 30 = 37
37 art and math books are left.

Question 9.
What’s the Question? It measures greater than 0° and less than 90°.
Type below:
_________

Answer: What is an acute angle?

Question 10.
Two angles, ∠A and ∠B, form a straight angle. ∠A measures 65°. For numbers 10a–10c, select True or False for the statement.
a. ∠B is an acute angle.
i. True
ii. False

Answer: False

Explanation:

Two angles, ∠A and ∠B, form a straight angle. ∠A measures 65°.
65° + ∠B = 180°
∠B = 180° – 65°
∠B = 115°
115° is not an acute angle.
So, the above statement is false.

Question 10.
b. The equation 180° – 65° = x° can be used to find the measure of ∠B.
i. True
ii. False

Answer: True

Question 10.
c. The measure of ∠B is 125°.
i. True
ii. False

Answer: False

65° + ∠B = 180°
∠B = 180° – 65°
∠B = 115°
So, the above statement is false.

Common Core – New – Page No. 631

Problem Solving Unknown Angle Measures

Solve each problem. Draw a diagram to help.

Question 1.
Go Math Grade 4 Answer Key Chapter 11 Angles Common Core - New img 84
Go Math Grade 4 Answer Key Chapter 11 Angles Common Core - New img 85

Question 2.
An artist is cutting a piece of metal as shown. What is the angle measure of the piece left over?
Go Math Grade 4 Answer Key Chapter 11 Angles Common Core - New img 86
x = ____ °

Answer: 95°

x + 130° = 225°
x = 225° – 130°
x = 95°
Therefore the angle of the piece leftover is 95°.

Question 3.
Joan has a piece of material for making a costume. She needs to cut it as shown. What is the angle measure of the piece left over?
Go Math Grade 4 Answer Key Chapter 11 Angles Common Core - New img 87
x = ____ °

Answer: 50°

Joan has a piece of material for making a costume. She needs to cut it as shown.
By seeing the above figure we can say that it is a right angle.|
The sum of two must be equal to 90°
Let the unknown angle be x
x + 40° = 90°
x = 90° – 40°
x = 50°
Thue the angle measure of the piece leftover is 50°

Common Core – New – Page No. 632

Lesson Check

Question 1.
Angelo cuts a triangle from a sheet of paper as shown. What is the measure of ∠x in the triangle?
Go Math Grade 4 Answer Key Chapter 11 Angles Common Core - New img 88
Options:
a. 15°
b. 25°
c. 75°
d. 105°

Answer: 15°

Explanation:

The above figure is a right angle.
So, to measure the ∠x we have to subtract 75° from 90°
∠x + 75° = 90°
∠x = 90° – 75°
∠x = 15°
Thus the correct answer is option A.

Question 2.
Cindy cuts a piece of wood as shown. What is the angle measure of the piece left over?
Go Math Grade 4 Answer Key Chapter 11 Angles Common Core - New img 89
Options:
a. 30°
b. 90°
c. 120°
d. 150°

Answer: 120°

Explanation:

x + 90° = 210°
x = 210° – 90°
x = 120°
120° is the measure of the piece leftover.
So, the correct answer is option C.

Spiral Review

Question 3.
Tyronne worked 21 days last month. He earned $79 each day. How much did Tyronne earn last month?
Options:
a. $869
b. $948
c. $1,659
d. $2,169

Answer: $1,659

Explanation:

Tyronne worked 21 days last month.
He earned $79 each day.
$79 × 21 = 1659
Thus Tyronne earned $1,659 last month.
So, the correct answer is option C.

Question 4.
Meg inline skated for \(\frac{7}{10}\) mile. Which shows this distance written as a decimal?
Options:
a. 0.07 mile
b. 0.1 mile
c. 0.7 mile
d. 7.1 miles

Answer: 0.7 mile

Explanation:

Meg inline skated for \(\frac{7}{10}\) mile.
The decimal of the fraction \(\frac{7}{10}\) is 0.7
So, the answer is option C.

Question 5.
Kerry ran 34 mile. Sherrie ran \(\frac{1}{2}\) mile. Marcie ran \(\frac{2}{3}\) mile. Which list orders the friends from least to greatest distance
run?
Options:
a. Kerry, Sherrie, Marcie
b. Kerry, Marcie, Sherrie
c. Sherrie, Kerry, Marcie
d. Sherrie, Marcie, Kerry

Answer: Sherrie, Marcie, Kerry

Explanation:

Kerry ran 34 miles. Sherrie ran \(\frac{1}{2}\) mile. Marcie ran \(\frac{2}{3}\) mile.
The order of the above fractions is Sherrie ran \(\frac{1}{2}\), \(\frac{2}{3}\), 34
The distance from least to greatest is Sherrie, Marcie, Kerry.
so, the correct answer is option D.

Question 6.
What is the measure of m∠ABC?
Go Math Grade 4 Answer Key Chapter 11 Angles Common Core - New img 90
Options:
a. 32°
b. 84°
c. 88°
d. 94°

Answer: 84°

Explanation:

m∠ABC = m∠ABD + m∠DBC
m∠ABC = 58° + 26°
m∠ABC = 84°
So, the correct answer is option B.

Page No. 633

Question 1.
An angle represents \(\frac{1}{12}\) of a circle. Use the numbers to show how to find the measure of the angle in degrees.
Go Math Grade 4 Answer Key Chapter 11 Angles img 91
Go Math Grade 4 Answer Key Chapter 11 Angles img 92
Go Math Grade 4 Answer Key Chapter 11 Angles img 93
The angle measure is ____ °

Answer: 30°
\(\frac{1}{12}\) × \(\frac{30}{30}\) = \(\frac{30}{360}\)
Thus the angle measure is 30°

Question 2.
Match the measure of each ∠C with the measure of ∠D that forms a straight angle.
Go Math Grade 4 Answer Key Chapter 11 Angles img 94
Type below:
_________

i. 122° + 58° = 180°
ii. 35° + 145° = 180°
iii. 62° + 118° = 180°
iv. 105° + 75° = 180°

Question 3.
Katie drew an obtuse angle. Which could be the measure of the angle she drew? Mark all that apply.
Options:
a. 35°
b. 157°
c. 180°
d. 92°

Answer: 157° and 92°
An obtuse angle has a measurement greater than 90 degrees but less than 180 degrees.
From the above options, B and D are more than 90°
So, the answer is option B and D.

Question 4.
Draw an angle that represents a \(\frac{1}{4}\) turn counterclockwise on the circle.
Go Math Grade 4 Answer Key Chapter 11 Angles img 95
Type below:
_________

Go Math Answer Key Grade 4 Chapter 11 solution image_8

Page No. 634

Question 5.
Renee drew the figure shown. For 5a–5c, select Yes or No to tell whether the statement is true.
Go Math Grade 4 Answer Key Chapter 11 Angles img 96
a. The measure of a straight angle is 180°.
i. yes
ii. no

Answer: Yes

By seeing the above figure we can say that the angle is a straight angle.
So, the above statement is true.

Question 5.
b. To find the measure of x, Renee can subtract 75° from 180°.
i. yes
ii. no

Answer: Yes

To know the value of x we have to subtract 75° from 180°.
x = 180° – 75°
Thus the above statement is true.

Question 5.
c. The measure of x is 115°.
i. yes
ii. no

Answer: No
x = 180° – 75°
x = 105°
Thus the above statement is false.
So, the answer is no.

Question 6.
Trey drew this figure with a protractor.
Go Math Grade 4 Answer Key Chapter 11 Angles img 97
Part A
Write an equation that can be used to find m∠KFG.
Type below:
_________

Answer: 55° + 80° + x = 180°

The figure is a straight angle.
So, the sum of the three angles must be equal to 180°
Let m∠KFG = x
55° + 80° + x = 180°

Question 6.
Part B
What is the measure of m∠KFG? Describe how you solved the equation and how you can check your answer.
____ °
Explain:
_________

Answer: 45°

Explanation:

55° + 80° + x = 180°
x = 180° – 80° – 55°
x = 45°

Question 7.
Use a protractor to find the measure of the angle.
Go Math Grade 4 Answer Key Chapter 11 Angles img 98
The angle measures ____ °

Answer: 40°
By using a protractor we can measure the angle.
The angle of the above figure is 40°

Page No. 635

Question 8.
Alex drew this angle on the circle. Which describes the angle? Mark all that apply.
Go Math Grade 4 Answer Key Chapter 11 Angles img 99
Options:
a. \(\frac{1}{4}\) turn
b. clockwise
c. \(\frac{1}{2}\) turn
d. counterclockwise

Answer: \(\frac{1}{2}\) turn

The above figure shows that it is straight angle. So, the fraction of the circle is \(\frac{1}{2}\) turn.
The correct answer is option C.

Question 9.
Miles has a piece of paper that is \(\frac{1}{4}\) of a large circle. He cuts the paper into three equal parts from the center point of the circle. What is the angle measure of each part?
Go Math Grade 4 Answer Key Chapter 11 Angles img 100
The angle measure is ____ °

Answer: 30°

Explanation:

Miles has a piece of paper that is \(\frac{1}{4}\) of a large circle. He cuts the paper into three equal parts from the center point of the circle.
\(\frac{1}{4}\) of a large circle = 90°
Given that he cut into 3 equal parts = \(\frac{90}{3}\) = 30°
So, the angle for each part is 30°

Question 10.
Use a protractor to find the measure of each angle. Write each angle and its measure in a box ordered by the measure of the angles from least to greatest.
Go Math Grade 4 Answer Key Chapter 11 Angles img 101
Go Math Grade 4 Answer Key Chapter 11 Angles img 102

Answer:

Go-Math-Grade-4-Answer-Key-Chapter-11-Angles-solution-img-9

Question 11.
Use the numbers and symbols to write an equation that can be used to find the measure of the unknown angle.
Go Math Grade 4 Answer Key Chapter 11 Angles img 103
What is the measure of the unknown angle?
____ °

Answer: 57°

Explanation:

Let the unknown angle be x
It is a straight angle.
The sum of three angles = 180°
90° + 33° + x = 180°
x = 180° – 90° – 33°
x = 57°

Page No. 636

Question 12.
Choose the word or number to complete a true statement about m∠JKL.
Go Math Grade 4 Answer Key Chapter 11 Angles img 104
Go Math Grade 4 Answer Key Chapter 11 Angles img 105
m∠JKL is a(n) ______ angle that has a measure of ____ °.

Answer: m∠JKL is an Obtuse angle that has a measure of 120°.

Question 13.
Vince began practicing piano at 5:15 p.m. He stopped at 5:35 p.m. How many degrees did the minute hand turn during Vince’s practice time?
Explain how you found your answer.
Go Math Grade 4 Answer Key Chapter 11 Angles img 106
____ °
Explain:
_________

Answer: 120°

I shaded the part of the clock that the minute hand turned from 5:15 p.m. to 5:35 p.m. anmd found that it is \(\frac{1}{3}\) of the circle.
Next, I multiplied \(\frac{1}{3}\) × 360° = 120°
Thus the minute hand moved 120°

Question 14.
An angle measures 125°. Through what fraction of a circle does the angle turn?
\(\frac{□}{□}\) of a circle

Answer: \(\frac{125}{360}\)

The fraction of the circle the angle turned is \(\frac{125}{360}\)

Question 15.
Write the letter for each angle measure in the correct box.
Go Math Grade 4 Answer Key Chapter 11 Angles img 107
Type below:
__________

Answer:

Go-Math-Grade-4-Answer-Key-Chapter-11-Angles-solution-img-10

Page No. 637

Question 16.
For numbers 16a–16b, select the fraction that makes a true statement about the figure.
Go Math Grade 4 Answer Key Chapter 11 Angles img 108

Question 16.
a. The angle in Figure 1 represents a Go Math Grade 4 Answer Key Chapter 11 Angles img 109 turn.
\(\frac{□}{□}\) turn

Answer: The angle in Figure 1 represents a \(\frac{3}{4}\) turn

The above figure shows that \(\frac{3}{4}\) part of the circle is shaded. So, the angle represents \(\frac{3}{4}\) turn.

Question 16.
b. The angle in Figure 2 represents a Go Math Grade 4 Answer Key Chapter 11 Angles img 110 turn.
\(\frac{□}{□}\) turn

Answer: The angle in Figure 2 represents a \(\frac{1}{2}\) turn.
From the second figure, we observe that half of the circle is shaded. So, The angle in Figure 2 represents a \(\frac{1}{2}\) turn.

Question 17.
Melanie cuts a rectangle out of a piece of scrap paper as shown. She wants to calculate the angle measure of the piece that is left over.
Go Math Grade 4 Answer Key Chapter 11 Angles img 111
Part A
Draw a bar model to represent the problem.

Go Math Grade 4 Chapter 11 Answer Key Angles Image_7

Question 17.
Part B
Write and solve an equation to find x.
The angle measures ____ °.

Answer: 36°
m∠RST = 90°
m∠RSN = 126°
m∠TSN = x°
x + 90° = 126°
x = 126° – 90°
x = 36°
m∠TSN = 36°

Page No. 638

Question 18.
Two angles, m∠A and m∠B, form a right angle. m∠A measures 32°.
For numbers, 18a–18c, select True or False for the statement.
a. m∠B is an acute angle.
i. True
ii. False

Answer: True

If the sum of two angles is 90°, if one angle is acute then the other angle will be acute.
So, the above statement is true.

Question 18.
b. The equation 180° − 32° = x° can be used to find the measure of m∠B.
i. True
ii. False

Answer: False

Explanation:

Given that the sum of 2 angles is 90°
The sum of m∠A and m∠B = 90°
90° – 32° = x°
So, the above statement is false.

Question 18.
c. The measure of m∠B is 58°.
i. True
ii. False

Answer: True

Explanation:

Let m∠B = x
x° + 32° = 90°
x = 90 – 32
x = 58°.
So, the above statement is true.

Question 19.
A circle is divided into parts. Which sum could represent the angle measures that make up the circle? Mark all that apply.
Options:
a. 120° + 120° + 120° + 120°
b. 25° + 40° + 80° + 105° + 110°
c. 33° + 82° + 111° + 50° + 84°
d. 40° + 53° + 72° + 81° + 90° + 34°

Answer: 25° + 40° + 80° + 105° + 110°; 33° + 82° + 111° + 50° + 84°

Explanation:

The sum of all the angles must be equal to 360°
i. 120° + 120° + 120° + 120° = 480° ≠ 360°
ii. 25° + 40° + 80° + 105° + 110° = 360°
iii. 33° + 82° + 111° + 50° + 84° = 360°
iv. 40° + 53° + 72° + 81° + 90° + 34° = 370° ≠ 360°
So, the correct answers are option B, C.

Question 20.
Use a protractor to find the measures of the unknown angles.
Go Math Grade 4 Answer Key Chapter 11 Angles img 112
What do you notice about the measures of the unknown angles? Is this what you would have expected? Explain your reasoning.
m∠x = ____ ° m∠y = ____ °

Answer: m∠x = 70°; m∠y = 110°
By using a protractor we can find the measure of m∠y
m∠y = 110°
Let m∠x = x°
Sum of supplementary angles = 180°
110° + x = 180°
x = 180° – 110°
x = 70°
Therefore m∠x = 70°

Page No. 643

Use benchmarks to choose the metric unit you would use to measure each.
Go Math Grade 4 Answer Key Chapter 11 Angles img 113

Question 1.
mass of a strawberry
__________

Answer: gram
The metric unit used to measure the mass of a strawberry is the gram.

Question 2.
length of a cell phone
__________

Answer: Centimeter
The metric unit used to measure the length of a cell phone is Centimeter.

Circle the better estimate.

Question 3.
width of a teacher’s desk
10 meters or 1 meter
__________

Answer: 1 meter
The estimation of the width of the teacher’s desk is 1 meter.

Question 4.
the amount of liquid a punch bowl holds
2 liters or 20 liters
__________

Answer: 2 liters

20 liters is greater than 2 liters.
The estimation of the amount of liquid a punch bowl holds is 2 liters.

Question 5.
distance between Seattle and San Francisco
6 miles or 680 miles
__________

Answer: 680 miles
The distance between Seattle and San Francisco is 680 miles.

Use benchmarks to choose the customary unit you would use to measure each.
Go Math Grade 4 Answer Key Chapter 11 Angles img 114

Question 6.
length of a football field
__________

Answer: Yard
The units to measure the length of a football field is Yards.

Question 7.
weight of a pumpkin
__________

Answer: Pound
The customary unit I use to measure the weight of a pumpkin is pounds.

Circle the better estimate.

Question 8.
weight of a watermelon
4 pounds or 4 ounces
__________

Answer: 4 pounds
The estimation of the weight of watermelon is 4 pounds.

Question 9.
the amount of liquid a fish tank holds
10 cups or 10 gallons
__________

Answer: 10 gallons
The estimation of the amount of liquid a fish tank holds is 10 gallons.

Complete the sentence. Write more or less.

Question 10.
Matthew’s large dog weighs ________ than one ton.
________

Answer: Less
1 ton = 1000 kgs
The weight of dogs can’t be more than a ton.
So, Matthew’s large dog weighs less than one ton.

Question 11.
The amount of liquid a sink can hold is _______ than one cup of water.
________

Answer: More
1 cup holds very less amount of water.
So, The amount of liquid a sink can hold is more than one cup of water.

Question 12.
A paper clip has a mass of _______ than one kilogram.
________

Answer: Less

The weight of a paper clip is about 1 gram.
So, A paper clip has a mass of less than one kilogram.

Page No. 644

For 13–15, use benchmarks to explain your answer.
Go Math Grade 4 Answer Key Chapter 11 Angles img 115

Question 13.
Cristina is making macaroni and cheese for her family. Would Cristina use 1 pound of macaroni or 1 ounce of macaroni?
__________

Answer: Cristina should use 1 pound of macaroni.

Question 14.
Which is the better estimate for the length of a kitchen table, 200 centimeters or 200 meters?
__________

Answer: 200 centimeters

Centimeters are less than meters. The length of the kitchen will be measured in centimeters.
So, the answer is 200 centimeters.

Question 15.
Jodi wants to weigh her cat and measure its standing height. Which two units should she use?
weight: ________
height: ________

Answer:
The weight of the cat should be measured in Kilograms.
The height of the cat should be measured in Centimeters.

Question 16.
Evaluate Reasonableness Dalton used benchmarks to estimate that there are more cups than quarts in one gallon. Is Dalton’s estimate reasonable? Explain.
Type below:
__________

Answer: Dalton’s reasoning is correct because the measurement of cups is smaller than the measurement of quarts, therefore there would be more cups in a gallon than quarts.

Question 17.
Select the correct word to complete the sentence. Justine is thirsty after running two miles.
She should Go Math Grade 4 Answer Key Chapter 11 Angles img 116 of water.
__________

Answer: 1 pint

The suitable word for the above sentence is the pint. Pint is a measure for liquid equal to about half a liter. There are eight pints in a gallon.

Conclusion

Hoping that the data provided here has shed some light on the students of grade 4. If teachers & parents want to make their kids learn all angles concepts properly then make sure they refer to these Go Math Grade 4 Chapter 11 Angles Answer Key. Need any assistance then check our other articles such as HMH Go Math 4th Grade Chapter 11 Homework Practice FL Angles.

Go Math Grade 6 Answer Key Chapter 7 Exponents

go-math-grade-6-chapter-7-exponents-answer-key

The solutions of Grade 6 Go Math Answer Key for Chapter 7 Exponents are available in simple PDFs here. With the help off the HMH Go Math Grade 6 Chapter 7 Exponents Answer Ley can be easily downloaded by the students by using the provided links. You can understand the concept of the standard form in this article. So, Download a free pdf of Go Math Grade 6 Answer Key Chapter 7 Exponents.

Go Math Grade 6 Answer Key Chapter 7 Exponents

Our main aim is to provide a brief explanation of all the questions. We have provided the table of contents of chapter 7 Exponents in the below section. So, once go through the topics before you start your preparation. This will help you to know in which topic you are lagging. Hence make use of the resources provided on this page and try to score good marks in the exams. After your preparation we suggest the students to test your skills by solving the questions in the mid-chapter checkpoint and review test.

Lesson 1: Exponents

Lesson 2: Evaluate Expressions Involving Exponents

Lesson 3: Write Algebraic Expressions

Lesson 4: Identify Parts of Expressions

Lesson 5: Evaluate Algebraic Expressions and Formulas

Mid-Chapter Checkpoint

Lesson 6: Use Algebraic Expressions

Lesson 7: Problem Solving • Combine Like Terms

Lesson 8: Generate Equivalent Expressions

Lesson 9: Identify Equivalent Expressions

Chapter 7 Review/Test

Share and Show – Page No. 359

Question 1.
Write 24 by using repeated multiplication. Then find the value of 24.
___________

Answer: 16

Explanation:
The repeated factor is 2
The number 2 is repeated 4 times.
The repeated multiplication of 24 is 2 × 2 × 2 × 2 = 16
Thus the value of 24 is 16.

Use one or more exponents to write the expression.

Question 2.
7 × 7 × 7 × 7
Type below:
_____________

Answer: 74

Explanation:
The repeated factor is 7.
7 is repeated four times.
The exponent of the repeated multiplication 7 × 7 × 7 × 7 is 74

Question 3.
5 × 5 × 5 × 5 × 5
Type below:
_____________

Answer: 55

Explanation:
The repeated factor is 5. The number 5 is repeated five times.
The exponent of the repeated multiplication 5 × 5 × 5 × 5 × 5 is 55

Question 4.
3 × 3 × 4 × 4
Type below:
_____________

Answer: 32 × 42

Explanation:
The exponent of the repeated multiplication 3 × 3 is 32
The exponent of the repeated multiplication 4 × 4 is 42
Thus the exponent for 3 × 3 × 4 × 4 is 32 × 42

On Your Own

Find the value.

Question 5.
202
______

Answer: 20 × 20 = 400

Explanation:
The repeated factor is 20
Write the factor 2 times.
20 × 20 = 400
The value of 202 = 400

Question 6.
821
______

Answer: 82

Explanation:
The repeated factor is 82
Write the factor 1 time.
The value of 821 is 82

Question 7.
35

Answer: 3 × 3 × 3 × 3 × 3 = 243

Explanation:
The repeated factor is 3
Write the factor 5 times.
The value of 35 is 343

Question 8.
Write 32 as a number with an exponent by using 2 as the base.
Type below:
_____________

Answer: 25

Explanation:
The exponent of 32 by using the base 2 is 2 × 2 × 2 × 2 × 2  = 25

Complete the statement with the correct exponent.

Question 9.
5? = 125
______

Answer: 53

Explanation:
The exponential form of 125 is 5 × 5 × 5 = 53
5? = 125
5? = 53
When bases are equal powers should be equated.
Thus the exponent is 3

Question 10.
16? = 16
______

Answer: 1

Explanation:
The exponential form of 16 is 161
16? = 161
When bases are equal powers should be equated.
Thus the exponent is 1.

Question 11.
30? = 900
______

Answer: 2

Explanation:
The exponential form of 900 is 30 × 30 = 302
30? = 302
When bases are equal powers should be equated.
Thus the exponent is 2.

Question 12.
Use Repeated Reasoning Find the values of 41, 42, 43, 44, and 45. Look for a pattern in your results and use it to predict the ones digit in the value of 46.
Type below:
_____________

Answer:
The value of 41 is 4.
The value of 42 is 4 × 4 = 16
The value of 43 is 4 × 4 × 4 = 64
The value of 44 is 4 × 4 × 4 × 4 = 256
The value of 45 is 4 × 4 × 4 × 4 × 4 = 1024
The value of 46 is 4 × 4 × 4 × 4 × 4 × 4 = 4096

Question 13.
Select the expressions that are equivalent to 32. Mark all that apply.
Options:
a. 25
b. 84
c. 23 × 4
d. 2 × 4 × 4

Answer: 25

Explanation:
The exponent of 32 by using the base 2 is 2 × 2 × 2 × 2 × 2  = 25
32 = 25
Thus the correct answer is option A.

Bacterial Growth – Page No. 360

Bacteria are tiny, one-celled organisms that live almost everywhere on Earth. Although some bacteria cause disease, other bacteria are helpful to humans, other animals, and plants. For example, bacteria are needed to make yogurt and many types of cheese.

Under ideal conditions, a certain type of bacterium cell grows larger and then splits into 2 “daughter” cells. After 20 minutes, the daughter cells split, resulting in 4 cells. This splitting can happen again and again as long as conditions remain ideal.

Complete the table.
Go Math Grade 6 Answer Key Chapter 7 Exponents img 1
Extend the pattern in the table above to answer 14 and 15.

Question 14.
What power of 2 shows the number of cells after 3 hours? How many cells are there after 3 hours?
Type below:
_____________

Answer: 29

Explanation:
So, each cell doubles every 20 mins. After 20 minutes, you have 1(2) = 2 cells. After 40 minutes, you have 2(2) = 4 cells, etc.
1 hour = 60 minutes
3 hours = 3 × 60 minutes = 180 minutes
180/20 = 9 divisions
Thus 29 cells are there after 3 hours.

Question 15.
How many minutes would it take to have a total of 4,096 cells?
_______ minutes

Answer: 240 minutes

Explanation:
First, convert the cells into the exponential form.
The exponential form of 4096 is 2 × 2 × 2 × 2 × 2 × 2× 2 × 2× 2 × 2× 2 × 2 = 212
Multiply the power with 20
12 × 20 = 240
Thus it would take 240 minutes to have a total of 4,096 cells

Exponents – Page No. 361

Use one or more exponents to write the expression.

Question 1.
6 × 6
Type below:
_____________

Answer:
The number 6 is used as a repeated factor.
6 is used as a factor 2 times.
Now write the base and exponent for 6 × 6 = 62

Question 2.
11 × 11 × 11 × 11
Type below:
_____________

Answer:
The number 11 is used as a repeated factor.
11 is used as a factor 4 times.
Now write the base and exponent for 11 × 11 × 11 × 11 = 114

Question 3.
9 × 9 × 9 × 9 × 7 × 7
Type below:
_____________

Answer:
The number 9 and 7 is used as a repeated factor.
9 is used as a factor 4 times and 7 is used 2 times.
Now write the base and exponent for 9 × 9 × 9 × 9 × 7 × 7 = 94 × 72

Question 4.
64
_______

Answer:
The repeated factor is 6.
Write the factor 4 times.
The value of 64 is 6 × 6 × 6 × 6 = 1296

Question 5.
16
_______

Answer:
The repeated factor is 1.
Write the factor 6 times.
The value of 16 is 1 × 1 × 1 × 1 × 1 × 1 = 1

Question 6.
105
_______

Answer:
The repeated factor is 10.
Write the factor 5 times.
The value of 105 is 10 × 10 × 10 × 10 × 10 = 1,00,000

Question 7.
Write 144 with an exponent by using 12 as the base.
Type below:
_____________

Answer: 12 × 12 = 122
The exponential form of 144 is 12 × 12 = 122

Question 8.
Write 343 with an exponent by using 7 as the base.
Type below:
_____________

Answer: The exponential form of 343 is 7 × 7 × 7 = 73

Question 9.
Each day Sheila doubles the number of push-ups she did the day before. On the fifth day, she does 2 × 2 × 2 × 2 × 2 push-ups. Use an exponent to write the number of push-ups Shelia does on the fifth day.
Type below:
_____________

Answer:
The number 2 is the repeated factor.
2 is repeated 5 times.
The exponential form of 2 × 2 × 2 × 2 × 2 is 25

Question 10.
The city of Beijing has a population of more than 107 people. Write 107 without using an exponent.
_______

Answer:
The repeated factor is 10.
Write the factor 7 times.
The value of 107 is 10 × 10 × 10 × 10 × 10 × 10 × 10 = 10,000,000

Question 11.
Explain what the expression 45 means and how to find its value.
Type below:
_____________

Answer:
The repeated factor is 4.
Write the factor 5 times.
The value of 45 is 4 × 4 × 4 × 4 × 4 = 1024

Lesson Check – Page No. 362

Question 1.
The number of games in the first round of a chess tournament is equal to 2 × 2 × 2 × 2 × 2 × 2. Write the number of games using an exponent.
Type below:
_____________

Answer: 26

Explanation:

The number 2 is the repeated factor.
2 is repeated 6 times.
2 × 2 × 2 × 2 × 2 × 2 = 26

Question 2.
The number of gallons of water in a tank at an aquarium is equal to 83. How many gallons of water are in the tank?
_______ gallons

Answer: 512 gallons

Explanation:
The repeated factor is 8.
Write the factor 3 times.
The value of 83 is 8 × 8 × 8 = 512 gallons
Therefore there are 512 gallons of water in the tank.

Spiral Review

Question 3.
The table shows the amounts of strawberry juice and lemonade needed to make different amounts of strawberry lemonade. Name another ratio of strawberry juice to lemonade that is equivalent to the ratios in the table.
Go Math Grade 6 Answer Key Chapter 7 Exponents img 2
Type below:
_____________

Answer: 5 : 15

Explanation:
By using the above table we can find the ratio of strawberry juice to lemonade.
2 : 6 = 1 : 3
The ratio of strawberry juice to lemonade next to 4 : 12 is 5 : 15

Question 4.
Which percent is equivalent to the fraction \(\frac{37}{50}\)?
_______ %

Answer: 74%

Explanation:
\(\frac{37}{50}\) × 100
0.74 × 100 = 74
Thus 74% is equivalent to the fraction \(\frac{37}{50}\)

Question 5.
How many milliliters are equivalent to 2.7 liters?
_______ milliliters

Answer: 2700 milliliters

Explanation:
Convert from liters to milliliters.
1 liter = 1000 milliliters
2.7 liters = 2.7 × 1000 milliliters = 2700 milliliters
2.7 liters is equivalent to 2700 milliliters.

Question 6.
Use the formula d = rt to find the distance traveled by a car driving at an average speed of 50 miles per hour for 4.5 hours.
_______ miles

Answer: 225 miles

Explanation:
Given,
r = 50 miles/hour
t = 4.5 hours
Use the formula d = rt
d = 50 × 4.5 = 225 miles
Thus the distance traveled by a car driving at an average speed of 50 miles per hour for 4.5 hours is 225 miles.

Share and Show – Page No. 365

Question 1.
Evaluate the expression 9 + (52 − 10)
_______

Answer: 24

Explanation:
First write the square for 52
52 is 25
Now simplify the expression 9 + (25 – 10)
9 + 15 = 24
So, 9 + (52 − 10) = 24

Evaluate the expression.

Question 2.
6 + 33 ÷ 9
_______

Answer: 9

Explanation:
6 + 33 ÷ 9
6 + (33 ÷ 9)
Write the factor for 33
33 = 3 × 3 × 3 = 27
6 + (27 ÷ 9)
27 ÷ 9 = 3
6 + 3 = 9
Thus 6 + 33 ÷ 9 = 9

Question 3.
(15 − 3)2 ÷ 9
_______

Answer: 16

Explanation:
First subtract 15 – 3 = 12
(12)2 ÷ 9
(12)2 = 12 × 12 = 144
144 ÷ 9
9 divides 144 16 times.
144 ÷ 9 = 16
Thus (15 − 3)2 ÷ 9 = 16

Question 4.
(8 + 92) − 4 × 10
_______

Answer: 49

Explanation:
First multiply 9 × 9 = 81
(8 + 81) – (4 × 10)
Multiply 4 and 10.
4 × 10 = 40
(8 + 81) – (40)
89 – 40 = 49
(8 + 92) − 4 × 10 = 49

On Your Own

Evaluate the expression

Question 5.
10 + 62 × 2 ÷ 9
_______

Answer: 18

Explanation:
10 + (62 × 2) ÷ 9
Multipley 6 × 6 = 36
10 + (36 × 2) ÷ 9
Multiply 36 and 2 and then divide by 9.
10 + (72 ÷ 9)
10 + 8 = 18
So, 10 + 62 × 2 ÷ 9 = 18

Question 6.
62 − (23 + 5)
_______

Answer: 23

Explanation:

The value of 62 is 6 × 6 = 36
The value of 23 is 2 × 2 × 2 = 8
36 – (8 + 5)
36 – 13 = 23
Thus the answer for the expression for 62 − (23 + 5) is 23.

Question 7.
16 + 18 ÷ 9 + 34
_______

Answer: 99

Explanation:
16 + (18 ÷ 9) + 34
First divide 18 by 9
16 + 2 + 34
18 + 34
The value of 34 is 3 × 3 × 3 × 3 = 81
18 + 81 = 99
Thus the answer for the expression 16 + (18 ÷ 9) + 34 is 99.

Place parentheses in the expression so that it equals the given value.

Question 8.
102 − 50 ÷ 5
value: 10
Type below:
_____________

Answer: 10

Explanation:
102 − 50 ÷ 5
The factor of 102 is 10 × 10 = 100
(102 − 50) ÷ 5
50 ÷ 5 = 10
102 − 50 ÷ 5 = 10
The value of 102 − 50 ÷ 5 = 10

Question 9.
20 + 2 × 5 + 41
value: 38
Type below:
_____________

Answer: 38

Explanation:
20 + 2 × 5 + 41
The value of 41 is 4.
20 + 2 × (5 + 4)
20 + 2 × 9
Now multiply 2 and 9.
20 + 18 = 38
The value of 20 + 2 × 5 + 41 = 38

Question 10.
28 ÷ 22 + 3
value: 4
Type below:
_____________

Answer: 4

Explanation:
28 ÷ 22 + 3
28 ÷ (22 + 3)
The value of 22 is 4
28 ÷ (4 + 3)
28 ÷ 7 = 4
The value of 28 ÷ 22 + 3 is 4.

Problem Solving + Applications – Page No. 366

Use the table for 11–13.
Go Math Grade 6 Answer Key Chapter 7 Exponents img 3

Question 11.
Write an Expression To find the cost of a window, multiply its area in square feet by the price per square foot. Write and evaluate an expression to find the cost of a knot window
$ _______

Answer: 108

Explanation:
To find the cost of the knot window multiply the area with the price per square foot.
Area per square feet is 22
Price per square foot is $27
Cost = 22 × 27 = 4 × 27 = 108
Thus the cost of a knot window is $108

Question 12.
A builder installs 2 rose windows and 2 tulip windows. Write and evaluate an expression to find the combined area of the windows.
_______ square feet

Answer: 50

Explanation:
The area of rose window is 32
The area of tulip windows is 22
Combines area of rose and tulip window is 32 + 22 = 52
52 × 2 = 25 × 2 = 50
Thus the area of the combined windows is 50 square feet.

Question 13.
DeShawn bought a tulip window. Emma bought a rose window. Write and evaluate an expression to determine how much more DeShawn paid for his window than Emma paid for hers.
$ _______

Answer: 258

Explanation:
Given that, DeShawn bought a tulip window.
DeShawn bought it for 42 × $33 = 16 × $33 = 528
Emma bought a rose window
Emma bought it for 32 × 30 = 9 × 30 = 270
$528 – $270 = $258
DeShawn paid $258 for his window than Emma paid for hers.

Question 14.
What’s the Error? Darius wrote 17 − 22 = 225. Explain his error.
Type below:
_____________

Answer: 17 – 4 is actually 13 but not 225.

Question 15.
Ms. Hall wrote the expression 2 × (3 + 5)2÷ 4 on the board. Shyann said the first step is to evaluate 52. Explain Shyann’s mistake. Then evaluate the expression
_______

Answer: 32

Explanation:
2 × (3 + 5)2÷ 4
First, add 3 and 5.
2 × (8)2÷ 4
The square of 8 × 8 is 64.
2 × (64 ÷ 4) = 2 × 16 = 32

Evaluate Expressions Involving Exponents – Page No. 367

Evaluate the expression.

Question 1.
5 + 17 − 102 ÷ 5
_______

Answer: 2

Explanation:
5 + 17 – (100 ÷ 5)
Divide 100 by 5
(5 + 17) – 20
22 – 20 = 2
So, the value for the expression 5 + 17 − 102 ÷ 5 = 2

Question 2.
72 − 32 × 4
_______

Answer: 13

Explanation:
72 − 32 × 4
72 − (32 × 4)
72 − (9 × 4)
49 – 36 = 13
Thus, 72 − 32 × 4 = 13

Question 3.
24 ÷ (7 − 5)
_______

Answer: 8

Explanation:
24 ÷ (7 − 5)
24 ÷ 2
24 = 2 × 2 × 2 × 2 = 16
16 ÷ 2 = 8
24 ÷ (7 − 5) = 8

Question 4.
(82 + 36) ÷ (4 × 52)
_______

Answer: 1

Explanation:
(82 + 36) ÷ (4 × 52)
82 = 8 × 8 = 64
52 = 5 × 5 = 25
(64 + 36) ÷ (4 × 25)
100 ÷ 100 = 1
So, (82 + 36) ÷ (4 × 52) = 1

Question 5.
12 + 21 ÷ 3 + (22 × 0)
_______

Answer: 19

Explanation:
12 + 21 ÷ 3 + 0
12 + (21 ÷ 3)
12 + 7 = 19
12 + 21 ÷ 3 + (22 × 0) = 19

Question 6.
(12 − 8)3 − 24 × 2
_______

Answer: 16

Explanation:
(12 − 8)3 − 24 × 2 = (4)3 − 24 × 2
64 – (24 × 2)
= 64 – 48 = 16
(12 − 8)3 − 24 × 2 = 16

Place parentheses in the expression so that it equals the given value.

Question 7.
12 × 2 + 23
value: 120
Type below:
_____________

Answer:
12 × (2 + 23)
12 × (2 + 8)
12 × 10 = 120
12 × 2 + 23 = 120

Question 8.
72 + 1 − 5 × 3
value: 135
Type below:
_____________

Answer:
(72 + 1 − 5) × 3
(49 + 1 – 5) × 3
(50 – 5) × 3
45 × 3 = 135
72 + 1 − 5 × 3 = 135

Problem Solving

Question 9.
Hugo is saving for a new baseball glove. He saves $10 the first week, and $6 each week for the next 6 weeks. The expression 10 + 62 represents the total amount in dollars he has saved. What is the total amount Hugo has saved?
$ _______

Answer: $46

Explanation:
Hugo is saving for a new baseball glove.
He saves $10 the first week, and $6 each week for the next 6 weeks.
The expression 10 + 62 represents the total amount in dollars he has saved.
10 + 62 = 10 + 36 = 46
total amount Hugo has saved is $46

Question 10.
A scientist placed 5 fish eggs in a tank. Each day, twice the number of eggs from the previous day hatch. The expression 5 × 26 represents the number of eggs that hatch on the seventh day. How many eggs hatch on the seventh day?
_______ eggs

Answer: 320 eggs

Explanation:
A scientist placed 5 fish eggs in a tank.
Each day, twice the number of eggs from the previous day hatch.
The expression 5 × 26 represents the number of eggs that hatch on the seventh day.
5 × 26 = 5 × 64 = 320 eggs
Therefore 320 eggs hatch on the seventh day.

Question 11.
Explain how you could determine whether a calculator correctly performs the order of operations.
Type below:
_____________

Answer: Create a problem that must use the order of operations and isn’t solved by just left to right. Solve it going left to right. Then solve it using the order of operations. Solve it on the calculator. Your answer on the calculator will match the one using the order of operations.

Lesson Check – Page No. 368

Question 1.
Ritchie wants to paint his bedroom ceiling and four walls. The ceiling and each of the walls are 8 feet by 8 feet. A gallon of paint covers 40 square feet. Write an expression that can be used to find the number of gallons of paint Ritchie needs to buy.
Type below:
_____________

Answer:
Ritchie wants to paint his bedroom ceiling and four walls.
The ceiling and each of the walls are 8 feet by 8 feet.
A gallon of paint covers 40 square feet.
8 × 8 × (4 + 1) ÷ 40
82 (4 + 1) ÷ 40
Thus the expression that can be used to find the number of gallons of paint Ritchie needs to buy is 82 (4 + 1) ÷ 40

Question 2.
A Chinese restaurant uses about 225 pairs of chopsticks each day. The manager wants to order a 30-day supply of chopsticks. The chopsticks come in boxes of 750 pairs. How many boxes should the manager order?
_______ boxes

Answer: 9 boxes

Explanation:
A Chinese restaurant uses about 225 pairs of chopsticks each day.
The manager wants to order a 30-day supply of chopsticks.
Multiply the number of pairs with the number of days
225 × 30 = 6750
The chopsticks come in boxes of 750 pairs.
Now divide the number of chopsticks by the number of pairs.
6750 ÷ 750 = 9 boxes.

Spiral Review

Question 3.
Annabelle spent $5 to buy 4 raffle tickets. How many tickets can she buy for $20?
_______ tickets

Answer: 16 tickets

Explanation:
Annabelle spent $5 to buy 4 raffle tickets.
To find the number of tickets she can buy for $20.
($20 ÷ $5) × 4
4 × 4 = 16 tickets
That means she can buy 16 tickets for $20.

Question 4.
Gavin has 460 baseball players in his collection of baseball cards, and 15% of the players are pitchers. How many pitchers are in Gavin’s collection?
_______ pitchers

Answer: 69 pitchers

Explanation:
Gavin has 460 baseball players in his collection of baseball cards, and 15% of the players are pitchers.
The decimal form of 15% is 0.15
Now multiply 460 with 0.15
460 × 0.15 = 69.00
Thus there are 69 pitchers in Gavin’s collection.

Question 5.
How many pounds are equivalent to 40 ounces?
_______ pounds

Answer: 2.5 pounds

Explanation:
Convert from ounces to pounds.
1 pound = 16 ounces
1 ounce = 1/16 pound
40 ounces = 40 × 1/16 pound
40 ounces = 2.5 pounds
Thus, 2.5 pounds are equivalent to 40 ounces

Question 6.
List the expressions in order from least to greatest.
15 33 42 81
Type below:
_____________

Answer:
15 33 42 81
15 = 1 × 1 × 1 × 1 × 1 = 1
33 = 3 × 3 × 3 = 27
42 = 4 × 4 = 16
81 = 8
Thus the order from least to greatest.
15 81 42 33

Share and Show – Page No. 371

Question 1.
Write an algebraic expression for the product of 6 and p.
What operation does the word “product” indicate?
Type below:
_____________

Answer: 6 × p
Explanation:
The word product indicates multiplication.
Multiply 6 with p.
The algebraic expression for the product of 6 and p is 6 × p.

Write an algebraic expression for the word expression.

Question 2.
11 more than e
Type below:
_____________

Answer: 11 + e

Explanation:
The word more than indicates addition operation.
So, the algebraic expression is 11 + e

Question 3.
9 less than the quotient of n and 5
Type below:
_____________

Answer: 9 – (n ÷ 5)

Explanation:
The word “less than” indicates subtraction and the “quotient” indicates division.
So, the expression is 9 – (n ÷ 5)

On Your Own

Write an algebraic expression for the word expression.

Question 4.
20 divided by c
Type below:
_____________

Answer: 20 ÷ c

Explanation:
Here we have to divide 20 by c.
The expression is 20 ÷ c

Question 5.
8 times the product of 5 and t
Type below:
_____________

Answer: 8 × (5t)

Explanation:
The word times indicate multiplication and the product indicates multiplication.
Here we have to multiply 8 with 5 and t.
Thus the expression is 8 × 5 × t = 8 × 5t

Question 6.
There are 12 eggs in a dozen. Write an algebraic expression for the number of eggs in d dozen.
Type below:
_____________

Answer: 12d

Explanation:
Given,
There are 12 eggs in a dozen.
d represents number of eggs in dozen
So, we have to multiply 12 with d.
Thus the algebraic expression is 12d.

Question 7.
A state park charges a $6.00 entry fee plus $7.50 per night of camping. Write an algebraic expression for the cost in dollars of entering the park and camping for n nights.
Type below:
_____________

Answer: $6.00 + $7.50 n

Explanation:
Given that, A state park charges a $6.00 entry fee plus $7.50 per night of camping.
Find the camping for n nights. The product of $7.50 camping for n nights.
$7.50 × n
Now add park charges to the camping nights.
$6.00 + $7.50 n
Thus the algebraic expression for the cost in dollars of entering the park and camping for n nights is $6.00 + $7.50 n

Question 8.
Look for Structure At a bookstore, the expression 2c + 8g gives the cost in dollars of c comic books and g graphic novels. Next month, the store’s owner plans to increase the price of each graphic novel by $3. Write an expression that will give the cost of c comic books and g graphic novels next month.
Type below:
_____________

Answer: 2c + 11g

Explanation:
Look for Structure At a bookstore, the expression 2c + 8g gives the cost in dollars of c comic books and g graphic novels.
Next month, the store’s owner plans to increase the price of each graphic novel by $3.
Here we have to add $3 to 8 g = 3g + 8g = 11g
Sum of cost of c comic books and g graphic novels
Thus the expression is 2c + 11g

Unlock the Problem – Page No. 372

Question 9.
Martina signs up for the cell phone plan described at the right. Write an expression that gives the total cost of the plan in dollars if Martina uses it for m months.
Go Math Grade 6 Answer Key Chapter 7 Exponents img 4
a. What information do you know about the cell phone plan?
Type below:
_____________

Answer: Pay a low monthly fee of $50. Receive $10 off your first month’s fee.

Question 9.
b. Write an expression for the monthly fee in dollars for m months.
Type below:
_____________

Answer:
M is the number of months.
50 × m
Given that $10 off on first-month fee.
50m + (50-10)
50m + $40

Question 9.
c. What operation can you use to show the discount of $10 for the first month?
Type below:
_____________

Answer: We have to use subtraction operations to show a discount of $10 for the first month.

Question 9.
d. Write an expression for the total cost of the plan in dollars for m months
Type below:
_____________

Answer: 50m + 40

Question 10.
A group of n friends evenly share the cost of dinner. The dinner costs $74. After dinner, each friend pays $11 for a movie. Write an expression to represent what each friend paid for dinner and the movie.
Type below:
_____________

Answer: 74 ÷ n + 11n

Explanation:
Given,
A group of n friends evenly share the cost of dinner.
The dinner costs $74. After dinner, each friend pays $11 for a movie.
The word share represents the division operation.
That means we have to divide 74 by n.
74 ÷ n
After that n friends paid $11 for movie
Multiply 11 with n.
Thus the expression to represent what each friend paid for dinner and the movie is 74 ÷ n + 11n

Question 11.
A cell phone company charges $40 per month plus $0.05 for each text message sent. Select the expressions that represent the cost in dollars for one month of cell phone usage and sending m text messages. Mark all that apply.
Options:
a. 40m + 0.05
b. 40 + 0.05m
c. 40 more than the product of 0.05 and m
d. the product of 40 and m plus 0.05

Answer: 40 + 0.05m

Explanation:
A cell phone company charges $40 per month plus $0.05 for each text message sent.
Let m represent the messages sent.
40m + 0.05m
Thus the answer is option B.

Write Algebraic Expressions – Page No. 373

Write an algebraic expression for the word expression.

Question 1.
13 less than p
Type below:
_____________

Answer: 13 – p

Explanation:
Less than is nothing but subtraction.
So the expression for 13 less than p is 13 – p

Question 2.
the sum of x and 9
Type below:
_____________

Answer: x + 9

Explanation:
The sum is nothing but an addition.
Thus the expression for the sum of x and 9 is x + 9.

Question 3.
6 more than the difference of b and 5
Type below:
_____________

Answer: 6 + (b – 5)

Explanation:
More than is nothing but addition and difference means subtraction.
The expression for 6 more than the difference of b and 5 is 6 + (b – 5)

Question 4.
the sum of 15 and the product of 5 and v
Type below:
_____________

Answer: 15 + 5v

Explanation:
Product is nothing but multiplication and sum is nothing but an addition.
So, the expression for the sum of 15 and the product of 5 and v is 15 + 5 × v

Question 5.
the difference of 2 and the product of 3 and k
Type below:
_____________

Answer: 2 – 3k

Explanation:
The difference means subtraction and Product is nothing but the multiplication
So, the difference of 2 and the product of 3 and k is 2 – 3 × k
2 – 3k

Question 6.
12 divided by the sum of h and 2
Type below:
_____________

Answer: 12 ÷ h + 2

Explanation:
12 divided by the sum of h and 2
Divide 12 by sum of h and 2.
12 ÷ (h + 2)

Question 7.
the quotient of m and 7
Type below:
_____________

Answer: m ÷ 7

Explanation:
Given the quotient of m and 7
That means we have to divide m by 7.
Thus the answer is m ÷ 7

Question 8.
9 more than 2 multiplied by f
Type below:
_____________

Answer: 9 + 2f

Explanation:
9 more than 2 multiplied by f
We have to add 9 to 2 × f
So, the expression is 9 + 2f

Question 9.
6 minus the difference of x and 3
Type below:
_____________

Answer: 6 – (x – 3)

Explanation:
First, subtract 3 from x
The expression for 6 minus the difference of x and 3 is 6 – (x – 3)

Question 10.
10 less than the quotient of g and 3
Type below:
_____________

Answer: 10 – (g ÷ 3)

Explanation:
The quotient of g and 3 is nothing but divide g by 3
g ÷ 3
Now subtract g ÷ 3 from 10.
So, the expression for 10 less than the quotient of g and 3 is 10 – (g ÷ 3)

Question 11.
the sum of 4 multiplied by a and 5 multiplied by b
Type below:
_____________

Answer: 4a + 5b

Explanation:
First, multiply 4 with a and then multiply 5 with b
After that add both the expressions.
4a + 5b
So, the sum of 4 multiplied by a and 5 multiplied by b is 4a + 5b

Question 12.
14 more than the difference of r and s
Type below:
_____________

Answer: 14 + (r – s)

Explanation:
Subtract r and s
And then add 14 to that r -s
14 + (r – s)

Problem Solving

Question 13.
Let h represent Mark’s height in inches. Suzanne is 7 inches shorter than Mark. Write an algebraic expression that represents Suzanne’s height in inches.
Type below:
_____________

Answer: h – 7

Explanation:
Let h represent Mark’s height in inches. Suzanne is 7 inches shorter than Mark.
That means we have to subtract 7 from h.
i.e., h – 7
Thus Suzanne’s height is h – 7 inches.

Question 14.
A company rents bicycles for a fee of $10 plus $4 per hour of use. Write an algebraic expression for the total cost in dollars for renting a bicycle for h hours.
Type below:
_____________

Answer: 10 + 4h

Explanation:
A company rents bicycles for a fee of $10 plus $4 per hour of use.
Multiply 4 with hours
And then 10 to 4h
10 + 4h
Thus the total cost in dollars for renting a bicycle for h hours is 10 + 4h

Question 15.
Give an example of a real-world situation involving two unknown quantities. Then write an algebraic expression to represent the situation.
Type below:
_____________

Answer:
Cooper bikes so many miles per day and does it for 7 months.
The expression for the question is 6m × 7

Lesson Check – Page No. 374

Question 1.
The female lion at a zoo weighs 190 pounds more than the female cheetah. Let c represent the weight in pounds of the cheetah. Write an expression that gives the weight in pounds of the lion.
Type below:
_____________

Answer: c + 190

Explanation:
Given that, The female lion at a zoo weighs 190 pounds more than the female cheetah.
Let c represent the weight in pounds of the cheetah.
We have to add 190 to the weight in pounds of the cheetah.
That means c + 190
Thus the expression that gives the weight in pounds of the lion is c + 190.

Question 2.
Tickets to a play cost $8 each. Write an expression that gives the ticket cost in dollars for a group of g girls and b-boys.
Type below:
_____________

Answer: 8 × (g + b)

Explanation:
First add girls group and boys group.
g + b
And then multiply 8 with the group of girls and boys.
8 × (g + b)
So, the expression that gives the ticket cost in dollars for a group of g girls and b-boys is 8 × (g + b).

Spiral Review

Question 3.
A bottle of cranberry juice contains 32 fluid ounces and costs $2.56. What is the unit rate?
$ _______ per fluid ounce

Answer: 0.08

Explanation:
A bottle of cranberry juice contains 32 fluid ounces and costs $2.56.
Divide the number of fluid ounces by the cost.
32 ÷ $2.56
32/2.56 = 0.08
The unit rate is 0.08 per fluid ounce.

Question 4.
There are 32 peanuts in a bag. Elliott takes 25% of the peanuts from the bag. Then Zaire takes 50% of the remaining peanuts. How many peanuts are left in the bag?
_______ peanuts

Answer: 12

Explanation:
First, we have to find 25% of 32.
25% of 32 its 0.25 × 32=8
Now we have to subtract 32 and 8
32 – 8=24
Now we have to find 50% of 24
50% of 24 = 12
24-12=12.
Thus 12 peanuts are left in the bag.

Question 5.
Hank earns $12 per hour for babysitting. How much does he earn for 15 hours of babysitting?
$ _______

Answer: 180

Explanation:
Hank earns $12 per hour for babysitting.
Multiply $12 with 15
12 × 15 = $180
He earned $180 for 15 hours of babysitting.

Question 6.
Write an expression using exponents that represent the area of the figure in square centimeters
Go Math Grade 6 Answer Key Chapter 7 Exponents img 5
Type below:
_____________

Answer: 72 – 22

Explanation:
The area of the square is 7 cm × 7 cm = 72
The area of the square is 2 cm × 2 cm = 22
Now subtract a small square from the large square.
The expression that represents the area of the figure is 72 – 22

Share and Show – Page No. 377

Identify the parts of the expression. Then, write a word expression for the numerical or algebraic expression.

Question 1.
7 × (9 ÷ 3)
Type below:
_____________

Answer:
The quotient of 9 and 3 and then multiply with 7.
Word expression: Product of 7 with the quotient of 9 and 3.

Question 2.
5m + 2n
Type below:
_____________

Answer:
Product of 5 and m and product of 2 and n
Now add both the product of 5 and m and 2 and n.
Word Expression: Product of 5 and m plus the product of 2 and n.

On Your Own

Practice: Copy and Solve Identify the parts of the expression. Then write a word expression for the numerical or algebraic expression.

Question 3.
8 + (10 − 7)
Type below:
_____________

Answer:
Subtraction is the difference between 10 and 7. Addition to the subtraction of 10 and 7.
Word expression: Add 8 to the difference between 10 and 7.

Question 4.
1.5 × 6 + 8.3
Type below:
_____________

Answer:
The addition is the sum of 6 and 8.3 and then multiply the sum to 1.5.
Word expression: 1.5 times the sum of 6 and 8.3

Question 5.
b + 12x
Type below:
_____________

Answer:
Product of 12 and x. Add b to the product of 12 and x.
Word expression: Sum of b to the product of 12 and x.

Question 6.
4a ÷ 6
Type below:
_____________

Answer:
The division is the quotient of 4a and 6. Multiply 4 and a. The expression is the product of 4 and a divided by 6.
Word expression: The quotient of the product 4 and a and 6.

Identify the terms of the expression. Then, give the coefficient of each term.

Question 7.
k − \(\frac{1}{3}\)d
Type below:
_____________

Answer:
The terms of the expression are k and \(\frac{1}{3}\)d
Coefficients – 1 and \(\frac{1}{3}\)

Question 8.
0.5x + 2.5y
Type below:
_____________

Answer:
The terms of the expression are 0.5x and 2.5y
Coefficients – 0.5 and 2.5

Question 9.
Connect Symbols and Words Ava said she wrote an expression with three terms. She said the first term has the coefficient 7, the second term has the coefficient 1, and the third term has the coefficient 0.1. Each term involves a different variable. Write an expression that could be the expression Ava wrote
Type below:
_____________

Answer:
Connect Symbols and Words Ava said she wrote an expression with three terms.
She said the first term has the coefficient 7, the second term has the coefficient 1, and the third term has the coefficient 0.1.
The expression for the first term is 7x
The expression for the second term is 1y
The expression for the third term is 0.1z
7x + y + 0.1z

Problem Solving + Applications – Page No. 378

Use the table for 10–12.
Go Math Grade 6 Answer Key Chapter 7 Exponents img 6

Question 10.
A football team scored 2 touchdowns and 2 extra points. Their opponent scored 1 touchdown and 2 field goals. Write a numerical expression for the points scored in the game.
Type below:
_____________

Answer:
A football team scored 2 touchdowns and 2 extra points.
2 touchdowns = 2 × 6
2 extra points = 2 × 1
Their opponent scored 1 touchdown and 2 field goals.
1 touchdown = 1 × 6
2 field goals = 2 × 3
Thue the numerical expression is 12 + 2 + 6 + 6
14 + 12
The numerical expression for the points scored in the game is 14 + 12.

Question 11.
Write an algebraic expression for the number of points scored by a football team that makes t touchdowns, f field goals, and e extra points
Type below:
_____________

Answer: 6t + 3f + e

Explanation:
The number of points scored by a football team that makes t touchdowns, f field goals, and e extra points.
The table shows that touchdown has 6 points, field goal has 3 points and extra point has 1 point.
So we need to add all the points to make the expressions
That means 6t + 3f + e

Question 12.
Identify the parts of the expression you wrote in Exercise 11.
Type below:
_____________

Question 13.
Give an example of an expression involving multiplication in which one of the factors is a sum. Explain why you do or do not need parentheses in your expression
Type below:
_____________

Answer: 6 × 2 + 3
In this expression, there is no need for parentheses because there are no exponents or multiple operations.

Question 14.
Kennedy bought a pounds of almonds at $5 per pound and p pounds of peanuts at $2 per pound. Write an algebraic expression for the cost of Kennedy’s purchase.
Type below:
_____________

Answer: 5 + 2p = x

Explanation:
Kennedy bought a pounds of almonds at $5 per pound and p pounds of peanuts at $2 per pound.
We have to multiply p with $2 per pound.
The algebraic expression for the cost of Kennedy’s purchase is the sum of 5 and the product of p and 2
Thus the expression is 5 + 2p = x

Identify Parts of Expressions – Page No. 379

Identify the parts of the expression. Then write a word expression for the numerical or algebraic expression.

Question 1.
(16 − 7) ÷ 3
Type below:
_____________

Answer:
Subtraction is the difference between 16 and 7. The division is the quotient of the difference and 3
Word expression: the quotient of the difference 16 and 7 and 3.

Question 2.
8 + 6q + q
Type below:
_____________

Answer:
Sum of 8 and the product of 6 and q added to q.
Addition – Sum of 8 plus the product of 6 and q plus q.
Addition – 6 times q and the sum of q.
Multiply – the product of 6 and q.

Identify the terms of the expression. Then give the coefficient of each term.

Question 3.
11r + 7s
Type below:
_____________

Answer:
The terms of the expression are 11r and 7s
The coefficient of each term is 11 and 7.

Question 4.
6g − h
Type below:
_____________

Answer:
The terms of the expression are 6g and h
The coefficient of each term is 6 and 1.

Problem Solving

Question 5.
Adam bought granola bars at the store. The expression 6p + 5n gives the number of bars in p boxes of plain granola bars and n boxes of granola bars with nuts. What are the terms of the expression?
Type below:
_____________

Answer:
Adam bought granola bars at the store.
The expression 6p + 5n gives the number of bars in p boxes of plain granola bars and n boxes of granola bars with nuts.
The terms of the expression are 6p and 5n.

Question 6.
In the sixth grade, each student will get 4 new books. There is one class of 15 students and one class of 20 students. The expression 4 × (15 + 20) gives the total number of new books. Write a word expression for the numerical expression.
Type below:
_____________

Answer:
In the sixth grade, each student will get 4 new books.
There is one class of 15 students and one class of 20 students.
The expression 4 × (15 + 20) gives the total number of new books.
The product of 4 the sum of 15 and 20.

Question 7.
Explain how knowing the order of operations helps you write a word expression for a numerical or algebraic expression.
Type below:
_____________

Answer: Because if you don’t know and use the order of operations you can get an entirely different answer.

Lesson Check – Page No. 380

Question 1.
A fabric store sells pieces of material for $5 each. Ali bought 2 white pieces and 8 blue pieces. She also bought a pack of buttons for $3. The expression 5 × (2 + 8) + 3 gives the cost in dollars of Ali’s purchase. How can you describe the term (2 + 8) in words?
Type below:
_____________

Answer: the sum of 2 and 8

Explanation:
A fabric store sells pieces of material for $5 each.
Ali bought 2 white pieces and 8 blue pieces.
She also bought a pack of buttons for $3.
The expression 5 × (2 + 8) + 3 gives the cost in dollars of Ali’s purchase.
The word expression for the term 2 + 8 is the sum of 2 and 8.

Question 2.
A hotel offers two different types of rooms. The expression k + 2f gives the number of beds in the hotel where k is the number of rooms with a king-size bed and f is the number of rooms with 2 full-size beds. What are the terms of the expression?
Type below:
_____________

Answer: k and 2f

Explanation:
The terms for the expression k + 2f is k and 2f.

Spiral Review

Question 3.
Meg paid $9 for 2 tuna sandwiches. At the same rate, how much does Meg pay for 8 tuna sandwiches?
$ _______

Answer: 36

Explanation:
Meg paid $9 for 2 tuna sandwiches.
To find how much does Meg pay for 8 tuna sandwiches
2 – $9
8 -?
$9 × 8/2 = 72/2 = 36
Thus Meg pays $36 for 8 tuna sandwiches.

Question 4.
Jan is saving for a skateboard. She has saved $30 already, which is 20% of the total price. How much does the skateboard cost?
$ _______

Answer: 150

Explanation:
Jan is saving for a skateboard. She has saved $30 already, which is 20% of the total price.
Divide $30 by 20%
30 ÷ 20%
30 ÷ 20 × 1/100
30 ÷ 1/5
30 × 5 = 150
Thus the cost of the skateboard is $150.

Question 5.
It took Eduardo 8 hours to drive from Buffalo, NY, to New York City, a distance of about 400 miles. Find his average speed.
_______ miles per hour

Answer: 50

Explanation:
Given,
It took Eduardo 8 hours to drive from Buffalo, NY, to New York City, a distance of about 400 miles.
We can use the formula d = rt
r = d/t
r = 400 miles/8 hours
r = 50 miles per hour

Question 6.
Write an expression that represents the value, in cents, of n nickels.
Type below:
_____________

Answer: 0.05n

Explanation:
An expression does not have an equal sign.
Since the value of a nickel is 5 cents and you want to find out the value of n nickels (which means if you had any number of nickels) the expression would be
.05n

Share and Show – Page No. 383

Question 1.
Evaluate 5k + 6 for k = 4.
_______

Answer: 26

Explanation:
The expression is 5k + 6
Substitute the value k = 4
5(4) + 6 = 20 + 6 = 26
5k + 6 = 26

Evaluate the expression for the given value of the variable.

Question 2.
m − 9 for m = 13
_______

Answer: 4

Explanation:
m – 9
Substitute the value of m in the expression
13 – 9 = 4
Thus m – 9 = 4

Question 3.
16 − 3b for b = 4
_______

Answer: 4

Explanation:
Given the expression 16 – 3b
Now substitute the value of b in the expression.
16 – 3b = 16 – 3(4) = 16 – 12 = 4
16 – 3b = 4

Question 4.
p2 + 4 for p = 6
_______

Answer: 40

Explanation:
Given the expression p2 + 4
Substitute the value of p in the expression
62 + 4 = 36 + 4 = 40
Thus the value of p2 + 4 is 40.

Question 5.
The formula A = lw gives the area A of a rectangle with length l and width w. What is the area in square feet of a United States flag with a length of 12 feet and a width of 8 feet?
_______ square feet

Answer: 96 square feet

Explanation:
Use the formula A = lw
Length = 12 feet
Width = 8 feet
A = lw
A = 12 feet × 8 feet = 96 square feet
Thus the area of the United States flag is 96 square feet.

On Your Own

Practice: Copy and Solve Evaluate the expression for the given value of the variable.

Question 6.
7s + 5 for s = 3
_______

Answer: 26

Explanation:
Given the expression 7s + 5
Substitute  the value of S in the above expression
7(3) + 5 = 21 + 5 = 26

Question 7.
21 − 4d for d = 5
_______

Answer: 1

Explanation:
Given the expression 21 – 4d
Substitute  the value d = 5 in the above expression
21 – 4(5) = 21 – 20 = 1

Question 8.
(t − 6)2 for t = 11
_______

Answer: 25

Explanation:
Given the expression (t − 6)2
Substitute the value t = 11
Thus (t − 6)2 = (11 − 6)2 = 5 × 5 = 25

9.6 × (2v − 3) for v = 5
_______

Answer: 42

Explanation:
Given the expression 6 × (2v – 3)
Substitute the value of v in the above expression.
6 × (2v – 3) = 6 × (2 × 5 – 3)
6 × (10 – 3)
6 × 7 = 42
Thus the value of 6 × (2v – 3) = 42

Question 10.
2 × (k2 − 2) for k = 6
_______

Answer: 68

Explanation:
Given the expression 2 × (k2 − 2)
Substitute the value of k in the above expression
2 × (k2 − 2) = 2 × (62 − 2)
2 × (36 – 2) = 2 × 34 = 68
Thus the value of 2 × (k2 − 2) is 68

Question 11.
5 × (f − 32) ÷ 9 for f = 95
_______

Answer: 35

Explanation:
The expression is 5 × (f – 32) ÷ 9
Substitute  the value f = 95
5 × (f – 32) ÷ 9 = 5 × (95 – 32) ÷ 9
5 × (63 ÷ 9) = 5 × 7 = 35
The value of 5 × (63 ÷ 9) = 35

Question 12.
The formula P = 4s gives the perimeter P of a square with side length s. How much greater is the perimeter of a square with a side length of 5 \(\frac{1}{2}\) inches than a square with a side length of 5 inches?
_______ inches

Answer: 2 inches

Explanation:
We have to use the formula P = 4s to find the perimeter of the square.
4 × 5 \(\frac{1}{2}\)
Convert the mixed fraction to the improper fraction.
4 × 11/2 = 2 × 11 = 22 inches
4 × 5 inches = 20 inches
To find which has the greater  perimeter  we have to subtract 20 inches from 22 inches
22 inches – 20 inches = 2 inches
Thus the perimeter of a square with 5 \(\frac{1}{2}\) inches is 2 inches greater than a square with a side length of 5 inches.

Problem Solving + Applications – Page No. 384

The table shows how much a company charges for skateboard wheels. Each pack of 8 wheels costs $50. Shipping costs $7 for any order. Use the table for 13−15.
Go Math Grade 6 Answer Key Chapter 7 Exponents img 7

Question 13.
Complete the table.
Type below:
_____________

Answer:
Go-Math-Grade-6-Answer-Key-Chapter-7-Exponents-img-7

Question 14.
A skateboard club has $200 to spend on new wheels this year. What is the greatest number of packs of wheels the club can order?
_______ packs

Answer: 3 packs

Explanation:
A skateboard club has $200 to spend on new wheels this year.
From the above table, we can say that the club can order 3 packs of wheels.

Question 15.
Make Sense of Problems A sporting goods store placed an order for 12 packs of wheels on the first day of each month last year. How much did the sporting goods store spend on these orders last year?
$ _______

Answer: 7284

Explanation:
Make Sense of Problems A sporting goods store placed an order for 12 packs of wheels on the first day of each month last year.
Substitute n = 7 in the expression 50 × n + 7
We get, 50 × 12 + 7
600 + 7 = 607
Now multiply 607 with 12
607 × 12 = 7284
Therefore the sporting goods store spent $7284 on these orders last year.

Question 16.
What’s the Error? Bob used these steps to evaluate 3m − 3 ÷ 3 for m = 8. Explain his error.
3 × 8 − 3 ÷ 3 = 24 − 3 ÷ 3
= 21 ÷ 3
= 7
Type below:
_____________

Answer:
First, he has to subtract 8 and 3. But he first multiplied and then subtracted 24 and 3.
3 × 8 − 3 ÷ 3 = 3 × (8 − 3) ÷ 3
3 × 5 ÷ 3
15 ÷ 3 = 5

Question 17.
The surface area of a cube can be found by using the formula 6s2, where s represents the length of the side of the cube.
The surface area of a cube that has a side length of 3 meters is _____ meters squared.
The surface area of a cube that has a side length                       meters
of 3 meters is _____________ squared

Answer: 5

Explanation:
The surface area of a cube can be found by using the formula 6s2
he surface area of a cube that has a side length of 3 meters
s2 = 32 = 9
6 × 9 = 54 square meters

Evaluate Algebraic Expressions and Formulas – Page No. 385

Evaluate the expression for the given values of the variables.

Question 1.
w + 6 for w = 11
_______

Answer: 17

Explanation:
Given the expression w + 6
Substitute the value w = 6 in the expression
w + 6 = 11 + 6 = 17

Question 2.
17 − 2c for c = 7
_______

Answer: 3

Explanation:
Substitute  the value c = 7 in the given expression
17 – 2(7) = 17 – 14 = 3
Thus the value for 17 – 2c is 3.

Question 3.
b2 − 4 for b = 5
_______

Answer: 21

Explanation:
Substitute the value b = 5 in the expression
b2 − 4 = 52 − 4 = 25 – 4 = 21
Thus the value for the expression b2 − 4 is 21.

Question 4.
(h − 3)2 for h = 5
_______

Answer: 4

Explanation:
We have to substitute the value h = 5
(h − 3)2 = (5 − 3)2
= (2)2 = 4
Therefore the value of (h − 3)2 is 4.

Question 5.
m + 2m + 3 for m = 12
_______

Answer: 39

Explanation:
Given the expression m + 2m + 3
Now substitute the value m = 12 in the above expression.
12 + 2(12) + 3 = 12 + 24 + 3 = 39.
The value for m + 2m + 3 = 39.

Question 6.
4 × (21 − 3h) for h = 5
_______

Answer: 24

Explanation:
Substitute h = 5 in the given expression.
4 × (21 – 3h) = 4 × (21 – 3(5))
4 × (21 – 15) = 4 × 6 = 24
Therefore the value for 4 × (21 – 3h) is 24.

Question 7.
7m − 9n for m = 7 and n = 5
_______

Answer: 4

Explanation:
Substitute the values m = 7 and n = 5 in the above expression.
7m – 9n = 7 × 7 – 9 × 5
= 49 – 45 = 4
Thus 7m – 9n = 4.

Question 8.
d2 − 9k + 3 for d = 10 and k = 9
_______

Answer: 22

Explanation:
Given the expression d2 − 9k + 3
Now substitute d = 10 and k = 9 in the expression.
d2 − 9k + 3 = 102 − 9(9) + 3
100 – 81 + 3 = 22
Thus the value for the expression d2 − 9k + 3 is 22.

Question 9.
3x + 4y ÷ 2 for x = 7 and y = 10
_______

Answer: 41

Explanation:
Substitute the values x = 7 and y = 10 in the expression.
3x + 4y ÷ 2 = 3(7) + 4(10) ÷ 2
21 + 40 ÷ 2 = 21 + 20 = 41
Thus the value for 3x + 4y ÷ 2 is 41.

Problem Solving

Question 10.
The formula P = 2l + 2w gives the perimeter P of a rectangular room with length l and width w. A rectangular living room is 26 feet long and 21 feet wide. What is the perimeter of the room?
_______ feet

Answer: 94 feet

Explanation:
Use the formula  of the perimeter  of a rectangle P = 2l + 2w
L = 26 feet
W = 21 feet
P = 2(26) + 2(21)
P = 52 feet + 42 feet
P = 94 feet
Therefore the perimeter of a room is 94 feet.

Question 11.
The formula C = 5(F − 32) ÷ 9 gives the Celsius temperature in C degrees for a Fahrenheit temperature of F degrees. What is the Celsius temperature for a Fahrenheit temperature of 122 degrees?
_______ degrees Celsius

Answer: 50

Explanation:
C = 5(F – 32) ÷ 9
We know that F = 122 degrees
Substitute the value of F in the formula
C = 5(122 – 32) ÷ 9
C = 5(90) ÷ 9
C = 450 ÷ 9 = 50
Thus the answer is 50 degrees Celsius.

Question 12.
Explain how the terms variable, algebraic expression, and evaluate are related.
Type below:
_____________

Answer: To evaluate an algebraic expression, you have to substitute a number for each variable and perform the arthematic operations. If we know the variables, we can replace the variables with their values and then evaluate the expression.

Lesson Check – Page No. 386

Question 1.
When Debbie baby-sits, she charges $5 to go to the house plus $8 for every hour she is there. The expression 5 + 8h gives the amount in dollars she charges. How much will she charge to baby-sit for 5 hours?
$ _______

Answer: 45

Explanation:
When Debbie baby-sits, she charges $5 to go to the house plus $8 for every hour she is there. The expression 5 + 8h gives the amount in dollars she charges.
If h = 5 hours
Substitute the value h in the above expression.
5 + 8h = 5 + 8(5) = 5 + 40 = 45
Thus she charges $45 to baby-sit for 5 hours.

Question 2.
The formula to find the cost C in dollars of a square sheet of glass is C = 25s2 where s represents the length of a side in feet. How much will Ricardo pay for a square sheet of glass that is 3 feet on each side?
$ _______

Answer: $225

Explanation:
Use the formula C = 25s2
s represents the length of a side in feet.
s = 3 feet
Substitute the value s in the above formula.
C = 25s2
C = 25(32)
C = 25(9) = 225
Ricardo pays $225 for a square sheet of glass that is 3 feet on each side.

Spiral Review

Question 3.
Evaluate using the order of operations.
\(\frac{3}{4}+\frac{5}{6} \div \frac{2}{3}\)
_______

Answer: 2

Explanation:
\(\frac{3}{4}\) + [/latex]\frac{5}{6}[/latex] ÷ [/latex]\frac{2}{3}[/latex]
[/latex]\frac{5}{6}[/latex] ÷ [/latex]\frac{2}{3}[/latex]
= [/latex]\frac{5}{6}[/latex] × [/latex]\frac{3}{2}[/latex] = [/latex]\frac{15}{12}[/latex] = [/latex]\frac{5}{4}[/latex]
Now convert the improper fraction to the mixed fraction.
[/latex]\frac{5}{4}[/latex] = 1 [/latex]\frac{1}{4}[/latex]
1 [/latex]\frac{1}{4}[/latex] + \(\frac{3}{4}\)
1 + [/latex]\frac{1}{4}[/latex] + \(\frac{3}{4}\) = 1 + 1 = 2
\(\frac{3}{4}+\frac{5}{6} \div \frac{2}{3}\) = 2

Question 4.
Patricia scored 80% on a math test. She missed 4 problems. How many problems were on the test?
_______ problems

Answer: 20

Explanation:
Patricia scored 80% on a math test. She missed 4 problems.
4 ÷ 80%
4 × [/latex]\frac{100}{80}[/latex] = 4 × 5 = 20
Therefore there are 20 questions in the test.

Question 5.
What is the value of 73?
_______

Answer: 343

Explanation:
73 = 7 × 7 × 7 = 49 × 7 = 343
Thus the value of 73 is 343.

Question 6.
James and his friends ordered b hamburgers that cost $4 each and f fruit cups that cost $3 each. Write an algebraic expression for the total cost in dollars of their purchases.
Type below:
_____________

Answer: 4b + 3f

Explanation:
Given that, James and his friends ordered b hamburgers that cost $4 each and f fruit cups that cost $3 each.
Multiply b with $4 and multiply $3 with f
Add 4b and 3f
Thus the expression is 4b + 3f.

Vocabulary – Page No. 387

Choose the best term from the box to complete the sentence.
Go Math Grade 6 Answer Key Chapter 7 Exponents img 8

Question 1.
A(n) _____ tells how many times a base is used as a factor.
Type below:
_____________

Answer: Exponent
An Exponent tells how many times a base is used as a factor.

Question 2.
The mathematical phrase 5+2×18 is an example of a(n) _____.
Type below:
_____________

Answer: Numerical expression
The mathematical phrase 5+2×18 is an example of Numerical expression.

Concepts and Skills

Find the value.

Question 3.
54
________

Answer: 5 × 5 × 5 × 5 = 625

Explanation:
The number 5 is the repeated factor.
5 is used 4 times.
Multiply 5 four times.
5 × 5 × 5 × 5 = 625

Question 4.
212
________

Answer: 21 × 21 = 441

Explanation:
The number 21 is the repeated factor.
21 is used 2 times.
Multiply 21 two times.
21 × 21 = 441

Question 5.
83
________

Answer: 8 × 8 × 8 = 512

Explanation:
The number 8 is the repeated factor.
8 is used 3 times.
8 × 8 × 8 = 512

Evaluate the expression.

Question 6.
92 × 2 − 42
________

Answer: 146

Explanation:
92 × 2 − 42
92 = 9 × 9 = 81
42 = 4 × 4 = 16
81 × 2 – 16 = 162 – 16 = 146
Thus 92 × 2 − 42 = 146

Question 7.
2 × (10 − 2) ÷ 22
________

Answer: 4

Explanation:
2 × (10 − 2) ÷ 22
2 × (10 − 2) ÷ 4
2 × 8 ÷ 4 = 16 ÷ 4 = 4
Thus 2 × (10 − 2) ÷ 22 = 4

Question 8.
30 − (33 − 8)
________

Answer: 11

Explanation:
33 = 3 × 3 × 3 = 27
30 − (33 − 8) = 30 – (27 – 8) = 30 – 19 = 11
30 − (33 − 8) = 11
So, 30 − (33 − 8) is 11.

Write an algebraic expression for the word expression.

Question 9.
the quotient of c and 8
Type below:
_____________

Answer: c ÷ 8
The quotient is nothing but the division of c by 8. So, the expression is c ÷ 8.

Question 10.
16 more than the product of 5 and p
Type below:
_____________

Answer: 16 + 5p

Explanation:
The operation for more than is addition. Here we have to add 16 to the product of 5 and p.
The product is the operation for multiplication. Multiply 5 and p and then add 16 to it.
The expression of the word is 16 + 5p.

Question 11.
9 less than the sum of x and 5
Type below:
_____________

Answer: 9 – x + 5

Explanation:
First, we have to evaluate the expression x and 5.
Sum of is nothing but adding x and 5.
Difference between 9 and x and 5
The expression is 9 – x + 5.

Evaluate the expression for the given value of the variable.

Question 12.
5 × (h + 3) for h = 7
________

Answer: 50

Explanation:
Given expression is 5 × (h + 3)
Substitute h = 7 in the above expression.
5 × (h + 3) = 5 × (7 + 3)
5 × 10 = 50
5 × (h + 3) = 50

Question 13.
2 × (c2 − 5) for c = 4
________

Answer: 22

Explanation:
Given 2 × (c2 − 5)
Substitute c = 4 in the expression
2 × (c2 − 5) = 2 × (42 − 5)
= 2 × (16 – 5) = 2 × 11 = 22
2 × (c2 − 5) = 22

Question 14.
7a − 4a for a = 8
________

Answer: 24

Explanation:
Given, 7a − 4a
Subtract the like terms
7a − 4a = 3a
Now substitute the value a = 8 in the above expression
3a = 3 × 8 = 24
7a − 4a = 24

Page No. 388

Question 15.
The greatest value of any U.S. paper money ever printed is 105 dollars. What is this amount written in standard form?
________

Answer: 100000

Explanation:
105 dollars = 10 × 10 × 10 × 10 × 10
10 is a repeated factor.
10 repeated 5 times.
10 × 10 × 10 × 10 × 10 = 100000 dollars

Question 16.
A clothing store is raising the price of all its sweaters by $3.00. Write an expression that could be used to find the new price of a sweater that originally cost d dollars.
Type below:
_____________

Answer: d + 3

Explanation:
A clothing store is raising the price of all its sweaters by $3.00.
The cost of the sweater is d dollars. The store is going to add $3.
So, the new price of a sweater is the sum of d dollars and $3.
The expression is d + 3.

Question 17.
Kendra bought a magazine for $3 and 4 paperback books for $5 each. The expression 3 + 4 × 5 represents the total cost in dollars of her purchases. What are the terms in this expression?
Type below:
_____________

Answer: 3 and 4 × 5

Explanation:
Kendra bought a magazine for $3 and 4 paperback books for $5 each. The expression 3 + 4 × 5 represents the total cost in dollars of her purchases.
The terms in the expression are 3, 4, and 5.

Question 18.
The expression 5c + 7m gives the number of people who can ride in c cars and m minivans. What are the coefficients in this expression?
Type below:
_____________

Answer: The coefficients in the expression 5c + 7m are 5 and 7.

Question 19.
The formula P = a + b + c gives the perimeter P of a triangle with side lengths a, b, and c. How much greater is the perimeter of a triangular field with sides that measure 33 yards, 56 yards, and 65 yards than the perimeter of a triangular field with sides that measure 26 yards, 49 yards, and 38 yards?
________ yards

Answer: 41 yards

Explanation:
First, we have to calculate the perimeter of the 1st triangle.
Given:
a = 33 yards
b = 56 yards
c = 65 yards
P1 = a + b + c
P1 = 33 + 56 + 65 = 154 yards
Now we have to calculate the perimeter of 2nd triangle.
Given:
a = 26 yards
b = 49 yards
c = 38 yards
P2 = a + b + c
P2 = 26 + 49 + 38 = 113 yards
Now we have to calculate which triangle has greater perimeter and how much greater.
P1 – P2 = 154 yards – 113 yards = 41 yards
Therefore, 41 yards greater is the perimeter of the 1st triangular field than the perimeter of the 2nd triangular field.

Share and Show – Page No. 391

Louisa read that the highest elevation of Mount Everest is 8,848 meters. She wants to know how much higher Mount Everest is than Mount Rainier. Use this information for 1–2.

Question 1.
Write an expression to represent the difference in the heights of the two mountains. Tell what the variable in your expression represents.
Type below:
_____________

Answer: 8848 – h, where h represents the height of the Mount Rainier

Explanation:
Given that, the height of the Mount Everest is 8848 meters
Let the height of the Mount Rainier is h
The difference in height of Mount Everest and height of the Mount Rainier is 8848 – h.

Question 2.
Louisa researches the highest elevation of Mount Rainier and finds that it is 4,392 meters. Use your expression to find the difference in the mountains’ heights.
________ meters

Answer: 4456 meters

Explanation:
The height of the Mount Rainier = 4392 meters
Replace the value of height of the Mount Rainier in the above expression.
8848 – h = 8848 meters – 4392 meters = 4456 meters
Thus the difference between the height of the two mountains is 4456 meters.

On Your Own

A muffin recipe calls for 3 times as much flour as sugar. Use this information for 3–5.

Question 3.
Write an expression that can be used to find the amount of flour needed for a given amount of sugar. Tell what the variable in your expression represents.
Type below:
_____________

Answer:
Let the amount of sugar used represents the variable is s.
The expression to find the amount of flour needed for a given amount of sugar is 3 × m i.e., 3m

Question 4.
Use your expression to find the amount of flour needed when \(\frac{3}{4}\) cup of sugar is used.
______ \(\frac{□}{□}\)

Answer: 2 \(\frac{1}{4}\)

Explanation:
Given that, A muffin recipe calls for 3 times as much flour as sugar.
The amount of flour needed when \(\frac{3}{4}\) cup of sugar used is 3 × \(\frac{3}{4}\) = \(\frac{9}{4}\)
Convert the improper fraction into the mixed fraction.
\(\frac{9}{4}\) = 2 \(\frac{1}{4}\)
Therefore 2 \(\frac{1}{4}\) amount of flour needed when \(\frac{3}{4}\) cup of sugar is used.

Question 5.
Reason Quantitatively Is the value of the variable in your expression restricted to a particular set of numbers? Explain.
Type below:
_____________

Answer: The values that make the denominator equal to zero for a rational expression are known as restricted values. The solutions are the restricted values since they result in a denominator of zero when replaced for the variable(s).

Practice: Copy and Solve Write an algebraic expression for each word expression. Then evaluate the expression for these values of the variable: \(\frac{1}{2}\), 4, and 6.5.

Question 6.
the quotient of p and 4
Type below:
_____________

Answer: p ÷ 4

Explanation:
The expression is p ÷ 4
p = \(\frac{1}{2}\)
\(\frac{1}{2}\) ÷ 4
\(\frac{1}{2}\)/4 = \(\frac{1}{8}\)
p ÷ 4 when p = \(\frac{1}{2}\) is \(\frac{1}{8}\)
p = 4
4 ÷ 4 = 1
p = 6.5
6.5 ÷ 4 = 1.625

Question 7.
4 less than the sum of x and 5
Type below:
_____________

Answer: 4 – (x + 5)

Explanation:
The expression is 4 – (x + 5)
x = 1/2
4 – (x + 5) = 4 – 1/2 + 5
3 1/2 + 5 = 8 1/2
x = 4
4 – x + 5
4 – 4 + 5 = 5
x = 6.5
4 – 6.5 + 5 = 2.5

Problem Solving + Applications – Page No. 392

Use the graph for 8–10.
Go Math Grade 6 Answer Key Chapter 7 Exponents img 9

Question 8.
Write expressions for the distance in feet that each animal could run at top speed in a given amount of time. Tell what the variable in your expressions represents.
Type below:
_____________

Answer:
The expression for distance in feet for Elephant = 22t
The expression for distance in feet for Cheetah = 103t
The expression for distance in feet for Giraffe = 51t
The expression for distance in feet for hippopotamus = 21t
Where t represents the time.

Question 9.
How much farther could a cheetah run in 20 seconds at top speed than a hippopotamus could?
______ feet

Answer: 1640 feet

Explanation:
The expression for distance in feet for Cheetah = 103t
where t = 20 sec
103t = 103 × 20 sec = 2060 feet
The expression for distance in feet for hippopotamus = 21t
where t = 20 sec
21t = 21 × 20 = 420 feet
Now we have to find How much farther could a cheetah run in 20 seconds at top speed than a hippopotamus could
2060 feet – 420 feet = 1640 feet

Question 10.
A giraffe runs at top speed toward a tree that is 400 feet away. Write an expression that represents the giraffe’s distance in feet from the tree after s seconds.
Type below:
_____________

Answer:
The expression representing the giraffe’s distance from tree after s seconds, if the rate is 51 ft per second.
7 43/60 seconds in all

Question 11.
A carnival charges $7 for admission and $2 for each ride. An expression for the total cost of going to the carnival and riding n rides is 7 + 2n.
Complete the table by finding the total cost of going to the carnival and riding n rides.
Go Math Grade 6 Answer Key Chapter 7 Exponents img 10
Type below:
_____________

Answer:
Go-Math-Grade-6-Answer-Key-Chapter-7-Exponents-img-10

Use Algebraic Expressions – Page No. 393

Jeff sold the pumpkins he grew for $7 each at the farmer’s market.

Question 1.
Write an expression to represent the amount of money in dollars Jeff made selling the pumpkins. Tell what the variable in your expression represents
Type below:
_____________

Answer: 7p, where p is the number of pumpkins

Question 2.
If Jeff sold 30 pumpkins, how much money did he make?
$ ________

Answer: 210

Explanation:
The expression is 7p
p = 30 pumpkins
7 × 30 = 210
Thus Jeff sold 30 pumpkins for $210.

An architect is designing a building. Each floor will be 12 feet tall.

Question 3.
Write an expression for the number of floors the building can have for a given building height. Tell what the variable in your expression represents.
Type below:
_____________

Answer: The expression for the number of floors is h/12, where h is the height of the building.

Question 4.
If the architect is designing a building that is 132 feet tall, how many floors can be built?
________ floors

Answer: 11 floors

Explanation:
Given the height of the building is 132 feet
Substitute h in the above expression
h/12 = 132/12 = 11 floors
Thus 11 floors can be built.

Write an algebraic expression for each word expression. Then evaluate the expression for these values of the variable: 1, 6, 13.5.

Question 5.
the quotient of 300 and the sum of b and 24
Type below:
_____________

Answer: 300 ÷ (b + 24)

Explanation:
For b = 1
300 ÷ (b + 24) = 300 ÷ (1 + 24)
300 ÷ 25 = 12
Thus 300 ÷ (b + 24) when b = 1 is 12.
For b = 6
300 ÷ (b + 24) = 300 ÷ (6 + 24)
300 ÷ 30 = 10
Thus 300 ÷ (b + 24) when b = 6 is 10.
For b = 13.5
300 ÷ (b + 24) = 300 ÷ (13.5 + 24)
300 ÷ 37.5 = 8
300 ÷ (b + 24) when b = 13.5 is 8.

Question 6.
13 more than the product of m and 5
Type below:
_____________

Answer: 13 + 5m

Explanation:
For m = 1
13 + 5m = 13 + 5(1) = 13 + 6 = 19
For m = 6
13 + 5m = 13 + 5(6) = 13 + 30 = 43
For m = 13.5
13 + 5m = 13 + 5(13.5) = 13 + 67.5 = 80.5

Problem Solving

Question 7.
In the town of Pleasant Hill, there is an average of 16 sunny days each month. Write an expression to represent the approximate number of sunny days for any number of months. Tell what the variable represents.
Type below:
_____________

Answer: 16m, m for months

Explanation:
In the town of Pleasant Hill, there is an average of 16 sunny days each month. Write an expression to represent the approximate number of sunny days for any number of months.
we have to multiply the number of months with 16
The expression will be 16 times of m = 16m

Question 8.
How many sunny days can a resident of Pleasant Hill expect to have in 9 months?
________ days

Answer: 144 days

Explanation:
The expression to represent the approximate number of sunny days for any number of months is 16m
m = 9
Substitute the value of m in the expression.
16m = 16 × 9 = 144 days

Question 9.
Describe a situation in which a variable could be used to represent any whole number greater than 0.
Type below:
_____________

Answer: To represent the number of people any answer can be accepted.

Lesson Check – Page No. 394

Question 1.
Oliver drives 45 miles per hour. Write an expression that represents the distance in miles he will travel for h hours driven.
Type below:
_____________

Answer: 45h

Explanation:
It is given that Oliver drives 45 miles per hour. Let the number of hours he drove be h. The distance is the product of speed and time. The distance travel by Oliver is defined by the expression is 45h.

Question 2.
Socks cost $5 per pair. The expression 5p represents the cost in dollars of p pairs of socks. Why must p be a whole number?
Type below:
_____________

Answer: p must be a whole number because in almost 100% of all stores it is not allowed to buy a single sock, you must always buy a pair of socks.

Spiral Review

Question 3.
Sterling silver consists of 92.5% silver and 7.5% copper. What decimal represents the portion of the silver in sterling silver?
________

Answer: 0.925

Explanation:
If Sterling silver is 92.5% silver, that means it has 92.5/100 * 100% silver
The fraction 92.5/100 can be simplified by just moving the decimal 2 places to the left:
92.5/100 = .925

Question 4.
How many pints are equivalent to 3 gallons?
________ pints

Answer: 24

Explanation:
Convert from gallons to pints.
1 gallon = 8 pints
3 gallons = 3 × 8 pints = 24 pints
24 pints are equivalent to 3 gallons.

Question 5.
Which operation should be done first to evaluate 10 + (66 – 62)?
Type below:
_____________

Answer: Square 6

Question 6.
Evaluate the algebraic expression h(m + n) ÷ 2 for h = 4, m = 5, and n = 6.
________

Explanation:
Given the expression h(m + n) ÷ 2
h = 4
m = 5
n = 6
h(m + n) ÷ 2 = 4 (5 + 6) ÷ 2
4 (11) ÷ 2 = 44 ÷ 2 = 22
h(m + n) ÷ 2 = 22

Share and Show – Page No. 397

Question 1.
Museum admission costs $7, and tickets to the mammoth exhibit cost $5. The expression 7p + 5p represents the cost in dollars for p people to visit the museum and attend the exhibit. Simplify the expression by combining like terms.
Type below:
_____________

Answer: 12p

Explanation:

7p+5p
When you combine like terms, you just add all the terms that have the same variable
so you get 7p + 5p = 12p

Question 2.
What if the cost of tickets to the exhibit was reduced to $3? Write an expression for the new cost in dollars for p people to visit the museum and attend the exhibit. Then, simplify the expression by combining like terms.
Type below:
_____________

Answer: 10p

Explanation:
Museum admission costs $7, and tickets to the mammoth exhibit cost $5.
The expression 7p + 5p represents the cost in dollars for p people to visit the museum and attend the exhibit.
The cost of tickets to the mammoth exhibit is $5.
If it is reduced to $3 then the cost will be $5 – $2 = $3
12p – 2p = 10p

Question 3.
A store receives tomatoes in boxes of 40 tomatoes each. About 4 tomatoes per box cannot be sold due to damage. The expression 40b − 4b gives the number of tomatoes that the store can sell from a shipment of b boxes. Simplify the expression by combining like terms.
Type below:
_____________

Answer: 36b

Explanation:
Given, A store receives tomatoes in boxes of 40 tomatoes each.
About 4 tomatoes per box cannot be sold due to damage.
The expression 40b − 4b gives the number of tomatoes that the store can sell from a shipment of b boxes.
Subtract 40b and 4b
40b – 4b = 36b

Question 4.
Each cheerleading uniform includes a shirt and a skirt. The shirts cost $12 each, and skirts cost $18 each. The expression 12u + 18u represents the cost in dollars of buying u uniforms. Simplify the expression by combining like terms.
Type below:
_____________

Answer: 30u

Explanation:
The expression 12u + 18u represents the cost in dollars of buying u uniforms.
12u and 18u are the like terms. So add the two terms
12u + 18u = 30u

Question 5.
A shop sells vases holding 9 red roses and 6 white roses. The expression 9v + 6v represents the total number of roses needed for v vases. Simplify the expression by combining like terms.
Type below:
_____________

Answer: 15v

Explanation:
A shop sells vases holding 9 red roses and 6 white roses.
The expression 9v + 6v represents the total number of roses needed for v vases.
The like terms are 9v and 6v
9v + 6v = 15v

On Your Own – Page No. 398

Question 6.
Marco received a gift card. He used it to buy 2 bike lights for $10.50 each. Then he bought a handlebar bag for $18.25. After these purchases, he had $0.75 left on the card. How much money was on the gift card when Marco received it?
$ _______

Answer:
Marco received a gift card. He used it to buy 2 bike lights for $10.50 each.
Then he bought a handlebar bag for $18.25.
After these purchases, he had $0.75 left on the card.
Add total amount = 2 × $10.50 + $18.25 + $0.75
$21 + $19 = $40
$40 was on the gift card when Marco received it.

Question 7.
Lydia collects shells. She has 24 sea snail shells, 16 conch shells, and 32 scallop shells. She wants to display the shells in equal rows, with only one type of shell in each row. What is the greatest number of shells Lydia can put in each row?
_______ shells

Answer: 8 shells

Explanation:
Lydia collects shells. She has 24 sea snail shells, 16 conch shells, and 32 scallop shells.
She wants to display the shells in equal rows, with only one type of shell in each row.
The possible shells in equal rows are 8 because 16, 24, and 32 are the multiples of 8.
Thus the greatest number of shells Lydia can put in each row is 8.

Question 8.
The three sides of a triangle measure 3x + 6 inches, 5x inches, and 6x inches. Write an expression for the perimeter of the triangle in inches. Then simplify the expression by combining like terms.
Type below:
_____________

Answer:
Perimeter of the triangle = a + b + c
Let a = 3x + 6 inches
b = 5x inches
c = 6x inches
P = a + b + c
P = 3x + 6 + 5x + 6x
Combine the like terms 3x, 5x, 6x
P = 14x + 6
Thus the perimeter of the triangle is 14x + 6.

Question 9.
Verify the Reasoning of Others Karina states that you can simplify the expression 20x + 4 by combining like terms to get 24x. Does Karina’s statement make sense? Explain.
Type below:
_____________

Answer: Karina’s statement doesn’t make sense. Because the 20x + 4 are not the like terms.
We can add only the like terms. 20x + 4 ≠ 24x

Question 10.
Vincent is ordering accessories for his surfboard. A set of fins costs $24 each and a leash costs $15. The shipping cost is $4 per order. The expression 24b + 15b + 4 can be used to find the cost in dollars of buying b fins and b leashes plus the cost of shipping.
For numbers 10a–10c, select True or False for each statement.
10a. The terms are 24b, 15b and 4.
10b. The like terms are 24b and 15b.
10c. The simplified expression is 43b.
10a. _____________
10b. _____________
10c. _____________

Answer:
10a. True
10b. True
10c. False

Explanation:
a. The terms of the expression 24b + 15b + 4 area 24b, 15b, 4.
b. The terms are said to be like if they have the common variable. So, the common terms are 24b, 15b.
c. Combine the like terms 24b and 15b
24b + 15b = 39b
Thus the statement is false.

Problem Solving Combine Like Terms – Page No. 399

Read each problem and solve.

Question 1.
A box of pens costs $3 and a box of markers costs $5. The expression 3p + 5p represents the cost in dollars to make p packages that include 1 box of pens and 1 box of markers. Simplify the expression by combining like terms.
Type below:
_____________

Answer: 3p + 5p = 8p

Explanation:
A box of pens costs $3 and a box of markers costs $5.
The expression 3p + 5p represents the cost in dollars to make p packages that include 1 box of pens and 1 box of markers.
Adding the like terms 3p + 5p is 8p.

Question 2.
Riley’s parents got a cell phone plan that has a $40 monthly fee for the first phone. For each extra phone, there is a $15 phone service charge and a $10 text service charge. The expression 40 + 15e + 10e represents the total phone bill in dollars, where e is the number of extra phones. Simplify the expression by combining like terms.
Type below:
_____________

Answer: 25e + 40

Explanation:
Given that,
Riley’s parents got a cell phone plan that has a $40 monthly fee for the first phone.
For each extra phone, there is a $15 phone service charge and a $10 text service charge.
The expression 40 + 15e + 10e represents the total phone bill in dollars,
We have to combine the like terms here
The like terms in the expression are 15e and 10e.
That means 40 + 15e + 10e = 25e + 40

Question 3.
A radio show lasts for h hours. For every 60 minutes of air time during the show, there are 8 minutes of commercials. The expression 60h – 8h represents the air time in minutes available for talk and music. Simplify the expression by combining like terms.
Type below:
_____________

Answer: 52h

Explanation:
A radio show lasts for h hours. For every 60 minutes of air time during the show, there are 8 minutes of commercials.
The expression 60h – 8h represents the air time in minutes available for talk and music.
Now we have to Subtract the like terms 60h – 8h = 52h

Question 4.
A sub shop sells a meal that includes an Italian sub for $6 and chips for $2. If a customer purchases more than 3 meals, he or she receives a $5 discount. The expression 6m + 2m – 5 shows the cost in dollars of the customer’s order for m meals, where m is greater than 3. Simplify the expression by combining like terms.
Type below:
_____________

Answer: 8m – 5

Explanation:
The expression is 6m + 2m – 5
Now combine the like terms 6m + 2m – 5 = 8m – 5

Question 5.
Explain how combining like terms is similar to adding and subtracting whole numbers. How are they different?
Type below:
_____________

Answer: It’s the same because you are adding or subtracting numbers but it’s different because they can only be added or subtracted if the variable attached is the same. There are no variables when adding/subtracting regular whole numbers.

Lesson Check – Page No. 400

Question 1.
For each gym class, a school has 10 soccer balls and 6 volleyballs. All of the classes share 15 basketballs. The expression 10c + 6c + 15 represents the total number of balls the school has for c classes. What is a simpler form of the expression?
Type below:
_____________

Answer: 16c + 15

Explanation:
For each gym class, a school has 10 soccer balls and 6 volleyballs.
All of the classes share 15 basketballs.
c represents classes.
The expression is 10c + 6c + 15
Combine the like terms 10c and 6c
Now add common terms 10c + 6c + 15 = 16c + 15

Question 2.
A public library wants to place 4 magazines and 9 books on each display shelf. The expression 4s + 9s represents the total number of items that will be displayed on s shelves. Simplify this expression.
Type below:
_____________

Answer: 13s

Explanation:
A public library wants to place 4 magazines and 9 books on each display shelf.
The expression is 4s + 9s
Combine the like terms 4s + 9s = 13s

Spiral Review

Question 3.
A bag has 8 bagels. Three of the bagels are cranberry. What percent of the bagels are cranberry?
________ %

Answer: 37.5%

Explanation:
[/latex]\frac{3}{8}[/latex] = 0.375
0.375 × 100 = 37.5 %
37.5% of the bagels are cranberry.

Question 4.
How many kilograms are equivalent to 3,200 grams?
________ kilograms

Answer: 3.2 kg

Explanation:
Convert from grams into kilograms
1000 grams = 1kg
3200 grams = 3200 × 1/1000 kg = 3.2 kg
3.2 kilograms are equivalent to 3,200 grams.

Question 5.
Toni earns $200 per week plus $5 for every magazine subscription that she sells. Write an expression that represents how much she will earn in dollars in a week in which she sells s subscriptions.
Type below:
_____________

Answer: 200 + 5s

Explanation:
Toni earns $200 per week plus $5 for every magazine subscription that she sells.
s represents subscriptions.
200 + 5 × s
Thus the expression that represents how much she will earn in dollars in a week is 200 + 5s

Question 6.
At a snack stand, drinks cost $1.50. Write an expression that could be used to find the total cost in dollars of d drinks.
Type below:
_____________

Answer: 1.5d

Explanation:
At a snack stand, drinks cost $1.50.
To find the total cost in dollars of d drinks we have to multiply 1.50 with d.
1.50 × d
Thus the expression that could be used to find the total cost in dollars of d drinks is 1.5d

Share and Show – Page No. 403

Use properties of operations to write an equivalent expression by combining like terms.

Question 1.
\(3 \frac{7}{10} r-1 \frac{1}{5} r\)
Type below:
_____________

Answer: 2 \(frac{5}{10}\)r

Explanation:
3 \(frac{7}{10}\)r – 1 \(frac{1}{5}\)r
3 + \(frac{7}{10}\)r – 1 – \(frac{1}{5}\)r
3 – 1 = 2
\(frac{7}{10}\)r – \(frac{1}{5}\)r
\(frac{7}{10}\)r – \(frac{2}{10}\)r = \(frac{5}{10}\)r
\(3 \frac{7}{10} r-1 \frac{1}{5} r\) = 2 \(frac{5}{10}\)r

Question 2.
20a + 18 + 16a
Type below:
_____________

Answer: 36a + 18

Explanation:
Combine  the like terms first
16a and 20a are like terms in the given expression.
Add 16a and 20a
16a + 20a +18 = 36a + 18

Question 3.
7s + 8t + 10s + 12t
Type below:
_____________

Answer: 17s + 20t

Explanation:
There are 4 terms in the expression they are 7s, 10s, 8t, 12t.
Now combine the like terms
7s + 10s + 8t + 12t = 17s + 20t

Use the Distributive Property to write an equivalent expression.

Question 4.
8(h + 1.5)
Type below:
_____________

Answer: 8h + 12

Explanation:
Here we have to use the distributive property for the above expression.
8(h + 1.5) = 8 × h + 1.5 × 8
= 8h + 12
Thus 8(h + 1.5) is 8h + 12.

Question 5.
4m + 4p
Type below:
_____________

Answer: 4(m + p)

Explanation:
Here we have to take 4 as a common factor from the expression.
4m + 4p = 4 × m + 4 × p
That implies 4 × (m + p)

Question 6.
3a + 9b
Type below:
_____________

Answer: 3(a + 3b)

Explanation:
Let us take 4 as a common factor from the expression.
3a + 9b = 3 × a + 9 × b
3(a + 3b)
3a + 9b = 3(a + 3b)

On Your Own

Practice: Copy and Solve Use the Distributive Property to write an equivalent expression.

Question 7.
3.5(w + 7)
Type below:
_____________

Answer: 3.5w + 24.5

Explanation:
Use the distributive property.
Multiply within the parentheses.
3.5(w + 7) = 3.5 × w + 3.5 × 7
3.5w + 24.5
Thus 3.5(w + 7) = 3.5w + 24.5

Question 8.
\(\frac{1}{2}\)(f + 10)
Type below:
_____________

Answer: \(\frac{1}{2}\)f + 5

Explanation:
\(\frac{1}{2}\)(f + 10)
Use the distributive property.
Multiply within the parentheses.
\(\frac{1}{2}\) × f + \(\frac{1}{2}\) × 10
= \(\frac{1}{2}\)f + 5
Thus \(\frac{1}{2}\)(f + 10) = \(\frac{1}{2}\)f + 5

Question 9.
4(3z + 2)
Type below:
_____________

Answer: 12z + 8

Explanation:
Use the distributive property.
Multiply within the parentheses.
4(3z + 2) = 4 × 3z + 4 × 2
= 12z + 8
So, 4(3z + 2) = 12z + 8

Question 10.
20b + 16c
Type below:
_____________

Answer: 4(5b + 4c)

Explanation:
20b + 16c
Use the distributive property.
Multiply within the parentheses.
Take 4 as a common factor.
20b + 16c = 4 × 5b + 4 × 4c = 4 (5b + 4c)
Thus the expression 20b + 16c = 4 (5b + 4c)

Question 11.
30d + 18
Type below:
_____________

Answer: 6(5d + 3)

Explanation:
30 and 18 are the factors of 6.
So, take 6 as a common factor.
30d + 18 = 6 × 5d + 6 × 3
6 (5d + 3)
30d + 18 = 6 (5d + 3)

Question 12.
24g − 8h
Type below:
_____________

Answer: 8(3g – h)

Explanation:
Given the expression 24g − 8h
24 and 8 are the factors of 8.
So, let us take 8 as a common factor.
24g − 8h = 8 × 3g – 8 × 1h
= 8(3g – h)

Question 13.
Write an Expression The lengths of the sides of a triangle are 3t, 2t + 1, and t + 4. Write an expression for the perimeter (sum of the lengths). Then, write an equivalent expression with 2 terms.
Type below:
_____________

Answer: 6t + 5

Explanation:
Given that, The lengths of the sides of a triangle are 3t, 2t + 1, and t + 4.
We know that the perimeter of the triangle is P = a + b + c
P = 3t + 2t + 1 + t + 4
Combine the like terms.
P = 6t + 5

Question 14.
Use properties of operations to write an expression equivalent to the sum of the expressions 3(g + 5) and 2(3g − 6).
Type below:
_____________

Answer: 3(3g + 1)

Explanation:
Given two expressions 3(g + 5) and 2(3g − 6).
Use the distributive property to simplify the expressions.
3(g + 5) = 3 × g + 3 × 5 = 3g + 15
2(3g − 6) = 2 × 3g – 2 × 6 = 6g – 12
Add both the expressions and combine the like terms
3g + 15 + 6g – 12 = 9g + 3 = 3(3g + 1)

Problem Solving + Applications – Page No. 404

Question 15.
Sense or Nonsense Peter and Jade are using what they know about properties to write an expression equivalent to 2 × (n + 6) + 3. Whose answer makes sense? Whose answer is nonsense? Explain your reasoning.
Peter’s Work:
Expression: 2 × (n + 6) + 3
Associative Property of Addition: 2 × n + (6 + 3)
Add within parentheses: 2 × n + 9
Multiply: 2n + 9

Jade’s Work:
Expression: 2 × (n + 6) + 3
Distributive Property: (2 × n) + (2 × 6) + 3
Multiply within parentheses: 2n + 12 + 3
Associative Property of Addition: 2n + (12 + 3)
Add within parentheses: 2n + 15
For the answer that is nonsense, correct the statement.
Type below:
_____________

Answer: Jade’s Work makes sense. Peter’s Work makes non-sense because
He must have multiplied n + 6 with 2 but he added 6 with 3.
2 × (n + 6) + 3
2 × n + 2 × 6 + 3 = 2n + 12 + 3
= 2n + 15

Question 16.
Write the algebraic expression in the box that shows an equivalent expression.
Go Math Grade 6 Answer Key Chapter 7 Exponents img 11
Type below:
_____________

Answer:
6(z + 5) = 6 × z + 6 × 5 = 6z + 30
6z + 5z = z(6 + 5) = 11z
2 + 6z + 3 = 6z + 5

Generate Equivalent Expressions – Page No. 405

Use properties of operations to write an equivalent expression by combining like terms.

Question 1.
7h − 3h
Type below:
_____________

Answer: 4h

Explanation:
Combine the like terms
7h and 3h are the common terms
Now subtract 3h from 7h
7h – 3h = 4h

Question 2.
5x + 7 + 2x
Type below:
_____________

Answer: 7x + 7

Explanation:
The given expression is 5x + 7 + 2x
The common terms are 5x and 2x
Combine the like terms 5x + 7 + 2x = 7x + 7

Question 3.
16 + 13p − 9p
Type below:
_____________

Answer: 16 + 4p

Explanation:

Combine the like terms for the above expressions.
The like terms are 13p and 9p
16 + 13p − 9p = 16 + 4p

Question 4.
y2 + 13y − 8y
Type below:
_____________

Answer: y2 + 5y

Explanation:
The given expression is y2 + 13y − 8y
The like terms are 13y and 8y
y2 + 13y − 8y = y2 + 5y

Question 5.
5(2h + 3) + 3h
Type below:
_____________

Answer: 13h + 15

Explanation:
5(2h + 3) + 3h = 10h + 15 + 3h
The like terms are 10h and 3h
10h + 15 + 3h = 13h + 15

Question 6.
12 + 18n + 7 − 14n
Type below:
_____________

Answer: 19 + 4n

Explanation:
The expression is 12 + 18n + 7 − 14n
The like terms are 18n and 14n
12 + 18n + 7 − 14n = 19 + 4n

Use the Distributive Property to write an equivalent expression.

Question 7.
2(9 + 5k)
Type below:
_____________

Answer: 18 + 10k

Explanation:
Use the Distributive property
Multiply within the parentheses.
2(9 + 5k) = (2 × 9) + (2 × 5k)
(2 × 9) + (2 × 5k) = 18 + 10k

Question 8.
4d + 8
Type below:
_____________

Answer: 4(d + 2)

Explanation:
Use the Distributive property
Multiply within the parentheses.
4d + 8 = 4 × d + 4 × 2
The common term is 4.
Take 4 as a common factor.
4d + 8 = 4 (d + 2)

Question 9.
21p + 35q
Type below:
_____________

Answer: 7(3p + 5q)

Explanation:
Use the Distributive property
Multiply within the parentheses.
7 × 3p + 7 × 5q
The common term is 7.
7(3p + 5q)
21p + 35q = 7(3p + 5q)

Problem Solving

Question 10.
The expression 15n + 12n + 100 represents the total cost in dollars for skis, boots, and a lesson for n skiers. Simplify the expression 15n + 12n + 100. Then find the total cost for 8 skiers.
Type below:
_____________

Answer: 27n + 100, $316

Explanation:
The terms that have n can be operated:
15n +12n + 100 = 27n +100. Then, we have that
total cost = 27n +100 for n skiers. So, for 8 skiers we have
total cost = 27(8) +100 = 216 + 100 = 316.
Then, the total cost of 8 skiers is $316.

Question 11.
Casey has n nickels. Megan has 4 times as many nickels as Casey has. Write an expression for the total number of nickels Casey and Megan have. Then simplify the expression.
Type below:
_____________

Answer: n + 4n; 5n

Explanation:
Casey has n nickels. Megan has 4 times as many nickels as Casey has.
Sum of n and 4n
Add the common terms n and 4n.
n + 4n = 5n

Question 12.
Explain how you would use properties to write an expression equivalent to 7y + 4b – 3y.
Type below:
_____________

Answer:
1st you combine like terms so subtract 7y and 3y and you get 4y.
So this is the final answer: 4y+4b.

Lesson Check – Page No. 406

Question 1.
A ticket to a museum costs $8. A ticket to the dinosaur exhibit costs $5. The expression 8n + 5n represents the cost in dollars for n people to visit the museum and the exhibit. What is a simpler form of the expression 8n + 5n?
Type below:
_____________

Answer: 13n

Explanation:
A ticket to a museum costs $8. A ticket to the dinosaur exhibit costs $5.
The expression is the sum of 8n and 5n.
Thus the simpler form of the expression is 8n + 5n = 13n

Question 2.
What is an expression that is equivalent to 3(2p – 3)?
Type below:
_____________

Answer: 6p – 9

Explanation:
Use the distributive property to find the equivalent expression.
3(2p – 3) = 3 × 2p – 3 × 3
= 6p – 9
Thus the expression that is equivalent to 3(2p – 3) is 6p – 9.

Question 3.
A Mexican restaurant received 60 take-out orders. The manager found that 60% of the orders were for tacos and 25% of the orders were for burritos. How many orders were for other items?
______ orders

Answer: 9 orders

Explanation:
Given,
A Mexican restaurant received 60 take-out orders.
The manager found that 60% of the orders were for tacos and 25% of the orders were for burritos.
The answer is 9 because 25% of 60 is 15 plus 60% of 60 is 36 so 36+15=51 and 60-51=9
Thus 9 orders were for other items.

Question 4.
The area of a rectangular field is 1,710 square feet. The length of the field is 45 feet. What is the width of the field?
______ feet

Answer: 38 feet

Explanation:
The area of a rectangular field is 1,710 square feet.
The length of the field is 45 feet.
The width of the field is x feet
A = l × w
1710 square feet = 45 feet × x
x = 1710/45 = 38 feet
Thus the width of the rectangular field is 38 feet.

Question 5.
How many terms are in 2 + 4x + 7y?
______ terms

Answer: 3

Explanation:
Given expression 2 + 4x + 7y
There are 3 terms in the expression 2, 4x, 7y.

Question 6.
Boxes of cereal usually cost $4, but they are on sale for $1 off. A gallon of milk costs $3. The expression 4b – 1b + 3 can be used to find the cost in dollars of buying b boxes of cereal and a gallon of milk. Write the expression in a simpler form.
Type below:
_____________

Answer: 3b + 3

Explanation:
Boxes of cereal usually cost $4, but they are on sale for $1 off. A gallon of milk costs $3.
The expression is 4b – 1b + 3
Combine the like terms for the above expression.
4b – 1b + 3 = 3b + 3 = 3(b + 1)

Share and Show – Page No. 409

Use properties of operations to determine whether the expressions are equivalent.

Question 1.
7k + 4 + 2k and 4 + 9k
The expressions are _____________

Answer: equivalent

Explanation:
7k + 4 + 2k
Combine the like terms 7k and 2k
Add the like terms 7k + 2k + 4 = 9k + 4
9k + 4 and 4 + 9k are equivalent.

Question 2.
9a × 3 and 12a
The expressions are _____________

Answer: not equivalent

Explanation:
Multiply 9a with 3.
9a × 3 = 27a
27a and 12a are not equivalent.
Thus the expressions are not equivalent.

Question 3.
8p + 0 and 8p × 0
The expressions are ______

Answer: not equivalent

Explanation:
8p + 0 = 8p
8p × 0 = 0
8p and 0 are not equivalent.
The expressions 8p + 0 and 8p × 0 are not equivalent.

Question 4.
5(a + b) and (5a + 2b) + 3b
The expressions are _____________

Answer: equivalent

Explanation:
5(a + b) = 5a + 5b
(5a + 2b) + 3b
The like terms are 5a and 2b, 3b
Add the combine terms 5a + 2b + 3b = 5a + 5b
Thus the expressions 5(a + b) and (5a + 2b) + 3b are equivalent.

On Your Own

Use properties of operations to determine whether the expressions are equivalent.

Question 5.
3(v + 2) + 7v and 16v
The expressions are _____________

Answer: not equivalent

Explanation:
3(v + 2) + 7v
Combine the like terms 3v and 7v
3(v + 2) + 7v = 3v + 6 + 7v = 10v + 6
The expressions 10v + 6 and 16v are not equivalent.

Question 6.
14h + (17 + 11h) and 25h + 17
The expressions are _____________

Answer: equivalent

Explanation:
14h + (17 + 11h)
Combine the like terms 14h and 11h.
14h + 17 + 11h = 25h + 17
The expressions 14h + (17 + 11h) and 25h + 17 are equivalent.

Question 7.
4b × 7 and 28b
The expressions are _____________

Answer: equivalent

Explanation:
Multiply 4b with 7.
4b × 7 = 28b
The expressions 4b × 7 and 28b are equivalent.

Question 8.
Each case of dog food contains c cans. Each case of cat food contains 12 cans. Four students wrote the expressions below for the number of cans in 6 cases of dog food and 1 case of cat food. Which of the expressions are correct?
6c + 12     6c × 12      6(c + 2)      (2c + 4) × 3
Type below:
_____________

Answer: The correct expressions are 6c + 12, 6(c + 2), (2c + 4) × 3
6(c + 2) is the distributive form of the expression.

Problem Solving + Applications – Page No. 410

Use the table for 9–11.
Go Math Grade 6 Answer Key Chapter 7 Exponents img 12

Question 9.
Marcus bought 4 packets of baseball cards and 4 packets of animal cards. Write an algebraic expression for the total number of cards Marcus bought.
Type below:
_____________

Answer: 4a + 4b

Explanation:
Marcus bought 4 packets of baseball cards and 4 packets of animal cards.
b represents the number per packet
Multiply 4 with b
4 × b = 4b
a represents the number per packet of animal cards.
Multiply 4 with a.
4 × a = 4a
Therefore the algebraic expression for the total number of cards Marcus bought is the sum of 4a and 4b.
The expression is 4a + 4b

Question 10.
Make Arguments Is the expression for the number of cards Marcus bought equivalent to 4(a + b)? Justify your answer.
Type below:
_____________

Answer: Yes
Use the distributive property to simplify the expression 4a + 4b.
Take 4 as the common factor for the expression 4a + 4b.
4a + 4b = 4(a + b)

Question 11.
Angelica buys 3 packets of movie cards and 6 packets of cartoon cards and adds these to the 3 packets of movie cards she already has. Write three equivalent algebraic expressions for the number of cards Angelica has now
Type below:
_____________

Answer: 3m + 6c + 3m

Explanation:
Angelica buys 3 packets of movie cards and 6 packets of cartoon cards and adds these to the 3 packets of movie cards she already has.
The expression for 3 packets of movie cards is 3m
The expression for 6 packets of cartoon cards is 6c.
Now we have to add 3m to the expression.
3m + 6c + 3m
Thus the three equivalent algebraic expressions for the number of cards Angelica has now is 3m + 6c + 3m

Question 12.
Select the expressions that are equivalent to 3(x + 2). Mark all that apply.
Options:
a. 3x + 6
b. 3x + 2
c. 5x
d. x + 5

Answer: 3x + 6

Explanation:
Use distributive property to solve the expression 3(x + 2).
3(x + 2) = 3 × x + 3 × 2 = 3x + 6
Thus the correct answer is option A.

Identify Equivalent Expressions – Page No. 411

Use properties of operations to determine whether the expressions are equivalent.

Question 1.
2s + 13 + 15s and 17s + 13
The expressions are _____________

Answer: equivalent

Explanation:
2s + 13 + 15s
Combine the like terms
2s + 13 + 15s = 17s + 13
17s + 13 = 17s + 13
Thus the expressions 2s + 13 + 15s and 17s + 13 are equivalent.

Question 2.
5 × 7h and 35h
The expressions are _____________

Answer: equivalent

Explanation:
5 × 7h = 35h
35h = 35h
The expressions 5 × 7h and 35h are equivalent.

Question 3.
10 + 8v − 3v and 18 − 3v
The expressions are _____________

Answer: not equivalent

Explanation:
Combine the like terms 8v and 3v
10 + 8v − 3v = 10 + 5v
10 + 5v ≠ 18 − 3v
Thus the expressions 10 + 8v − 3v and 18 − 3v are not equivalent.

Question 4.
(9w × 0)−12 and 9w – 12
The expressions are _____________

Answer: not equivalent

Explanation:
(9w × 0)−12 = 0 – 12 = – 12
– 12 ≠ 9w – 12
So, the expressions (9w × 0)−12 and 9w – 12 are not equivalent.

Question 5.
11(p + q) and 11p + (7q + 4q)
The expressions are _____________

Answer: equivalent

Explanation:
11(p + q) = 11p + 11q
Combine the terms 7q and 4q
11p + (7q + 4q) = 11p + 11q = 11(p + q)
So, the expressions 11(p + q) and 11p + (7q + 4q) are equivalent.

Question 6.
6(4b + 3d) and 24b + 3d
The expressions are _____________

Answer: not equivalent

Explanation:
6(4b + 3d) = 24b + 18d
24b + 18d ≠ 24b + 3d
So, the expressions 6(4b + 3d) and 24b + 3d are not equivalent.

Question 7.
14m + 9 − 6m and 8m + 9
The expressions are _____________

Answer: equivalent

Explanation:
Combine the like terms 14m and 6m
14m + 9 − 6m = 8m + 9
8m + 9 = 8m + 9
Thus the expressions are equivalent.

Question 8.
(y × 1) + 2 and y + 2
The expressions are _____________

Answer: equivalent

Explanation:
(y × 1) + 2 = y + 2
y + 2 = y + 2
Thus the expressions (y × 1) + 2 and y + 2 are equivalent.

Question 9.
4 + 5(6t + 1) and 9 + 30t
The expressions are _____________

Answer: equivalent

Explanation:
4 + 5(6t + 1) = 4 + 30t + 5 = 9 + 30t
9 + 30t = 9 + 30t
Thus the expressions 4 + 5(6t + 1) and 9 + 30t are equivalent.

Question 10.
9x + 0 + 10x and 19x + 1
The expressions are _____________

Answer: not equivalent

Explanation:
9x + 0 + 10x
Combine the like terms 9x and 10x.
9x + 10x = 19x
19x ≠ 19x + 1
Thus the expressions 9x + 0 + 10x and 19x + 1 are not equivalent.

Question 11.
12c − 3c and 3(4c − 1)
The expressions are _____________

Answer: not equivalent

Explanation:
12c − 3c
Take 3 as a common factor.
3c(4 – 1) or 3 (4c – 1c)
3 (4c – 1c) ≠ 3(4c − 1)
Thus the expressions 12c − 3c and 3(4c − 1) are not equivalent.

Question 12.
6a × 4 and 24a
The expressions are _____________

Answer: equivalent

Explanation:
6a × 4 = 24a
24a = 24a
The expressions 6a × 4 and 24a are equivalent.

Problem Solving

Question 13.
Rachel needs to write 3 book reports with b pages and 3 science reports with s pages during the school year. Write an algebraic expression for the total number of pages Rachel will need to write.
Type below:
_____________

Answer: 3b + 3s

Explanation:
Rachel needs to write 3 book reports with b pages and 3 science reports with s pages during the school year.
Multiply 3 book reports with b pages = 3b.
Multiply 3 science books with s pages = 3s.
The algebraic expression for the total number of pages Rachel will need to write is 3b + 3s.

Question 14.
Rachel’s friend Yassi has to write 3(b + s) pages for reports. Use properties of operations to determine whether this expression is equivalent to the expression for the number of pages Rachel has to write.
This expression is _____________

Answer: equivalent

Explanation:
Rachel’s friend Yassi has to write 3(b + s) pages for reports.
The equivalent expression of 3(b + s) = 3b + 3s

Question 15.
Use properties of operations to show whether 7y + 7b + 3y and 7(y + b) + 3b are equivalent expressions. Explain your reasoning.
Type below:
_____________

Answer:
Use Distributive property to simplify the expressions.
The equivalent expression of 7y + 7b + 3y = 7(y + b) + 3y
Thus 7y + 7b + 3y and 7(y + b) + 3b are equivalent.

Lesson Check – Page No. 412

Question 1.
Ian had 4 cases of comic books and 6 adventure books. Each case holds c comic books. He gave 1 case of comic books to his friend. Write an expression that gives the total number of books Ian has left.
Type below:
_____________

Answer: 3c + 6

Explanation:
Ian had 4 cases of comic books and 6 adventure books. Each case holds c comic books. He gave 1 case of comic books to his friend.
4c + 6 – 1c
Combine the like terms
3c + 6

Question 2.
In May, Xia made 5 flower planters with f flowers in each planter. In June, she made 8 flower planters with f flowers in each planter. Write an expression in the simplest form that gives the number of flowers Xia has in the planters.
Type below:
_____________

Answer: 13f

Explanation:
In May, Xia made 5 flower planters with f flowers in each planter.
The expression is 5f
In June, she made 8 flower planters with f flowers in each planter.
The expression is 8f.
Sum of 5f and 8f is 8f + 5f = 13f

Spiral Review

Question 3.
Keisha wants to read for 90 minutes. So far, she has read 30% of her goal. How much longer does she need to read to reach her goal?
________ minutes

Answer: 63 min

Explanation:
Keisha wants to read for 90 minutes.
So far, she has read 30% of her goal.
30% = 30/100 = 0.3
Multiply 90 with 0.3
90 × 0.3 = 27
Subtract 27 from 90
90 – 27 = 63
She needs to read 63 minutes to reach her goal.

Question 4.
Marvyn travels 105 miles on his scooter. He travels for 3 hours. What is his average speed?
________ miles per hour

Answer: 35 miles per hour

Explanation:
Divide the number of miles by hours traveled.
Average speed = 105 miles/3 hours = 35 miles per hour
Thus the average speed is 35 miles per hour.

Question 5.
The expression 5(F − 32) ÷ 9 gives the Celsius temperature for a Fahrenheit temperature of F degrees. The noon Fahrenheit temperature in Centerville was 86 degrees. What was the temperature in degrees Celsius?
________ degrees Celsius

Answer: 30 degrees Celsius

Explanation:
The expression is 5(F − 32) ÷ 9
F = 86 degrees
Substitute F in the above expression.
5(86 − 32) ÷ 9 = 5(54) ÷ 9
270 ÷ 9 = 30
The temperature is 30 degrees Celsius

Question 6.
At the library book sale, hardcover books sell for $4 and paperbacks sell for $2. The expression 4b + 2b represents the total cost for b hardcover books and b paperbacks. Write a simpler expression that is equivalent to 4b + 2b.
Type below:
_____________

Answer: 6b

Explanation:
Given expression is 4b + 2b
The terms are 4b and 2b
Now combine the like terms
That means 4b + 2b = 6b

Chapter 7 Review/Test – Page No. 413

Question 1.
Use exponents to rewrite the expression.
3 × 3 × 3 × 3 × 5 × 5
Type below:
_____________

Answer: 34 × 52

Explanation:
3 is a repeated factor.
The number 3 is repeated four times.
5 is a repeated factor.
The number 5 is repeated two times.
The exponential form of 3 × 3 × 3 × 3 × 5 × 5 is 34 × 52

Question 2.
A plumber charges $10 for transportation and $55 per hour for repairs. Write an expression that can be used to find the cost in dollars for a repair that takes h hours.
Type below:
_____________

Answer: 10 + 55h

Explanation:
A plumber charges $10 for transportation and $55 per hour for repairs.
Multiply 55 with an hour
Sum of 10 and product of 55 and h.
The expression is 10 + 55h.

Question 3.
Ellen is 2 years older than her brother Luke. Let k represent Luke’s age. Identify the expression that can be used to find Ellen’s age.
Options:
a. k−2
b. k+2
c. 2k
d. \(\frac{k}{2}\)

Answer: k+2

Explanation:
Given, Ellen is 2 years older than her brother Luke. Let k represent Luke’s age.
Older is nothing but more so we have to add 2 years to k.
That means k + 2.
Thus the correct answer is option B.

Question 4.
Write 43 using repeated multiplication. Then find the value of 43.
________

Answer:
43 = 4 × 4 × 4 = 64
The value of 43 is 64.

Question 5.
Jasmine is buying beans. She bought r pounds of red beans that cost $3 per pound and b pounds of black beans that cost $2 per pound. The total amount of her purchase is given by the expression 3r + 2b. Select the terms of the expression. Mark all that apply
Options:
a. 2
b. 2b
c. 3
d. 3r

Answer: B, D

Explanation:
The expression is 3r + 2b
The terms of the expressions are 3r and 2b.
Thus the correct answers are B and D.

Chapter 7 Review/Test – Page No. 414

Question 6.
Choose the number that makes the sentence true. The formula V= s3 gives the volume V of a cube with side length s.
The volume of a cube that has a side length of 8 inches
inches is _____________ cubed

Answer: 512

Explanation:
Use the formula V= s3
s = 8
V = 83 = 8 × 8 × 8 = 512

Question 7.
Liang is ordering new chairs and cushions for his dining room table. A new chair costs $88 and a new cushion costs $12. Shipping costs $34. The expression 88c + 12c + 34 gives the total cost for buying c sets of chairs and cushions. Simplify the expression by combining like terms.
Type below:
_____________

Answer: 100c + 34

Explanation:
Liang is ordering new chairs and cushions for his dining room table.
A new chair costs $88 and a new cushion costs $12. Shipping costs $34.
The expression is 88c + 12c + 34.
Combine the like terms
88c + 12c + 34 = 100c + 34

Question 8.
Mr. Ruiz writes the expression 5 × (2 + 1)2 ÷ 3 on the board. Chelsea says the first step is to evaluate 12. Explain Chelsea’s mistake. Then, evaluate the expression
_____________

Answer:
She should have done what was in the parentheses (2 + 1) and then the exponent 32= 9
5 × (2 + 1)2 ÷ 3 = 5 × 9 ÷ 3
5 × 3 = 15

Question 9.
Jake writes this word expression.
Go Math Grade 6 Answer Key Chapter 7 Exponents img 13
Write an algebraic expression for the word expression. Then, evaluate the expression for m = 4. Show your work.
________

Answer:
The expression is 7m
Replace m = 4 with m
7m = 7 × 4 = 28

Chapter 7 Review/Test – Page No. 415

Question 10.
Sora has some bags that each contain 12 potatoes. She takes 3 potatoes from each bag. The expression 12p – 3p represents the number of potatoes p left in the bags. Simplify the expression by combining like terms. Draw a line to match the expression with the simplified expression.
Go Math Grade 6 Answer Key Chapter 7 Exponents img 14
Type below:
_____________

Answer: 9p
Go-Math-Grade-6-Answer-Key-Chapter-7-Exponents-img-14

Question 11.
Logan works at a florist. He earns $600 per week plus $5 for each floral arrangement he delivers. Write an expression that gives the amount in dollars that Logan earns for delivering f floral arrangements. Use the expression to find the amount Logan will earn if he delivers 45 floral arrangements in one week. Show your work.
$ ________

Answer: $825

Explanation:
Logan works at a florist. He earns $600 per week plus $5 for each floral arrangement he delivers. Write an expression that gives the amount in dollars that Logan earns for delivering f floral arrangements.
The expression is 600 + 5f
f = 45
600 + 5f = 600 + 5(45)
600 + 225 = 825
Thus Logan earned $825 for delivering f floral arrangements.

Question 12.
Choose the word that makes the sentence true.
Dara wrote the expression 7 × (d + 4) in her notebook. She used the _____ Property to write the equivalent expression 7d + 28.
Answer: Dara wrote the expression 7 × (d + 4) in her notebook. She used the Distributive Property to write the equivalent expression 7d + 28.
Use the distributive property to simplify the expression.
7 × (d + 4) = 7d + 28

Chapter 7 Review/Test – Page No. 416

Question 13.
Use properties of operations to determine whether 5(n + 1) + 2n and 7n + 1 are equivalent expressions.
The expressions are _____________

Answer:
5n + 5 + 2n is Distributive property
5n + 2n + 5 is Commutative property of addition
7n + 5 combine like term
5n + 5 + 2n is equivalent to 7n + 5
Since it is not equivalent to 7n + 1, 7n + 5 is not equivalent to 7n.

Question 14.
Alisha buys 5 boxes of peanut butter granola bars and 5 boxes of cinnamon granola bars. Let p represent the number of bars in a box of peanut butter granola bars and c represents the number of bars in a box of cinnamon granola bars. Jaira and Emma each write an expression that represents the total number of granola bars Alisha bought. Are the equivalent of the expression? Justify your answer
Jaira
5p + 5c
Emma
5(p + c)
Type below:
_____________

Answer:
They are equivalent statements.
5p + 5c = 5(p + c) by the distributive property.

Question 15.
Abe is 3 inches taller than Chen. Select the expressions that represent Abe’s height if Chen’s height is h inches. Mark all that apply
Options:
a. h−3
b. h+3
c. the sum of h and 3
d. the difference between h and 3

Answer:
Abe is 3 inches taller than Chen.
Let Chen’s height is h.
The expression is the sum of Chen’s height and 3.
So, the suitable answers are h + 3 and the sum of h and 3.
Thus the correct answers are option B and C.

Question 16.
Write the algebraic expression in the box that shows an equivalent expression.
Go Math Grade 6 Answer Key Chapter 7 Exponents img 15
Type below:
_____________

Answer:
3(k + 2) = 3k + 6
3k + 2k = 5k
2 + 6k + 3 = 6k + 5

Chapter 7 Review/Test – Page No. 417

Question 17.
Draw a line to match the property with the expression that shows the property.
Go Math Grade 6 Answer Key Chapter 7 Exponents img 16
Type below:
_____________

Answer:
Go-Math-Grade-6-Answer-Key-Chapter-7-Exponents-img-16

Question 18.
A bike rental company charges $10 to rent a bike plus $2 for each hour the bike is rented. An expression for the total cost of renting a bike for h hours is 10 + 2h. Complete the table to find the total cost of renting a bike for h hours.
Go Math Grade 6 Answer Key Chapter 7 Exponents img 17
Type below:
_____________

Answer:
Go-Math-Grade-6-Answer-Key-Chapter-7-Exponents-img-17

Question 19.
An online sporting goods store charges $12 for a pair of athletic socks. Shipping is $2 per order
Part A
Write an expression that Hana can use to find the total cost in dollars for ordering n pairs of socks.
Type below:
_____________

Answer: 12n + 2

Explanation:
Let n represents a pair of socks.
Multiply the price of pair of athletic socks with pair of socks = 12 × n
Shipping is $2 per order
The expression for the total cost in dollars for ordering n pairs of socks is 12n + 2

Question 19.
Part B
Hana orders 3 pairs of athletic socks and her friend, Charlie, order 2 pairs of athletic socks. What is the total cost, including shipping, for both orders? Show your work.
$ ________

Answer:
The cost of Hannah’s order is 12 × 3 + 2 = 36 + 2 = 38
The cost of Charlie’s order is 12 × 2 + 2 = 24 + 2 = 26
The total cost for both is 38 + 26 = 64

Chapter 7 Review/Test – Page No. 418

Question 20.
Fernando simplifies the expression (6 + 2)2 – 4 × 3.
Part A
Fernando shows his work on the board. Use numbers and words to explain his mistake.
(6 + 2)2 – 4 × 3
(6 + 4) – 4 × 3
10 − 4 × 3
6 × 3
18
Type below:
_____________

Answer: Fernando did not use the correct order of operations. He should have added 6 and 2, then evaluate the exponent. He also subtracted before multiplying. He should have multiplied first.

Question 20.
Part B
Simplify the expression (6 + 2)2 − 4 × 3 using the order of operations.
_______

Answer: 52

Explanation:
(6 + 2)2 − 4 × 3
First, add 6 and 2 and then subtract with 12.
82 – 4 × 3
= 64 – 12 = 52
(6 + 2)2 − 4 × 3 = 52

Conclusion:

In addition to the exercise and homework problems, we have given the solutions for the mid-chapter and review test. Hence the students of grade 6 can check whether the answers are right or wrong. Feel free to post your comments in the below comment box if you have any queries. Bookmark our ccssmathanswers.com to get the go math answer key for all grade 6 chapters.

Go Math Grade 5 Answer Key Chapter 7 Multiply Fractions

go-math-grade-5-chapter-7-multiply-fractions-answer-key

If you are looking for Chapter 7 Go Math Grade 5 Answer Key then you have reached the right destination.  We have mentioned Go Math Grade 5 Answer Key Chapter 7 Multiply Fractions with straight forward explanation for all the Problems. Parents who wanted to give their kids enough practice on concepts of Multiplying Fractions can give them the Go Math 5th Grade Answer Key Chapter 7 Multiply Fractions. Enhance your Math Skills by practicing from the HMH Go Math Grade 5 Answer Key Ch 7 Multiply Fractions.

Go Math Grade 5 Answer Key Chapter 7 Multiply Fractions

Do you feel the concept of Multiplying Fractions difficult? Not anymore with our Go Math Answer Key Grade 5 Chapter 7 Multiply Fractions as they have concepts explained in detail. Start learning the basics involved right from the basic level with our 5th Grade Go Math Answer Key Ch 7 Multiply Fractions. Test your preparation level by answering the questions at the end of the Go Math Grade 5 Answer Key for Multiply Fractions. By doing so, you can have an estimation of your preparation standards.

Chapter 7 – Lesson 1: Find Part of a Group

Chapter 7 – Lesson 2: Investigate • Multiply Fractions and Whole Numbers

Chapter 7 – Lesson 3: Fraction and Whole Number Multiplication

Chapter 7 – Lesson 4: Investigate • Multiply Fractions

Chapter 7 – Lesson 5: Compare Fraction Factors and Products

Chapter 7 – Lesson 6: Fraction Multiplication

Chapter 7 – Mid-Chapter Checkpoint

Chapter 7 – Lesson 7: Investigate • Area and Mixed Numbers

Chapter 7 – Lesson 8: Compare Mixed Number Factors and Products

Chapter 7 – Lesson 9: Multiply Mixed Numbers

Chapter 7 – Lesson 10: Problem Solving • Find Unknown Lengths

Chapter 7 – Chapter 7 Review/Test

Share and Show – Page No. 293

Question 1.
Complete the model to solve.
Go Math Grade 5 Answer Key Chapter 7 Multiply Fractions img 1
\(\frac{7}{8}\) of 16, or \(\frac{7}{8}\) × 16
How many rows of counters are there?
_____ rows

Answer: 8
By seeing the above figure we can say that the number of counters is 8 rows.

Question 1.
How many counters are in each row?
_____ counters

Answer: 2
There are 2 counters in each row.

Question 1.
Circle ____ rows to solve the problem.
_____ rows

Answer: 7

• • • • • • •
• • • • • • •
• • • • • • •
• • • • • • •
• • • • • • •
• • • • • • •
• • • • • • •
• • • • • • •

Question 1.
How many counters are circled?
\(\frac{7}{8}\) of 16=
or \(\frac{7}{8}\) × 16 =
_____ counters

Answer: 14
\(\frac{7}{8}\) × 16
8 divides 16 two times.
So, \(\frac{7}{8}\) × 16 = 7 × 2 = 14
Therefore 14 counters are circled.

Use a model to solve.

Question 2.
\(\frac{2}{3}\) × 18 = _____

Answer: 12

Explanation:
\(\frac{2}{3}\) × 18
3 divides 18 six times.
2 × 6 = 12

Question 3.
\(\frac{2}{5}\) × 15 = _____

Answer: 6

Explanation:
\(\frac{2}{5}\) × 15
5 divides 15 three times.
2 × 3 = 6
Thus \(\frac{2}{5}\) × 15 = 6

Question 4.
\(\frac{2}{3}\) × 6 = _____

Answer: 4

Explanation:
\(\frac{2}{3}\) × 6
3 divides 6 two times.
\(\frac{2}{3}\) × 6
2 × 2 = 4
\(\frac{2}{3}\) × 6 = 4

On Your Own

Use a model to solve.

Question 5.
\(\frac{5}{8}\) × 24 = _____

Answer: 15

Explanation:
\(\frac{5}{8}\) × 24
8 divides 24 three times.
5 × 3 = 15
\(\frac{5}{8}\) × 24 = 15

Question 6.
\(\frac{3}{4}\) × 24 = _____

Answer: 18

Explanation:
\(\frac{3}{4}\) × 24
4 divides 24 six times.
\(\frac{3}{4}\) × 24 = 3 × 6 = 18
So, \(\frac{3}{4}\) × 24 = 18

Question 7.
\(\frac{4}{7}\) × 21 = _____

Answer: 12

Explanation:
\(\frac{4}{7}\) × 21
7 divides 21 three times.
4 × 3 = 12
\(\frac{4}{7}\) × 21 = 12

Question 8.
\(\frac{2}{9}\) × 27 = _____

Answer: 6

Explanation:
\(\frac{2}{9}\) × 27
9 divides 27 three times.
2 × 3 = 6
\(\frac{2}{9}\) × 27 = 6

Question 9.
\(\frac{3}{5}\) × 20 = _____

Answer: 12

Explanation:
\(\frac{3}{5}\) × 20
5 divides 20 four times.
3 × 4 = 12
Thus \(\frac{3}{5}\) × 20 = 12

Question 10.
\(\frac{7}{11}\) × 22 = _____

Answer: 14

Explanation:
\(\frac{7}{11}\) × 22
11 divides 22 two times.
7 × 2 = 14
\(\frac{7}{11}\) × 22 = 14

Problem Solving – Page No. 294

Use the table for 11-12.
Go Math Grade 5 Answer Key Chapter 7 Multiply Fractions img 2

Question 11.
Four-fifths of Zack’s stamps have pictures of animals. How many stamps with pictures of animals does Zack have? Use a model to solve.
_____ stamps

Answer: 24 stamps

Explanation:
Given that, Four-fifths of Zack’s stamps have pictures of animals.
Number of stamps that Zack collected is 30
30 × \(\frac{4}{5}\)
5 divides 30 six times.
6 × 4 = 24
Zack has 24 stamps with pictures of animals.

Question 12.
Zack, Teri, and Paco combined the foreign stamps from their collections for a stamp show. Out of their collections, \(\frac{3}{10}\) of Zack’s stamps, \(\frac{5}{6}\) of Teri’s stamps, and \(\frac{3}{8}\) of Paco’s stamps were from foreign countries. How many stamps were in their display? Explain how you solved the problem.
_____ stamps

Answer: 33 stamps

Explanation:
Zack, Teri, and Paco combined the foreign stamps from their collections for a stamp show.
Out of their collections, \(\frac{3}{10}\) of Zack’s stamps, \(\frac{5}{6}\) of Teri’s stamps, and \(\frac{3}{8}\) of Paco’s stamps were from foreign countries.
Number of stamps Zack collected = 30
Number of stamps Teri collected = 18
Number of stamps Paco collected = 24
\(\frac{3}{10}\) of 30
\(\frac{3}{10}\) × 30 = 3 × 3 = 9
\(\frac{5}{6}\) × 18 = 5 × 3 = 15
\(\frac{3}{8}\) × 24 = 3 × 3 = 9
Now add all the stamps = 9 + 9 + 15 = 33

Question 13.
Paula has 24 stamps in her collection. Among her stamps, \(\frac{1}{3}\) have pictures of animals. Out of her stamps with pictures of animals, \(\frac{3}{4}\) of those stamps have pictures of birds. How many stamps have pictures of birds on them?
_____ stamps

Answer: 6 stamps

Explanation:
Paula has 24 stamps in her collection. Among her stamps, \(\frac{1}{3}\) have pictures of animals.
Out of her stamps with pictures of animals, \(\frac{3}{4}\) of those stamps have pictures of birds.
\(\frac{1}{3}\) × \(\frac{3}{4}\) × 24 = 24/4 = 6
Therefore 6 stamps have pictures of birds.

Question 14.
Test Prep Barry bought 21 stamps from a hobby shop. He gave \(\frac{3}{7}\) of them to his sister. How many stamps did he have left?
Go Math Grade 5 Answer Key Chapter 7 Multiply Fractions img 3
Options:
a. 3 stamps
b. 6 stamps
c. 9 stamps
d. 12 stamps

Answer: 9 stamps

Explanation:
Test Prep Barry bought 21 stamps from a hobby shop. He gave \(\frac{3}{7}\) of them to his sister.
\(\frac{3}{7}\) × 21
7 divides 21 three times.
3 × 3 = 9 stamps.
Thus the correct answer is option C.

Share and Show – Page No. 297

Use the model to find the product.

Question 1.
\(\frac{5}{6}\) × 3
Go Math Grade 5 Answer Key Chapter 7 Multiply Fractions img 4
______ \(\frac{□}{□}\)

Answer: 2 \(\frac{1}{2}\)

Explanation:
Place three whole fractions strips side by side.
Find six fraction strips all with the same denominator that fits exactly under the three whole numbers.
Circle \(\frac{5}{6}\) of 3 on the model you drew.
Complete the number sentence. \(\frac{5}{6}\) of 3
\(\frac{5}{6}\) × 3 = \(\frac{5}{2}\)
2 \(\frac{1}{2}\)

Question 2.
2 × \(\frac{5}{6}\)
Go Math Grade 5 Answer Key Chapter 7 Multiply Fractions img 5
______ \(\frac{□}{□}\)

Answer: 1 \(\frac{2}{3}\)

Explanation:
Place two whole fractions strips side by side.
Find six fraction strips all with the same denominator that fits exactly under the two whole numbers.
2 of \(\frac{5}{6}\) = \(\frac{5}{6}\) × 2
\(\frac{5}{3}\)
The mixed fraction of \(\frac{5}{3}\) is 1 \(\frac{2}{3}\)

Find the product.

Question 3.
\(\frac{5}{12}\) × 3 = ______ \(\frac{□}{□}\)

Answer: 1 \(\frac{1}{4}\)

Explanation:
\(\frac{5}{12}\) × 3
Place three whole fractions strips side by side.
Find six fraction strips all with the same denominator that fits exactly under the two whole numbers.
\(\frac{5}{12}\) × 3
3 divides 12 four times
\(\frac{5}{12}\) × 3 = \(\frac{5}{4}\)
The mixed fraction of \(\frac{5}{4}\) is 1 \(\frac{1}{4}\)
\(\frac{5}{12}\) × 3 = 1 \(\frac{1}{4}\)

Question 4.
9 × \(\frac{1}{3}\) = ______

Answer: 3

Explanation:
9 × \(\frac{1}{3}\)
Place nine whole fractions strips side by side.
Find three fraction strips all with the same denominator that fits exactly under the two whole numbers.
9 × \(\frac{1}{3}\)
3 divides 9 three times.
9 × \(\frac{1}{3}\) = 3
Thus 9 × \(\frac{1}{3}\) = 3

Question 5.
\(\frac{7}{8}\) × 4 = ______ \(\frac{□}{□}\)

Answer: 3 \(\frac{1}{2}\)

Explanation:
\(\frac{7}{8}\) × 4
Place four whole fractions strips side by side.
\(\frac{7}{8}\) × 4
4 divides 8 two times.
\(\frac{7}{8}\) × 4 = \(\frac{7}{2}\)
The mixed fraction of  \(\frac{7}{2}\) is 3 \(\frac{1}{2}\)
\(\frac{7}{8}\) × 4 = 3 \(\frac{1}{2}\)

Question 6.
4 × \(\frac{3}{5}\) = ______ \(\frac{□}{□}\)

Answer: 2 \(\frac{2}{5}\)

Explanation:
4 × \(\frac{3}{5}\)
Place four whole fractions strips side by side.
Place three \(\frac{1}{5}\) fraction strips all with the same denominator that fits exactly under the two whole numbers.
4 of \(\frac{3}{5}\)
4 × \(\frac{3}{5}\) = \(\frac{12}{5}\)
The mixed fraction of \(\frac{12}{5}\) is 2 \(\frac{2}{5}\)

Question 7.
\(\frac{7}{8}\) × 2 = ______ \(\frac{□}{□}\)

Answer: 1 \(\frac{3}{4}\)

Explanation:
\(\frac{7}{8}\) × 2
Place two whole fractions strips side by side.
Place seven \(\frac{1}{8}\) fraction strips all with the same denominator that fits exactly under the two whole numbers.
\(\frac{7}{8}\) of 2
\(\frac{7}{8}\) × 2 = \(\frac{7}{4}\)
The mixed fraction of \(\frac{7}{4}\) is 1 \(\frac{3}{4}\)

Question 8.
7 × \(\frac{2}{5}\) = ______ \(\frac{□}{□}\)

Answer: 2 \(\frac{4}{5}\)

Explanation:
7 × \(\frac{2}{5}\)
Place seven whole fractions strips side by side.
Place two \(\frac{1}{5}\) fraction strips all with the same denominator that fits exactly under the two whole numbers.
7 × \(\frac{2}{5}\) = \(\frac{14}{5}\)
The mixed fraction of \(\frac{14}{5}\) = 2 \(\frac{4}{5}\)

Question 9.
\(\frac{3}{8}\) × 4 = ______

Answer: \(\frac{3}{2}\)

Explanation:
\(\frac{3}{8}\) × 4
Place four whole fractions strips side by side.
Place three \(\frac{1}{8}\) fraction strips all with the same denominator that fits exactly under the two whole numbers.

Question 10.
11 × \(\frac{3}{4}\) = ______ \(\frac{□}{□}\)

Answer: 8 \(\frac{1}{4}\)

Explanation:
11 × \(\frac{3}{4}\)
Place Eleven whole fractions strips side by side.
Place three \(\frac{1}{4}\) fraction strips all with the same denominator that fits exactly under the two whole numbers.
11 of \(\frac{3}{4}\)
11 × \(\frac{3}{4}\) = \(\frac{33}{4}\)
Convert the improper fraction to the mixed fraction.
\(\frac{33}{4}\) = 8 \(\frac{1}{4}\)
11 × \(\frac{3}{4}\) = 8 \(\frac{1}{4}\)

Question 11.
\(\frac{4}{15}\) × 5 = ______ \(\frac{□}{□}\)

Answer: 5 \(\frac{1}{3}\)

Explanation:
\(\frac{4}{15}\) × 5
Place five whole fractions strips side by side.
Place four \(\frac{1}{15}\) fraction strips all with the same denominator that fits exactly under the two whole numbers.
\(\frac{4}{15}\) of 5
\(\frac{4}{15}\) × 5 = \(\frac{4}{3}\)
Convert the improper fraction to the mixed fraction.
\(\frac{4}{3}\) = 5 \(\frac{1}{3}\)

Question 12.
Matt has a 5-pound bag of apples. To make a pie, he needs to use \(\frac{3}{5}\) of the bag. How many pounds of apples will he use for the pie? Explain what a model for this problem might look like.
______ pound(s)

Answer: 3 pounds

Explanation:
Given, Matt has a 5-pound bag of apples.
To make a pie, he needs to use \(\frac{3}{5}\) of the bag.
\(\frac{3}{5}\) × 5 = 3
Therefore Matt used 3 pounds of apples to make a pie.

Problem Solving – Page No. 298

Pose a Problem

Question 13.
Tarique drew the model below for a problem. Write 2 problems that can be solved using this model. One of your problems should involve multiplying a whole number by a fraction and the other problem should involve multiplying a fraction by a whole number.
Go Math Grade 5 Answer Key Chapter 7 Multiply Fractions img 6
Pose problems.                                       Solve your problems.
How could you change the model to give you an answer of 4 \(\frac{4}{5}\)?
Explain and write a new equation.
Type below:
_________

Answer:
The five children in the Smith family each spend 2/5 of an hour doing household chores on Saturday. How much time did the spend altogether on their chores?
Multiply the numerator with the whole number.
5 × \(\frac{2}{5}\) = \(\frac{10}{5}\) = 2

Share and Show – Page No. 301

Find the product. Write the product in simplest form.

Question 1.
3 × \(\frac{2}{5}\) =
• Multiply the numerator by the whole number. Write the product over the denominator.
Go Math Grade 5 Answer Key Chapter 7 Multiply Fractions img 7
• Write the answer as a mixed number in simplest form.
Go Math Grade 5 Answer Key Chapter 7 Multiply Fractions img 8
______ \(\frac{□}{□}\)

Answer: 1 \(\frac{1}{5}\)

Explanation:
Multiply the whole number with the numerator.
3 \(\frac{2}{5}\) = \(\frac{6}{5}\)
Now write the improper fraction in the form of the mixed fraction.
\(\frac{6}{5}\) = 1 \(\frac{1}{5}\)

Question 2.
\(\frac{2}{3}\) × 5 = ______ \(\frac{□}{□}\)

Answer: 3 \(\frac{1}{3}\)

Explanation:
Multiply the whole number with the numerator.
\(\frac{2}{3}\) × 5 = \(\frac{10}{3}\)
Now write the improper fraction in the form of the mixed fraction.
\(\frac{10}{3}\) = 3 \(\frac{1}{3}\)

Question 3.
6 × \(\frac{2}{3}\) = ______

Answer: 4

Explanation:
6 × \(\frac{2}{3}\)
Multiply the whole number with the numerator.
6 × \(\frac{2}{3}\) = \(\frac{12}{3}\)
Now write the improper fraction in the form of the mixed fraction.
\(\frac{12}{3}\) = 4

Question 4.
\(\frac{5}{7}\) × 4 = ______ \(\frac{□}{□}\)

Answer: 2 \(\frac{6}{7}\)

Explanation:
\(\frac{5}{7}\) × 4
Multiply the whole number with the numerator.
\(\frac{5}{7}\) × 4 = \(\frac{20}{7}\)
Now write the improper fraction in the form of the mixed fraction.
2 \(\frac{6}{7}\)
Thus, \(\frac{5}{7}\) × 4 = 2 \(\frac{6}{7}\)

On Your Own

Find the product. Write the product in simplest form.

Question 5.
5 × \(\frac{2}{3}\) = ______ \(\frac{□}{□}\)

Answer: 3 \(\frac{1}{3}\)

Explanation:
5 × \(\frac{2}{3}\)
Multiply the whole number with the numerator.
5 × \(\frac{2}{3}\) = \(\frac{10}{3}\)
Now write the improper fraction in the form of the mixed fraction.
3 \(\frac{1}{3}\)
5 × \(\frac{2}{3}\) = 3 \(\frac{1}{3}\)

Question 6.
\(\frac{1}{4}\) × 3 = ______ \(\frac{□}{□}\)

Answer: \(\frac{3}{4}\)

Explanation:
\(\frac{1}{4}\) × 3
Multiply the whole number with the numerator.
\(\frac{1}{4}\) × 3 = \(\frac{3}{4}\)

Question 7.
7 × \(\frac{7}{8}\) = ______ \(\frac{□}{□}\)

Answer: 6 \(\frac{1}{8}\)

Explanation:
7 × \(\frac{7}{8}\)
Multiply the whole number with the numerator.
\(\frac{49}{8}\)
Now write the improper fraction in the form of the mixed fraction.
\(\frac{49}{8}\) = 6 \(\frac{1}{8}\)
Thus, 7 × \(\frac{7}{8}\) = 6 \(\frac{1}{8}\)

Question 8.
2 × \(\frac{4}{5}\) = ______ \(\frac{□}{□}\)

Answer: 1 \(\frac{3}{5}\)

Explanation:
2 × \(\frac{4}{5}\)
Multiply the whole number with the numerator.
2 × \(\frac{4}{5}\) = \(\frac{8}{5}\)
Now write the improper fraction in the form of the mixed fraction.
\(\frac{8}{5}\) = 1 \(\frac{3}{5}\)

Question 9.
4 × \(\frac{3}{4}\) = ______

Answer: 3

Explanation:
Multiply the whole number with the numerator.
4 × \(\frac{3}{4}\) = \(\frac{12}{4}\)
4 divides 12 three times.
So, \(\frac{12}{4}\) = 3
4 × \(\frac{3}{4}\) = 3

Question 10.
\(\frac{7}{9}\) × 2 = ______ \(\frac{□}{□}\)

Answer: 1 \(\frac{5}{9}\)

Explanation:
\(\frac{7}{9}\) × 2
Multiply the whole number with the numerator.
\(\frac{7}{9}\) × 2 = \(\frac{14}{9}\)
Now write the improper fraction in the form of the mixed fraction.
\(\frac{14}{9}\) = 1 \(\frac{5}{9}\)

Practice: Copy and Solve. Find the product. Write the product in simplest form.

Question 11.
\(\frac{3}{5}\) × 11 = ______ \(\frac{□}{□}\)

Answer: 6 \(\frac{3}{5}\)

Explanation:
\(\frac{3}{5}\) × 11
Multiply the whole number with the numerator.
\(\frac{3}{5}\) × 11 = \(\frac{33}{5}\)
Now write the improper fraction in the form of the mixed fraction.
\(\frac{33}{5}\) = 6 \(\frac{3}{5}\)

Question 12.
3 × \(\frac{3}{4}\) = ______ \(\frac{□}{□}\)

Answer: 2 \(\frac{1}{4}\)

Explanation:
3 × \(\frac{3}{4}\)
Multiply the whole number with the numerator.
3 × \(\frac{3}{4}\) = \(\frac{9}{4}\)
Now write the improper fraction in the form of the mixed fraction.
\(\frac{9}{4}\) = 2 \(\frac{1}{4}\)

Question 13.
\(\frac{5}{8}\) × 3 = ______ \(\frac{□}{□}\)

Answer: 1 \(\frac{7}{8}\)

Explanation:
\(\frac{5}{8}\) × 3
Multiply the whole number with the numerator.
\(\frac{5}{8}\) × 3 = \(\frac{15}{8}\)
Now write the improper fraction in the form of the mixed fraction.
\(\frac{15}{8}\) = 1 \(\frac{7}{8}\)

Algebra Find the unknown digit.

Question 14.
\(\frac{■}{2}\) × 8 = 4
■ = ______

Answer: 1

Explanation:
\(\frac{■}{2}\) × 8 = 4
\(\frac{■}{2}\) = 4/8
■ = 4 × 2/8 = 1
■ = 1

Question 15.
■ × \(\frac{5}{6}\) = \(\frac{20}{6}\) or 3 \(\frac{1}{3}\)
■ = ______

Answer: 4

Explanation:
■ × \(\frac{5}{6}\) = \(\frac{20}{6}\)
■ = 20/6 × 6/5
■ = 20/5 = 4
■ = 4

Question 16.
\(\frac{1}{■}\) × 18 = 3
■ = ______

Answer: 6

Explanation:
\(\frac{1}{■}\) × 18 = 3
\(\frac{1}{3}\) × 18 = ■
■ = 18/3 = 6
■ = 6

UNLOCK the Problem – Page No. 302

Go Math Grade 5 Answer Key Chapter 7 Multiply Fractions img 9

Question 17.
The caterer wants to have enough turkey to feed 24 people. If he wants to provide \(\frac{3}{4}\) of a pound of turkey for each person, how much turkey does he need?
a. What do you need to find?
Type below:
__________

Answer: I need to find How much turkey the caterer needs to provide for each person.

Question 17.
b. What operation will you use?
Type below:
__________

Answer: I will use the multiplication operation to solve the problem.

Question 17.
c. What information are you given?
Type below:
__________

I am given the information about the number of people to feed and the fraction of pounds of turkey each person gets.

Question 17.
d. Solve the problem.
Type below:
__________

Answer:
The caterer wants to serve 24 people
\(\frac{3}{4}\) × 24
4 divides 24 six times.
3 × 6 = 18
Thus the caterer needs 18 pounds of Turkey.

Question 17.
e. Complete the sentences.
The caterer wants to serve 24 people _____ of a pound of turkey each.
He will need ____ × ____ , or ______ pounds of turkey.
Type below:
__________

Answer: \(\frac{3}{4}\) × 24

Question 17.
f. Fill in the bubble for the correct answer choice.
Options:
a. 72 pounds
b. 24 pounds
c. 18 pounds
d. 6 pounds

Answer: 18 pounds

Explanation:
The caterer wants to serve 24 people
\(\frac{3}{4}\) × 24
4 divides 24 six times.
3 × 6 = 18
The correct answer is option C.

Question 18.
Patty wants to run \(\frac{5}{6}\) of a mile every day for 5 days. How far will she run in that time?
Options:
a. 25 miles
b. 5 miles
c. 4 \(\frac{1}{6}\) miles
d. 1 \(\frac{2}{3}\) miles

Answer: 4 \(\frac{1}{6}\) miles

Explanation:
Patty wants to run \(\frac{5}{6}\) of a mile every day for 5 days.
\(\frac{5}{6}\) × 5 = \(\frac{25}{6}\)
Convert the improper fraction to the mixed fraction.
\(\frac{25}{6}\) = 4 \(\frac{1}{6}\) miles
Thus the correct answer is option C.

Question 19.
Doug has 33 feet of rope. He wants to use \(\frac{2}{3}\) of it for his canoe. How many feet of rope will he use for his canoe?
Options:
a. 11 feet
b. 22 feet
c. 33 feet
d. 66 feet

Answer: 22 feet

Explanation:
Doug has 33 feet of rope. He wants to use \(\frac{2}{3}\) of it for his canoe.
\(\frac{2}{3}\) × 33 feet
3 divides 33 eleven times.
2 × 11 = 22 feet
The correct answer is option B.

Share and Show – Page No. 304

Use the model to find the product.

Question 1.
Go Math Grade 5 Answer Key Chapter 7 Multiply Fractions img 10
\(\frac{3}{5} \times \frac{1}{3}=\)
\(\frac{□}{□}\)

Answer: \(\frac{1}{5}\)

Explanation:
The fraction \(\frac{3}{5}\) represents the rows and columns.
The fraction \(\frac{1}{3}\) indicates the shaded part of the figure.
\(\frac{3}{5}\) × \(\frac{1}{3}\) = \(\frac{1}{5}\)

Question 2.
Go Math Grade 5 Answer Key Chapter 7 Multiply Fractions img 11
\(\frac{2}{3} \times \frac{3}{5}=\)
\(\frac{□}{□}\)

Answer: \(\frac{2}{5}\)

Explanation:

The above figure shows that the circle is divided into 5 parts in which 2 parts are non shaded and 3 parts are shaded.
So, the fraction of the circle is \(\frac{2}{3}\)
The fraction for the shaded part of the circle is \(\frac{3}{5}\)
\(\frac{2}{3}\) × \(\frac{3}{5}\) = \(\frac{2}{5}\)

Share and Show – Page No. 305

Find the product. Draw a model.

Question 3.
\(\frac{2}{3} \times \frac{1}{5}=\) \(\frac{□}{□}\)

Answer: \(\frac{2}{15}\)

Explanation:
\(\frac{2}{3}\) × \(\frac{1}{5}\)
Multiply the denominators of both the fractions.
\(\frac{2}{15}\)
\(\frac{2}{3} \times \frac{1}{5}=\) \(\frac{2}{15}\)

Question 4.
\(\frac{1}{2} \times \frac{5}{6}=\) \(\frac{□}{□}\)

Answer: \(\frac{5}{12}\)

Explanation:
\(\frac{1}{2}\) × \(\frac{5}{6}\)
Multiply the numerators and the denominators.
\(\frac{1}{2}\) × \(\frac{5}{6}\) = \(\frac{5}{12}\)
\(\frac{1}{2} \times \frac{5}{6}=\) \(\frac{5}{12}\)

Question 5.
\(\frac{3}{5} \times \frac{1}{3}=\) \(\frac{□}{□}\)

Answer: \(\frac{1}{5}\)

Explanation:
\(\frac{3}{5}\) × \(\frac{1}{3}\)
Multiply the denominators and the numerators of the fractions.
\(\frac{3}{5}\) × \(\frac{1}{3}\) = \(\frac{3}{15}\)
\(\frac{3}{15}\) = \(\frac{1}{5}\)
\(\frac{3}{5} \times \frac{1}{3}=\) \(\frac{1}{5}\)

Question 6.
\(\frac{3}{4} \times \frac{1}{6}=\) \(\frac{□}{□}\)

Answer: \(\frac{1}{8}\)

Explanation:
\(\frac{3}{4}\) × \(\frac{1}{6}\)
Multiply the denominators and the numerators of the fractions.
\(\frac{3}{4}\) × \(\frac{1}{6}\) = \(\frac{3}{24}\)
3 divides 24 eight times.
So, \(\frac{3}{24}\) = \(\frac{1}{8}\)
Thus, \(\frac{3}{4} \times \frac{1}{6}=\) \(\frac{1}{8}\)

Question 7.
\(\frac{2}{5} \times \frac{5}{6}=\) \(\frac{□}{□}\)

Answer: \(\frac{1}{3}\)

Explanation:
\(\frac{2}{5}\) × \(\frac{5}{6}\)
Multiply the denominators and the numerators of the fractions.
\(\frac{2}{5}\) × \(\frac{5}{6}\) = \(\frac{10}{30}\)
10 divides 30 three times.
\(\frac{10}{30}\) = \(\frac{1}{3}\)
\(\frac{2}{5} \times \frac{5}{6}=\) \(\frac{1}{3}\)

Question 8.
\(\frac{5}{6} \times \frac{3}{5}=\) \(\frac{□}{□}\)

Answer: \(\frac{1}{2}\)

Explanation:
\(\frac{5}{6}\) × \(\frac{3}{5}\)
Multiply the denominators and the numerators of the fractions.
\(\frac{5}{6}\) × \(\frac{3}{5}\) = \(\frac{15}{30}\)
\(\frac{5}{6}\) × \(\frac{3}{5}\) = \(\frac{15}{30}\) = \(\frac{1}{2}\)
\(\frac{5}{6} \times \frac{3}{5}=\) \(\frac{1}{2}\)

Problem Solving – Page No. 306

What’s the Error?
Go Math Grade 5 Answer Key Chapter 7 Multiply Fractions img 12

Question 9.
Cheryl and Marcus are going to make a two-tiered cake. The smaller tier is \(\frac{2}{3}\) the size of the larger tier. The recipe for the bottom tier calls for \(\frac{3}{5}\) cup of water. How much water will they need to make the smaller tier?

They made a model to represent the problem. Cheryl says they need \(\frac{6}{9}\) cup of water. Marcus says they need \(\frac{2}{5}\) cup water. Who is correct? Explain.
Go Math Grade 5 Answer Key Chapter 7 Multiply Fractions img 13
Cheryl’s answer               Marcus’ answer
Type below:
_________

Answer: Marcus’ answer is correct.

Explanation:
Cheryl and Marcus are going to make a two-tiered cake.
The smaller tier is \(\frac{2}{3}\) the size of the larger tier.
The recipe for the bottom tier calls for \(\frac{3}{5}\) cup of water.
\(\frac{3}{5}\) × \(\frac{2}{3}\) = \(\frac{2}{5}\)

Share and Show – Page No. 309

Complete the statement with equal to, greater than, or less than.

Question 1.
4 × \(\frac{7}{8}\) will be ___________ \(\frac{7}{8}\)
Go Math Grade 5 Answer Key Chapter 7 Multiply Fractions img 14
_________

Answer: Greater than

Explanation:
4 × \(\frac{7}{8}\) = \(\frac{7}{2}\)
The denominator with a greater number will be the smallest number.
So, \(\frac{7}{2}\) is greater than \(\frac{7}{8}\)

Question 2.
\(\frac{3}{5} \times \frac{2}{7}\) will be ___________ \(\frac{3}{5}\)

Answer: Less than

Explanation:
\(\frac{3}{5}\) × \(\frac{2}{7}\) = \(\frac{6}{35}\)
The denominator with the greatest number will be the smallest fraction.
So, \(\frac{6}{35}\) is less than \(\frac{3}{5}\)

Question 3.
\(\frac{5}{8} \times 6\) will be ___________ \(\frac{5}{8}\)

Answer: Greater than

Explanation:
\(\frac{5}{8}\) × 6 = \(\frac{15}{4}\)
\(\frac{15}{4}\) = 3 \(\frac{3}{4}\)
3 \(\frac{3}{4}\) is greater than \(\frac{5}{8}\)

Question 4.
\(\frac{2}{3} \times \frac{5}{5}\) will be ___________ \(\frac{2}{3}\)

Answer: Equal to

Explanation:
\(\frac{2}{3}\) × \(\frac{5}{5}\) = \(\frac{2}{3}\)
\(\frac{2}{3}\) is equal to \(\frac{2}{3}\)

Question 5.
\(8 \times \frac{7}{8}\) will be ___________ 8

Answer: Less than

Explanation:
8 × \(\frac{7}{8}\)= 7
7 is less than 8.
\(8 \times \frac{7}{8}\) will be less than 8.

On Your Own

Complete the statement with equal to, greater than, or less than.

Question 6.
\(\frac{4}{9} \times \frac{3}{8}\) will be ___________ \(\frac{3}{8}\)

Answer: Less than

Explanation:
\(\frac{4}{9}\) × \(\frac{3}{8}\) = \(\frac{12}{72}\)
= \(\frac{1}{6}\)
\(\frac{1}{6}\) is less than \(\frac{3}{8}\)
\(\frac{4}{9} \times \frac{3}{8}\) will be less than \(\frac{3}{8}\)

Question 7.
\(7 \times \frac{9}{10}\) will be ___________ \(\frac{9}{10}\)

Answer: Greater than

Explanation:
7 × \(\frac{9}{10}\) = \(\frac{63}{10}\)
Denominators are same so compare the numerators.
\(\frac{63}{10}\) is greater than \(\frac{9}{10}\)

Question 8.
\(5 \times \frac{1}{3}\) will be ___________ \(\frac{1}{3}\)

Answer: Greater than

Explanation:
5 × \(\frac{1}{3}\) = \(\frac{5}{3}\)
Denominators are same so compare the numerators.
\(\frac{5}{3}\) is greater than \(\frac{1}{3}\)

Question 9.
\(\frac{6}{11} \times 1\) will be ___________ \(\frac{6}{11}\)

Answer: Equal to

Explanation:
\(\frac{6}{11}\) × 1 = \(\frac{6}{11}\)
\(\frac{6}{11}\) is equal to \(\frac{6}{11}\).

Question 10.
\(\frac{1}{6} \times \frac{7}{7}\) will be ___________ 1

Answer: Less than

Explanation:
\(\frac{1}{6}\) × \(\frac{7}{7}\) = \(\frac{1}{6}\)
\(\frac{1}{6}\) is less than 1

Question 11.
\(4 \times \frac{3}{5}\) will be ___________ \(\frac{3}{5}\)

Answer: Greater than

Explanation:
4 × \(\frac{3}{5}\) = \(\frac{12}{5}\)
Denominators are same so compare the numerators.
\(\frac{12}{5}\) is greater than \(\frac{3}{5}\)

Problem Solving – Page No. 310

Question 12.
Lola is making cookies. She plans to multiply the recipe by 3 so she can make enough cookies for the whole class. If the recipe calls for \(\frac{2}{3}\) cup of sugar, will she need more than \(\frac{2}{3}\) or less than \(\frac{2}{3}\) cup of sugar to make all the cookies?
_________ \(\frac{2}{3}\) cup of sugar

Answer: More than

Explanation:
ola is making cookies. She plans to multiply the recipe by 3 so she can make enough cookies for the whole class.
3 × \(\frac{2}{3}\) = 2
So, Lola needs more than \(\frac{2}{3}\) cup of sugar.

Question 13.
Peter is planning on spending \(\frac{2}{3}\) as many hours watching television this week as he did last week. Is Peter going to spend more hours or fewer hours watching television this week?
_________ hours

Answer: Fewer

Explanation:
Peter is planning on spending \(\frac{2}{3}\) as many hours watching television this week as he did last week.
7 × \(\frac{2}{3}\) = \(\frac{14}{3}\)
\(\frac{14}{3}\) = 4 \(\frac{2}{3}\)
Thus peter going to spend more hours or fewer hours watching television this week.

Question 14.
Test Prep Rochelle saves \(\frac{1}{4}\) of her allowance. If she decides to start saving \(\frac{1}{2}\) as much, which statement below is true?
Options:
a. She will be saving the same amount.
b. She will be saving more.
c. She will be saving less.
d. She will be saving twice as much.

Answer: She will be saving more

Explanation:
Test Prep Rochelle saves \(\frac{1}{4}\) of her allowance.
\(\frac{1}{4}\) is greater than \(\frac{1}{2}\)
So, the answer is option B.

Connect to Art

A scale model is a representation of an object with the same shape as the real object. Models can be larger or smaller than the actual object but are often smaller.

Architects often make scale models of the buildings or structures they plan to build. Models can give them an idea of how the structure will look when finished. Each measurement of the building is scaled up or down by the same factor.
Go Math Grade 5 Answer Key Chapter 7 Multiply Fractions img 15

Bob is building a scale model of his bike. He wants his model to be \(\frac{1}{5}\) as long as his bike.

Question 15.
If Bob’s bike is 60 inches long, how long will his model be?
_____ in.

Answer: 12 inches

Explanation:
Given that, Bob is building a scale model of his bike. He wants his model to be \(\frac{1}{5}\) as long as his bike.
If Bob’s bike is 60 inches long then multiply with the fraction \(\frac{1}{5}\)
\(\frac{1}{5}\) × 60 = 12 inches
The model will be 12 inches long.

Question 16.
If one wheel on Bob’s model is 4 inches across, how many inches across is the actual wheel on his bike? Explain.
\(\frac{□}{□}\) in.

Answer: \(\frac{4}{5}\) in.

Explanation:
Given that, one wheel on Bob’s model is 4 inches across.
4 × \(\frac{1}{5}\) = \(\frac{4}{5}\) in.

Share and Show – Page No. 313

Find the product. Write the product in simplest form.

Question 1.
\(6 \times \frac{3}{8}\)
\(\frac{6}{1} \times \frac{3}{8}\) = \(\frac{■}{■}\)
______ \(\frac{□}{□}\)

Answer: 2 \(\frac{1}{4}\)

Explanation:
\(\frac{6}{1} \times \frac{3}{8}\) = \(\frac{■}{■}\)
6 × \(\frac{3}{8}\) = \(\frac{18}{8}\) = \(\frac{9}{4}\)
\(\frac{9}{4}\) = 2 \(\frac{1}{4}\)
2 \(\frac{1}{4}\) = \(\frac{■}{■}\)
\(\frac{■}{■}\) = 2 \(\frac{1}{4}\)

Question 2.
\(\frac{3}{8} \times \frac{8}{9}\) = \(\frac{□}{□}\)

Answer: \(\frac{1}{3}\)

Explanation:
\(\frac{3}{8} \times \frac{8}{9}\) = \(\frac{□}{□}\)
\(\frac{3}{8}\) × \(\frac{8}{9}\) = \(\frac{1}{3}\)
Thus, \(\frac{3}{8} \times \frac{8}{9}\) = \(\frac{1}{3}\)

Question 3.
\(\frac{2}{3} \times 27\) = ______

Answer: 18

Explanation:
27 × \(\frac{2}{3}\)
3 divides 27 nine times.
Thus, 27 × \(\frac{2}{3}\) = 18

Question 4.
\(\frac{5}{12} \times \frac{3}{5}\) = \(\frac{□}{□}\)

Answer: \(\frac{1}{4}\)

Explanation:
\(\frac{5}{12}\) × \(\frac{3}{5}\) = \(\frac{3}{12}\)
3 divides 12 four times.
\(\frac{3}{12}\) = \(\frac{1}{4}\)
\(\frac{5}{12} \times \frac{3}{5}\) = \(\frac{1}{4}\)

Question 5.
\(\frac{1}{2} \times \frac{3}{5}\) = \(\frac{□}{□}\)

Answer: \(\frac{3}{10}\)

Explanation:
\(\frac{1}{2}\) × \(\frac{3}{5}\)
Multiply the numerators and the denominators.
\(\frac{1}{2}\) × \(\frac{3}{5}\)  = \(\frac{3}{10}\)
\(\frac{1}{2} \times \frac{3}{5}\) = \(\frac{3}{10}\)

Question 6.
\(\frac{2}{3} \times \frac{4}{5}\) = \(\frac{□}{□}\)

Answer: \(\frac{8}{15}\)

Explanation:
\(\frac{2}{3}\) × \(\frac{4}{5}\)
Multiply the numerators and the denominators.
\(\frac{2}{3}\) × \(\frac{4}{5}\) = \(\frac{8}{15}\)

Question 7.
\(\frac{1}{3} \times \frac{5}{8}\) = \(\frac{□}{□}\)

Answer: \(\frac{5}{24}\)

Explanation:
\(\frac{1}{3}\) × \(\frac{5}{8}\)
Multiply the numerators and the denominators.
\(\frac{1}{3} \times \frac{5}{8}\) = \(\frac{5}{24}\)

Question 8.
\(4 \times \frac{1}{5}\) = \(\frac{□}{□}\)

Answer: \(\frac{4}{5}\)

Explanation:
Multiply the numerator with the whole number.
4 × \(\frac{1}{5}\) = \(\frac{4}{5}\)
\(4 \times \frac{1}{5}\) = \(\frac{4}{5}\)

On Your Own

Find the product. Write the product in simplest form.

Question 9.
\(2 \times \frac{1}{8}\) = \(\frac{□}{□}\)

Answer: \(\frac{1}{4}\)

Explanation:
Multiply the whole number with the numerator.
2 × \(\frac{1}{8}\)
2 divides 8 four times.
2 × \(\frac{1}{8}\) = \(\frac{1}{4}\)
\(2 \times \frac{1}{8}\) = \(\frac{1}{4}\)

Question 10.
\(\frac{4}{9} \times \frac{4}{5}\) = \(\frac{□}{□}\)

Answer: \(\frac{16}{45}\)

Explanation:
\(\frac{4}{9}\) × \(\frac{4}{5}\)
Multiply the numerators and the denominators.
\(\frac{4}{9}\) × \(\frac{4}{5}\) = \(\frac{16}{45}\)
\(\frac{4}{9} \times \frac{4}{5}\) = \(\frac{16}{45}\)

Question 11.
\(\frac{1}{12} \times \frac{2}{3}\) = \(\frac{□}{□}\)

Answer: \(\frac{1}{18}\)

Explanation:
\(\frac{1}{12}\) × \(\frac{2}{3}\)
Multiply the numerators and the denominators.
\(\frac{1}{12}\) × \(\frac{2}{3}\) = \(\frac{2}{36}\)
\(\frac{2}{36}\) = \(\frac{1}{18}\)
\(\frac{1}{12} \times \frac{2}{3}\) = \(\frac{1}{18}\)

Question 12.
\(\frac{1}{7} \times 30\) = _____ \(\frac{□}{□}\)

Answer: 4 \(\frac{2}{7}\)

Explanation:
30 × \(\frac{1}{7}\) = \(\frac{30}{7}\)
Convert improper fraction to the mixed fraction.
\(\frac{30}{7}\) = 4 \(\frac{2}{7}\)
\(\frac{1}{7} \times 30\) = 4 \(\frac{2}{7}\)

Question 13.
Of the pets in the pet show, \(\frac{5}{6}\) are cats. \(\frac{4}{5}\) of the cats are calico cats. What fraction of the pets are calico cats?
\(\frac{□}{□}\) calico cats

Answer: \(\frac{2}{3}\)

Explanation:
Of the pets in the pet show, \(\frac{5}{6}\) are cats. \(\frac{4}{5}\) of the cats are calico cats.
\(\frac{5}{6}\) × \(\frac{4}{5}\) = \(\frac{20}{30}\) = \(\frac{2}{3}\)
\(\frac{2}{3}\) fraction of the pets are calico cats.

Question 14.
Five cats each ate \(\frac{1}{4}\) cup of food. How much food did they eat altogether?
_____ \(\frac{□}{□}\) cups of food

Answer: 1 \(\frac{1}{4}\)

Explanation:
Five cats each ate \(\frac{1}{4}\) cup of food.
5 × \(\frac{1}{4}\) = \(\frac{5}{4}\)
The mixed fraction of \(\frac{5}{4}\) is 1 \(\frac{1}{4}\)

Algebra Evaluate for the given value.

Question 15.
\(\frac{2}{5}\) × c for c = \(\frac{4}{7}\)
\(\frac{□}{□}\)

Answer: \(\frac{8}{35}\)

Explanation:
\(\frac{2}{5}\) × c = \(\frac{4}{7}\)
c = \(\frac{4}{7}\) × \(\frac{2}{5}\)
c = \(\frac{8}{35}\)

Question 16.
m × \(\frac{4}{5}\) for m = \(\frac{7}{8}\)
\(\frac{□}{□}\)

Answer: \(\frac{7}{10}\)

Explanation:
m = \(\frac{4}{5}\) × \(\frac{7}{8}\)
Multiply the numerators and denominators.
\(\frac{4}{5}\) × \(\frac{7}{8}\) = \(\frac{7}{10}\)

Question 17.
\(\frac{2}{3}\) × t for t = \(\frac{1}{8}\)
\(\frac{□}{□}\)

Answer: \(\frac{1}{12}\)

Explanation:
\(\frac{2}{3}\) × t for t = \(\frac{1}{8}\)
t = \(\frac{1}{8}\) × \(\frac{2}{3}\)
t = \(\frac{1}{12}\)

Question 18.
y × \(\frac{2}{3}\) for y = 5
_______

Answer: 4

Explanation:
y × \(\frac{2}{3}\) for y = 5
6 × \(\frac{2}{3}\) = 4

Problem Solving – Page No. 314

Speedskating is a popular sport in the Winter Olympics. Many young athletes in the U.S. participate in speedskating clubs and camps.
Go Math Grade 5 Answer Key Chapter 7 Multiply Fractions img 16

Question 19.
At a camp in Green Bay, Wisconsin, \(\frac{7}{9}\) of the participants were from Wisconsin. Of that group, \(\frac{3}{5}\) were 12 years old. What fraction of the group was from Wisconsin and 12 years old?
\(\frac{□}{□}\)

Answer: \(\frac{7}{15}\)

Explanation:
Given that,
At a camp in Green Bay, Wisconsin, \(\frac{7}{9}\) of the participants were from Wisconsin.
Of that group, \(\frac{3}{5}\) were 12 years old.
To find the fraction of the group was from Wisconsin and 12 years old
We have to multiply the fraction \(\frac{7}{9}\) and \(\frac{3}{5}\)
\(\frac{7}{9}\) × \(\frac{3}{5}\) = \(\frac{21}{45}\)
\(\frac{21}{45}\) = \(\frac{7}{15}\)
Thus the fraction of the group was from Wisconsin and 12 years old is \(\frac{7}{15}\).

Question 20.
Maribel wants to skate 1 \(\frac{1}{2}\) miles on Monday. If she skates \(\frac{9}{10}\) mile Monday morning and \(\frac{2}{3}\) of that distance Monday afternoon, will she reach her goal? Explain.
_____

Answer: Yes

Explanation:
Maribel wants to skate 1 \(\frac{1}{2}\) miles on Monday.
To find whether Maribel reached her goal we have to multiply the fractions \(\frac{9}{10}\) and \(\frac{2}{3}\)
\(\frac{9}{10}\) × \(\frac{2}{3}\) = \(\frac{3}{5}\)
By this we can say that Maribel reaches her goal.
So, the answer is yes.

Question 21.
On the first day of camp, \(\frac{5}{6}\) of the skaters were beginners. Of the beginners, \(\frac{1}{3}\) were girls. What fraction of the skaters were girls and beginners? Explain why your answer is reasonable.
\(\frac{□}{□}\)

Answer: \(\frac{5}{18}\)

Explanation:
On the first day of camp, \(\frac{5}{6}\) of the skaters were beginners. Of the beginners, \(\frac{1}{3}\) were girls.
Multiply the fraction of the skaters were beginning and the fraction of skaters were girls.
\(\frac{5}{6}\) × latex]\frac{1}{3}[/latex] = latex]\frac{5}{18}[/latex]
The fraction of the skaters were girls and beginners are latex]\frac{5}{18}[/latex]

Question 22.
Test Prep On Wednesday, Danielle skated \(\frac{2}{3}\) of the way around the track in 2 minutes. Her younger brother skated \(\frac{3}{4}\) of Danielle’s distance in 2 minutes. What fraction of the track did Danielle’s brother finish in 2 minutes?
Options:
a. \(\frac{1}{3}\)
b. \(\frac{1}{2}\)
c. \(\frac{5}{7}\)
d. \(\frac{3}{4}\)

Answer: \(\frac{1}{2}\)

Explanation:
Test Prep On Wednesday, Danielle skated \(\frac{2}{3}\) of the way around the track in 2 minutes.
Her younger brother skated \(\frac{3}{4}\) of Danielle’s distance in 2 minutes.
Multiply the fraction of Danielle skated and her younger brother skated.
\(\frac{2}{3}\) × \(\frac{3}{4}\) = \(\frac{1}{2}\)
Thus the correct answer is option B.

Mid-Chapter Checkpoint – Page No. 315

Concept and Skills

Question 1.
Explain how you would model 5 × \(\frac{2}{3}\)
Type below:
__________

Answer: \(\frac{10}{3}\)

Question 2.
When you multiply \(\frac{2}{3}\) by a fraction less than one, how does the product compare to the factors?
Type below:
__________

Answer: \(\frac{2}{3}\) × \(\frac{1}{2}\)
= \(\frac{1}{3}\)

Find the product. Write the product in simplest form.

Question 3.
\(\frac{2}{3} \times 6\)
______

Answer: 4

Explanation:
6 × \(\frac{2}{3}\)
Multiply the numerator with the whole numbers.
\(\frac{1}{3}\)

Question 4.
\(\frac{4}{5} \times 7\)
______ \(\frac{□}{□}\)

Answer: 5 \(\frac{3}{5}\)

Explanation:
Multiply the numerator with the whole numbers.
\(\frac{4}{5} \times 7\)
7 × \(\frac{4}{5}\) = \(\frac{28}{5}\)
Convert the improper fraction to the mixed fraction.
\(\frac{28}{5}\) = 5 \(\frac{3}{5}\)
\(\frac{4}{5} \times 7\) = 5 \(\frac{3}{5}\)

Question 5.
\(8 \times \frac{5}{7}\)
______ \(\frac{□}{□}\)

Answer: 5 \(\frac{5}{7}\)

Explanation:
8 × \(\frac{5}{7}\)
Multiply the numerator with the whole numbers.
8 × \(\frac{5}{7}\) = \(\frac{40}{7}\)
Convert the improper fraction to the mixed fraction.
\(\frac{40}{7}\) = 5 \(\frac{5}{7}\)

Question 6.
\(\frac{7}{8} \times \frac{3}{8}\)
\(\frac{□}{□}\)

Answer: \(\frac{21}{64}\)

Explanation:
\(\frac{7}{8}\) × \(\frac{3}{8}\)
Multiply the numerators and denominators of the fractions.
\(\frac{7}{8}\) × \(\frac{3}{8}\) = \(\frac{21}{64}\)

Question 7.
\(\frac{1}{2} \times \frac{3}{4}\)
\(\frac{□}{□}\)

Answer: \(\frac{3}{8}\)

Explanation:
Multiply the numerators and denominators of the fractions.
\(\frac{1}{2} \times \frac{3}{4}\)
\(\frac{1}{2}\) × \(\frac{3}{4}\) = \(\frac{3}{8}\)
\(\frac{1}{2} \times \frac{3}{4}\) = \(\frac{3}{8}\)

Question 8.
\(\frac{7}{8} \times \frac{4}{7}\)
\(\frac{□}{□}\)

Answer: \(\frac{1}{2}\)

Explanation:
Multiply the numerators and denominators of the fractions.
7 in the numerator and 7 in the denominator will be canceled.
4 divides 8 two times.
Thus the fraction is \(\frac{1}{2}\)
\(\frac{7}{8} \times \frac{4}{7}\) = \(\frac{1}{2}\)

Question 9.
\(2 \times \frac{3}{11}\)
\(\frac{□}{□}\)

Answer: \(\frac{6}{11}\)

Explanation:
Multiply the numerator with the whole numbers.
2 × \(\frac{3}{11}\)
2 × 3 = 6
2 × \(\frac{3}{11}\) = \(\frac{6}{11}\)
Thus, \(2 \times \frac{3}{11}\) = \(\frac{6}{11}\)

Question 10.
\(\frac{5}{8} \times \frac{2}{3}\)
\(\frac{□}{□}\)

Answer: \(\frac{5}{12}\)

Explanation:
\(\frac{5}{8} \times \frac{2}{3}\)
Multiply the numerators and denominators of the fractions.
\(\frac{5}{8}\) × \(\frac{2}{3}\) = \(\frac{10}{24}\)
\(\frac{10}{24}\) = \(\frac{5}{12}\)
\(\frac{5}{8} \times \frac{2}{3}\) = \(\frac{5}{12}\)

Question 11.
\(\frac{7}{12} \times 8\)
______ \(\frac{□}{□}\)

Answer: 4 \(\frac{2}{3}\)

Explanation:
8 × \(\frac{7}{12}\)
Multiply the numerator with the whole numbers.
8 × \(\frac{7}{12}\) = \(\frac{56}{12}\) = \(\frac{14}{3}\)
Convert the improper fraction to the mixed fraction.
\(\frac{14}{3}\) = 4 \(\frac{2}{3}\)

Complete the statement with equal to, greater than, or less than.

Question 12.
3 × \(\frac{2}{3}\) _________ 3

Answer: Less Than

Explanation:
3 × \(\frac{2}{3}\)
Multiply the numerator with the whole numbers.
3 in the denominator will be canceled.
3 × \(\frac{2}{3}\) = 2
2 is less than 3.
3 × \(\frac{2}{3}\) less than 3.

Question 13.
\(\frac{5}{7}\) × 3 _________ \(\frac{5}{7}\)

Answer: Greater than

Explanation:
\(\frac{5}{7}\) × 3
Multiply the numerator with the whole numbers.
\(\frac{5}{7}\) × 3 = \(\frac{15}{7}\)
Convert it into mixed fraction.
\(\frac{15}{7}\) = 2 \(\frac{1}{4}\)
2 \(\frac{1}{4}\) is greater than \(\frac{5}{7}\)

Mid-Chapter Checkpoint – Page No. 316

Question 14.
There is \(\frac{5}{6}\) of an apple pie left from dinner. Tomorrow, Victor plans to eat \(\frac{1}{6}\) of the pie that was left. How much of the whole pie will be left after he eats tomorrow?
\(\frac{□}{□}\) of the whole pie

Answer: \(\frac{25}{36}\) of the whole pie

Explanation:
Gīven that,
An apple pie left from the dinner is \(\frac{5}{6}\)
Victor plans to eat pie which was left is \(\frac{1}{6}\)
The whole pie will be left after Victor eats tomorrow =?
Pie left from dinner = \(\frac{5}{6}\)
Victor plans to eat pie which was left  = \(\frac{1}{6}\)
\(\frac{5}{6}\) × \(\frac{1}{6}\) = \(\frac{5}{36}\)
To find the whole pie will be left after he eats tomorrow:
\(\frac{5}{6}\) – \(\frac{5}{36}\)
LCD = 36
\(\frac{5}{6}\) × \(\frac{6}{6}\) – \(\frac{5}{36}\)
\(\frac{30}{36}\) – \(\frac{5}{36}\) = \(\frac{25}{36}\)
Therefore, whole pie left after Victor eats tomorrow is \(\frac{25}{36}\)

Question 15.
Everett and Marie are going to make fruit bars for their family reunion. They want to make 4 times the amount the recipe makes. If the recipe calls for \(\frac{2}{3}\) cup of oil, how much oil will they need?
______ \(\frac{□}{□}\) cup of oil

Answer: 2 \(\frac{2}{3}\)

Explanation:
Everett and Marie are going to make fruit bars for their family reunion.
They want to make 4 times the amount the recipe makes.
4 × \(\frac{2}{3}\) = \(\frac{8}{3}\)
The mixed fraction of \(\frac{8}{3}\) is 2 \(\frac{2}{3}\)
Thus Everett and Marie need Everett and Marie of oil.

Question 16.
Matt made the model below to help him solve his math problem. Write an expression that matches Matt’s model.
Go Math Grade 5 Answer Key Chapter 7 Multiply Fractions Mid-Chapter Checkpoint img 17
Type below:
__________

Answer: \(\frac{3}{4}\) × \(\frac{1}{3}\)

Explanation:
By seeing the above figure we can say that the fraction for the Matt’s model is \(\frac{3}{4}\) and \(\frac{2}{3}\).
Multiply the fractions \(\frac{3}{4}\) × \(\frac{2}{3}\) = \(\frac{1}{4}\)

Share and Show – Page No. 319

Use the grid to find the area. Let each square represent \(\frac{1}{3}\) meter by \(\frac{1}{3}\) meter.

Question 1.
1 \(\frac{2}{3}\) × 1 \(\frac{1}{3}\)
• Draw a diagram to represent the dimensions.
Go Math Grade 5 Answer Key Chapter 7 Multiply Fractions img 18
• How many squares cover the diagram?
• What is the area of each square?
• What is the area of the diagram?
______ \(\frac{□}{□}\)

Answer: 2 \(\frac{2}{9}\)

Explanation:
20 squares cover the diagram.
Each square represents \(\frac{1}{9}\) square meter
20 × \(\frac{1}{9}\) = \(\frac{20}{9}\)
Convert the fraction into the mixed fraction.
\(\frac{20}{9}\) = 2 \(\frac{2}{9}\)
Thus the area of the diagram is 2 \(\frac{2}{9}\)

Use the grid to find the area. Let each square represent \(\frac{1}{4}\) meter by \(\frac{1}{4}\) meter.

Question 2.
1 \(\frac{3}{4}\) × 1 \(\frac{2}{4}\)
Go Math Grade 5 Answer Key Chapter 7 Multiply Fractions img 19
______ \(\frac{□}{□}\)

Answer: 2 \(\frac{5}{8}\)

Explanation:
42 squares cover the diagram.
Each square represents \(\frac{1}{16}\) square meters.
42 × \(\frac{1}{16}\) = \(\frac{21}{8}\)
Convert the fraction into the mixed fraction.
\(\frac{21}{8}\) = 2 \(\frac{5}{8}\)
The area of the diagram is 2 \(\frac{5}{8}\) square meter.

Question 3.
1 \(\frac{1}{4}\) × 1 \(\frac{1}{2}\)
Go Math Grade 5 Answer Key Chapter 7 Multiply Fractions img 20
______ \(\frac{□}{□}\)

Answer: 1 \(\frac{7}{8}\)

Explanation:
30 squares cover the diagram.
Each square represents \(\frac{1}{16}\) square meters.
30 × \(\frac{1}{16}\) = \(\frac{15}{8}\)
Convert the fraction into the mixed fraction.
\(\frac{15}{8}\) = 1 \(\frac{7}{8}\)

Use an area model to solve.

Question 4.
1 \(\frac{3}{4}\) × 2 \(\frac{1}{2}\)
______ \(\frac{□}{□}\)

Answer: 4 \(\frac{3}{8}\)

Explanation:

54 squares covers the diagram.
Each square represents \(\frac{1}{16}\) square meters.
54 × \(\frac{1}{16}\) = \(\frac{27}{8}\)
Convert the fraction into the mixed fraction.
\(\frac{27}{8}\) = 4 \(\frac{3}{8}\)

Question 5.
1 \(\frac{3}{8}\) × 2 \(\frac{1}{2}\)
______ \(\frac{□}{□}\)

Answer: 3 \(\frac{7}{16}\)

Explanation:
55 squares cover the diagram.
Each square represents \(\frac{1}{16}\) square meters.
55 × \(\frac{1}{16}\) = \(\frac{55}{16}\)
Convert the fraction into the mixed fraction.
\(\frac{55}{16}\) = 3 \(\frac{7}{16}\)

Question 6.
1 \(\frac{1}{9}\) × 1 \(\frac{2}{3}\)
______ \(\frac{□}{□}\)

Answer: 1 \(\frac{23}{27}\)

Explanation:
130 squares the diagram.
Each square represents \(\frac{1}{16}\) square meters.
1 \(\frac{1}{9}\) × 1 \(\frac{2}{3}\)
\(\frac{10}{9}\) × \(\frac{5}{3}\) = \(\frac{50}{27}\)
Convert the fraction into the mixed fraction.
\(\frac{50}{27}\) = 1 \(\frac{23}{27}\)

Question 7.
Explain how finding the area of a rectangle with whole-number side lengths compares to finding the area of a rectangle with fractional side lengths.
Type below:
__________

Answer:
Go-Math-Grade-5-Answer-Key-Chapter-7-Multiply-Fractions-img-20
15 squares cover the diagram.
Each square is \(\frac{1}{16}\) square unit.
The area of the diagram is \(\frac{15}{16}\) square units.

Problem Solving – Page No. 320

Pose a Problem

Question 8.
Terrance is designing a garden. He drew the following diagram of his garden. Pose a problem using mixed numbers that can be solved using his diagram.
Go Math Grade 5 Answer Key Chapter 7 Multiply Fractions img 21
Go Math Grade 5 Answer Key Chapter 7 Multiply Fractions img 22
Pose a Problem.                 Solve your problem.
Describe how you decided on the dimensions of Terrance’s garden.
Type below:
__________

Answer:
how finding the area of a rectangle with mixed fractions side compares to finding the area of the rectangle with the fractional side lengths.
6 × 1 \(\frac{1}{8}\) = \(\frac{□}{□}\)
Let each square represent \(\frac{1}{2}\) meter by \(\frac{1}{2}\)
From the above figure, we can say that the number of squares is 27.
So, 27 squares cover the diagram.
Each square is \(\frac{1}{4}\) square unit.
27 × \(\frac{1}{4}\)  = \(\frac{27}{4}\)
Convert the fraction into the mixed fraction.
\(\frac{27}{4}\) = 6 \(\frac{3}{4}\)

Share and Show – Page No. 323

Complete the statement with equal to, greater than, or less than.

Question 1.
\(\frac{5}{6}\) × 2 \(\frac{1}{5}\) will be __________ 2 \(\frac{1}{5}\)
Go Math Grade 5 Answer Key Chapter 7 Multiply Fractions img 23 Shade the model to
show \(\frac{5}{6}\) × 2 \(\frac{1}{5}\) .
__________

Answer: Less than

Explanation:
\(\frac{5}{6}\) × 2 \(\frac{1}{5}\)
Convert the mixed fraction to the improper fraction.
2 \(\frac{1}{5}\) = \(\frac{11}{5}\)
\(\frac{5}{6}\) × \(\frac{11}{5}\) = \(\frac{55}{30}\)
1 \(\frac{25}{30}\) = 1 \(\frac{5}{6}\)
Thus 1 \(\frac{5}{6}\) is less than 2 \(\frac{1}{5}\)

Question 2.
1 \(\frac{1}{5}\) × 2 \(\frac{2}{3}\) will be __________ 2 \(\frac{2}{3}\)

Answer: Greater than

Explanation:
1 \(\frac{1}{5}\) × 2 \(\frac{2}{3}\)
First, Convert the mixed fraction to the improper fraction.
\(\frac{6}{5}\) × \(\frac{8}{3}\) = \(\frac{48}{15}\)
\(\frac{48}{15}\) = 3 \(\frac{3}{15}\)
3 \(\frac{3}{15}\) is greater than 2 \(\frac{2}{3}\)

Question 3.
\(\frac{4}{5}\) × 2 \(\frac{2}{5}\) will be __________ 2 \(\frac{2}{5}\)

Answer: Less than

Explanation:
\(\frac{4}{5}\) × \(\frac{12}{5}\)
First, Convert the mixed fraction to the improper fraction.
\(\frac{4}{5}\) × \(\frac{12}{5}\) = \(\frac{48}{5}\)
\(\frac{48}{5}\) = 9 \(\frac{3}{5}\)
9 \(\frac{3}{5}\) is less than 2 \(\frac{2}{5}\)

On Your Own

Complete the statement with equal to, greater than, or less than.

Question 4.
\(\frac{2}{2}\) × 1 \(\frac{1}{2}\) will be __________ 1 \(\frac{1}{2}\)

Answer: Equal to

Explanation:
\(\frac{2}{2}\) × \(\frac{3}{2}\) = \(\frac{6}{4}\)
\(\frac{6}{4}\) = 1 \(\frac{1}{2}\)
1 \(\frac{1}{2}\) is equal to 1 \(\frac{1}{2}\)
\(\frac{2}{2}\) × 1 \(\frac{1}{2}\) will be equal to 1 \(\frac{1}{2}\)

Question 5.
\(\frac{2}{3}\) × 3 \(\frac{1}{6}\) will be __________ 3 \(\frac{1}{6}\)

Answer: Less than

Explanation:
\(\frac{2}{3}\) × 3 \(\frac{1}{6}\)
First, Convert the mixed fraction to the improper fraction.
\(\frac{2}{3}\) × \(\frac{19}{6}\) = \(\frac{38}{18}\)
\(\frac{38}{18}\) = 2 \(\frac{2}{18}\)
2 \(\frac{2}{18}\) is less than 3 \(\frac{1}{6}\)
\(\frac{2}{3}\) × 3 \(\frac{1}{6}\) will be less than 3 \(\frac{1}{6}\)

Question 6.
2 × 2 \(\frac{1}{4}\) will be __________ 2 \(\frac{1}{4}\)

Answer: Greater than

Explanation:
2 × 2 \(\frac{1}{4}\)
First, Convert the mixed fraction to the improper fraction.
2 × \(\frac{9}{4}\) = \(\frac{18}{4}\)
\(\frac{18}{4}\) = 4 \(\frac{2}{4}\)
4 \(\frac{1}{2}\) is greater than 2 \(\frac{1}{4}\)

Question 7.
4 × 1 \(\frac{3}{7}\) will be __________ 1 \(\frac{3}{7}\)

Answer: Greater than

Explanation:
4 × 1 \(\frac{3}{7}\)
First, Convert the mixed fraction to the improper fraction.
4 × \(\frac{10}{7}\) = \(\frac{40}{7}\)
4 × 1 \(\frac{3}{7}\) = 5 \(\frac{5}{7}\)
5 \(\frac{5}{7}\) is greater than 1 \(\frac{3}{7}\)

Algebra Tell whether the unknown factor is less than 1 or greater than 1.

Question 8.
■ × 1 \(\frac{2}{3}\) = \(\frac{5}{6}\)
The unknown factor is __________ 1.

Answer: Less than

Explanation:
■ × 1 \(\frac{2}{3}\) = \(\frac{5}{6}\)
■ × \(\frac{5}{3}\) = \(\frac{5}{6}\)
■ = \(\frac{1}{2}\)
Thus the unknown factor is \(\frac{1}{2}\)
\(\frac{1}{2}\) is less than 1.

Question 9.
■ × 1 \(\frac{1}{4}\) = 2 \(\frac{1}{2}\)
The unknown factor is __________ 1.

Answer: Greater than

Explanation:
■ × 1 \(\frac{1}{4}\) = 2 \(\frac{1}{2}\)
■ = 2 \(\frac{1}{2}\) ÷ 1 \(\frac{1}{4}\)
■ = 2 × 2 = 4
■ = 4
Thus the unknown factor is 4
4 is greater than 1.

Problem Solving – Page No. 324

Question 10.
Kyle is making a scale drawing of his math book. The dimensions of his drawing will be \(\frac{1}{3}\) the dimensions of his book. If the width of his book is 8 \(\frac{1}{2}\) inches, will the width of his drawing be equal to, greater than, or less than 8 \(\frac{1}{2}\) inches?
__________

Answer: Less than

Explanation:
Given that,
Kyle is making a scale drawing of his math book.
The dimensions of his drawing will be \(\frac{1}{3}\) the dimensions of his book.
\(\frac{1}{3}\) × 8 \(\frac{1}{2}\)
First, Convert the mixed fraction to the improper fraction.
\(\frac{1}{3}\) × \(\frac{17}{2}\) = \(\frac{17}{6}\)
Convert the fraction into the mixed fraction.
\(\frac{17}{6}\) = 2 \(\frac{5}{6}\)
2 \(\frac{5}{6}\) is less than 8 \(\frac{1}{2}\) inches.

Question 11.
Sense or Nonsense?
Penny wants to make a model of a beetle that is larger than life-size. Penny says she is going to use a scaling factor of \(\frac{7}{12}\). Does this make sense or is it nonsense? Explain.
Type below:
__________

Answer: It is nonsense because Penny wants to make beetle Larger than life size. So, the scaling factor \(\frac{7}{12}\) is not corresponding, because when we multiply any value with the number less than 1 we get a smaller number.

Question 12.
Shannon, Mary, and John earn a weekly allowance. Shannon earns an amount that is \(\frac{2}{3}\) of what John earns. Mary earns an amount that is 1 \(\frac{2}{3}\) of what John earns. John earns $20 a week. Who earns the greatest allowance? Who earns the least?
__________ earns the greatest allowance.
__________ earns the least allowance

Answer:
Mary earns the greatest allowance.
Shannon earns the least allowance.

Explanation:

Shannon, Mary, and John earn a weekly allowance.
Shannon earns an amount that is \(\frac{2}{3}\) of what John earns.
Mary earns an amount that is 1 \(\frac{2}{3}\) of what John earns.
John earns $20 a week.
\(\frac{2}{3}\) ________ 1 \(\frac{2}{3}\)
Convert the mixed fraction into the improper fraction.
1 \(\frac{2}{3}\) = \(\frac{5}{3}\)
\(\frac{2}{3}\) is less than 1 \(\frac{2}{3}\)
Thus Shannon earns the least allowance and Mary earns the greatest allowance.

Question 13.
Test Prep Addie’s puppy weighs 1 \(\frac{2}{3}\) times what it weighed when it was born. It weighed 1 \(\frac{1}{3}\) pounds at birth. Which statement below is true?
Options:
a. The puppy weighs the same as it did at birth.
b. The puppy weighs less than it did at birth.
c. The puppy weighs more than it did at birth.
d. The puppy weighs twice what it did at birth.

Answer: The puppy weighs more than it did at birth.

Explanation:
Test Prep Addie’s puppy weighs 1 \(\frac{2}{3}\) times what it weighed when it was born.
It weighed 1 \(\frac{1}{3}\) pounds at birth.
1 \(\frac{2}{3}\) is greater than 1 \(\frac{1}{3}\).
So, the puppy weighs more than it did at birth.
Thus the correct answer is option C.

Share and Show – Page No. 327

Find the product. Write the product in simplest form.

Question 1.
1 \(\frac{2}{3}\) × 3 \(\frac{4}{5}\) = \(\frac{■}{3}\) × \(\frac{■}{5}\)
= \(\frac{■}{■}\)
=?
_____ \(\frac{□}{□}\)

Answer: 6 \(\frac{1}{3}\)

Explanation:

1 \(\frac{2}{3}\) × 3 \(\frac{4}{5}\)
\(\frac{5}{3}\) × \(\frac{19}{5}\)
\(\frac{■}{3}\) × \(\frac{■}{5}\) = \(\frac{5}{3}\) × \(\frac{19}{5}\)
\(\frac{5}{3}\) × \(\frac{19}{5}\) = 6 \(\frac{1}{3}\)

Question 2.
\(\frac{1}{2}\) × 1 \(\frac{1}{3}\)
Go Math Grade 5 Answer Key Chapter 7 Multiply Fractions img 24
Shade the model to find the product.
\(\frac{□}{□}\)

Answer: \(\frac{2}{3}\)

Explanation:
\(\frac{1}{2}\) × 1 \(\frac{1}{3}\)
1 \(\frac{1}{3}\) = \(\frac{4}{3}\)
\(\frac{1}{2}\) × \(\frac{4}{3}\) = \(\frac{4}{6}\)

Question 3.
\(1 \frac{1}{8} \times 2 \frac{1}{3}\) = ______ \(\frac{□}{□}\)

Answer: 2 \(\frac{5}{8}\)

Explanation:
1 \(\frac{1}{8}\) × 2 \(\frac{1}{3}\)
\(\frac{9}{8}\) × \(\frac{7}{3}\) = \(\frac{63}{24}\)
\(\frac{63}{24}\) = 2 \(\frac{63}{24}\) = 2 \(\frac{15}{24}\)
2 \(\frac{15}{24}\) = 2 \(\frac{5}{8}\)
\(1 \frac{1}{8} \times 2 \frac{1}{3}\) = 2 \(\frac{5}{8}\)

Question 4.
\(\frac{3}{4} \times 6 \frac{5}{6}\) = ______ \(\frac{□}{□}\)

Answer: 5 \(\frac{1}{8}\)

Explanation:
\(\frac{3}{4}\) × 6 \(\frac{5}{6}\)
\(\frac{3}{4}\) × \(\frac{41}{6}\)
\(\frac{123}{24}\) = \(\frac{41}{8}\)
Convert the fraction to the mixed fraction.
\(\frac{41}{8}\) = 5 \(\frac{1}{8}\)

Question 5.
\(1 \frac{2}{7} \times 1 \frac{3}{4}\) = ______ \(\frac{□}{□}\)

Answer: 2 \(\frac{1}{4}\)

Explanation:
1 \(\frac{2}{7}\) × 1 \(\frac{3}{4}\)
Multiply the numerators and the denominators.
Convert the mixed fraction to the improper fraction.
\(\frac{9}{7}\) × \(\frac{7}{4}\) = \(\frac{63}{28}\)
\(\frac{63}{28}\) = 2 \(\frac{1}{4}\)
\(1 \frac{2}{7} \times 1 \frac{3}{4}\) = 2 \(\frac{1}{4}\)

Question 6.
\(\frac{3}{4} \times 1 \frac{1}{4}\) = ______ \(\frac{□}{□}\)

Answer: \(\frac{15}{16}\)

Explanation:
\(\frac{3}{4}\) × 1 \(\frac{1}{4}\)
\(\frac{3}{4}\) × \(\frac{5}{4}\) = \(\frac{15}{16}\)
\(\frac{3}{4} \times 1 \frac{1}{4}\) = \(\frac{15}{16}\)

Use the Distributive Property to find the product.

Question 7.
\(16 \times 2 \frac{1}{2}\) = ______

Answer: 40

Explanation:
\(16 \times 2 \frac{1}{2}\)
(16 × 2) + (16 × \(\frac{1}{2}\))
32 + 8 = 40
\(16 \times 2 \frac{1}{2}\) = 40

Question 8.
\(1 \frac{4}{5} \times 15\) = ______

Answer: 27

Explanation:
\(1 \frac{4}{5} \times 15\)
15 × 1 \(\frac{4}{5}\)
(1 × 15) + (15 × \(\frac{4}{5}\))
15 + \(\frac{60}{5}\)
15 + 12 = 27
Thus \(1 \frac{4}{5} \times 15\) = 27

On Your Own

Find the product. Write the product in simplest form.

Question 9.
\(\frac{3}{4} \times 1 \frac{1}{2}\) = ______ \(\frac{□}{□}\)

Answer: 1 \(\frac{1}{8}\)

Explanation:
\(\frac{3}{4}\) × 1 \(\frac{1}{2}\)
\(\frac{3}{4}\) × \(\frac{3}{2}\) = \(\frac{9}{8}\)
Now convert the improper fraction to the mixed fraction.
\(\frac{9}{8}\) = 1 \(\frac{1}{8}\)

Question 10.
\(4 \frac{2}{5} \times 1 \frac{1}{2}\) = ______ \(\frac{□}{□}\)

Answer: 6 \(\frac{3}{5}\)

Explanation:
4 \(\frac{2}{5}\) × 1 \(\frac{1}{2}\)
Convert the mixed fraction to the improper fraction.
\(\frac{22}{5}\) × \(\frac{3}{2}\) = \(\frac{66}{10}\)
The mixed fraction of \(\frac{66}{10}\) is 6 \(\frac{3}{5}\)

Question 11.
\(5 \frac{1}{3} \times \frac{3}{4}\) = ______

Answer: 4

Explanation:
5 \(\frac{1}{3}\) × \(\frac{3}{4}\)
Convert the mixed fraction to the improper fraction.
\(\frac{16}{3}\) × \(\frac{3}{4}\) = \(\frac{48}{12}\)
12 divides 48 four times.
Thus \(5 \frac{1}{3} \times \frac{3}{4}\) = 4

Question 12.
\(2 \frac{1}{2} \times 5 \frac{1}{5}\) = ______

Answer: 13

Explanation:
2 \(\frac{1}{2}\) × 5 \(\frac{1}{5}\)
\(\frac{5}{2}\) × \(\frac{26}{5}\) = \(\frac{130}{10}\)
10 divides 130 thirteen times.
\(\frac{130}{10}\) = 13
\(2 \frac{1}{2} \times 5 \frac{1}{5}\) = 13

Question 13.
\(12 \frac{3}{4} \times 2 \frac{2}{3}\) = ______

Answer: 34

Explanation:
12 \(\frac{3}{4}\) × 2 \(\frac{2}{3}\)
\(\frac{51}{4}\) × \(\frac{6}{3}\)
3 divides 51 seventeen times.
17 × 2 = 34

Question 14.
\(3 \times 4 \frac{1}{2}\) = ______ \(\frac{□}{□}\)

Answer: 13 \(\frac{1}{2}\)

Explanation:
3 × 4 \(\frac{1}{2}\)
3 × \(\frac{9}{2}\) = \(\frac{27}{2}\)
Convert the fraction to the mixed fraction
\(\frac{27}{2}\) = 13 \(\frac{1}{2}\)

Question 15.
\(2 \frac{3}{8} \times \frac{4}{9}\) = ______ \(\frac{□}{□}\)

Answer: 1 \(\frac{1}{18}\)

Explanation:
2 \(\frac{3}{8}\) × \(\frac{4}{9}\)
\(\frac{19}{8}\) × \(\frac{4}{9}\) = \(\frac{76}{72}\)
\(\frac{76}{72}\) = 1 \(\frac{1}{18}\)
\(2 \frac{3}{8} \times \frac{4}{9}\) = 1 \(\frac{1}{18}\)

Question 16.
\(1 \frac{1}{3} \times 1 \frac{1}{4} \times 1 \frac{1}{5}\) = ______

Answer: 2

Explanation:

1 \(\frac{1}{3}\) × 1 \(\frac{1}{4}\) × 1 \(\frac{1}{5}\)
\(\frac{4}{3}\) × \(\frac{5}{4}\) × \(\frac{6}{5}\) = 2
\(1 \frac{1}{3} \times 1 \frac{1}{4} \times 1 \frac{1}{5}\) = 2

Use the Distributive Property to find the product.

Question 17.
\(10 \times 2 \frac{3}{5}\) = ______

Answer: 26

Explanation:
10 × 2 \(\frac{3}{5}\)
Now use the Distributive Property to find the product.
(10 × 2) + (10 × \(\frac{3}{5}\))
20 + \(\frac{30}{5}\)
5 divides 30 6 times.
20 + 6 = 26

Question 18.
\(3 \frac{3}{4} \times 12\) = ______

Answer: 45

Explanation:
3 \(\frac{3}{4}\) × 12
Now use the Distributive Property to find the product.
(12 × 3) + (12 × \(\frac{3}{4}\))
36 + \(\frac{36}{4}\)
36 + 9 = 45
\(3 \frac{3}{4} \times 12\) = 45

Share and Show Connect to health – Page No. 328

Changing Recipes

You can make a lot of recipes more healthful by reducing the amounts of fat, sugar, and salt.
Go Math Grade 5 Answer Key Chapter 7 Multiply Fractions img 25

Kelly has a muffin recipe that calls for 1 \(\frac{1}{2}\) cups of sugar. She wants to use \(\frac{1}{2}\) that amount of sugar and more cinnamon and vanilla. How much sugar will she use?
Multiply 1 \(\frac{1}{2}\) by \(\frac{1}{2}\) to find what part of the original amount of sugar to use.
Write the mixed number as a fraction greater than 1. Then, multiply.
\(\frac{1}{2} \times 1 \frac{1}{2}=\frac{1}{2} \times \frac{3}{2}\)
= \(\frac{3}{4}\)
So, Kelly will use \(\frac{3}{4}\) cup of sugar.

Question 19.
Michelle has a recipe that calls for 2 \(\frac{1}{2}\) cups of vegetable oil. She wants to use \(\frac{2}{3}\) that amount of oil and use applesauce to replace the rest. How much vegetable oil will she use?
______ \(\frac{□}{□}\) cups

Answer: 1 \(\frac{2}{3}\)

Explanation:
Michelle has a recipe that calls for 2 \(\frac{1}{2}\) cups of vegetable oil.
She wants to use \(\frac{2}{3}\) that amount of oil and use applesauce to replace the rest
Multiply 2 \(\frac{1}{2}\) by \(\frac{2}{3}\) to find how much vegetable oil she will use.
2 \(\frac{1}{2}\) × \(\frac{2}{3}\)
Convert the mixed fractions into the fractions.
\(\frac{5}{2}\) × \(\frac{2}{3}\) = \(\frac{10}{6}\)
\(\frac{10}{6}\) = \(\frac{5}{3}\) = 1 \(\frac{2}{3}\)
She will use 1 \(\frac{2}{3}\) cups of vegetable oil.

Question 20.
Tony’s recipe for soup calls for 1 \(\frac{1}{4}\) teaspoons of salt. He wants to use \(\frac{1}{2}\) that amount. How much salt will he use?
\(\frac{□}{□}\) teaspoon

Answer: \(\frac{5}{8}\)

Explanation:
Tony’s recipe for soup calls for 1 \(\frac{1}{4}\) teaspoons of salt.
He wants to use \(\frac{1}{2}\) that amount.
Multiply the fractions to find how much salt he will use in the recipe for soup.
1 \(\frac{1}{4}\) × \(\frac{1}{2}\)
Convert the mixed fractions to the improper fractions.
\(\frac{5}{4}\) × \(\frac{1}{2}\) = \(\frac{5}{8}\)
Thus Tony use \(\frac{5}{8}\) teaspoon of salt for soup.

Question 21.
Jeffrey’s recipe for oatmeal muffins calls for 2 \(\frac{1}{4}\) cups of oatmeal and makes one dozen muffins. If he makes 1 \(\frac{1}{2}\) dozen muffins for a club meeting, how much oatmeal will he use?
_____ \(\frac{□}{□}\) cups

Answer: 3 \(\frac{3}{8}\)

Explanation:
Jeffrey’s recipe for oatmeal muffins calls for 2 \(\frac{1}{4}\) cups of oatmeal and makes one dozen muffins.
To find how much oatmeal he will use we need to multiply the fractions.
2 \(\frac{1}{4}\) × 1 \(\frac{1}{2}\)
Convert the mixed fractions to the improper fractions.
\(\frac{9}{4}\) × \(\frac{3}{2}\)
\(\frac{27}{8}\) = 3 \(\frac{3}{8}\)
Thus he will use 3 \(\frac{3}{8}\) cups of oatmeal to make oatmeal muffins.

Question 22.
Cara’s muffin recipe calls for 1 \(\frac{1}{2}\) cups of flour for the muffins and \(\frac{1}{4}\) cup of flour for the topping. If she makes \(\frac{1}{2}\) of the original recipe, how much flour will she use?
\(\frac{□}{□}\) cup of flour

Answer: \(\frac{7}{8}\)

Explanation:
Convert mixed fractions into improper fractions.
1 \(\frac{1}{2}\) = \(\frac{3}{2}\)
\(\frac{3}{2}\) + \(\frac{1}{4}\) = \(\frac{7}{4}\)
Now we can find how much flour she will use to make \(\frac{1}{2}\) of the original recipe, when multiply
\(\frac{7}{4}\) by \(\frac{1}{2}\)
\(\frac{7}{4}\) × \(\frac{1}{2}\) = \(\frac{7}{8}\)

Share and Show – Page No. 331

Go Math Grade 5 Answer Key Chapter 7 Multiply Fractions img 26

Question 1.
When Pascal built a dog house, he knew he wanted the floor of the house to have an area of 24 square feet. He also wanted the width to be \(\frac{2}{3}\) the length. What are the dimensions of the dog house?
First, choose two numbers that have a product of 24.
Guess: ____ feet and ____ feet
Then, check those numbers. Is the greater number \(\frac{2}{3}\) of the other number?
Check: \(\frac{2}{3}\) × _____ = _____
My guess is ______.
Finally, if the guess is not correct, revise it and check again. Continue until you find the correct answer.
_____ feet by _____ feet

Answer: 4 feet by 6 feet

Explanation:
When Pascal built a dog house, he knew he wanted the floor of the house to have an area of 24 square feet.
He also wanted the width to be \(\frac{2}{3}\) the length.
My guess for 24 square feet is 4 feet and 6 feet.
Now let us check the numbers.
6 × \(\frac{2}{3}\) = 4
So my guess is correct.
Thus the dimensions are 4 feet by 6 feet

Question 2.
What if Pascal wanted the area of the floor to be 54 square feet and the width still to be \(\frac{2}{3}\) the length? What would the dimensions of the floor be?
_____ feet by _____ feet

Answer: 6 feet by 9 feet

Explanation:
My guess for 54 square feet is  6 feet and 9 feet.
9 × \(\frac{2}{3}\)
3 divides 9 three times.
9 × \(\frac{2}{3}\) = 6
So, my guess is correct.
Therefore the dimensions of the will be 6 feet by 9 feet

Question 3.
Leo wants to paint a mural that covers a wall with an area of 1,440 square feet. The height of the wall is \(\frac{2}{5}\) of its length. What is the length and the height of the wall?
_____ feet by _____ feet

Answer: 24 feet by 60 feet

Explanation:
Leo wants to paint a mural that covers a wall with an area of 1,440 square feet. The height of the wall is \(\frac{2}{5}\) of its length.
Guess: 1,440 square feet = 24 feet × 60 feet
\(\frac{2}{5}\) × 60 = 24
So, our guess is correct.
.Thus the dimensions of the wall are 24 feet by 60 feet.

On Your Own – Page No. 332

Question 4.
Barry wants to make a drawing that is \(\frac{1}{4}\) the size of the original. If a tree in the original drawing is 14 inches tall, how tall will the tree in Barry’s drawing be?
_____ \(\frac{□}{□}\) inches

Answer: 3 \(\frac{1}{2}\) inches

Explanation:
Given:
Barry wants to make a drawing that is \(\frac{1}{4}\) the size of the original.
The tree is 14 inches tall in the drawing.
14 × \(\frac{1}{4}\) = \(\frac{14}{4}\) = \(\frac{7}{2}\)
Convert the fraction to the mixed fraction.
\(\frac{7}{2}\) = 3 \(\frac{1}{2}\) inches

Question 5.
A blueprint is a scale drawing of a building. The dimensions of the blueprint for Penny’s doll house are \(\frac{1}{4}\) of the measurements of the actual doll house. The floor of the doll house has an area of 864 square inches. If the width of the doll house is \(\frac{2}{3}\) the length, what are the dimensions of the floor on the blueprint of the doll house?
_____ inches by _____ inches

Answer: 9 inches by 6 inches

Explanation:
A blueprint is a scale drawing of a building.
The dimensions of the blueprint for Penny’s dollhouse are \(\frac{1}{4}\) of the measurements of the actual dollhouse.
The floor of the dollhouse has an area of 864 square inches.
The area of the dollhouse is 54 square inches.
My guess is 9 inches by 6 inches
Let us check the numbers
9 × \(\frac{2}{3}\) = 6
My guess is correct.
Therefore the dimensions of the floor on the blueprint of the dollhouse is 9 inches by 6 inches

Question 6.
Pose a Problem Look back at Exercise 4. Write a similar problem using a different measurement and a different fraction. Then solve your problem.
Type below:
__________

Answer:
Kyle is making reusable grocery bags and lunch bags. She used a 3/4 yard of cloth to make the grocery bag. A lunch bag requires 2/3 of the amount of cloth of a grocery bag’s needs. How much does she need to make the lunch bag?
\(\frac{3}{4}\) × \(\frac{2}{3}\) = \(\frac{1}{2}\)
Thus Kyle needs \(\frac{1}{2}\) of the grocery bag to make the lunch bag.

Question 7.
Test Prep Albert’s photograph has an area of 80 square inches. The length of the photo is 1 \(\frac{1}{4}\) the width. Which of the following could be the dimensions of the photograph?
Options:
a. 5 inches by 16 inches
b. 12 inches by 10 inches
c. 6 inches by 5 inches
d. 10 inches by 8 inches

Answer: 10 inches by 8 inches

Explanation:
Albert’s photograph has an area of 80 square inches.
The length of the photo is 1 \(\frac{1}{4}\) the width.
My guess for 80 square inches is 10 inches by 8 inches.
Now let us check the numbers.
8 × 1 \(\frac{1}{4}\) = 8 × \(\frac{5}{4}\) = 10
Thus the correct answer is option D.

Chapter Review/Test – Page No. 333

Concepts and Skills

Question 1.
When you multiply 3 \(\frac{1}{4}\) by a number greater than one, how does the product compare to 3 \(\frac{1}{4}\)? Explain.
Type below:
__________

Answer:
Your product will be greater than 3 1/4 because anytime you multiply a fraction times a whole number less than 1 you get a fraction less than one and any time you multiply by a fraction and a whole number greater than 1 your answer is greater than 1.

Use a model to solve.

Question 2.
\(\frac{2}{3}\) × 6 = _____

Answer: 4

Explanation:
\(\frac{2}{3}\) × 6
3 divides 6 two times.
2 × 2 = 4
\(\frac{2}{3}\) × 6 = 4

Question 3.
\(\frac{3}{7}\) × 14 = _____

Answer: 6

Explanation:
\(\frac{3}{7}\) × 14
7 divides 14 two times.
3 × 2 = 6
\(\frac{3}{7}\) × 14 = 6

Question 4.
\(\frac{5}{8}\) × 24 = _____

Answer: 15

Explanation:

\(\frac{5}{8}\) × 24
8 divides 24 three times.
5 × 3 = 15
\(\frac{5}{8}\) × 24 = 15

Find the product. Write the product in simplest form.

Question 5.
\(\frac{3}{5}\) × 8 = _____ \(\frac{□}{□}\)

Answer: 4 \(\frac{4}{5}\)

Explanation:
\(\frac{3}{5}\) × 8 = \(\frac{24}{5}\)
The mixed fraction of \(\frac{24}{5}\) is 4 \(\frac{4}{5}\)
\(\frac{3}{5}\) × 8 = 4 \(\frac{4}{5}\)

Question 6.
\(\frac{1}{4}\) × 10 = _____ \(\frac{□}{□}\)

Answer: 2 \(\frac{1}{2}\)

Explanation:
\(\frac{1}{4}\) × 10
2 divides 10 five times.
\(\frac{1}{2}\) × 5 = \(\frac{5}{2}\)
The mixed fraction of \(\frac{5}{2}\) is 2 \(\frac{1}{2}\)
\(\frac{1}{4}\) × 10 = 2 \(\frac{1}{2}\)

Question 7.
\(\frac{7}{5}\) × 15 = _____

Answer: 21

\(\frac{7}{5}\) × 15
5 divides 15 three times.
\(\frac{7}{5}\) × 15 = 7 × 3 = 21
\(\frac{7}{5}\) × 15 = 21

Question 8.
\(\frac{5}{6}\) × \(\frac{2}{3}\) = \(\frac{□}{□}\)

Answer: \(\frac{5}{9}\)

Explanation:
\(\frac{5}{6}\) × \(\frac{2}{3}\) = \(\frac{10}{18}\)
\(\frac{10}{18}\) = \(\frac{5}{9}\)
Thus \(\frac{5}{6}\) × \(\frac{2}{3}\) = \(\frac{5}{9}\)

Question 9.
\(\frac{1}{5}\) × \(\frac{5}{7}\) = \(\frac{□}{□}\)

Answer: \(\frac{1}{7}\)

Explanation:
\(\frac{1}{5}\) × \(\frac{5}{7}\)
5 in the numerator and 5 in the denominator gets canceled.
= \(\frac{1}{7}\)
Thus \(\frac{1}{5}\) × \(\frac{5}{7}\) = \(\frac{1}{7}\)

Question 10.
\(\frac{3}{8}\) × \(\frac{1}{6}\) = \(\frac{□}{□}\)

Answer: \(\frac{1}{16}\)

Explanation:
\(\frac{3}{8}\) × \(\frac{1}{6}\)
3 divides 6 two times
\(\frac{3}{8}\) × \(\frac{1}{6}\) = \(\frac{1}{8}\) × \(\frac{1}{2}\)
Multiply the denominators.
= \(\frac{1}{16}\)
Thus \(\frac{3}{8}\) × \(\frac{1}{6}\) = \(\frac{1}{16}\)

Complete the statement with equal to, greater than, or less than.

Question 11.
\(\frac{7}{8}\) × \(\frac{6}{6}\) will be __________ \(\frac{7}{8}\)

Answer: Equal to

Explanation:
\(\frac{7}{8}\) × \(\frac{6}{6}\)
\(\frac{6}{6}\) = 1
\(\frac{7}{8}\) × 1 = \(\frac{7}{8}\)
\(\frac{7}{8}\) = \(\frac{7}{8}\)
Thus \(\frac{7}{8}\) × \(\frac{6}{6}\) will be equal to \(\frac{7}{8}\)

Question 12.
\(\frac{1}{2}\) × \(\frac{8}{9}\) will be __________ \(\frac{8}{9}\)

Answer: Less than

Explanation:
\(\frac{1}{2}\) × \(\frac{8}{9}\)
Multiply the numerators and denominators
\(\frac{1}{2}\) × \(\frac{8}{9}\) = \(\frac{4}{9}\)
\(\frac{4}{9}\) is less than \(\frac{8}{9}\)
So, \(\frac{1}{2}\) × \(\frac{8}{9}\) will be less than \(\frac{8}{9}\)

Chapter Review/Test – Page No. 334

Fill in the bubble completely to show your answer.

Question 13.
Wolfgang wants to enlarge a picture he developed. Which factor listed below would scale up (enlarge) his picture the most if he used it to multiply its current dimensions?
Options:
a. \(\frac{7}{8}\)
b. \(\frac{14}{14}\)
c. 1 \(\frac{4}{9}\)
d. \(\frac{3}{2}\)

Answer: 1 \(\frac{4}{9}\)

Explanation:
The greatest fraction among all the fractions is 1 \(\frac{4}{9}\).
1 \(\frac{4}{9}\) is greater than 1.
Thus the correct answer is option C.

Question 14.
Rachel wants to reduce the size of her photo. Which factor listed below would scale down (reduce) the size of her picture the most?
Options:
a. \(\frac{5}{8}\)
b. \(\frac{11}{16}\)
c. 1 \(\frac{3}{4}\)
d. \(\frac{8}{5}\)

Answer: \(\frac{5}{8}\)

Explanation:
Compare to all the fractions \(\frac{5}{8}\) is smaller.
So, Rachel would reduce the size of her picture to \(\frac{5}{8}\)
So, the correct answer is option A.

Question 15.
Marteen wants to paint \(\frac{2}{3}\) of her room today. She wants to paint \(\frac{1}{4}\) of that before lunch. How much of her room will she paint today before lunch?
Options:
a. \(\frac{1}{12}\)
b. \(\frac{1}{6}\)
c. 1 \(\frac{5}{12}\)
d. \(\frac{11}{12}\)

Answer: \(\frac{1}{6}\)

Explanation:
Marteen wants to paint \(\frac{2}{3}\) of her room today.
She wants to paint \(\frac{1}{4}\) of that before lunch.
\(\frac{2}{3}\) × \(\frac{1}{4}\) = \(\frac{1}{6}\)
So, the answer is option B.

Chapter Review/Test – Page No. 335

Fill in the bubble completely to show your answer.

Question 16.
Gia’s bus route to school is 5 \(\frac{1}{2}\) miles. The bus route home is 1 \(\frac{3}{5}\) times as long. How long is Gia’s bus route home?
Options:
a. 5 \(\frac{3}{10}\) miles
b. 8 miles
c. 8 \(\frac{4}{5}\) miles
d. 17 \(\frac{3}{5}\) miles

Answer: 8 \(\frac{4}{5}\) miles

Explanation:
Gia’s bus route to school is 5 \(\frac{1}{2}\) miles. The bus route home is 1 \(\frac{3}{5}\) times as long.
5 \(\frac{1}{2}\) × 1 \(\frac{3}{5}\)
Convert the mixed fractions to improper fractions.
\(\frac{11}{2}\) × \(\frac{8}{5}\) = \(\frac{88}{10}\) = \(\frac{44}{5}\)
The mixed fraction of \(\frac{44}{5}\) is 8 \(\frac{4}{5}\) miles
Therefore the answer is option C.

Question 17.
Carl’s dog weighs 2 \(\frac{1}{3}\) times what Judy’s dog weighs. If Judy’s dog weighs 35 \(\frac{1}{2}\) pounds, how much does Carl’s dog weigh?
Options:
a. 88 \(\frac{3}{4}\) pounds
b. 82 \(\frac{5}{6}\) pounds
c. 81 \(\frac{2}{3}\) pounds
d. 71 pounds

Answer: 82 \(\frac{5}{6}\) pounds

Explanation:
Carl’s dog weighs 2 \(\frac{1}{3}\) times what Judy’s dog weighs.
To find the weigh of Carl’s dog we need to multiply the fractions
2 \(\frac{1}{3}\) and 35 \(\frac{1}{2}\)
\(\frac{7}{3}\) × \(\frac{71}{2}\) = \(\frac{497}{6}\)
The mixed fraction of \(\frac{497}{6}\) is 82 \(\frac{5}{6}\) pounds.
Thus the correct answer is option B.

Question 18.
In a fifth grade class, \(\frac{4}{5}\) of the girls have brown hair. Of the brown-haired girls, \(\frac{3}{5}\) of the girls have long hair. What fraction of the girls in the class have long brown hair?
Options:
a. \(\frac{1}{20}\)
b. \(\frac{1}{5}\)
c. \(\frac{3}{5}\)
d. \(\frac{1}{4}\)

Answer: \(\frac{1}{5}\)

Explanation:
In a fifth grade class, \(\frac{4}{5}\) of the girls have brown hair. Of the brown-haired girls, \(\frac{3}{5}\) of the girls have long hair.
\(\frac{4}{5}\) – \(\frac{3}{5}\) = \(\frac{1}{5}\)
The correct answer is option B.

Chapter Review/Test – Page No. 336

Constructed Response

Question 19.
Tasha plans to tile the floor in her room with square tiles that are \(\frac{1}{4}\) foot long. Will she use more or fewer tiles if she is only able to purchase square tiles that are \(\frac{1}{3}\) foot long? Explain.
_________ tiles

Answer: Fewer

Explanation:
\(\frac{1}{4}\) is less than \(\frac{1}{3}\)
So, Tasha will use fewer tiles if she is only able to purchase square tiles that are \(\frac{1}{3}\) long.

Performance Task

Question 20.
For a bake sale, Violet wants to use the recipe below.
Go Math Grade 5 Answer Key Chapter 7 Multiply Fractions Chapter Review/Test img 27
A). If she wants to double the recipe, how much flour will she need?
_____ \(\frac{□}{□}\) cups flour

Answer: 5 \(\frac{1}{2}\) cups flour

Explanation:
To bake the sugar cookies she needs 2 \(\frac{3}{4}\) cups flour.
If she wants to double the recipe, she needs to multiply the 2 \(\frac{3}{4}\) cups flour by 2.
2 \(\frac{3}{4}\) + 2 \(\frac{3}{4}\) = 5 \(\frac{1}{2}\) cups flour

Question 20.
B). Baxter wants to make 1 \(\frac{1}{2}\) times the recipe. Will he need more or less sugar than Violet needs if she doubles the recipe? Explain.
__________ sugar

Answer: less

Explanation:
If violet doubles the recipe he need 2 × 1 \(\frac{1}{2}\) = 3 cups of sugar
Baxter wants to make 1 \(\frac{1}{2}\) times the recipe.
1 \(\frac{1}{2}\) × \(\frac{1}{2}\) = 1 \(\frac{3}{4}\)
Baxter needs less sugar when compared to Violet’s recipe.

Question 20.
C). As shown, the recipe makes 60 cookies. Jorge wants to bring 150 cookies. How much flour will he need to make 150 cookies? Explain how you got your answer. (Hint: what can you multiply 60 by to get 150?)
_____ \(\frac{□}{□}\) cups flour

Answer: 2 \(\frac{1}{2}\) cups flour

Explanation:
The recipe makes 60 cookies. Jorge wants to bring 150 cookies.
Let the amount of flour be x.
60 × x = 150
x = 150/60 = 5/2
The mixed fraction of \(\frac{5}{2}\) is 2 \(\frac{1}{2}\)
Thus, Jorge need 2 \(\frac{1}{2}\) cups flour to make 150 cookies.

Conclusion

Make the most out of the Go Math Answer Key for Grade 5 Ch 7 Multiply Fractions. Our guide on 5th Grade Chapter 7 Multiply Fractions helps you to clarify the concepts related in no time. Keep in touch with our site to avail Go Math Answer Key for different Grades all in one place.

Eureka Math Algebra 1 Answer Key | Algebra 1 Engage NY Eureka Solutions

eureka-math-algebra-1-answer-key

Learn math in a fun way by using the Eureka Math Algebra 1 Answer Key. Avail of the Engage NY Math Textbook Algebra 1 Answer Key and be prepared for your tests. Solve the Eureka Math Algebra 1 Solution Key PDF and take the tests with the utmost confidence. Get module-wise Solutions for Algebra 1 provided here through quick links attached in the below section and learn the topics thoroughly.

Engage NY Eureka Math Algebra 1 Solution Key | Eureka Math Algebra 1 Answer Key

With the help of Eureka Math Algebra 1 Answer Key students can analyze and explain the process of solving the equations. By repeated reasoning, students can develop fluency in solving the problems. We provide the solutions in different methods in an easy way. Get the Eureka Math Algebra 1 Solution Key Student Materials here. Click on the links you wish to practice and Download Engage NY Eureka Math Algebra 1 Solutions Pdf for free of cost.

  • Eureka Math Algebra 1 Module 1 Answer Key
  • Eureka Math Algebra 1 Module 2 Answer Key
  • Eureka Math Algebra 1 Module 3 Answer Key
  • Eureka Math Algebra 1 Module 4 Answer Key
  • Eureka Math Algebra 1 Module 5 Answer Key

Why you should refer to Algebra 1 Eureka Math Textbook Answers?

You will have plenty of benefits by going through the Module-wise Engage NY Eureka Math Algebra 1 Answers PDF. They are listed as follows

  1. Develop your level of math skills by practicing from the Engage NY Math Algebra 1 Answer Key Pdf.
  2. By using this material students can learn the concepts easily and cover their knowledge gap.
  3. You can secure the highest marks in the exams by referring to the Engage NY Eureka Math Answer Key of Algebra 1.

FAQs on Engage NY Math Algebra 1 Solution Key

  1. Which website offers Engage NY Math Algebra 1 Answer Key for free?
    ccssmathanswers.com is a trustworthy portal that offers Eureka Math Book Algebra 1 Solution Key free of cost. Kick start your preparation using the study material available and make fun learning.
  2. How to Download Algebra 1 Engage NY Eureka Math Answer Key Modulewise?
    Students can download Engage NY Eureka Math Algebra 1 Answer Key by just tapping the links provided in the ccssmathanswers.com website.
  3. Can I get the Engage NY Math Algebra 1 Solutions in pdf format?
    Yes of course the students can get the Engage NY Algebra 1 Math Answers in pdf format so that you prepared in the offline mode.

Eureka Math Algebra 2 Answer Key | Algebra 2 Engage NY Eureka Solutions

eureka-math-geometry-answer-key

Wanna become a master in maths? This is the right destination to learn and score maximum marks in the exams. Eureka Math Algebra 2 Answer Key provides students unlimited practice, real-time feedback, and also different question types and learning aids. So, the students can grab this wonderful opportunity of learning math problems easily with the free Engage NY Eureka Math Algebra 2 Answer Key pdf. Engage NY Math Algebra 2 Solution Key PDF Modulewise and start your learnings in a fun and engaging way.

Download Engage NY Eureka Math Algebra 2 Answer Key Free Pdf

All the question and answers covered in this Engage NY Eureka Math Algebra 2 Answer Key are arranged by award-winning Mathematics educators in a systematic way to understand the concepts deeply. Hence make use of the given handy solutions of all chapters covered in our Engage NY Math Algebra 2 Solutions and learn all math concepts easily whenever you want. All you have to do is to click on the links provided below. We have provided the Answer Key for Eureka Math Algebra 2 from Module 1 to Module 4.

  • Eureka Math Algebra 2 Module 1 Answer Key
  • Eureka Math Algebra 2 Module 2 Answer Key
  • Eureka Math Algebra 2 Module 3 Answer Key
  • Eureka Math Algebra 2 Module 4 Answer Key

What are the benefits of solving Eureka Algebra 2 Solution Key?

Modulewise Algebra 2 Eureka Math Answers PDF provided here are very essential for students to prepare well for the exams.

  1. Practicing the Math concepts from Engage NY Eureka Math Textbook Algebra 2 Answer Key will be a great help for students and teachers in their Math Journey.
  2. With the help of Eureka Math Algebra 2 Answer Key you can finish your homework in time.
  3. You can enhance your performance in the practice test, assignments by using the Engage NY Algebra 2 Answer key.
  4. It helps to improve your logical thinking and become master in math.

FAQs on Eureka Math Algebra 2 Answer Key Modulewise

  1. How do I Use Engage NY Eureka Math Algebra 2 Answer Key to Score better marks?
    You can utilize Eureka Math Algebra 2 Answer Key and learn the tricks to solve all types of Math problems from all modules. By practicing and answering the Math sums provided in the Engage NY Eureka Math Textbook Algebra 2 Answer Key Pdf will help you to score the highest marks in math exams.
  2. Where do I get Module 1 to Module 4 Eureka Math Algebra 2 Solutions?
    You can get Module Wise Engage NY Eureka Math Algebra 2 Answers in PDF Format via quick links available on our site ie, on ccssmathanswers.com
  3. How to Download Eureka Math Algebra 2 Answer Key for all modules?
    Simply tap the direct links provided on our page and download the Engage NY Eureka Math Algebra 2 Answers in pdf format for all chapters for free of cost. You can prepare online or offline mode from ccssmathanswers.com

Eureka Math Geometry Answer Key | Geometry Engage NY Eureka Math Solution Key

eureka-math-algebra-2-answer-key

Students who are in search of Eureka Math Geometry Answer Key can get them on this page. You can find Modulewise Engage NY Eureka Math Geometry Solution Key (from module 1 to module 5) here. Know precise definitions of angles, circles, parallelograms, trapezoid, regular polygon, and so on from Engage NY Math Geometry Answer Key. Hence Download Geometry Engage NY Eureka Math Answer Key for free. Motivate your kids to improve in their Math Skills and make their learning fun using our Eureka Math Geometry Answer Key. The engaging and interactive approach present here meets up student’s requirements.

Free Eureka Math Geometry Answer Key Pdf Download | Engage NY Eureka Math Textbook Geometry Solutions

Get free step by step explanations for all the modules in Eureka Math Geometry Answer Key. The solutions provided on this page are prepared by the professional experts. So, the students who are lagging in geometry can go through our Engage NY Eureka Math Textbook Geometry Solutions. All you have to do is to click on the Eureka Math Geometry Answer Key from Module 1 to Module 5. This material is a collection of module assessments, lessons, practice tests, etc. Thus enhance your math skills by referring to our Geometry Engage NY Eureka Math Solution Key.

  • Eureka Math Geometry Module 1 Answer Key
  • Eureka Math Geometry Module 2 Answer Key
  • Eureka Math Geometry Module 3 Answer Key
  • Eureka Math Geometry Module 4 Answer Key
  • Eureka Math Geometry Module 5 Answer Key

Benefits of Solving Eureka Math Geometry Modulewise Answer Key

Both students and teachers can have a plethora of benefits in their study with Geometry Engage NY Eureka Math Solution Key pdf. Some of the advantages are shown below.

  1. Engage NY Eureka Math Textbook Geometry Solutions will be of extreme help for students to lay a deeper understanding of geometry.
  2. Solutions prepared for Eureka Math Geometry Answer Key are as per the Leading Curriculum and helps you score better scores in the exams.
  3. Even teachers can make use of these Eureka Math Book Geometry Answer Key in the classrooms to clarify the doubts of students.

FAQs on Eureka Math Geometry Modulewise Answer Key

  1. Where can I find the Eureka Math Geometry Answer Key for all modules?
    You can find the Engage NY Eureka Math Textbook Geometry Solutions for all Chapters from our page i.e., ccssmathanswers.com. Click on the corresponding chapter link and prepare as per your wish.
  2. How to download Engage NY Eureka Math Geometry Answer Key?
    Just tap the quick links available to access the Eureka Math Geometry Modulewise Answer Key. Then, you will be directed to a new page having the download option.
  3. How do I use the Engage NY Eureka Math Textbook Geometry Solutions to help me teach math?
    You can use the Engage NY Math Book Geometry Answer Key Modulewise to teach maths by practicing from them.

Eureka Math Grade 8 Answer Key | Engage NY Math 8th Grade Answer Key Solutions

eureka-math-grade-8-answer-key

Eureka Math Grade 8 Answer Key provides a road map for implementing modules across a school year. We have curated the common collection of Eureka Math Book Solutions for Grade 8 Students on this page. Get Module wise Engage NY Math Grade 8 Answers Download pdf for free of cost. Students of 8th Class can get the complete knowledge of math and learn various problem solving math. Learn, Practice, and Succeed using the Extra Practice, Assessment Tests, Chapter Test, Review Tests available in Grade 8 Eureka Math Answers.

EngageNY Math Grade 8 Answer Key | Eureka Math 8th Grade Answers Key PDF Free Download

Every concept seen in Eureka Math Grade 8 Solution Key is provided with a step-by-step solution in order to make your preparation effective. You can solve the number of questions with the help of Eureka Engage NY Math Book Solutions for Grade 8. Tap the links given below to Download Engage NY Math Grade 8 Answer Key Pdf and start learning the concepts.

Preparation Tips for Grade 8 Students

  1. First, prepare the list of the topics
  2. Go through the previous question papers to start the preparation
  3. Prepare the timetable
  4. Try to study in chunks if possible
  5. Rewrite your notes

What are the benefits of using Eureka Engage NY Math Book 8th Grade Solutions?

There are several advantages of referring to Engage NY Grade 8 Math Answer Key. Some of the benefits are given in the below section.

  1. Parents who want to help their children to complete homework in time can make use of the Eureka Math Grade 8 Solution Key Pdf.
  2. You can learn the concepts in-depth with the help of Eureka Grade 8 Math Solution Key.
  3. You can download the Eureka Math Textbook Grade 8 Answer Key in PDF Format for free of cost and prepare it anywhere and anytime.

FAQs on Eureka Math Answers For Grade 8

1. Where do I find Module 1 to Module 7 Eureka Math Grade 8 Answers?
You can get Modulewise Eureka Math Grade 8 Answers in pdf Format via quick links available on our website i.e., ccssmathanswers.com

2. How to download Eureka Math Grade 8 Answer Key?
Click on the links to download Engage NY Math 8th Grade Solution Key of all modules for free of cost.

3. Is there any portal that provides the Eureka Math Solutions for Grade 8?
ccssmathanswers.com is a trusted portal that provides the Grade 8 Eureka Math Answers from Module 1 to Module 7.

Eureka Math Grade 7 Answer Key | Engage NY Math 7th Grade Answer Key Solutions

eureka-math-grade-7-answer-key

Quick and easy learning is possible with our Eureka Math Grade 7 Answer Key. The students who are willing to become master in math by solving the problems from Eureka Math Answer Key for 7th Grade. Overcome all your difficulties in maths and fill up the knowledge gap by practicing the problems from our Eureka Math 7th Grade Solution Key. Hit the link you wish and start practicing the problems from Engage NY Eureka Math Grade 7 Answers. Also, you will find some chapter tests, quizzes, standard assessments for each Module in Eureka Math Grade 7 Solution Key.

EngageNY Math Grade 7 Answer Key | Eureka Math 7th Grade Answers Key PDF Free Download

In order to assist the students with the implementation of common core, we have provided the solutions in an easy manner. See more ideas about the Eureka math from this page. The resources consist of the links of the Eureka Math Grade 7 Answers from Module 1 to Module 6. Go through the below sections to know the advantages of using Eureka Math Grade 7 Answer Key. Learn math in a fun way with the help of Engage NY Math Textbook Grade 7 Answer Key. Here is the list of all the math skills that students in the 7th class to start practicing.

Advantages of Solving Module Wise Engage NY Math Book Answer Key

Compiled resources of Eureka Math Grade 7 Solutions Module wise helps the students to enhance their skills. We have highlighted some of the benefits of referring Engage NY Eureka Math Grade 7 Answer Key.

  • You can discover more questions from Practice Test in Eureka Math Grade 7 Answer Key for better practice and enhance the math skills.
  • Not only students teachers can also use Eureka Math 7th Grade Solution Key in the classroom to clarify the doubts.
  • The solutions provided in the Engage NY Eureka Math Grade 7 Answer Key are prepared by the subject experts as per the latest common core curriculum.
  • With the help of Eureka Math Grade 7 Solutions Pdf, you can score maximum marks in the exams.

FAQs on Eureka Math Grade 7 Answer Key

1. Can I get the Answer Key for Eureka Math Grade 7 in pdf format?
Yes, the students of the 7th class can download Eureka Math Grade 7 Answer Key in pdf format for free of cost.
2. Can I get the Engage NY Math Grade 7 Answer Key for all the Modules?
You can get Module Wise Eureka Math Grade 7 Answers in PDF Format via quick links available on our website ie., ccssmathAnswers.com
3. How to access the pdf formatted Engage NY Math Grade 7 Solution Key offline?
In order to access the pdf formatted Engage NY Math Grade 7 Solution Key offline you just need to click on the links available here and download them to practice online or offline mode.